TAP Vol 6 Issue 13

Page 1

Relapsed Multiple Myeloma

3

| Novel EGFR Inhibitors for NSCLC

37, 41

| Hematology Quiz

VOLUME 6, ISSUE 13

60

JULY 25, 2015

Editor-in-Chief, James O. Armitage, MD | ASCOPost.com

ASCO Annual Meeting

Obinutuzumab Moves Into Lymphoma Treatment

Collaborating Toward a Cure

By Alice Goodman

T

he addition of obinutuzumab (Gazyva) to standard bendamustine (Treanda) chemotherapy followed by obinutuzumab maintenance therapy delays the progression of indolent non-Hodgkin lymphoma (NHL), almost doubling progression-free survival in rituximab (Rituxan)-refractory patients, according to the results of the phase III GADOLIN trial presented at the 2015 ASCO Annual Meeting.1 Median progression-free survival was 29.2 months with obinutuzumab/bendamustine plus maintenance obinutuzumab vs 14 months with bendamustine monotherapy and no maintenance. These results were so robust that the trial was halted early. “Indolent NHL is incurable with standard therapy. Rituximab improves overall survival and progressionfree survival, but some patients do not respond to rituximab. Others become resistant to rituximab, and these patients have a poor outcome and few treatment options,” explained lead author Laurie Sehn, MD, of the British Columbia Cancer Agency and the University of British Columbia, Vancouver, Canada.

“This study is a remarkable first demonstration of the benefit of a novel antibody for patients who are rituximab-refractory. Results are statistically significant and clinically meaningful, with no new safety signals. Based on these results, bendamustine plus obinutuzumab followed by obinutuzumab maintenance is a novel and effective treatment option Laurie Sehn, MD for patients with indolent NHL refractory to rituximab,” she stated.

Study Details

W

e’ve seen how dramatically patients’ lives can change when they are matched with the right treatment at the right time in their disease course. Although this is still an exception and not the rule, we believe collaborative research approaches will make this kind of precision medicine a reality for all patients with cancers. At the Multiple Myeloma Research Foundation (MMRF), collaboration is not just a priority; it is the only way we do things. We’ve worked shoulder to shoulder with the best partners in the continued on page 98

GADOLIN randomized 413 patients to receive obinutuzumab/bendamustine followed by obinutuzumab maintenance vs bendamustine alone with no maintenance. In the combination arm, bendamustine continued on page 6

Perspective

ASTRO Guidelines on Radiotherapy in Locally Advanced NSCLC: A Good Place to Start By Andreas Rimner, MD

A

By Kathy Giusti

SCO’s endorsement of the American Society for Radiation Oncology (ASTRO) guidelines on the role of radiotherapy in locally advanced non–small cell lung cancer (NSCLC) is an important summary of the best evidence on the appropriate role and techniques for radiotherapy in this disease.1 These guidelines—reviewed in this issue of The ASCO Post (page 42)—have been developed through a rigorous ASCO Clinical Practice Guideline Committee review process to ensure

the highest quality of guideline endorsement possible. Clinical guidelines serve practitioners by improving the consistency of care, avoiding unnecessary or harmful interventions, informing patients about their choices, and providing orientation points for policy makers and insurance carriers. If followed, clinical guidelines should benefit patients and the health-care system as a whole.2 However, clinical guidelines cannot replace the individual treatment approach to a given patient, Clinical guidelines are the summary taking into consideration his or her unique set of of the best evidence on the use of comorbidities, beliefs, and radiotherapy, but they cannot replace willingness to undergo more- or less-aggressive the individual treatment approach to treatment. Guidelines also a given patient.... cannot provide compre—Andreas Rimner, MD hensive details of ­radiation

Ms. Giusti is a multiple myeloma survivor and the Founder and Executive Chairman of the Multiple ­Myeloma Research Foundation. She is also a member of the President’s Advisory Board for the Precision Medicine Initiative. Disclaimer: This commentary represents the views of the author and may not necessarily reflect the views of ASCO.

MORE IN THIS ISSUE Oncology Meetings Coverage ASCO Annual Meeting Hematology ��������������������������������� 1, 20–22 Multiple Myeloma ��������������������������������������� 3 Genitourinary Cancer ��������������������������������� 4 Pancreatic Cancer ��������������������������������������� 9 Colorectal Cancer ������������������������������������� 16 Brain Tumors ��������������������������������������������� 19 Melanoma ��������������������������������������������������23 Precision Medicine �����������������������������������25 Direct From ASCO �������������������������� 46–49 Geriatrics for the Oncologist �������������������53 Inside the Black Box �������������������������������� 68 In Memoriam ������������������������������������ 92–94

continued on page 44

Send your comments to editor@ASCOPost.com

A Harborside Press® Publication


The ASCO Post  |   JULY 25, 2015

PAGE 2

Harborside Press® Publishing Staff

Editorial Board

Conor Lynch, Executive Editor Conor@harborsidepress.com

James O. Armitage, MD Editor-in-Chief

Bishoy Morris Faltas, MD Weill Cornell Medical College

George W. Sledge, MD Indiana University

Cara H. Glynn, Director of Editorial Cara@harborsidepress.com

Elizabeth Reed, MD Deputy Editor University of Nebraska Medical Center

John A. Fracchia, MD New York Urological Associates

Thomas J. Smith, MD Sidney Kimmel Comprehensive Cancer Center at Johns Hopkins

Andrew Nash, Associate Director of Editorial Andrew@harborsidepress.com

Associate Editors

Alison Freifeld, MD University of Nebraska Medical Center

Jame Abraham, MD Cleveland Clinic

Louis B. Harrison, MD Moffitt Cancer Center

Syed A. Abutalib, MD Cancer Treatment Centers of America

Jimmie C. Holland, MD Memorial Sloan Kettering Cancer Center

Manmeet Ahluwalia, MD, FACP Cleveland Clinic

Clifford A. Hudis, MD, FACP Memorial Sloan Kettering Cancer Center

Chandrakanth Are, MD University of Nebraska Medical Center

Nora Janjan, MD, MPSA, MBA National Center for Policy Analysis

Joseph S. Bailes, MD Texas Oncology

Hagop M. Kantarjian, MD MD Anderson Cancer Center

Laurence H. Baker, DO University of Michigan Health System

Mario E. Lacouture, MD Memorial Sloan Kettering Cancer Center

Richard R. Barakat, MD Memorial Sloan Kettering Cancer Center

Theodore S. Lawrence, MD, PhD University of Michigan Comprehensive Cancer Center

Charles L. Bennett, MD, PhD, MPP University of South Carolina, Columbia

Jamie Von Roenn, MD American Society of Clinical Oncology Lynn D. Wilson, MD Yale University School of Medicine Stanley H. Winokur, MD Singer Island, Florida William C. Wood, MD Winship Cancer Institute, Emory University

International Editors Clement Adebamowo, BM, ChB (Hons), ScD University of Ibadan, Nigeria Eduardo Cazap, MD, PhD International Union Against Cancer (UICC) Buenos Aires, Argentina

Stephen J. Lemon, MD, MPH Oncology Associates, PC, Omaha

Rakesh Chopra, MD Artemis Healthsciences Institute Gurgaon, Haryana, India

Philip D. Bonomi, MD Rush University Medical Center

Stuart Lichtman, MD Memorial Sloan-Kettering Cancer Center Commack, New York

Nagi El-Saghir, MD American University of Beirut, Lebanon

Richard Boxer, MD University of Wisconsin School of Medicine

Michael P. Link, MD Stanford University Medical Center

Harold J. Burstein, MD Dana-Farber Cancer Institute

John L. Marshall, MD Ruesch Center for the Cure of GI Cancer at Georgetown University

Douglas W. Blayney, MD Stanford University Medical Center

Robert W. Carlson, MD National Comprehensive Cancer Network Barrie R. Cassileth, PhD Memorial Sloan Kettering Cancer Center Jay S. Cooper, MD Maimonides Medical Center John Cox, DO Texas Oncology E. David Crawford, MD University of Colorado Nancy E. Davidson, MD University of Pittsburgh Cancer Institute George D. Demetri, MD Dana-Farber Cancer Institute Paul F. Engstrom, MD Fox Chase Cancer Center David S. Ettinger, MD Sidney Kimmel Comprehensive Cancer Center at Johns Hopkins

The ASCO Post (ISSN 2154-3283), USPS Publicaton Number 6885, is published semi-monthly, except monthly in January by Harborside Press®, LLC, 37 Main Street, Cold Spring Harbor, NY 11724, under a license arrangement with the American Society of Clinical Oncology, Inc. (ASCO®). Periodicals Postage paid at Cold Spring Harbor, NY, and additional mailing offices. Change of Address: Postmaster send address changes to The ASCO Post, c/o Harborside Press®, LLC, 37 Main Street, Cold Spring Harbor, NY 11724. ASCO Members: If you would like to cancel your subscription to The ASCO Post or need to update your mailing address, please visit your personalized page on ASCO.org. For personalized service, please contact ASCO Member Services at (888) 282-2552, (703) 299-0158, or via email at membermail@asco.org. Non ASCO Members: To initiate or cancel a subscription or to update your mailing address, please email subscriptions@harborsidepress.com or fax (631) 692-0805. Copyright ©2015 by Harborside Press®, LLC. All rights reserved. Reproduction in whole or in part, in any form, without prior written permission of the publisher is pro-

Mary Gospodarowicz, MD Princess Margaret Hospital Toronto, Ontario, Canada Jacek Jassem, MD Medical University of Gdansk, Poland

Mary S. McCabe, RN, MA Memorial Sloan Kettering Cancer Center

David Khayat, MD Pitie-Salpetriere Hospital, Paris, France

William T. McGivney, PhD Philadelphia, Pennsylvania

Tony Mok, MD The Chinese University of Hong Kong Shatin, Hong Kong

James L. Mulshine, MD Rush University Medical Center Derek Raghavan, MD, PhD Levine Cancer Institute Carolinas HealthCare System Steven T. Rosen, MD City of Hope National Medical Center Lee S. Schwartzberg, MD University of Tennessee Health Science Center Andrew D. Seidman, MD Memorial Sloan Kettering Cancer Center Samuel Silver, MD, PhD University of Michigan Health System

hibited. For permission inquiries, contact permissions@ harborsidepress.com. Editorial Mission: The ASCO Post communicates timely information to a broad audience of oncology specialists, helping to advance the highest quality multidisciplinary cancer care. The ASCO Post publishes highly validated coverage of cancer research and policy news, patient care and clinical practice issues, and thoughtful commentary from leaders in the field and others with an interest in clinical oncology. Circulation: The ASCO Post is sent free of charge to approximately 27,000 physicians and nurses, including all US-based ASCO members. Medical, surgical, pediatric, and gynecologic oncologists, hematologists, and hematologist/oncologists in the United States who are not members of ASCO will be eligible for a complimentary subscription. ASCO members outside of the United States receive complimentary access to The ASCO Post online at www.ASCOPost.com.

Paid subscriptions to The ASCO Post are available for all other interested individuals. Individual Domestic: $300;

Eliezer Robinson, MD National Council for Oncology Israeli Cancer Association, Haifa, Israel Nagahiro Saijo, MD, PhD Kinki University School of Medicine Osaka, Japan John F. Smyth, MD University of Edinburgh Edinburgh, Scotland Daniel A. Vorobiof, MD Sandton Oncology Centre Johannesburg, South Africa

Canada: $436; Individual International: $575; Institutional Domestic: $370; Canada: $507; Institutional International: $645. Single Copy Domestic: $57; Canada: $65; International: $72. Contact subscriptions@harborsidepress.com.

Jo Cavallo, Senior Editor and Correspondent Jo@harborsidepress.com Randi Londer Gould and Susan Reckling, Senior Editors Randi@harborsidepress.com Susan@harborsidepress.com Sarah McGullam, Web Editor Sarah@harborsidepress.com Michael Buckley, Art Director Michael@harborsidepress.com Regine M. Lombardo, Senior Graphic Designer Regine@harborsidepress.com Terri Caivano, Layout Artist Terri@harborsidepress.com Gail van Koot, Editorial Coordinator Gail@harborsidepress.com Elizabeth Janetschek, Editorial Assistant Elizabeth@harborsidepress.com Norman Virtue, Production Manager Norman@harborsidepress.com Shannon Meserve, Circulation Manager Shannon@harborsidepress.com Jeannine Coronna, Vice President, Director of Operations Jeannine@harborsidepress.com Frank Buchner, Chief Technology Officer Frank@harborsidepress.com Leslie Dubin, Vice-President, Director of Sales Leslie@harborsidepress.com Anthony Cutrone, President Anthony@harborsidepress.com John A. Gentile, Jr, Chairman Jack@harborsidepress.com

Contributing Writers: Charlotte Bath, Margot Fromer, Alice Goodman, Caroline Helwick, Susan London, Caroline McNeil, Eileen O’Gara-Kurtis, Ronald Piana, Matthew Stenger

Contributing Artists: Portraits by Keith Witmer, Keith Witmer Illustrations. Disclosure information available at ASCOPost.com.

Advertising: For information on advertising rates, reprints, or supplements, contact Leslie Dubin, phone: 631.935.7660; e-mail: leslie@harborsidepress.com.

Advertiser and advertising agency accept and assume liability for all content (including text, representations, illustrations, opinions, and facts) of advertisements printed, and also assume responsibility for any claims made against the publisher or ASCO arising from or related to such advertisements. In the event that legal action or a claim is made against the publisher or ASCO arising from or related to such advertisements, advertiser and advertising agency agree to fully defend, indemnify, and hold harmless the publisher and ASCO, and to pay any judgment, expenses, and legal fees incurred by the publisher and by ASCO as a result of said legal action or claim. The publisher reserves the right to reject any advertising that it believes is not in keeping with the publication’s standards.

Notice to Advertisers: Advertiser and advertising agency recognize and accept that the following language appears within the publication: “All statements, including product claims, are those of the person or organization making the statement or claim. Neither the publisher nor ASCO adopts any such statement or claim as its own, and any such statement or claim does not necessarily reflect the opinion of the publisher or ASCO.”

The publisher is not liable for delays in delivery and/or non-delivery in the event of Act of God, action by any government or quasi-governmental entity, fire, flood, insurrection, riot, explosion, embargo, strikes (whether legal or illegal), labor or material shortage, transportation interruption of any kind, work slow-down, or any condition beyond the control of the publisher affecting production or delivery in any manner.

Correspondence: Address general inquiries to Harborside Press®, LLC, 37 Main Street, Cold Spring Harbor, NY 11724. Phone: 631.692.0800; Fax: 631.692.0805. Address editorial correspondence to James O. Armitage, MD, Editor-in-Chief, c/o Cara Glynn, phone: 631.935.7654; e-mail: cara@harborsidepress.com.

Disclaimer: The ideas and opinions expressed in The ASCO Post™ do not necessarily reflect those of Harborside Press®, LLC, HSP News Service, LLC, or the American Society of Clinical Oncology, Inc. (ASCO®). The mention of any product, service, or therapy in this publication should not be construed as an endorsement of the products mentioned. It is the responsibility of the treating physician or other health-care provider, relying on independent experience and knowledge of the patient, to determine the appropriate treatment for the patient. Readers are advised to check the appropriate medical literature and the product information currently provided by the manufacturer of each product or therapy to be administered to verify the dosage, method, and duration of administration, or contraindications. Readers are also encouraged to contact the manufacturer with questions about the features or limitations of any products. Harborside Press®, HSP News Service, LLC, and ASCO® assume no responsibility for any injury or damage to persons or property arising out of or related to any use of material contained in this publication or to any errors or omissions.


ASCOPost.com  |   JULY 25, 2015

PAGE 3

ASCO Annual Meeting Hematology

Carfilzomib/Dexamethasone Doubles Progression-Free Survival vs Bortezomib/Dexamethasone in Patients With Relapsed Multiple Myeloma By Caroline Helwick

I

n the first head-to-head study comparing two proteasome inhibitors in relapsed multiple myeloma, carfilzomib (Kyprolis)/dexamethasone provided a doubling in progression-free survival, compared with bortezomib (Velcade)/ dexamethasone.1 Results of the phase III ENDEAVOR trial of the two regimens in previously treated patients were reported at the 2015 ASCO Annual Meeting by Meletios A. ­Dimopoulos, MD, of the National and Kapodistrian University of Athens in Greece. “In what is one of the largest trials ever done in multiple myeloma, carfilzomib/dexamethasone resulted in a twofold decrease in the risk of progression or death, compared with bortezomib/ dexamethasone,” Dr. Dimopoulos said. Median progression-free survival was 18.7 months with carfilzomib/ dexamethasone vs 9.4 months with

bortezomib/dexamethasone, for a hazard ratio (HR) of 0.53 (P < .0001).

Carfilzomib/dexamethasone was superior to bortezomib/ dexamethasone, regardless of age or prior bortezomib exposure, and represents a new standard of care.

Study Details and Results ENDEAVOR included 929 multiple myeloma patients who had received one to three prior treatments. Treatment involved a higher dose of carfilzomib (56 mg/m2 on a 28-day cycle) than the U.S. Food and Drug Administration–approved dose for the drug as a single agent (27 mg/m2). This was based on observations that, in combination with dexamethasone, the 56-mg dose produced higher response rates, Dr. Dimopoulos pointed out. The ­bortezomib dose was 1.3 mg/m2 by intravenous bolus or subcutaneous injection, and the dexamethasone dose was 20 mg in each arm. Approximately half the patients in each arm had received prior bortezo-

—Meletios A. Dimopoulos, MD

mib and prior thalidomide (Thalomid); 38% had been treated with lenalidomide (Revlimid), and fewer than 1% had received carfilzomib. At a median follow-up of 11.2 months, risk of progression was reduced by 47% in the carfilzomib/dexamethasone arm (P < .0001). All subgroups benefited more from the carfilzomib/ dexamethasone regimen, with the ex-

EXPERT POINT OF VIEW

J

effrey L. Wolf, MD, Director of the Myeloma Program at the University of California, San Francisco, formally discussed the ENDEAVOR study at the ASCO Annual Meeting. He commented that the “doubling” in progression-free survival is “impressive” and said that “time will tell if this difference persists for overall survival.”

Jeffrey L. Wolf, MD

He said he was “reassured” that the progression-free survival of ­bortezomib (Velcade)/dexamethasone in ENDEAVOR was consistent with that seen in the PANORAMA trial (8 months). It was also important that prior exposure to bortezomib did not compromise the efficacy of the carfilzomib (Kyprolis)/dexamethasone regimen and, moreover, that for bortezomib-naive patients, median progression-free survival had not been reached.

The greater toxicity in terms of dyspnea, cardiac events, and hypertension with carfilzomib/dexamethasone was expected, he said, and the higher incidence of peripheral neuropathy with bortezomib is “similar to what we were used to seeing before subcutaneous injections and easily explained by the twice-weekly schedule in the 21% of patients who received bortezomib by intravenous bolus.” Dr. Wolf was impressed that the doubling of the carfilzomib dose did not significantly increase toxicity over that observed in ASPIRE, where the standard dose was used. He suggested, “If 56 mg/m2 is the new optimal dose of carfilzomib, we will have to start looking at this dose in other combinations and watch for additive toxicity.”

New Treatment Landscape In the “new landscape” of treatment for myeloma patients with one to three relapses, regimens are producing response rates of 75% or higher, with many patients achieving very good partial responses and even stringent complete responses. “We can hope for progression-free survival in this arena to exceed a year and a half, or even 26 months, as

seen in ASPIRE,” he said. “This is a wonderful place to be for our patients, but at what cost?” he asked. He estimated the most effective regimens will cost around $22,000 per month, which (based on trial data) translates into a “cost per progression-free month” ranging from around $1,000 to nearly $2,000. “If the progression-free survival is good enough and the tail of the curve significant enough, the increased cost may be worth it,” he commented, “and if a regimen is curative in the front-line setting, almost any cost would be acceptable.” But expensive drugs used in unselected patients are causing toxicity and increasing the cost of care, “often with little benefit,” he suggested. “The costs of these complex regimens are increasing and must be weighed against their performance,” Dr. Wolf concluded. Meanwhile, until this happens, “we will continue to mix and match these drugs, looking for the best combinations that we hope will ultimately cure our patients.” n

Disclosure: Dr. Wolf reported stock ownership in Celgene; having a consulting or advisory role for Onyx, Amgen, Celgene, and Janssen; receiving speaker’s fees from Millennium; and receiving travel expenses from Celgene, Onyx, Millennium, and Janssen.

ception of patients with baseline creatinine clearance < 30 mL/min, for whom the results were comparable between the arms. “We see an early separation of the [progression-free survival] curve that improves over time, indicating the continuous favorable effect of [carfilzomib/dexamethasone] in all subgroups,” he commented. Objective response rates were also significantly improved with carfilzomib/dexamethasone: 77%, vs 63% (P < .0001). Complete responses or better occurred in 13% vs 6% (P < .0001), and very good partial responses or better were seen in 54% and 29%, respectively (P < .0001). Median duration of response was 21.3 months with carfilzomib/dexamethasone and 10.4 months with bortezomib/dexamethasone. Overall survival, a secondary endpoint, is not yet mature. Median overall survival at this point is 24.3 months with bortezomib/dexamethasone but has not been reached with carfilzomib/ dexamethasone (HR = 0.79; P = .066).

Toxicity Profile Adverse events grade ≥ 3 were seen in 73% with carfilzomib/dexamethasone and 67% with bortezomib/dexamethasone. Serious adverse events were seen in 48% and 36%, respectively, and four and three patients in each group died due to toxicity. Discontinuations related to toxicity were observed in about 15% per arm. The carfilzomib/dexamethasone arm demonstrated higher rates of grade ≥ 3 hypertension (9% vs 3%), dyspnea (5% vs 2.2%), and cardiac failure (5% vs 1.8%) but significantly lower rates of peripheral neuropathy. Patients on carfilzomib/dexamethasone also had more pyrexia and cough. “Patients stayed on [carfilzomib/ continued on page 4


The ASCO Post  |   JULY 25, 2015

PAGE 4

ASCO Annual Meeting Genitourinary Oncology

Immunotherapy Marches On, Making Headway in Advanced Urothelial Bladder Cancer By Alice Goodman

T

wo immunotherapy agents show promise in preliminary studies of advanced urothelial bladder cancer: the anti–PD-1 (programmed cell death protein 1) antibody pembrolizumab (Keytruda) and the anti–PD-L1 (programmed cell death ligand 1) antibody atezolizumab. Separate phase I studies of each drug showed excellent activity and durable responses in poor-prognosis patients with urothelial bladder cancer and were reported at the 2015 ASCO Annual Meeting.1,2

months in a cohort of patients with advanced urothelial bladder cancer treated with pembrolizumab. These findings compare favorably with results of historical trials in this population, noted lead author Elizabeth Plimack, MD, of Fox Chase Cancer Center, Philadelphia. About 50% of patients in the urothelial bladder cancer cohort were alive at 12 months, and about 85% experienced only mild or no adverse events. “These results support the further development of pembrolizumab in urothelial

These results support the further development of pembrolizumab in urothelial bladder cancer and continuing investigation of novel biomarkers. —Elizabeth Plimack, MD

Metastatic urothelial bladder cancer is associated with poor outcomes. There are no U.S. Food and Drug Administration–approved therapies for relapse after platinum-based therapy, and overall survival is 5 to 7 months in the second-line setting.

Pembrolizumab Updated safety and efficacy data from the phase Ib KEYNOTE-012 study showed an overall response rate of 27.6%, a disease control rate of 38%, and median overall survival of 12.7

Carfilzomib/Dexamethasone continued from page 3

dexamethasone] longer, for a median of 40 weeks vs 27 weeks, yet the adverse event rates were the same between the groups,” he noted. Delving further into the occurrence of neuropathy, Dr. Dimopoulos reported

bladder cancer and continuing investigation of novel biomarkers,” Dr. Plimack said. Of 95 patients screened, 62.4% were PD-L1–positive at baseline, according to expression of at least 1% PD-L1– positive tumor cells in tumor nests or a PD-L1–positive band in stroma by a prototype immunohistochemistry assay. A total of 33 patients with PD-L1– positive recurrent or metastatic urothelial bladder cancer were enrolled in the phase Ib trial and treated with pembrolizumab (10 mg/kg every 2 weeks) until complete remission, disease progresgrade ≥ 3 neuropathy in 5% of the bortezomib/dexamethasone arm compared to 1.3% of the carfilzomib/dexamethasone arm. Of particular interest, grade ≥ 2 neuropathy occurred in 32% and 6%, respectively (odds ratio [OR] = 0.14; P < .0001), “despite the fact that 79% of the bortezomib/dexamethasone group

Optimizing Multiple Myeloma Treatment ■■ The phase III ENDEAVOR trial compared two proteasome inhibitors plus dexamethasone, head to head, in one of the largest myeloma trials ever. ■■ Carfilzomib/dexamethasone doubled the median progression-free survival time (18.7 months), compared to bortezomib/dexamethasone (9.4 months). ■■ The dose of carfilzomib was 56 mg/m2, which is double the FDA-indicated dose as a single agent.

Immune Checkpoint Inhibitors in Urothelial Bladder Cancer ■■ In separate preliminary studies, immunotherapy with pembrolizumab—an anti–PD-1 agent—and atezolizumab—an anti–PD-L1 agent—achieved good activity in advanced urothelial bladder cancer. ■■ Both agents will be studied further in urothelial bladder cancer. ■■ PD-L1 appears to be of limited utility as a predictive biomarker.
Better biomarkers are needed to select patients who will and will not benefit from this type of therapy.

sion, or unacceptable toxicity. The median age of study patients was 70 years; patients were predominantly male; the majority had Eastern Cooperative Oncology Group (ECOG) 1 performance status; 24% had liver metastasis; and the only metastatic site in 9% was the lymph node. No brain metastasis was allowed. About one-quarter of patients had no prior therapy, and 33% had at least three prior therapies. Immune-related events occurred in only a few patients; they included colitis, myositis, rhabdomyolysis, rash, and uveitis. Five patients experienced grade 3 or 4 treatment-related adverse events. There was one treatment-related discontinuation. Radiographic responses were observed in 8 patients (27.6%); 3 (10%) had complete remission; 5 (17.2%) had a partial response; 3 patients (10%) had stable disease; 14 patients (48.3%) had progressive disease; and 4 patients could not be assessed. The disease control rate was 38% (11 patients). Sixty-four percent of patients treated with pembrolizumab had a decrease in target lesions.

The median duration of follow-up was 15 months. The median time to response was 9 weeks; response duration ranged from 8.1 weeks to 64.1 weeks, and some patients are still in response. Three patients remain on therapy. Median progression-free survival is 2 months. Progression-free survival at 12 months is 19.1%. Median overall survival is 13 months, and 1-year survival is 52.9%. An investigative analysis of PD-L1 revealed that the best way to maximize detection of response and minimize false-positive results is to measure PDL1 expression in both tumor cells and tumor-associated inflammatory cells, Dr. Plimack explained. Among 18 patients with PD-L1– positive tumors in tumor cells only, using a different immunohistochemical method than that used for screening, 33% had an objective response according to RECIST (Response Evaluation Criteria in Solid Tumors) criteria; in 11 PD-L1–negative tumors in tumor cells only, 9% had RECIST response.

received subcutaneous bortezomib throughout their treatment,” he noted.

were 0.48 for the group naive to bortezomib and 0.56 for the group with prior bortezomib treatment. “Carfilzomib/dexamethasone was superior to bortezomib/dexamethasone, regardless of age or prior bortezomib exposure, and represents a new standard of care,” Dr. Dimopoulos concluded. n

Other Key Data The advantage of carfilzomib/dexamethasone was observed across all age groups, with hazard ratios of 0.58 for patients under age 65 and 0.53 for patients 65 to 74 years old. Interestingly, patients ≥ age 75 maintained a long progressionfree survival, 18.7 months vs 8.9 months with bortezomib/dexamethasone, a 62% risk reduction with carfilzomib/dexamethasone. Adverse events were also consistent across the age groups, he added. “The effect of prior exposure to bortezomib was a key question in the study,” he continued, “but this had no effect on our results.” Hazard ratios

Disclosure: Dr. Dimopoulos reported receiving fees for consulting or advising, and honoraria, from Celgene and Onyx.

Reference 1. Dimopoulos MA, Moreau P, Palumbo A, et al: Carfilzomib and dexamethasone vs bortezomib and dexamethasone in patients with relapsed multiple myeloma: Results from the phase III study ENDEAVOR. 2015 ASCO Annual Meeting. Abstract 8509. Presented June 2, 2015.


ASCOPost.com  |   JULY 25, 2015

PAGE 5

ASCO Annual Meeting According to analysis of PD-L1 expression in both tumor cells and tumor-associated inflammatory cells, the response rate was 29% in 24 PD-L1– postive patients and 0% among 4 PDL1–negative patients. Analysis of four gene expression signatures derived from prior studies in melanoma found that a signature related to T-cell receptor signaling was associated with improved clinical benefit and prolonged response, whereas no association was found for the other three gene expression signatures analyzed: interferon gamma, expanded immune, and de novo (33 genes) signatures.

Atezolizumab Atezolizumab (formerly known as ­ PDL3280A) had encouraging activM ity in a cohort of heavily pretreated patients with metastatic urothelial bladder cancer. PD-L1 status, as measured by an SP142 assay, appears to be predictive of benefit but not survival of atezolizumab in urothelial bladder cancer.

Daniel P. Petrylak, MD

“Survival is encouraging, and responses are clinically meaningful in this phase Ia study,” said Daniel P. ­Petrylak, MD, of Yale Cancer Center, New Haven, Connecticut.

“[Urothelial bladder cancer] is a disease of high mutational complexity and immunogenicity,” Dr. Petrylak explained. “Since PD-L1 is expressed in many tumors, we were justified in doing a phase I trial of atezolizumab in many solid tumors, including triple-negative breast cancer, melanoma, non–small cell lung cancer, renal carcinoma, and [urothelial bladder cancer]. At first, the phase I study enrolled only PD-L1 expressors, and then we took all comers,” he said. He reported the results of an ongoing dose-expansion phase of the study that included 205 patients who were screened using the highly specific SP142 to measure PD-L1 using four immunohistochemistry scoring levels. Twenty-seven percent of patients had immunohistochemistry 2 or 3 expression (high levels). “We see long-term responses in both low and high expressors of PD-L1,” Dr. Petrylak said. Adverse events were generally well tolerated, with no treatment-related deaths. Five percent of patients had immune-mediated grade 3-4 events. Overall, 40% had a grade 3 or 4 adverse event of any cause. Median time to response was 62 days. The overall response rate was 50% for immunohistochemistry 2/3 patients. Patients with low levels of PD-L1 had an overall response rate of 17%. Responders included patients with visceral metastases at baseline, with an overall response rate of 32%. A waterfall plot showed that 44 patients (55%) had a reduced tumor burden. Median duration of treatment has not yet been reached in immuno-

EXPERT POINT OF VIEW

C

ommenting on the pembrolizumab (Keytruda) phase Ib study, Noah Hahn, MD, of Johns Hopkins Sidney Kimmel Cancer

Noah Hahn, MD

Center, Baltimore, called the overall response rate of 28% with pembrolizumab “pretty impressive. Nearly two-thirds of patients had some response or tumor reduction, and overall survival was also impressive,” Dr. Hahn noted. “Over the last 25 years, response rates to new agents have ranged from 10% to 15%, and median overall survival was about 10 months. Pembrohistochemistry 2/3 patients. Median overall survival is between 10 and 14 months, Dr. Petrylak said. n Disclosure: Dr. Plimack has participated in scientific advisory boards for Merck and Genentech, and her institution has received funding from Merck related to the conduct of clinical trials, including the trial featured in this article. Dr. Petrylak has received travel expenses and honoraria from Genentech and research funding from Genentech and Merck.

lizumab more than doubles overall response rate, and overall survival of 13 months stacks up nicely against historical treatments post platinum,” Dr. Hahn noted. Pembrolizumab was well tolerated, Dr. Hahn continued, with low rates of grade 3 and 4 toxicities. Turning to the atezolizumab study presented by Dr. Petrylak, Dr. Hahn said that the overall response rate of 34% for the entire poor-prognosis, heavily pretreated cohort is “independent of marker status and impressive.” “PD-L1 is not panning out as a biomarker for atezolizumab. For right now, we have no marker for treatment selection,” Dr. Hahn said. “With new agents like pembrolizumab and atezolizumab, therapeutic options are expanded, and outcomes are improving in [urothelial bladder cancer],” he said. n Disclosure: Dr. Hahn reported no potential conflicts of interest.

References 1. Plimack ER, Bellmunt J, Gupta S, et al: Pembrolizumab (MK-3475) for advanced urothelial cancer: Updated results and biomarker analysis for KEYNOTE-012. 2015 ASCO Annual Meeting. Abstract 4502. 2. Petrylak DP, Powles T, Bellmunt J, et al: A phase Ia study of MPDL3280A (anti-PDL1): Updated response and survival data in urothelial bladder cancer. 2015 ASCO Annual Meeting. Abstract 4501.

Don’t Miss These Important Reports in This Issue of The ASCO Post Smita Bhatia, MD, MPH, on NCCS Advocacy for Medicare Survivorship Planning Legislation see page 36

Pasi A. Jänne, MD, on EGFR Inhibitors Rociletinib and AZD9291 in T790M-Positive NSCLC see page 37

Ramaswamy Govindan, MD, on Progress in EGFR-Mutated NSCLC see page 41

Amina Ahmed, MD, on Gynecologic Cancer in Older Patients see page 53

Jyothirmai Gubili, MS, on American Ginseng see page 72

Alexander M.M. Eggermont, MD, PhD, on Adjuvant Ipilimumab After Resection of High-Risk Stage III Melanoma see page 74

Visit The ASCO Post online at ASCOPost.com


The ASCO Post  |   JULY 25, 2015

PAGE 6

ASCO Annual Meeting Obinutuzumab in Lymphoma continued from page 1

was dosed at 90 mg/m2, a dose used commonly in combination with other drugs. In the monotherapy arm, the dose was 120 mg/m2 for up to six cycles. The clinical characteristics were comparable in both arms of the trial. The median age was 63 years, with a median of two prior lines of therapy. More than 90% of patients in each arm were refractory to their previous therapy. Between 76% and 81% were double-refractory to both rituximab and an akylating agent.

Reduced Disease Progression At a planned interim analysis, the study met its primary endpoint of progression-free survival. Median progression-free survival, as assessed by an independent radiology facility, was 14.9 months for bendamustine vs not yet reached in the combination arm, representing a 45% reduction in the rate of disease progression (P = .0001). Investigator-assessed progression-free survival was similar. Progression-free survival favored the obinutuzumab combination in the majority of subgroups analyzed. Response rates were similar in the two arms. The best overall response rate to 12 months was 78.7% for the combination arm vs 76% for the bendamustine arm. Median overall survival is not yet reached in either arm, but the numbers trend toward favoring the combination arm. However, longer follow-up is needed.

Adverse Events No difference was observed between the two treatment arms in the rate of adverse events, grade 3 or higher adverse events, deaths, and withdrawal from

Obinutuzumab in Rituximab-Refractory Indolent Lymphoma ■■ Results from the GADOLIN trial offer the first phase III evidence that obinutuzumab plus bendamustine followed by maintenance obinutuzumab prolongs progression-free survival in relapsed or refractory indolent non-Hodgkin lymphoma. ■■ Obinutuzumab/bendamustine doubled progression-free survival compared with bendamustine alone. ■■ The combination was well tolerated, with no new safety signals.

therapy. The most common hematologic toxicities of any grades in both arms were neutropenia (35% for the combination and 29% for bendamustine alone) and thrombocytopenia (15% vs 24%). The most common nonhematologic toxicities of any grades were infusion-related reactions (69% vs 63%), nausea (54%

vs 61%), fatigue (39% vs 33%), and diarrhea (27% vs 30%). The rates of grade 3/4 neutropenia were 33% with the combination vs 26.3% with bendamustine; grade 3/4 thrombocytopenia occurred in 10.8% vs 16.2%, respectively. The most common grade 3/4 nonhematologic toxicity was infusion-related reac-

tion, occurring in 10.8% of the combination arm vs 5.6% with bendamustine alone. Serious adverse events were febrile neutropenia in the combination arm (4.1%) and pneumonia in the bendamustine arm (5.1%). “I sometimes think of obinutuzumab as a super-rituximab. These robust results for this novel agent in a rituximab-refrac-


ASCOPost.com  |   JULY 25, 2015

PAGE 7

ASCO Annual Meeting tory population are remarkable. This study may open up treatment options for this difficult-to-treat population, and maybe it will even allow these patients with incurable lymphoma to live longer. Obinutuzumab/bendamustine is not yet standard of care but has the potential to be standard of care for those patients who haven’t re-

ceived bendamustine in the front-line setting,” said Merry-­Jennifer Markham, MD, ASCO expert, of the University of Florida, Gainesville. n

Merry-Jennifer Markham, MD

Disclosure: Dr. Sehn has received honoraria from and served as a consultant or advisor to Amgen, Celgene, Gilead Sciences, Janssen, Lundbeck, Pfizer, and Roche/Genentech. Her institution also received

research funding from Roche Pharma AG. Dr. Markham reported no potential conflicts of interest.

Reference 1. Sehn LH, Chua NS, Mayer J, et al: GADOLIN: Primary results. 2015 ASCO Annual Meeting. Abstract LBA8502. Presented on June 1, 2015.


The ASCO Post  |   JULY 25, 2015

PAGE 8

ASCO Annual Meeting EXPERT POINT OF VIEW

J

eremy Abramson, MD, Clinical Director of the Center for Lymphoma at Massachusetts General Hospital, Boston, agrees that obinutumab is an advance over rituximab.

Like rituximab, obinutuzumab targets CD-20, but rituximab does not directly induce cell death. Obinutuzumab is a type 2 antibody and does cause direct cell death, so it has the

potential to be a better “mouse trap,” he commented. “Response rates were similar in the two arms, but progression-free survival was remarkably improved

with the addition of obinutuzumab, almost doubled from 15 months to 30 months,” he said. “This is the first study to show the benefit of obinutuzumab or any additional monoclonal antibody in the treatment of rituximab-refractory disease. This study could be practice-changing, with the caveat that

Jeremy Abramson, MD

results apply to patients with refractory disease who did not get bendamustine upfront as part of treatment. The study is practice-changing for patients with relapsed/rituximabrefractory indolent non-Hodgkin lymphoma who have never had bendamustine, but the truth is that bendamustine/rituximab has moved into upfront therapy in the United States in the past 2 years, and those patients were not studied in this ­trial,” Dr. Abramson commented. n Disclosure: Dr. Abramson reported no potential conflicts of interest.

The ASCO Post

Follow us on

@ASCOPost


ASCOPost.com  |   JULY 25, 2015

PAGE 9

ASCO Annual Meeting Gastrointestinal Oncology

Novel Enzyme Improves Outcomes in Subset of Patients With Pancreatic Cancer By Caroline Helwick

P

atients with stage IV pancreatic cancer who received a novel enzyme along with standard chemotherapy experienced a doubling in progression-free survival, compared to those given chemotherapy alone, in a study presented at the 2015 ASCO Annual Meeting.1 Response rates and median duration of response were also doubled, and a trend toward improved survival was observed as well, reported Sunil R. Hingorani, MD, PhD, of Fred Hutchinson Cancer Research Center, Seattle.

Sunil R. Hingorani, MD, PhD

“Collectively, these data have prompted the planning of a global phase III trial,” Dr. Hingorani indicated.

Study 202 Dr. Hingorani presented interim results from a randomized, multicenter phase II trial testing a novel combination therapy dubbed PAG—pegylated recombinant human hyaluronidase ­ (PEGPH20) plus nab-paclitaxel (Abraxane) and gemcitabine—in pa­ tients with stage IV pancreatic cancer. Poor outcomes in pancreatic cancer are associated in part with tumor stroma that limits chemotherapy perfusion. These tumors express high levels of hyaluronan, which has a number of detrimental effects on the tumor and mi-

croenvironment. PEGPH20 depletes hyaluronan in tumors, he explained. The ongoing phase II Study 202 has a target enrollment of 260 patients. It had enrolled 146 patients between March 2013 and April 2014, before a clinical hold was placed on the study after a higher thromboembolic event rate was observed in the PEGPH20 arm compared with the control arm. At that time, the 29 patients still receiving PEGPH20 were maintained on nabpaclitaxel/gemcitabine alone. At the time of the clinical hold, 28.4% of the PAG arm and 14.8% of the chemotherapy-alone arm had experienced a thromboembolic event, mainly pulmonary embolism, deep-vein thrombosis, and in a few patients, arterial events. As a result, the protocol was amended to exclude patients with a history of or increased risk for thromboembolic events, and patients in both arms were given low– molecular-weight heparin as prophylaxis. Dr. Hingorani presented the efficacy and safety data for patients enrolled up to the clinical hold and the thromboembolic rates after institution of the protocol amendment. He reported on 135 patients with a median follow-up of 7 months. Of the 135 patients, 106 were able to be evaluated for hyaluronan expression and were classified as hyaluronanhigh or hyaluronan-low.

Hyaluronan Expression “In these overall populations, the suggestion of improved efficacy did not reach statistical significance. However, when patients were stratified by [hyaluronan] content, there were two observations,” he reported. In hyaluronan-low patients, there ap-

peared to be no benefit to adding PEGPH20, “which might be expected, if hyaluronan is a specific target,” he pointed out. In contrast, in the hyaluronan-high patients, the results were striking. In the hyaluronan-high subset, the overall response rate was 52% in the PAG arm, compared to 24% with chemotherapy alone (P = .038), and median duration of response was 8.1 months vs 3.7 months, respectively. Restricting the evaluation to patients treated before the clinical hold (and therefore not contaminated by crossover), the investigators observed an even larger difference, with response rates of 73% and 27%, respectively (P = .01). Median progression-free survival was 9.2 months in the hyaluronan-high patients receiving PAG, vs 4.3 months with chemotherapy alone (hazard ratio [HR] = 0.39; P = .05). Again, no differences were observed in the hyaluronanlow subset. “A trend in overall survival in the triplet arm reflected the differences noted by the other measures and does include data on 12 of the 23 PAG patients who discontinued the enzyme and got nab-paclitaxel/gemcitabine alone, after the hold,” Dr. Hingorani said. Median overall survival was 12 months with PAG and 9 months with chemotherapy alone (HR = 0.62).

Thromboembolic Events Treatment-related adverse events seen more frequently with PAG included peripheral edema (58% vs 31%), muscle spasms (55% vs 2%), and neutropenia (32% vs 18%). After initiation of prophylaxis with enoxaparin at 40 mg/d, the rate of thromboembolic events “normalized” between

PEGPH20 Plus Chemotherapy ■■ Pegylated recombinant human hyaluronidase (PEGPH20) added to nab-paclitaxel/ gemcitabine doubled response rates and significantly improved progression-free survival in patients with stage IV pancreatic cancer who had high levels of hyaluronan. ■■ Median progression-free survival was 9.2 months in the hyaluronan-high patients receiving the triplet, vs 4.3 months with chemotherapy alone (HR = 0.39; P = .05). ■■ The drug works by lowering interstitial fluid pressures and increasing perfusion, enhancing the delivery of chemotherapy.

the arms, at approximately 29% for both groups. For patients initiated on enoxaparin at 1 mg/kg/d, the risk of thromboembolic events was ameliorated even further, to 0% with PAG and 10% with nab-paclitaxel and gemcitabine. These findings suggest that the thromboembolic risk can be adequately managed, he said. n Disclosure: Dr. Hingorani is a consultant for Halozyme Therapeutics.

Reference 1. Hingorani SR, Harris WP, Hendifar AE, et al: High response rate and PFs with PEGPH20 to nab-paclitaxel/gemcitabine in stage IV previously untreated pancreatic cancer patients with high-HA tumors: Interim results of a randomized phase II study. 2015 ASCO Annual Meeting. Abstract 4006. Presented May 31, 2015.

EXPERT POINT OF VIEW

K

enneth Yu, MD, MSc, of Memorial Sloan Kettering Cancer Center and Weill Cornell Medical College, New York, called investigational PEGPH20 “a very promising tumor ­ microenvironment-targeted approach” that appears to be well tolerated. “The results are preliminary but there appears to be a biomarker to predict response— high [hyaluronan] expression.” Dr. Yu reiterated that PEGPH20 is a

Kenneth Yu, MD, MSc

pegylated enzyme that hydrolyzes molecular hyaluronic acid, which is a major component of pancreatic tumors. Preclinical data suggest that it can improve the delivery of chemotherapy by creating holes within the tumor vasculature. Based on the data, he said, a number of research questions can be raised: Why are some tumors hyaluronanhigh or hyaluronan-low? How reliable is hyaluronan testing? How heteroge-

neous is hyaluronan level among tumors of individual patients? Is biopsy of the primary tumor more predictive? “This study highlights the potential that we may be entering an era when pancreatic cancer is a biomarker-directed disease,” he said. “We eagerly await results of a prospective validation randomized phase III trial.” n Disclosure: Dr. Yu reported no potential conflicts of interest.


THE POWER OF SEcOND-GENERATION PROTEASOmE INHIbITION:

SHAPING THE WAY FORWARD

Indication KyprolisÂŽ (carfilzomib) for Injection is indicated for the treatment of patients with multiple myeloma who have received at least two prior therapies including bortezomib and an immunomodulatory agent and have demonstrated disease progression on or within 60 days of completion of the last therapy. Approval is based on response rate. Clinical benefit, such as improvement in survival or symptoms, has not been verified.

Important Safety Information CONTRAINDICATIONS None.

WARNINGS AND PRECAUTIONS The safety of KYPROLIS was evaluated in clinical studies of 526 patients with relapsed and/or refractory multiple myeloma. Cardiac Arrest, Congestive Heart Failure, Myocardial Ischemia: Death due to cardiac arrest has occurred within a day of KYPROLIS administration. New onset or worsening of pre-existing congestive heart failure with decreased left ventricular function or myocardial ischemia have occurred following administration of KYPROLIS. Cardiac failure events (e.g., cardiac failure congestive, pulmonary edema, ejection fraction decreased) were reported in 7% of patients. Monitor for cardiac complications and manage promptly. Withhold KYPROLIS for Grade 3 or 4 cardiac events until recovery and consider whether to restart KYPROLIS based on a benefit/risk assessment. Patients with New York

Onyx, Onyx Pharmaceuticals, Onyx Pharmaceuticals logo, Kyprolis, and Kyprolis logo are all trademarks of Onyx Pharmaceuticals, Inc. Š2015 Onyx Pharmaceuticals, Inc., South San Francisco, CA TROPIC-KYPR-100826J(1) May 2015 Printed in USA

Heart Association Class III and IV heart failure, myocardial infarction in the preceding 6 months, and conduction abnormalities uncontrolled by medications may be at greater risk for cardiac complications. Pulmonary Hypertension: Pulmonary arterial hypertension (PAH) was reported in 2% of patients treated with KYPROLIS and was Grade 3 or greater in less than 1% of patients. Evaluate with cardiac imaging and/or other tests as indicated. Withhold KYPROLIS for pulmonary hypertension until resolved or returned to baseline and consider whether to restart KYPROLIS based on a benefit/risk assessment. Pulmonary Complications: Dyspnea was reported in 35% of patients enrolled in clinical trials. Grade 3 dyspnea occurred in 5%; no Grade 4 events, and 1 death (Grade 5) was reported. Monitor and manage dyspnea immediately; interrupt KYPROLIS until symptoms have resolved or returned to baseline. Infusion Reactions: Infusion reactions were characterized by a spectrum of systemic symptoms including fever, chills, arthralgia, myalgia, facial flushing, facial edema, vomiting, weakness, shortness of breath, hypotension, syncope, chest tightness, or angina. These reactions can occur immediately following infusion or up to 24 hours after administration of KYPROLIS. Administer dexamethasone prior to KYPROLIS to reduce the incidence and severity of reactions. Inform patients of the risk and symptoms, and to contact physician if symptoms of an infusion reaction occur. Tumor Lysis Syndrome: Tumor lysis syndrome (TLS) occurred following KYPROLIS administration in < 1% of patients. Patients with multiple myeloma and a high tumor burden should be considered to be at greater risk for TLS. Prior to receiving KYPROLIS, ensure that patients are well hydrated. Monitor for evidence of TLS during treatment, and manage promptly. Interrupt KYPROLIS until TLS is resolved. Thrombocytopenia: KYPROLIS causes thrombocytopenia with platelet nadirs occurring around Day 8 of each 28-day cycle and recovery to baseline by the start of the next 28-day cycle. In patients with multiple myeloma, 36% of patients experienced thrombocytopenia, including Grade 4 in 10%. Thrombocytopenia following KYPROLIS administration


Kyprolis® (carfilzomib) for Injection: 003-A1 Phase 2 Study Results* n

22.9% oveRAll ReSPonSe RAte (ORR) (95% CI: 18.0, 28.5)1

n

7.8-month medIAn duRAtIon of ReSPonSe (95% CI: 5.6, 9.2)1

n

Most patients across all phase 2 studies (85%) did not need to discontinue therapy due to an adverse event1,2 - Adverse reactions leading to discontinuation included congestive heart failure (2%), cardiac arrest, dyspnea, increased blood creatinine, and acute renal failure (1% each)1

*Study PX-171-003 was a single-arm, multicenter clinical trial of KYPROLIS in 266 patients with relapsed multiple myeloma who had received at least two prior therapies (including bortezomib and thalidomide and/or lenalidomide) and whose disease had a ≤25% response to the most recent therapy or had disease progression during or within 60 days of the most recent therapy. At the time of study entry, patients had received a median of 5 prior lines of therapy. The primary endpoint was ORR. Response was determined by Independent Review Committee assessment using International Myeloma Working Group criteria.

resulted in a dose reduction in 1% of patients and discontinuation of treatment with KYPROLIS in < 1% of patients. Monitor platelet counts frequently during treatment with KYPROLIS. Reduce or interrupt dose as clinically indicated.

potential should be advised to avoid becoming pregnant while being treated with KYPROLIS.

Hepatic Toxicity and Hepatic Failure: Cases of hepatic failure, including fatal cases, have been reported (< 1%). KYPROLIS can cause elevations of serum transaminases and bilirubin. Withhold KYPROLIS in patients experiencing Grade 3 or greater elevations of transaminases, bilirubin, or other liver abnormalities until resolved or returned to baseline. After resolution, consider if restarting KYPROLIS is appropriate. Monitor liver enzymes frequently. Thrombotic Thrombocytopenic Purpura/Hemolytic Uremic Syndrome: Cases of thrombotic thrombocytopenic purpura/hemolytic uremic syndrome (TTP/HUS) including fatal outcome have been reported in patients who received KYPROLIS. Monitor for signs and symptoms of TTP/HUS. If the diagnosis is suspected, stop KYPROLIS and evaluate. If the diagnosis of TTP/HUS is excluded, KYPROLIS can be restarted. The safety of reinitiating KYPROLIS therapy in patients previously experiencing TTP/HUS is not known. Posterior Reversible Encephalopathy Syndrome (PRES): PRES, formerly termed Reversible Posterior Leukoencephalopathy Syndrome (RPLS), is a neurological disorder, which can present with seizure, headache, lethargy, confusion, blindness, altered consciousness, and other visual and neurological disturbances, along with hypertension, and the diagnosis is confirmed by neuro-radiological imaging (MRI). Cases of PRES have been reported in patients receiving KYPROLIS. Discontinue KYPROLIS if PRES is suspected and evaluate. The safety of reinitiating KYPROLIS therapy in patients previously experiencing PRES is not known. Embryo-fetal Toxicity: KYPROLIS can cause fetal harm when administered to a pregnant woman based on its mechanism of action and findings in animals. There are no adequate and well-controlled studies in pregnant women using KYPROLIS. Carfilzomib caused embryo-fetal toxicity in pregnant rabbits at doses that were lower than in patients receiving the recommended dose. Females of reproductive

Serious adverse reactions were reported in 45% of patients. The most common serious adverse reactions were pneumonia (10%), acute renal failure (4%), pyrexia (3%), and congestive heart failure (3%). Adverse reactions leading to discontinuation of KYPROLIS occurred in 15% of patients and included congestive heart failure (2%), cardiac arrest, dyspnea, increased blood creatinine, and acute renal failure (1% each). The most common adverse reactions (incidence ≥ 30%) were fatigue (56%), anemia (47%), nausea (45%), thrombocytopenia (36%), dyspnea (35%), diarrhea (33%), and pyrexia (30%).

ADVERSE REACTIONS

USE IN SPECIFIC POPULATIONS Since dialysis clearance of KYPROLIS concentrations has not been studied, the drug should be administered after the dialysis procedure.

Please see Brief Summary of full Prescribing Information on adjacent pages.

References: 1. KYPROLIS [prescribing information]. South San Francisco, CA: Onyx Pharmaceuticals, Inc.; 2015. 2. Siegel D, Martin T, Nooka A, et al. Integrated safety profile of single-agent carfilzomib: experience from 526 patients enrolled in 4 phase II clinical studies. Haematologica. 2013;98(11):1753-1761.


The ASCO Post  |   JULY 25, 2015

PAGE 12

Announcements

George Wilding, MD, Named Vice Provost for Clinical and Interdisciplinary Research at MD Anderson

T

he University of Texas MD Anderson Cancer Center is pleased to announce the appointment of George Wilding, MD, as Vice Provost for Clin-

ical and Interdisciplinary Research, effective September 1. Prior to joining MD Anderson, Dr. Wilding served on the institution’s ex-

ternal advisory board. He also served as Chair of the Moon Shots Program Scientific Advisory Board. The Moon Shots Program was launched in fall

2012 to accelerate the conversion of scientific discoveries into clinical advances and significantly reduce cancer deaths.

B:16.75” T:16.25” S:14.625”

continued on page 13

Table 2: Dose Modifications for Toxicitya during KYPROLIS Treatment (continued) a Table 2: Dose Modifications Treatment (continued) Withhold until renal function has recovered to Grade 1 Renal Toxicity for Toxicity during• KYPROLIS • orWithhold until renal function recovered Grade 1 Renal Toxicity or to baseline andhas monitor renal tofunction. • Serum creatinine equal to and monitorto renal function. • Serum creatinine • If attributable KYPROLIS, restart at the next scheduled greaterequal than to2 or × baseline or to baseline to KYPROLIS, restart atdose the (from next scheduled greater than 2 × baseline at a reduced 27 mg/m2 to [see Adverse Reactions] • If attributabletreatment 2 dose (from 27 mg/m 2 treatment at20 mg/m a reduced to , OR from 20 mg/m2 to2 15 mg/m ). KYPROLIS™ (carfilzomib) for Injection [see Adverse Reactions] from 20 mg/m2totoKYPROLIS, 15 mg/m2).restart at the dose used 20 mg/m•2, OR KYPROLIS™ (carfilzomib) Injection Information. Please see the KYPROLIS package insert for full Brief Summary offor Prescribing If not attributable Brief Summary of Prescribing Information. Please see the KYPROLIS package insert for full • If not attributable KYPROLIS, prescribing information. prior totothe event. restart at the dose used prescribing information. prior to the INDICATIONS AND USAGE: KYPROLIS is indicated for the treatment of patients with multiple myeloma • Ifevent. tolerated, the reduced dose may be escalated to the INDICATIONS indicated for the treatment of patients multiple myeloma agent and whoAND haveUSAGE: received KYPROLIS at least twois prior therapies including bortezomib andwith an immunomodulatory • If tolerated, previous the reduced may be escalated the dosedose at the discretion of thetophysician. who have received at least two prior therapies including and an agent and Approval is have demonstrated disease progression on orbortezomib within 60 days of immunomodulatory completion of the last therapy. previous•dose at the discretion of the physician.to baseline. Withhold until resolved or returned Peripheral Neuropathy have demonstrated progression or within 60 days of completion the lastbenefit, therapy. Approval is based ondisease response rate [seeon Clinical Studies section of full PI].ofClinical such as improvement in • Withhold•until resolved or dose returned baseline. Peripheral•Neuropathy Restart at the usedtoprior to the event or reduced Grade 3 or 4 based on response rate [see Clinical of full PI]. Clinical benefit, such as improvement in survival or symptoms, has Studies not beensection verified. 2 dose used prior to2the event or reduced • Grade 3[see or 4Adverse Reactions] • Restart at the dose (from 27 mg/m to 20 mg/m , OR from 20 mg/m2 survival or symptoms, has ADMINISTRATION: not been verified. Dosing Guidelines. KYPROLIS is administered intravenously over 2 to DOSAGE AND 2 2 2 2 dose (from 27 mg/m to 20 mg/m , OR from to 15 mg/m ), at the discretion of 20 mg/m the physician. [see Adverse Reactions] DOSAGE AND ADMINISTRATION: Dosing Guidelines. KYPROLIS is administered intravenously over 2 to 2 10 minutes, on two consecutive days, each week for three weeks (Days 1, 2, 8, 9, 15, and 16), followed by to 15 mg/m at the discretion of thedose physician. • If),tolerated, the reduced may be escalated to the 10 minutes,aon12‑day two consecutive eachtoweek three weeksperiod (Daysis1,considered 2, 8, 9, 15,one andtreatment 16), followed rest perioddays, (Days 17 28).for Each 28‑day cycleby(Table 1). In • If tolerated, previous the reduced may be escalated the dosedose at the discretion of thetophysician. a 12‑day rest period (Days 17 tois 28). Each 28‑day is considered treatment In should be Cycle 1, KYPROLIS administered at aperiod dose of 20 mg/m2. one If tolerated in cycle Cycle (Table 1, the1). dose previous•dose at the discretion of the physician.to baseline. Withhold until resolved or returned Other 2 the dose should be Cycle 1, KYPROLIS at a doseinofCycle 2 20 mg/m If toleratedat in escalatedistoadministered 27 mg/m2 beginning and2.continued 27Cycle mg/m1, in subsequent cycles. Treatment • Withhold•until resolved or returned to next baseline. Other 2 2 Consider restarting at the scheduled treatment • Grade 3 or 4 non‑hematological escalated tomay 27 mg/m beginning in Cycle 2 continued or at 27 in subsequent cycles. Treatment be continued until diseaseand progression untilmg/m unacceptable toxicity occurs [see Dosage and the next treatment • with Grade 3 or 4toxicities non‑hematological • Consider restarting with oneatdose level scheduled reduction (from 27 mg/m2 to may be continued until disease progression until the unacceptable toxicity Dosage andPatients Administration]. The dose is calculatedorusing patient’s actual bodyoccurs surface [see area at baseline. 2 2 2 2 with one dose level reduction 27 mg/m to 2 2 toxicities , OR from(from 20 mg/m to 15 mg/m ). 20 mg/m Administration]. Thesurface dose isarea calculated patient’s actual bodyasurface area upon at baseline. a body greaterusing thanthe 2.2 m should receive dose based a bodyPatients surfacewith area of 2.2 m . 2 2 2 2 2 , OR from 20 mg/m to 15 mg/m ). 20 mg/m • If tolerated, the reduced dose may be escalated to the a body surface greater than 2.2 m uponofaless bodythan surface areatoof20%. 2.2 m . Dosearea adjustments do not needshould to be receive made fora dose weightbased changes or equal • If tolerated, the reduced dose may be escalated to the previous dose at the discretion of the physician. Dose adjustments doKYPROLIS not need toDosage be madeRegimen for weightfor changes of less than or equalMyeloma to 20%. Table 1: Patients with Multiple previous dose theAdverse discretion theCTCAE) physician. a National Cancer Institute Common Terminology Criteriaatfor Eventsof(NCI Version 3.0. Table 1: KYPROLIS Dosage Regimen for Patients with Multiple Myeloma Cycle 1 a National Cancer Institute Common Terminology Criteria for Adverse (NCI CTCAE) Version 3.0. Administration Precautions. The quantity Events of KYPROLIS contained in one single‑use vial (60 mg Cycle 1 Week 1 Week 2 Week 3 Week 4Administration Precautions. Thethequantity KYPROLIS onein single‑use (60 mgdelivered to carfilzomib) may exceed requiredofdose. Cautioncontained should be inused calculating vial the quantity Week 1 Day DaysWeek 3 4 Day2 Day DaysWeekDay Day Week Days Days carfilzomib)prevent Day may exceed the required Caution should beadminister used in calculating the quantity delivered to products. overdosing. Do notdose. mix KYPROLIS with or as an infusion with other medicinal Day Day 1 Days2 Day3–7Day 8 Days9 Day 10–14Day15 Days 16 Days 17–21 22–28prevent overdosing. Do not mix KYPROLIS with or administer as an infusion with other medicinal products. The intravenous administration line should be flushed with normal saline or 5% Dextrose Injection, USP KYPROLIS 15 No 16 2017–21 2 20 3–720 8 No 9 2010–14 22–28 1 20 20 No No The intravenous administration line after should be flushed with normalKYPROLIS saline or 5% Dextrose USP as a bolus. immediately before and KYPROLIS administration. should not beInjection, administered 2 KYPROLIS (20 mg/m No 20 No 20 No No 20 ):20 Dosing20 Dosing 20 Dosing Dosingimmediately before and after KYPROLIS administration. KYPROLIS should not be administered as a bolus. KYPROLIS should be administered over 2 to 10 minutes. Reconstitution and Preparation for (20 mg/m2): a Dosing Dosing Dosing Dosing Cycles 2 and Beyond KYPROLIS Intravenous should be administered over 2KYPROLIS to 10 minutes. Reconstitution andpreservatives Preparationandforare intended Administration. vials contain no antimicrobial a Beyond KYPROLISvials vialsofcontain no antimicrobial preservatives and areonintended Week 1 Cycles 2 and Week 2 Week 3 Week 4Intravenous onlyAdministration. for single use. Unopened KYPROLIS are stable until the date indicated the package when Week 1 Day DaysWeek 3 4 use.inUnopened of KYPROLIS stable the date on thesolution package when carfilzomib stored the originalvials package at 2°C toare 8°C (36°Funtil to 46°F). Theindicated reconstituted contains Day2 Day DaysWeekDay Day Week Days Days only for single Day package at of 2°C2 tomg/mL. 8°C (36°F to 46°F). The reconstituted solution containsprior carfilzomib Day Day 1 Days2 Day3–7Day 8 Days9 Day 10–14Day15 Days 16 Days 17–21 22–28stored in theatoriginal a concentration Read the complete preparation instructions to reconstitution. 2 27 3–727 8 No 9 2710–14 1 of 2 mg/mL. Read the complete preparation instructions prior reconstitution. KYPROLIS 27 15 No 16 2717–21 27 22–28 No No at a concentration Reconstitution/Preparation Steps: 1. Remove vial from refrigerator justtoprior to use. 2. Aseptically KYPROLIS (27 mg/m No 27 No 27 No No 27 2):27 Dosing27 Dosing 27 Dosing DosingReconstitution/Preparation 1. Remove fromSterile refrigerator justInjection, prior to USP, use.directing 2. Aseptically reconstitute each vialSteps: by slowly injectingvial 29 mL Water for the solution onto Dosing Dosing Dosing (27 mg/ma If2):previous cycle dosageDosing reconstitute each vial by slowly injecting 29 mL Sterile Water for Injection, USP, directing the solution onto for about the INSIDE WALL OF THE VIAL to minimize foaming. 3. Gently swirl and/or invert the vial slowly is tolerated. a If previous cycle dosage is tolerated. the INSIDE WALL OF THE VIALcomplete to minimize foaming.of3.any Gently swirl and/or invert the vial slowly for about 1 minute, or until dissolution cake or powder occurs. DO NOT SHAKE to avoid foam Hydration and Fluid Monitoring. Hydrate patients to reduce the risk of renal toxicity and of tumor lysis or until complete dissolution of any cake or powder occurs. DO NOT SHAKE to avoid foam Hydration syndrome and Fluid(TLS) Monitoring. Hydratetreatment patients [see to reduce the risk renal toxicityMaintain and of tumor lysisfluid1 minute, with KYPROLIS Warnings andofPrecautions]. adequate volume generation. If foaming occurs, allow solution to rest in vial for about 2 to 5 minutes, until foaming subsides. generation. If foaming occurs, allow solution to rest in vial for about 2 to 5 minutes, until foaming subsides. syndrome (TLS) with KYPROLIStreatment treatmentand [seemonitor Warnings and chemistries Precautions].closely. Maintain adequate volume status throughout blood Prior to eachfluid dose in Cycle 1, give 4. After reconstitution, KYPROLIS is ready for intravenous administration. The reconstituted product should 4. After reconstitution, ready forIf intravenous administration. The reconstituted product should be a clear,KYPROLIS colorless issolution. any discoloration or particulate matter is observed, do not use the status throughout blood chemistries Prior to each dose in Cycle 1, givean additional 250 mLtreatment to 500 mLand of monitor intravenous normal saline orclosely. other appropriate intravenous fluid. Give be a clear, reconstituted colorless solution. If any discoloration or particulate matter isbag, observed, dothe notcalculated use the dose [see product. 5. When administering in an intravenous withdraw 250 mL to 500 mL normal saline orfluids other appropriate Giveadministration. an additional Continue 250 mL oftointravenous 500 mL of intravenous as needed intravenous fluid. following KYPROLIS reconstituted product. 5. When administering in an intravenous bag, withdraw the calculated dose [seeintravenous Dosage and Administration] from the vial and dilute into 50 mL 5% Dextrose Injection, USP 250 mL tointravenous 500 mL ofhydration, intravenous fluids as needed following KYPROLIS as needed, in subsequent cycles. Also monitoradministration. patients duringContinue this period for fluid and Administration] from the vial and dilute into 50 mL 5% Dextrose Injection, USP intravenous intravenousoverload hydration,[see as needed, in subsequent cycles. Also monitor patients Premedication. during this period Pre‑medicate for fluid Dosage Warnings and Precautions]. Dexamethasone with bag. 6. Immediately discard the vial containing the unused portion. The stabilities of reconstituted bag. 6. Immediately discard the vial containing the unused portion. The stabilities of reconstituted overload [see Warnings and Dexamethasone dexamethasone 4 mgPrecautions]. orally or intravenously prior to all Premedication. doses of KYPROLISPre‑medicate during Cycle 1with and prior to all KYPROLIS under various temperature and container conditions are shown in Table 3. KYPROLIS under various temperature and container conditions are shown in Table 3. 2 Cycle 1 and prior to all dexamethasone 4 mg doses orally or intravenously prioroftodose all doses of KYPROLIS during KYPROLIS during the first cycle escalation to 27 mg/m to reduce the incidence and severity Table 3: Stability of Reconstituted KYPROLIS 2 KYPROLIS doses duringreactions the first cycle dose escalation to 27 mg/mReinstate to reducedexamethasone the incidence and severity Table 3: of infusion [see ofWarnings and Precautions]. premedication (4 mg Stability of Reconstituted KYPROLIS Stabilitya per Container of infusion orally reactions [see Warnings ifand Precautions]. premedication (4 mg or intravenously) these symptomsReinstate develop dexamethasone or reappear during subsequent cycles. Dose Storage Conditions of Reconstituted Stabilitya per Container orally or intravenously) these symptoms develop or reappearactions during and subsequent cycles. Dose Modificationsif based on Toxicities. Recommended dose modifications are presentedStorage Conditions of Reconstituted IV Bag KYPROLIS Modifications based on Toxicities. Recommended actions and dose modifications are presented in Table 2. IV Bag (D5Wb) Vial Syringe KYPROLIS in Table 2. Table 2: Dose Modifications for Toxicitya during KYPROLIS Treatment Vial Syringe (D5Wb) Refrigerated (2°C to 8°C; 36°F to 46°F) 24 hours 24 hours 24 hours Table 2: Dose Modifications for Toxicitya during KYPROLIS Treatment Hematologic Toxicity Recommended Action Refrigerated (2°C to 8°C; 36°F to 46°F) 24 hours 24 hours 24 hours a Hematologic Recommended Action Room Temperature (15°C to 30°C; 59°F to 86°F) 4 hours 4 hours 4 hours • Withhold dose. or 4 Neutropenia • Grade 3Toxicity Room Temperature (15°C to 30°C; 59°F to 86°F) 4 hours 4 hours 4 hours • Withhold•dose. 4 Neutropenia • Grade 3a•or Grade a If fully recovered before next scheduled dose, continue 4 Thrombocytopenia Total time from reconstitution to administration should not exceed 24 hours. b 5% Dextrose Injection, USP. a b • If fully recovered before scheduled dose, continue • Grade 4 [see Thrombocytopenia at same dosenext level. Total time from reconstitutionAND to administration should Cardiac not exceedArrest, 24 hours. 5% DextroseHeart Injection, USP. Myocardial Ischemia. Warnings and Precautions] WARNINGS PRECAUTIONS: Congestive Failure, at same •dose level. [see Warnings and Precautions] If recovered to Grade 2 neutropenia or Grade 3 WARNINGSDeath ANDdue PRECAUTIONS: Cardiac Arrest,within Congestive Failure, MyocardialNew Ischemia. to cardiac arrest has occurred a day ofHeart KYPROLIS administration. onset or worsening • If recoveredthrombocytopenia, to Grade 2 neutropenia Gradeby3one dose level reduceordose Death due toofcardiac arrest congestive has occurred within a daywith of KYPROLIS administration. New onset ororworsening pre‑existing heart failure decreased left ventricular function myocardial ischemia 2 2 2 thrombocytopenia, reduce dose by one dose level (from 27 mg/m to 20 mg/m , OR from 20 mg/m to of pre‑existing failureadministration with decreased ventricular functionfailure or myocardial ischemia havecongestive occurred heart following of left KYPROLIS. Cardiac events (e.g., cardiac failure 2 2 2 (from 27 mg/m to 20 mg/m , OR from 20 mg/m2 to 15 mg/m ). have occurred following administration KYPROLIS. failure (e.g., cardiac failure Monitor for congestive, pulmonary edema, of ejection fractionCardiac decreased) wereevents reported in 7% of patients. 15 mg/m•2).If tolerated, the reduced dose may be escalated to thecongestive, cardiac pulmonary edema, ejection fraction decreased) were reported in 7% of patients. Monitor forevents until complications and manage promptly. Withhold KYPROLIS for Grade 3 or 4 cardiac • If tolerated, previous the reduced may be escalated the dosedose at the discretion of thetophysician. cardiac complications and managewhether promptly. Withhold KYPROLIS for on Grade 3 or 4 cardiac events[see untilDosage and recovery and consider to restart KYPROLIS based a benefit/risk assessment previous dose at the discretion of the physician. recovery andAdministration]. consider whether to restart KYPROLIS based on a benefit/risk Dosage and infarction Patients with New York Heart Association Class IIIassessment and IV heart [see failure, myocardial Non-Hematologic Toxicity Recommended Action Administration]. with6New York and Heartconduction Association Class III anduncontrolled IV heart failure, in thePatients preceding months, abnormalities by myocardial medicationsinfarction were not eligible for Non-Hematologic Toxicity Recommended Action • Withhold until resolved or returned to baseline. Cardiac Toxicity in the preceding 6 months, and These conduction abnormalities were not eligible forPulmonary the clinical trials. patients may be uncontrolled at greater by riskmedications for cardiac complications. • Withhold resolved or returned Cardiac Toxicity After resolution, considertoifbaseline. restarting KYPROLIS at the clinical trials. These patients may be at greater risk for cardiac complications. Pulmonary Grade 3 or 4, new onset or worsening of: •until Hypertension. Pulmonary arterial hypertension (PAH) was reported in 2% of patients treated with 2 consider KYPROLIS at Grade 3 or 4,•new onset or worsening of: • After resolution, a reduced doseif restarting is appropriate (from 27 mg/m to congestive heart failure; Hypertension. Pulmonary arterial hypertension (PAH) was reported in 2% of patients treated KYPROLIS and was Grade 3 or greater in less than 1% of patients. Evaluate with cardiac with imaging and/or 2 2 2 2 is appropriate 27 mg/m to , OR from(from 20 mg/m to 15 mg/m ). 20 mg/m • congestive failure; • heart decreased left ventricular a reduced2 dose KYPROLIS and Grade 3 or greaterWithhold in less than 1% patients. Evaluate with cardiac imaging and/or otherwas tests as indicated. KYPROLISof for pulmonary hypertension until resolved or returned to 2 2 , OR from 20 mg/m to 15 mg/m ). 20 mg/m • If tolerated, the reduced dose may be escalated to the • decreased left ventricular function; other tests baseline as indicated. Withholdwhether KYPROLIS for pulmonary until resolved or returned and consider to restart KYPROLIShypertension based on a benefit/risk assessment [seetoDosage and the reduced may be escalated the dosedose at the discretion of thetophysician. function;• or myocardial ischemia • If tolerated, previous baseline andAdministration]. consider whether to restart KYPROLIS based on a benefit/risk assessment [see Dosage and Pulmonary Complications. Dyspnea was reported in 35% of patients enrolled in clinical • or myocardial ischemia and Precautions]previous dose at the discretion of the physician. Administration]. Complications. reported in 35% of patients enrolled in clinical trials.Pulmonary Grade 3 dyspnea occurred inDyspnea 5%; no was Grade 4 events, and 1 death (Grade 5) was reported. Monitor [see Warnings trials. Grade 3 dyspnea occurred inimmediately; 5%; no Grade 4 events,KYPROLIS and 1 death 5) washave reported. Monitor [see Warnings and Precautions] and manage dyspnea interrupt until(Grade symptoms resolved or returned to Pulmonary Hypertension • Withhold until resolved or returned to baseline. dyspnea KYPROLIS until symptoms have resolved or returned to reactions baseline [seeimmediately; Dosage and interrupt Administration and Adverse Reactions]. Infusion Reactions. Infusion Pulmonary[see Hypertension • Withhold•until resolved or dose returned baseline. Restart at the usedtoprior to the event or reduced and manage Warnings and Precautions] [see Dosage and Administration and Adverse Reactions]. Infusion Reactions. Infusion reactions were characterized by a spectrum of systemic symptoms including fever, chills, arthralgia, myalgia, facial 2 2 • Restart at the dose used prior to2the event or reduced [see Warnings and Precautions] to 20 mg/m , OR from 20 mg/mbaseline dose (from 27 mg/m were characterized by a spectrum of systemic symptoms including fever, chills, arthralgia, myalgia, facial 2 2 2 2 flushing, facial edema, vomiting, weakness, shortness of breath, hypotension, syncope, chest tightness, or , OR from dose (from 27 mg/m to 15 mg/mto ),20 mg/m at the discretion of 20 mg/m the physician. flushing, facial edema, vomiting, weakness, of breath, hypotension, chest tightness, orof KYPROLIS. 2 angina. These reactions can occurshortness immediately following or up to 24syncope, hours after administration to 15 mg/m at the discretion of thedose physician. • If),tolerated, the reduced may be escalated to the angina. These reactions can occur immediately to 24 the hours after administration Administer dexamethasone prior to following KYPROLISortoupreduce incidence and severity of KYPROLIS. reactions [see Dosage • If tolerated, previous the reduced may be escalated the dosedose at the discretion of thetophysician. Administer dexamethasone prior Inform to KYPROLIS to of reduce the and incidence and severity of reactions [see Dosage and Administration]. patients the risk symptoms and to contact physician if symptoms of an at the discretion of the physician.to baseline. Withhold until resolved or returned Pulmonary Complications previous•dose and Administration]. the Patient risk andCounseling symptoms and to contactTumor physician if symptoms of anTumor lysis infusion Inform reactionpatients occur of[see Information]. Lysis Syndrome. • Withhold•until resolved or returned to next baseline. Pulmonary•Complications Consider restarting at the scheduled treatment infusion reaction Grade 3 or 4 occur [see Patient Counseling Information]. Tumor Lysis Syndrome. Tumor lysiswith multiple syndrome (TLS) occurred following KYPROLIS administration in < 1% of patients. Patients • Consider restarting the next treatment • Grade 3[see or 4Warnings and Precautions] with oneatdose level scheduled reduction (from 27 mg/m2 to syndrome (TLS) occurred following KYPROLIS administration in < 1% of patients. Patients with multiple myeloma and a high tumor burden should be considered to be at greater risk for TLS. Prior to receiving 2 2 2 2 with one dose level reduction (from 27 mg/m to , OR from 20 mg/m to 15 mg/m ). 20 mg/m [see Warnings and Precautions] myeloma and a high tumor burden should be to be at Dosage greater risk TLS. Prior to Monitor receivingfor evidence KYPROLIS, ensure that patients areconsidered well hydrated [see and for Administration]. 2 2 from 20 mg/m to 15 mg/m ). be escalated to theKYPROLIS, ensure 20 mg/m•2, OR If tolerated, the reduced dose may that patients are well hydratedpromptly. [see Dosage and Administration]. Monitor for evidence of TLS during treatment, and manage Interrupt KYPROLIS until TLS is resolved [see Dosage and • If tolerated, previous the reduced may be escalated the dosedose at the discretion of thetophysician. of TLS during treatment, and manage promptly. Interrupt KYPROLIS TLS is resolvedwith [seeplatelet Dosagenadirs and occurring KYPROLIS causes until thrombocytopenia Administration].Thrombocytopenia. previous•dose at the discretion of the physician.to baseline. Administration].Thrombocytopenia. KYPROLIS thrombocytopenia withtheplatelet occurring Hepatic Toxicity Withhold until resolved or returned around Day 8 of each 28‑day cycle causes and recovery to baseline by start ofnadirs the next 28‑day cycle. In Hepatic Toxicity resolved or returned baseline. KYPROLIS is around Day 8 of each cycle and recovery baselineexperienced by the start thrombocytopenia, of the next 28‑dayincluding cycle. In Grade 4 in patients with28‑day multiple myeloma, 36% ofto patients After resolution, considertoif restarting • Grade 3 or 4 elevation of• Withhold•until multiple myeloma, 36% of patients experienced thrombocytopenia, Gradein41% in of patients considermay if restarting KYPROLIS is • Grade 3 or 4transaminases, elevation of bilirubin •or After 10%. Thrombocytopenia following KYPROLIS administration resulted in a including dose reduction appropriate; be reinitiated at a reduced dose (from patients with otherresolution, 2 2 2 2 following reduction in 1% of patients appropriate;27 mg/m may be reinitiated at a ,reduced (from to 15 mg/m10%. transaminases, or other to 20 mg/m OR fromdose 20 mg/m ) Thrombocytopenia and discontinuation of KYPROLIS treatment administration with KYPROLISresulted in < 1%in ofa dose patients. Monitor platelet counts frequently liver bilirubin abnormalities 2 2 2 2 to 20 mg/m , OR from 20 mg/m to 15 mg/m ) 27 mg/m and discontinuation of treatment with KYPROLIS in < 1% of patients. Monitor platelet counts frequently with frequent monitoring of liver function. liver abnormalities during treatment with KYPROLIS. Reduce or interrupt dose as clinically indicated [see Dosage and [see Warnings and Precautions] with frequent monitoringtheof reduced liver function. with KYPROLIS. interrupt doseFailure. as clinically Dosage andfatal cases, Administration]. HepaticReduce Toxicityor and Hepatic Cases indicated of hepatic [see failure, including • If tolerated, dose may be escalated to theduring treatment [see Warnings and Precautions] Administration]. Toxicity(< and Failure. hepatic failure, including fatal cases, • If tolerated, previous the reduced may be escalated the have Hepatic been reported 1%).Hepatic KYPROLIS can Cases cause of elevations of serum transaminases and bilirubin. dosedose at the discretion of thetophysician. have been Withhold reported KYPROLIS (< 1%). KYPROLIS cause elevations serum transaminases and bilirubin. bilirubin, or previous dose at the discretion of the physician. in patientscan experiencing Grade 3 of or greater elevations of transaminases, Withhold KYPROLIS in patients experiencing Grade 3 or greater elevations of transaminases, bilirubin, or other liver abnormalities until resolved or returned to baseline. After resolution, consider if restarting (continued) other liver abnormalities until resolved returned baseline.[see After resolution, consider ifand restarting KYPROLIS is appropriate. Monitororliver enzymestofrequently Dosage and Administration Adverse Reactions]. (continued) KYPROLIS isThrombotic appropriate. Monitor liver enzymes frequently [see Dosage and Administration and AdverseCases Reactions]. Thrombocytopenic Purpura/Hemolytic Uremic Syndrome: of thrombotic Thromboticthrombocytopenic Thrombocytopenic Purpura/Hemolytic Uremic(TTP/HUS) Syndrome: Casesfatalofoutcome thrombotic purpura/hemolytic uremic syndrome including have been thrombocytopenic purpura/hemolytic uremic syndrome (TTP/HUS) including fatal outcome have been

B

T

S


ASCOPost.com  |   JULY 25, 2015

PAGE 13

Announcements

Jonathan D. Licht, MD, Appointed Director of the University of Florida Health Cancer Center, Gainesville

B:16.75”

T:16.25”

S:14.625”

J

onathan D. Licht, MD, has been appointed Director of the University of Florida (UF) Health Cancer Center, Gainesville. His appointment

will go into effect on October 1. Dr. Licht comes to UF Health from Northwestern University. He is currently Associate Director for Clinical

Sciences at the Robert H. Lurie Comprehensive Cancer Center of Northwestern University, the Johanna Dobe Professor of Hematology/Oncology,

Chief of the Division of Hematology/ Oncology, and Professor of Biochemistry and Molecular Genetics at the continued on page 14

George Wilding, MD continued from page 12

In his role at MD Anderson, Dr. Wilding will oversee strategic planning, conduct, approval, and regulation of all clinical research conducted at MD Anderson and its global operations. He will also provide oversight and academic leadership for its multidisciplinary research institutes.

George Wilding, MD

B:11.25”

S:10” B:11.25”

T:10.875”

S:10”

“I am confident that Dr. Wilding’s professional expertise and leadership experience have prepared him for this vital role,” said Ethan Dmitrovsky, MD, MD Anderson Provost and Executive Vice President. “Additionally, his leadership approach and many other talents, including deep strategic thinking and thoughtfulness, will contribute substantially to our efforts to build upon our incredible foundation of critical clinical research a­ dvances.” Dr. Wilding’s interests focus on genitourinary cancers, particularly prostate cancer. In addition to his administrative appointment, he will serve as Professor of Genitourinary Medical Oncology at MD Anderson. His research interests concern the role of androgen-induced oxidative stress in prostate carcinogenesis and cancer progression. Two agents targeting this pathway for prostate cancer were developed in his laboratory. One of the agents has entered clinical testing, and the other is approaching this critical phase. Dr. Wilding currently serves as the Donald and Marilyn Anderson Professor of Clinical Oncology at the University of Wisconsin School of Medicine and Public Health and Director Emeritus of the University of Wisconsin Carbone Cancer Center, Madison. n T:10.875”

reported in patients who received KYPROLIS. Monitor for signs and symptoms of TTP/HUS. If the diagnosis Description of Selected Adverse Drug Reactions. Renal Events: The most common renal adverse of Selected most(9%), common reported in patients who received KYPROLIS. for Ifsigns symptoms of TTP/HUS. If the diagnosis were Adverse increase inDrug bloodReactions. creatinine Renal (24%) Events: and renalThe failure whichrenal wereadverse mostly Grade 1 is suspected, stop KYPROLIS andMonitor evaluate. the and diagnosis of TTP/HUS is excluded, KYPROLIS Description can be reactions increaseininseverity. blood creatinine (24%)adverse and renal failure occurred (9%), which were is suspected, stop KYPROLIS evaluate.KYPROLIS If the diagnosis TTP/HUSpreviously is excluded, KYPROLIS can beis notreactions or Grade 2 Grade 3 renal reactions in 6% of mostly patientsGrade 1 and Grade 4 events restarted. The safetyand of reinitiating therapy of in patients experiencing TTP/HUS known. were or Grade 2 in severity. Grade 3 renal adverse reactions occurred in 6% of patients and Grade 4 events restarted. The safety of reinitiating KYPROLIS therapy in patients previously experiencing TTP/HUS is not known. Posterior Reversible Encephalopathy Syndrome (PRES): PRES, formerly termed Reversible Posterior occurred in 1%. Discontinuations due to increased blood creatinine and acute renal failure were 1% 1%. Discontinuations due tooccurred increased creatinine andand acute renal failure 1% [see Dosage Posterior Reversible Encephalopathy Syndrome PRES, formerly termed Reversible Posterior In one patient, death withblood concurrent sepsis worsening renalwere function Leukoencephalopathy Syndrome (RPLS),(PRES): is a neurological disorder, which can present with occurred seizure, in each. each. In oneand patient, death occurred with concurrent sepsis and worsening renal(including function [see Dosageof peripheral Leukoencephalopathy (RPLS), is blindness, a neurological which canandpresent Administration]. Peripheral Neuropathy: Peripheral neuropathy all events headache, Syndrome lethargy, confusion, altereddisorder, consciousness, other with visualseizure, and neurological and Administration]. Peripheraland Neuropathy: neuropathy (including all events of peripheral headache, disturbances, lethargy, confusion, blindness, alteredandconsciousness, other visual and neurological sensory neuropathy peripheralPeripheral motor neuropathy) occurred in 14% of patients enrolled in clinical along with hypertension, the diagnosis isand confirmed by neuro‑radiological imaging (MRI). sensory and 3peripheral neuropathy) occurred of patients in clinical disturbances, alongofwith hypertension, the diagnosis is confirmed neuro‑radiological imaging (MRI). if PRES trials. Grade peripheralmotor neuropathy occurred in 1%inof14% patients. Seriousenrolled peripheral neuropathy events Cases PRES have beenand reported in patients receivingbyKYPROLIS. Discontinue KYPROLIS is neuropathy trials. Gradeoccurred 3 peripheral neuropathy occurred 1% of patients. Serious peripheral neuropathy events Cases of PRES have been reported The in patients KYPROLIS. Discontinue if PRES isexperiencing in < 1% of patients, whichinresulted in dose reduction in < 1% and treatment discontinuation suspected and evaluate. safety ofreceiving reinitiating KYPROLIS therapy in KYPROLIS patients previously occurred of Withhold patients, which resulted treatment in dose reduction in < 1% and discontinuation suspected and The safety of reinitiatingToxicity. KYPROLIS therapy can in patients previously experiencing < 1%. or discontinue as recommended [seetreatment Dosage and Administration]. Herpes PRESevaluate. is not known. Embryo-fetal KYPROLIS cause fetal harm when administered to a in in< 1% in < 1%. or discontinue treatment as recommended [see Dosage Administration]. PRES is notpregnant known.woman Embryo-fetal KYPROLIS can and cause fetal inharm whenThere administered to a and Virus Infection: Herpes zoster reactivation was reported in 2% and of patients. ConsiderHerpes antiviral prophylaxis based onToxicity. its mechanism of action findings animals. are no adequate well‑ Withhold Virus Infection: Herpeswho zoster reactivation was reported in 2% of patients. Consider antiviral prophylaxis pregnant woman basedstudies on its in mechanism of actionusing and findings in animals. There are noembryo‑fetal adequate and well‑ in pregnant for patients have a history of herpes zoster infection. controlled pregnant women KYPROLIS. Carfilzomib caused toxicity for patients who have a history of herpes zoster infection. controlled studies in pregnant women using KYPROLIS. Carfilzomib caused embryo‑fetal toxicity in pregnant rabbits at doses that were lower than in patients receiving the recommended dose. Females of reproductive DRUG INTERACTIONS: Carfilzomib is primarily metabolized via peptidase and epoxide hydrolase primarily metabolizedprofile via peptidase and isepoxide rabbits at doses that should were lower than in to patients recommended dose.treated Females reproductive activities, and Carfilzomib as a result,isthe pharmacokinetic of carfilzomib unlikelyhydrolase to be affected by potential be advised avoid receiving becomingthe pregnant while being withofKYPROLIS. If thisDRUG drug is INTERACTIONS: activities, as a result, the pharmacokinetic profile carfilzomib unlikely Carfilzomib to be affected potential should advised to avoid or becoming pregnant while being treated KYPROLIS. If this is should concomitant administration of cytochrome P450of inhibitors andisinducers. is notbyexpected to used be during pregnancy, if the patient becomes pregnant whilewith taking this drug, thedrug patient be and administration of of cytochrome P450 and inducers.section Carfilzomib used duringapprised pregnancy, or potential if the patient pregnant while taking this drug, the patient should be concomitantinfluence exposure other drugs [seeinhibitors Clinical Pharmacology of fullisPI].not expected to of the hazardbecomes to the fetus [see Use in Specific Populations]. influence exposure other drugs [see Clinical Pharmacology section of full PI]. D [see Warnings and Precautions]. apprised of ADVERSE the potentialREACTIONS: hazard to the The fetusfollowing [see Useadverse in Specific Populations]. USE IN of SPECIFIC POPULATIONS: Pregnancy. Pregnancy Category reactions are discussed in greater detail in other sections Pregnancy. Pregnancy Category D becoming [see Warnings and Precautions]. ADVERSE REACTIONS: FemalesPOPULATIONS: of reproductive potential should be advised to avoid pregnant while being treated with of the labeling:The following adverse reactions are discussed in greater detail in other sections USE IN SPECIFIC reproductiveBased potential should be advised to avoid while being treated with fetal harm of the labeling:• Cardiac Arrest, Congestive Heart Failure, Myocardial Ischemia [seeWarnings and Precautions] Females of KYPROLIS. on its mechanism of action andbecoming findings pregnant in animals, KYPROLIS can cause KYPROLIS. when Based administered on its mechanism of action woman. and findings in animals, KYPROLIS can cause fetal • Cardiac Arrest, Congestive Heart Failure, Myocardial [seeWarnings and Precautions] to a pregnant Carfilzomib caused embryo‑fetal toxicity in harm pregnant rabbits • Pulmonary Hypertension [seeWarnings andIschemia Precautions] when administered a pregnant woman. caused the embryo‑fetal toxicity in pregnant rabbits • Pulmonary• Hypertension [seeWarnings[see andWarnings Precautions] at doses tothat were lower than inCarfilzomib patients receiving recommended dose. If KYPROLIS is used during Pulmonary Complications and Precautions] at doses that were lower patients receiving the recommended If KYPROLIS is used during • Pulmonary• Complications [seeWarnings and and Precautions] pregnancy, or than if theinpatient becomes pregnant while takingdose. this drug, the patient should be apprised of Infusion Reactions [seeWarnings Precautions] pregnancy, the or ifpotential the patient becomes while taking drug, the intravenously patient shouldtobepregnant apprisedrats of and rabbits • Infusion Reactions [seeWarnings and Precautions] hazard to thepregnant fetus. Carfilzomib wasthis administered • Tumor Lysis Syndrome [see Warnings and Precautions] the potentialduring hazard the fetus. Carfilzomib was administered to pregnant • Tumor Lysis Syndrome [seeWarnings and Precautions] thetoperiod of organogenesis at doses of 0.5, 1,intravenously and 2 mg/kg/day in ratsrats andand 0.2,rabbits 0.4, and 0.8 mg/ • Thrombocytopenia [seeWarnings and Precautions] during the period at doses 0.5, 1, and 2 mg/kg/day rats andIn 0.2, 0.4,there and 0.8 • Thrombocytopenia Warnings and Precautions] kg/dayofinorganogenesis rabbits. Carfilzomib wasofnot teratogenic at any doseintested. rabbits, wasmg/ an increase in • Hepatic[see Toxicity and Hepatic Failure [seeWarnings and Precautions] kg/day Carfilzomibloss wasatnot≥ teratogenic at any tested. Inin rabbits, there wasand an increase in • HepaticThe Toxicity Hepatic Failurereactions [seeWarnings and Precautions] pre‑implantation 0.4 mg/kg/day anddose an increase early resorptions post‑implantation loss mostand common adverse (incidence of 30% or greater) to KYPROLIS observed in clinical trialsin rabbits. pre‑implantation loss at ≥ 0.4 mg/kg/day and an increase in early resorptions and post‑implantation loss The most common adverse reactions (incidence of 30% or greater) to KYPROLIS observed in clinical trials of patients with multiple myeloma were fatigue, anemia, nausea, thrombocytopenia, dyspnea, diarrhea, and a decrease in fetal weight at the maternally toxic dose of 0.8 mg/kg/day. The doses of 0.4 and 0.8 mg/ a decrease in fetal weightare at the maternally toxic of 0.8 mg/kg/day. doses of 0.4 and 0.8 mg/ of patients and with pyrexia. multiple Clinical myelomaTrials were fatigue, nausea, thrombocytopenia, diarrhea, kg/day in rabbits approximately 20% dose and 40%, respectively,The of the recommended dose in humans of Safety anemia, Experience. Because clinical trialsdyspnea, are conducted underand widely are 2approximately and area. 40%, Nursing respectively, of the recommended in humans of is excreted and pyrexia.varying Clinical Trials Safety Because clinical trials are conducted widely be kg/day 27 mg/m conditions, adverseExperience. reaction rates observed in the clinical trials of a under drug cannot directlyin rabbits based on body20% surface Mothers. It is not known dose whether KYPROLIS mg/m2 based varying conditions, reaction in the clinical trials a reflect drug cannot beobserved directly in 27 on body area. Nursing not known KYPROLIS excreted for serious comparedadverse with rates in the rates clinicalobserved trials of another drug, and mayofnot the rates medical in human milk.surface Since many drugs areMothers. excreted Itinishuman milk whether and because of theis potential compared with rates in the clinical trials of another drug, and may not reflect the rates observed in medical in human milk. Since many drugs are excreted in human milk and because of the potential for serious practice. A total of 526 patients with relapsed and/or refractory multiple myeloma received KYPROLIS as adverse reactions in nursing infants from KYPROLIS, a decision should be made whether to discontinue practice. A monotherapy total of 526 patients with relapsed and/or refractory multiple myeloma received KYPROLIS as cycles adverse infants the fromdrug, KYPROLIS, a decision be madeofwhether or with pre‑dose dexamethasone. Patients received a median of four treatment with reactions nursinginornursing to discontinue taking into account should the importance the drugtotodiscontinue the mother. Pediatric monotherapy or with cumulative pre‑dose dexamethasone. a median of four treatment cycles nursing discontinue theand drug, taking into account the importance the drughave to the Pediatric Geriatric a median KYPROLIS dose Patients of 993.4received mg. Deaths due to all causes within 30 dayswith of the last dose or toUse. The safety effectiveness of KYPROLIS in pediatricofpatients notmother. been established. a median cumulative KYPROLIS doseinof37/526 993.4 mg. due to Deaths all causes 30 days the lastprogression dose Use. The safety effectiveness of KYPROLIS in pediatric patients have not been established. of KYPROLIS occurred (7%)Deaths of patients. notwithin attributed to of disease were Use.and In studies of KYPROLIS there were no clinically significant differences observed Geriatric in safety and efficacy of KYPROLIS occurred 37/526(acute (7%) coronary of patients. Deaths cardiac not attributed disease progression were failure Use.inIn4 studies of KYPROLIS therethan were no clinically differences safetyRenal and efficacy cardiac in 5 inpatients syndrome, arrest, to cardiac disorder), end‑organ between patients less 65 years of agesignificant and patients 65 yearsobserved of age andinolder. Impairment. The cardiac in 5patients patients(multi‑organ (acute coronary syndrome, cardiac arrest, cardiac disorder), end‑organ failure in 4 between patients less than 65 years of age and patients 65 years of age and older. Renal Impairment. Thenormal renal failure, hepatic failure, renal failure), infection in 4 patients (sepsis, pneumonia, pharmacokinetics and safety of KYPROLIS were evaluated in a Phase 2 trial in patients with patients (multi‑organ hepaticinfection), failure, renal failure), infection in hemorrhage 4 patients (sepsis, pneumonia, KYPROLIS were evaluated in a Phase 2 trial in patients with normal renal dialysis. On respiratory failure, tract bacterial dyspnea and intracranial in 1 patient each, and 1pharmacokinetics patient functionand andsafety thoseofwith mild, moderate, and severe renal impairment and patients on chronic respiratory tract intracranial in 1 patient each, and 1patients. patient The function those with mild, were moderate, andforsevere renal using impairment and patients foundbacterial dead ofinfection), unknowndyspnea causes.and Serious adversehemorrhage reactions were reported in 45% most andaverage, patients treated 5.5 cycles KYPROLIS doses ofon 15chronic mg/m2dialysis. on CycleOn1, 20 mg/m2 2 2 found deadcommon of unknown causes. Serious adverse were(10%), reported 45% failure patients. Thepyrexia most (3%), average, were 5.5 cycles using and KYPROLIS doses of 15 mg/m2 on Cycle 1, 20 mg/m serious adverse reactions werereactions pneumonia acutein renal (4%), and patients on Cycle 2, treated and 27 for mg/m on Cycles 3 beyond. The pharmacokinetics and safety of KYPROLIS were 2 common serious adverse wereAdverse pneumonia (10%), acutetorenal failure (4%), pyrexia (3%), and in on Cycle andinfluenced 27 mg/m byontheCycles 3 beyond.renal The impairment, pharmacokinetics andthe safety of KYPROLIS were congestive heartreactions failure (3%). reactions leading discontinuation of KYPROLIS occurred 15% of 2,not degree and of baseline including patients on dialysis. Since dialysis congestive heart failure Adverse reactions leading discontinuation of KYPROLIS 15%creatinine, of not influenced by the degree of baseline renal impairment, the patients on dialysis. Since dialysis after the patients and(3%). included congestive heart failureto(2%), cardiac arrest, dyspnea, occurred increasedinblood and clearance of KYPROLIS concentrations has notincluding been studied, the drug should be administered patients andacute included (2%), cardiacoccurring arrest, dyspnea, and in Table clearance KYPROLIS concentrations hasPharmacology not been studied, should be administered the efficacy renalcongestive failure (1%heart each).failure Adverse reactions at a rateincreased of 10% orblood greatercreatinine, are presented 4. ofdialysis procedure [see Clinical sectiontheof drug full PI]. Hepatic Impairment.after The safety, acute renal failure each). Adverse reactions Reactions occurring at Occurring a rate of 10% greaterofare presented in Table 4. dialysis procedure [see Clinical Pharmacology of full PI].evaluated Hepatic Impairment. safety,hepatic efficacyimpairment. and pharmacokinetics of KYPROLISsection have not been in patients withThe baseline Table (1% 4: Incidence of Adverse inor≥ 10% Multiple Myeloma and pharmacokinetics of KYPROLIS havelaboratory not been values evaluated in excluded patients with hepatic impairment. Patients with the following were frombaseline the KYPROLIS clinical trials: ALT/AST Table 4: Incidence of Adverse Patients Treated withReactions KYPROLISOccurring in ≥ 10% of Multiple Myeloma Patients with excluded≥ from the KYPROLIS clinical trials: ALT/AST ≥ 3the× following upper limitlaboratory of normalvalues (ULN) were and bilirubin 2 × ULN [see Clinical Pharmacology section of full PI]. Patients Treated with KYPROLIS Patients (N = 526) ≥ 3 × upperCardiac limit of normal (ULN) and bilirubin 2 × York ULN [see Pharmacology section of fullfailure PI]. were not Impairment. Patients with≥New HeartClinical Association Class III and IV heart Patients (N = 526) [n (%)] Cardiac Impairment. Patients New YorkinHeart AssociationhasClass III andevaluated. IV heart failure were not eligible for the clinicalwith trials. Safety this population not been [n (%)] Grade 3 Grade 4 eligible for the All clinical trials. Safety has not been evaluated. OVERDOSAGE: There inis this no population known specific antidote for KYPROLIS overdosage. In the event of an All GradesaGrade 3 Events Grade 4 Events OVERDOSAGE: Event There monitor is no known specific for KYPROLIS overdosage. overdosage, the patient andantidote provide appropriate supportive care. In the event of an Gradesa 292 (55.5)Events 38 (7.2) Events Event overdosage,NONCLINICAL monitor the patient and provide appropriate supportive care. Fatigue 2 (0.4) TOXICOLOGY: Carcinogenesis, Mutagenesis, and Impairment of Fertility. Fatigue Anemia 292 (55.5) 246 (46.8)38 (7.2) 111 (21.1) 2 (0.4) NONCLINICAL TOXICOLOGY: Carcinogenesis, Mutagenesis, and Impairment Carcinogenicity studies have not been conducted with carfilzomib. CarfilzomibofwasFertility. clastogenic in the 7 (1.3) Carcinogenicity studies have not aberration been conducted with carfilzomib. Carfilzomib was clastogenic in vitro chromosomal test in peripheral blood lymphocytes. Carfilzomib was in notthe mutagenic in Anemia Nausea 246 (46.8) 236 (44.9) 111 (21.1) 7 (1.3) 7 (1.3) 0 in vitro chromosomal in peripheral blood lymphocytes. was not the in vitroaberration bacterial test reverse mutation (Ames) test and wasCarfilzomib not clastogenic in mutagenic the in vivo inmouse bone Nausea Thrombocytopenia 236 (44.9) 191 (36.3) 7 (1.3) 69 (13.1) 0 54 (10.3) the in vitro marrow bacterialmicronucleus reverse mutation (Ames) test and was not clastogenic in the in vivo mouse bone assay. Fertility studies with carfilzomib have not been conducted. No effects on Thrombocytopenia 191 (36.3) 182 (34.6) 69 (13.1) 25 (4.8)54 (10.3) Dyspnea 1 (0.2)b marrow micronucleus Fertility studiesduring with 28‑day carfilzomib have notratbeen No effects reproductiveassay. tissues were noted repeat‑dose and conducted. monkey toxicity studies on or in 6‑month b Dyspnea Diarrhea 182 (34.6) 172 (32.7)25 (4.8) reproductiverattissues were noted during 28‑daytoxicity repeat‑dose and monkey toxicityand/or studiesPharmacology. or in 6‑month Monkeys 4 (0.8) 1 (0.2) 1 (0.2) and 9‑month monkey chronic studies.ratAnimal Toxicology Diarrhea Pyrexia 172 (32.7) 160 (30.4) 4 (0.8) rat and 9‑month monkeyachronic toxicity studies. dose Animal Toxicology Pharmacology. administered single bolus intravenous of carfilzomib at 3and/or mg/kg (approximately 1.3Monkeys times recommended 7 (1.3) 1 (0.2) 2 (0.4) 2 administereddose a single bolus intravenous of carfilzomib at 3 mg/kg 1.3 times recommended in humans of 27 mg/mdose based on body surface area)(approximately experienced hypotension, increased heart rate, and Pyrexia Upper respiratory tract infection160 (30.4) 149 (28.3) 7 (1.3) 17 (3.2) 2 (0.4) 0 2 dose in humans of 27 mg/m based on body surface area) experienced hypotension, increased heart rate, and at ≥ 2 mg/ increased serum levels of troponin‑T. The repeated bolus intravenous administration of carfilzomib Upper respiratory tract infection 149 (28.3) 145 (27.6)17 (3.2) Headache 7 (1.3) 0 0 increased serum levels troponin‑T. The repeatedinbolus intravenous administration of carfilzomib ≥ 2 mg/ kg/dose in of rats and 2 mg/kg/dose monkeys using dosing schedules similar toatthose used clinically Headache Cough 145 (27.6) 137 (26.0) 7 (1.3) 0 1 (0.2) 0 kg/dose in rats and in2 mortalities mg/kg/dosethat in were monkeys dosingoccurring schedules similar to those used clinically resulted due using to toxicities in the cardiovascular (cardiac failure, cardiac Cough 137 (26.0) 1 (0.2) 0 resulted in mortalities that were fluid due toaccumulation, toxicities occurring the cardiovascular (cardiac failure, cardiac (necrosis/ Blood creatinine increased 127 (24.1) 13 (2.5) 1 (0.2) fibrosis, pericardial cardiacin hemorrhage/degeneration), gastrointestinal Blood creatinine increased 127 (24.1) 126 (24.0)13 (2.5) 84 (16.0) 1 (0.2) 11 (2.1) fibrosis, pericardial fluid renal accumulation, cardiac hemorrhage/degeneration), gastrointestinal (necrosis/ hemorrhage), (glomerulonephropathy, tubular necrosis, dysfunction), and pulmonary (hemorrhage/ Lymphopenia hemorrhage), renal (glomerulonephropathy, tubular necrosis, dysfunction), and pulmonary (hemorrhage/ inflammation) systems. The dose of 2 mg/kg/dose in rats is approximately half the recommended dose in Lymphopenia 126 (24.0) 126 (24.0) 84 (16.0) Edema peripheral 3 (0.6) 11 (2.1) 0 2of 2 mg/kg/dose in rats is approximately half the recommended dose in inflammation) systems. The dose humans of 27 mg/m based on body surface area. The dose of 2 mg/kg/dose in monkeys is approximately Edema peripheral 126 (24.0) 117 (22.2) 3 (0.6) Vomiting 5 (1.0) 0 0 2 humans of 27 mg/m based body surfacedose area.inThe dose of 2 mg/kg/dose in monkeys equivalent to the on recommended humans based on body surface area.is approximately Vomiting Constipation 117 (22.2) 110 (20.9) 5 (1.0) 1 (0.2) 0 0 equivalent toPATIENT the recommended dose INFORMATION: in humans based Discuss on body surface area. with patients prior to treatment with COUNSELING the following ConstipationNeutropenia 110 (20.9) 109 (20.7) 1 (0.2) 50 (9.5) 0 PATIENT COUNSELING INFORMATION: Discuss the following with patients prior with symptoms: 4 (0.8) KYPROLIS: Instruct patients to contact their physician if they develop anytooftreatment the following NeutropeniaBack pain 109 (20.7) 106 (20.2)50 (9.5) 15 (2.9) 4 (0.8) KYPROLIS: fever, Instruct patients contact their physician if theyof develop of the following symptoms: chills, rigors,tochest pain, cough, or swelling the feet any or legs. Advise patients that KYPROLIS may 0 fever, chills,cause rigors,fatigue, chest pain, cough,fainting, or swelling of the or legs. Advise patients that KYPROLIS may or operate dizziness, and/or dropfeet in blood pressure. Advise patients not to drive Back pain Insomnia 106 (20.2) 94 (17.9)15 (2.9) 0 0 0 cause fatigue, dizziness, fainting, and/or any dropofinthese bloodsymptoms. pressure.Advise Advisepatients patientsthat not they to drive operate shortness machinery if they experience mayorexperience Insomnia Chills 94 (17.9) 84 (16.0) 0 1 (0.2) 0 0 machinery ifofthey experience anyduring of these symptoms. patients may experience shortness breath (dyspnea) treatment with Advise KYPROLIS. This that mostthey commonly occurs within a day of dosing. Chills 84 (16.0) 83 (15.8) 1 (0.2) Arthralgia 7 (1.3) 0 0 of breath (dyspnea) during treatment mostexperience commonly occurs within a day ofCounsel dosing. patients to Advise patients to contactwith theirKYPROLIS. physiciansThis if they shortness of breath. Arthralgia Muscle spasms 83 (15.8) 76 (14.4) 7 (1.3) Advise patients contact theirsince physicians they experience shortness breath. Counsel patients to diarrhea. 2 (0.4) 0 0 avoidtodehydration, patientsifreceiving KYPROLIS therapy of may experience vomiting and/or Muscle spasms 76 (14.4) 75 (14.3) 2 (0.4) 15 (2.9) 0 avoid dehydration, patients receiving may experience vomiting and/or diarrhea. Instruct since patients to seek medicalKYPROLIS advice if therapy they experience symptoms of dizziness, lightheadedness, or Hypertension 2 (0.4) Instruct patients to spells. seek medical if they experience symptoms of dizziness, lightheadedness, fainting Counseladvice females of reproductive potential to use effective contraceptive ormeasures to Hypertension 75 (14.3) 73 (13.9)15 (2.9) 12 (2.3) 2 (0.4) Asthenia 1 (0.2) fainting spells. Counsel females of reproductive potential to useAdvise effective to pregnant prevent pregnancy during treatment with KYPROLIS. the contraceptive patient that if measures she becomes Asthenia Hypokalemia 73 (13.9) 72 (13.7)12 (2.3) 14 (2.7) 1 (0.2) 3 (0.6) prevent pregnancy during treatment with the patient if shenotbecomes pregnant treatment during treatment, to contact herKYPROLIS. physician Advise immediately. Advisethat patients to take KYPROLIS HypokalemiaHypomagnesemia 72 (13.7) 71 (13.5)14 (2.7) 2 (0.4) 3 (0.6) 0 during treatment, to contactorher physician immediately. to take KYPROLIS treatment advise her while pregnant breastfeeding. If a patient Advise wishespatients to restartnotbreastfeeding after treatment, Hypomagnesemia 71 (13.5) 71 (13.5) 2 (0.4) 27 (5.1) 0 while pregnant or breastfeeding. If a patient restart breastfeeding aftertotreatment, advise Leukopenia 1 (0.2) to discuss the appropriate timing wishes with hertophysician. Advise patients discuss with theirher physician any LeukopeniaPain in extremity 71 (13.5) 70 (13.3)27 (5.1) to discuss the appropriate with her physician. patients to discuss with theirorphysician any any new medication theytiming are currently taking prior toAdvise starting treatment with KYPROLIS, prior to starting 7 (1.3) 1 (0.2) 0 medication they are currently taking prior to starting treatment with KYPROLIS, or prior to starting any new medication(s) during treatment with KYPROLIS. Pain in extremity 70 (13.3) 67 (12.7) 7 (1.3) 52 (9.9) 0 Pneumonia 3 (0.6)b medication(s) during treatment with KYPROLIS. Pneumonia Aspartate aminotransferase increased 67 (12.7) 66 (12.5)52 (9.9) 15 (2.9) 3 (0.6)b 1 (0.2) Aspartate aminotransferase increased 66 (12.5) 66 (12.5)15 (2.9) Dizziness 5 (1.0) 1 (0.2) 1 (0.2) Dizziness Hypoesthesia 66 (12.5) 64 (12.2) 5 (1.0) 3 (0.6) 1 (0.2) 0 Hypoesthesia 64 (12.2) 63 (12.0) 3 (0.6) Anorexia 1 (0.2) 0 0 Anorexia Pain 63 (12.0) 63 (12.0) 1 (0.2) 12 (2.3) 0 0 Pain 63 (12.0) 62 (11.8)12 (2.3) 16 (3.0) 0 Hyperglycemia 3 (0.6) Hyperglycemia 62 (11.8) 60 (11.4)16 (3.0) Chest wall pain 3 (0.6) 3 (0.6) 0 Chest wall pain 60 (11.4) 58 (11.0) 3 (0.6) 13 (2.5) 0 Hypercalcemia 8 (1.5) Hypercalcemia 58 (11.0) 55 (10.5)13 (2.5) 24 (4.6) 8 (1.5) Hypophosphatemia 3 (0.6) Hypophosphatemia 55 (10.5) 54 (10.3)24 (4.6) 31 (5.9) 3 (0.6) Hyponatremia 3 (0.6) Hyponatremia 54 (10.3) 31 (5.9) (0.6) 3.0. a National Cancer Institute Common Terminology Criteria for Adverse Events (NCI CTCAE)3Version a Manufactured for: Onyx Pharmaceuticals, Inc., 249 East Grand Avenue, b National Cancer Institute Common Terminology Criteria for Adverse Events (NCI CTCAE) Version 3.0. One event was Grade 5 severity. Manufactured Pharmaceuticals, b Southfor: SanOnyx Francisco, CA 94080 Inc., 249 East Grand Avenue, One event was Grade 5 severity. South San Francisco, CANumbers: 94080 7,232,818; 7,417,042; 7,491,704; 7,737,112 05‑1088‑00 U.S. Patent U.S. Patent©2015 Numbers: 7,417,042; 7,491,704; 7,737,112 05‑1088‑00 Onyx7,232,818; Pharmaceuticals, Inc. TROPIC‑KYPR‑100826J(1) May 2015 ©2015 Onyx Pharmaceuticals, Inc. TROPIC‑KYPR‑100826J(1) May 2015


The ASCO Post  |   JULY 25, 2015

PAGE 14

Announcements Jonathan D. Licht, MD continued from page 13

Northwestern University School of Medicine.

Feinberg

mias and lymphomas by developing inhibitors against abnormally expressing epigenetic enzymes and proteins. In 2012, he received a $6.25 million grant from the Leukemia and Lymphoma Society to forward this line of research. He is also one of the principal investigators behind a $9.6 million grant Northwest-

ern University received this year from the National Cancer Institute. He has published 170 peer-reviewed articles, and his research has been cited more than 10,000 times. As UF Health Cancer Center Director, Dr. Licht succeeds Paul Okunieff, MD.

Before arriving at Northwestern, Dr. Licht was Professor at Icahn School of Medicine at Mount Sinai. He earned his medical degree at Columbia University and completed a residency at the Beth Israel Deaconess Medical Center and a fellowship at the Dana-Farber Cancer Institute. n

Jonathan D. Licht, MD

He is also the Chief Scientific Officer of the Samuel Waxman Cancer Research Foundation and on the editorial boards of numerous peer-reviewed cancer journals. He is a member of the Executive Committee of the American Society of Hematology (ASH) and is on the faculty of the ASH/European Hematology Association Translational Research Training in Hematology joint program. Dr. Licht sits on the Medical/ Scientific Board of the Leukemia and Lymphoma Society and is a member of the Basic Mechanisms of Cancer Therapeutics Study Section of the National Institutes of Health. Dr. Licht’s research efforts aim to combat blood cancers such as leuke-

The ASCO Post Wants to Hear From You

We encourage readers to share their opinions and thoughts on issues of interest to the oncology community.

Write to The ASCO Post at editor@ASCOPost.com Harborside Press Fax: 631.692.0805 www.ASCOPost.com © 2015 Genentech USA, Inc. All rights reserved. COB/092414/0002(1) Printed in USA.


ASCOPost.com  |   JULY 25, 2015

PAGE 15

Announcements

Kevin Fitzpatrick Named CEO of CancerLinQ LLC

K

evin Fitzpatrick has been named CEO of CancerLinQ LLC, a wholly owned nonprofit subsidiary of ASCO. Mr. Fitzpatrick, who is currently the Executive Vice President and Chief Innovation Officer of the Ameri-

can College of Cardiology (ACC), will begin his new role on August 3. CancerLinQ LLC was established for the development and operation of CancerLinQ™, a big data project aimed to improve the quality of cancer care.

“Mr. Fitzpatrick has the expertise and experience to make CancerLinQ the learning health system of choice for oncologists,” said ASCO CEO and CancerLinQ LLC Board of Governors Chair Allen S. Lichter, MD, FASCO.

IN ONCOLOGY, HAVE WE

MAXIMIZED THE POTENTIAL OF TARGETING THE MAPK PATHWAY?

Research has found that abnormal MAPK signaling may lead to increased or uncontrolled cell proliferation and resistance to apoptosis. Studies have shown that the MAPK pathway plays an important role in some cancers.1 Based on these findings, Genentech is investigating further ways to target the MAPK pathway.

Learn more at TargetMAPK.com.

REFERENCE: 1. Santarpia L, Lippman SM, El-Naggar AK. Targeting the MAPK-RAS-RAF signaling pathway in cancer therapy. Expert Opin Ther Targets. 2012;16:103-119.

“The ACC is a leader in developing and implementing electronic health registries for outpatient care, and he was a driving force in their expansion and utilization. We are thrilled to have Mr. Fitzpatrick bringing this same focus and commitment to CancerLinQ.”

Kevin Fitzpatrick

CancerLinQ will unlock real-world patient care data from millions of electronic health records and securely process and analyze the data to provide immediate quality feedback and personalized insights on a scope that was previously unattainable. Earlier this year, ASCO announced that CancerLinQ will be developed using SAP HANA®, a flexible, multipurpose data management and application platform created by SAP. Fifteen vanguard oncology practices from across the United States will adopt the first version of CancerLinQ beginning late in 2015. With oversight from the CancerLinQ LLC Board of Governors, the CEO will lead both CancerLinQ and ASCO staff throughout the organization to drive the mission, operations, and financial sustainability of CancerLinQ. Mr. Fitzpatrick will build upon a highly dynamic and collaborative effort within CancerLinQ, ASCO, and SAP to develop a system that is a nimble driver of change in the ever-evolving field of data and analytics. “I am honored to lead this transformational initiative,” said Mr. Fitzpatrick. “CancerLinQ truly has the potential to change the cancer care landscape, and I look forward to working with the oncology and health information technology communities to make that happen.” In his current role, Mr. Fitzpatrick jointly leads the ACC’s overall financial management and operational and strategic planning. He also serves as the Chief Senior Liaison between the ACC and its major corporate electronic health records, health information technology, and institutional partners, and oversees efforts to identify earlystage entrepreneurs tackling some of the biggest challenges facing healthcare today. n


The ASCO Post  |   JULY 25, 2015

PAGE 16

ASCO Annual Meeting Gastrointestinal Oncology

Triple Inhibition May Cripple BRAF-Mutated Colorectal Cancer By Caroline Helwick

T

riplet therapy that inhibits the BRAF, MEK, and EGFR pathways appears promising in BRAF-mutated colorectal cancer, a malignancy that typically does not respond to BRAF inhibition alone.1 At the 2015 ASCO Annual Meeting, Chloe Evelyn Atreya, MD, of the University of California, San Francisco, presented results on 67 patients with BRAF-mutated colorectal cancer, a generally difficult patient subset to treat. “These patients have a poor prognosis. With current chemotherapy regimens, median overall survival for BRAF-mutated colorectal cancer is only about 1 year, compared with 2.5 years

melanomas, BRAF-mutated colorectal tumors are much less responsive to BRAF inhibition. Although about 80% of patients with BRAF-mutated melanoma respond to vemurafenib (Zelboraf), the response rate to the same drug is just 5% in colorectal cancer. “BRAF inhibition alone is not effective in BRAFmutated colorectal cancer,” she noted. Dr. Atreya and colleagues previously evaluated the benefit of dually targeting the MAP kinase pathway with both the BRAF inhibitor dabrafenib (Tafinlar) and the MEK inhibitor trametinib (Mekinist). This strategy only slightly improved outcomes, although one patient had a complete response that was

BRAF inhibition alone is not effective in BRAF-mutated colorectal cancer. —Chloe Evelyn Atreya, MD

in BRAF wild-type tumors,” she said. Although BRAF is mutated in 50% of advanced melanomas, these mutations are observed in only about 8% of colorectal cancers. However, because colorectal cancer is common, there are about as many deaths associated with BRAF mutations in colorectal cancer as in melanoma, Dr. Atreya estimated. Also, in contrast to BRAF-mutated

ongoing for more than 3 years. Biopsies indicated that dual targeting of the MAP kinase pathway failed to shut down phosphorylated ERK signaling completely. Therefore, their current study tests the strategy of also blocking upstream with the EGFR-targeted antibody panitumumab (Vectibix). Targeting the EGFR pathway may overcome resistance to BRAF and MEK inhibi-

Triple Therapy for BRAF-Mutated Colorectal Cancer ■■ Combination treatment with the BRAF inhibitor dabrafenib, the MEK inhibitor trametinib, and the EGFR inhibitor panitumumab yielded a 26% response rate in BRAF-mutated colorectal cancers, in a phase I/II trial. ■■ Targeting EGFR may overcome resistance to BRAF and MEK inhibitors in these tumors. ■■ Median progression-free survival was 3.4 months with the doublet and 4.1 months with the triplet. Median progression-free survival was considerably higher in the dose-escalation cohorts that included panitumumab at 4.8 mg/kg, approximately 9.5 months.

tors in these tumors, the investigators hypothesized.

Study Details The phase I/II trial evaluated the combination of dabrafenib (150 mg twice daily) plus panitumumab (6 mg/kg every 2 weeks) in 20 patients with BRAF V600E-mutated metastatic colorectal cancer. The study also enrolled 35 patients on the triplet arm, which included dabrafenib, panitumumab, and trametinib in a dose-escalation scheme. Of these patients, 24 received the recommended phase II regimen of dabrafenib (150 mg twice daily) plus panitumumab (6 mg/kg every 2 weeks) and trametinib (2 mg daily). The primary endpoint was overall response rate, and progression-free survival was a secondary endpoint. “For completeness, we added two panitumumab-plus-trametinib cohorts, because secondary resistance to panitumumab or cetuximab is often mediated

by alterations upstream of MEK in the MAP kinase pathway,” she said. She presented safety data on 14 of these patients.

Increased Response With Triplet The evaluable population included 69 patients altogether who had received at least two prior treatment regimens. A total of 2 of 20 patients (10%) responded to the doublet of dabrafenib and panitumumab; 1 was a confirmed complete response. With triplet therapy, however, 9 of 35 patients (26%) responded, including 1 complete response. The unconfirmed response rate was 34%, and the stable disease rate was 60%, Dr. Atreya reported. “Triplet therapy produced more durable disease control than dabrafenib plus panitumumab. Four triplet patients (11%) have been on study for more than 1 year,” she said. Median progression-free survival was 3.4 months with the doublet and continued on page 18

EXPERT POINT OF VIEW

K

eith T. Flaherty, MD, Associate Professor of Medicine, Harvard Medical School, and Director of the Termeer Center for Targeted Therapy, Boston, was interviewed by The ASCO Post for his thoughts on the findings from Atreya and colleagues. He believes the study’s outcomes are sufficient for pursuing a “definitive trial” of this triplet in BRAF-mutated colorectal cancer. His likely comparator would be anti-EGFR therapy, which remains a standard, though imperfect, treatment for these tumors. Outcomes with triple inhibition may be much better, he suggested. “BRAF-mutated colon cancer is one of the most aggressive forms of

Keith T. Flaherty, MD

colon cancer,” said Dr. Flaherty. “The data from this early-phase study are swimming up against a pretty strong current.”

‘Lineage-Specific Biology’ Asked how the triplet might be effectively tackling BRAF-mutated

colorectal cancer, Dr. Flaherty explained, “What was quickly discovered in the lab is that when BRAFmutated colorectal cancer cell lines are treated with a BRAF inhibitor, signaling through EGFR would almost immediately engage and rescue the cells, both by restoring MAP kinase pathway signaling, which is where BRAF sits, but also by activating other pathways. Intriguingly, this rescue loop is available to colorectal cancer but not melanoma or other BRAF-mutated cancers,” he said. Such observations have made clear to Dr. Flaherty that “It is not only genetic information—ie, forget

about site of origin—or only site of origin that matters. It’s a mix of the two.” He refers to this as “lineagespecific biology.” He adds, “When you open the door with a single drug targeting a single vulnerability, the tumor system will tell you what is its next most important feature.” The BRAF-mutated colorectal cancer story is a perfect example of this approach, where “we promulgate all the single-agent opportunities but then quickly try to cycle to the combinations…. We pick the genetic lowhanging fruit, but then engage the lineage-specific biology.” n Disclosure: Dr. Flaherty is a consultant for Roche, Novartis, and Array BioPharma.


Trusted to take a bite out * of G-CSF acquisition costs GRANIX® has gained >34% share of the US short-acting G-CSF hospital market in its first 17 months1 » A 71% reduction in duration of severe neutropenia vs placebo (1.1 days vs 3.8 days, p<0.0001)2 – Efficacy was evaluated in a multinational, multicenter, randomized, controlled, Phase III study of chemotherapy-naïve patients with high-risk breast cancer receiving doxorubicin (60 mg/m2 IV bolus)/docetaxel (75 mg/m2)2 » The safety of GRANIX was established in 3 Phase III trials, with 680 patients receiving chemotherapy for either breast cancer, lung cancer, or non-Hodgkin lymphoma (NHL)2 » Offering a presentation for self-administration

Indication

» GRANIX is a leukocyte growth factor indicated for reduction in the duration of severe neutropenia in patients with nonmyeloid malignancies receiving myelosuppressive anticancer drugs associated with a clinically significant incidence of febrile neutropenia.

Important Safety Information

» Splenic rupture: Splenic rupture, including fatal cases, can occur following the administration of human granulocyte colonystimulating factors (hG-CSFs). Discontinue GRANIX and evaluate for an enlarged spleen or splenic rupture in patients who report upper abdominal or shoulder pain after receiving GRANIX. » Acute respiratory distress syndrome (ARDS): ARDS can occur in patients receiving hG-CSFs. Evaluate patients who develop fever and lung infiltrates or respiratory distress after receiving GRANIX, for ARDS. Discontinue GRANIX in patients with ARDS. » Allergic reactions: Serious allergic reactions, including anaphylaxis, can occur in patients receiving hG-CSFs. Reactions can occur on initial exposure. Permanently discontinue GRANIX in patients with serious allergic reactions. Do not administer GRANIX to patients with a history of serious allergic reactions to filgrastim or pegfilgrastim. » Use in patients with sickle cell disease: Severe and sometimes fatal sickle cell crises can occur in patients with sickle cell disease receiving hG-CSFs. Consider the potential risks and benefits prior to the administration of GRANIX in patients with sickle cell disease. Discontinue GRANIX in patients undergoing a sickle cell crisis. » Capillary leak syndrome (CLS): CLS can occur in patients receiving hG-CSFs and is characterized by hypotension, hypoalbuminemia, edema and hemoconcentration. Episodes vary in frequency, severity and may be life-threatening if treatment is delayed. Patients who develop symptoms of CLS should be closely monitored and receive standard symptomatic treatment, which may include a need for intensive care. » Potential for tumor growth stimulatory effects on malignant cells: The granulocyte colony-stimulating factor (G-CSF) receptor, through which GRANIX acts, has been found on tumor cell lines. The possibility that GRANIX acts as a growth factor for any tumor type, including myeloid malignancies and myelodysplasia, diseases for which GRANIX is not approved, cannot be excluded. » Most common treatment-emergent adverse reaction: The most common treatment-emergent adverse reaction that occurred in patients treated with GRANIX at the recommended dose with an incidence of at least 1% or greater and two times more frequent than in the placebo group was bone pain. Please see brief summary of Full Prescribing Information on adjacent page.

For more information, visit GRANIXhcp.com. *Based on wholesale acquisition cost (WAC) of all short-acting G-CSF products as of March 2015. WAC represents published catalogue or list prices and may not represent actual transactional prices. Please contact your supplier for actual prices. References: 1. This information is an estimate derived from the use of information under license from the following IMS Health Information Service: IMS National Sales Perspective, GRANIX micrograms by non-federal hospital channel March 2015. IMS expressly reserves all rights, including rights of copying, distribution, and republication (micrograms calculated as eaches x strength). 2. GRANIX® (tbo-filgrastim) Injection Prescribing Information. North Wales, PA: Teva Pharmaceuticals; 2014.

©2015 Cephalon, Inc., a wholly-owned subsidiary of Teva Pharmaceutical Industries Ltd. GRANIX is a registered trademark of Teva Pharmaceutical Industries Ltd. All rights reserved. GRX-40681 May 2015.


The ASCO Post  |   JULY 25, 2015

PAGE 18

ASCO Annual Meeting Triple Inhibition continued from page 16

4.1 months with the triplet. Median duration of response for all patients receiving the triplet was 5.4 months. Median progression-free survival was considerably higher in the doseescalation cohorts that included panitu-

mumab at 4.8 mg/kg—approximately 9.5 months. “Although the numbers are very small, we note that these patients achieved higher panitumumab drug compliance and were exposed to fewer lines of prior therapy. The role of panitumumab dose and line of treatment, therefore, will be further investigated,”

BRIEF SUMMARY OF PRESCRIBING INFORMATION FOR GRANIX® (tbo-filgrastim) injection, for subcutaneous use SEE PACKAGE INSERT FOR FULL PRESCRIBING INFORMATION 1 INDICATIONS AND USAGE GRANIX is indicated to reduce the duration of severe neutropenia in patients with nonmyeloid malignancies receiving myelosuppressive anti-cancer drugs associated with a clinically significant incidence of febrile neutropenia. 4 CONTRAINDICATIONS None. 5 WARNINGS AND PRECAUTIONS 5.1 Splenic Rupture Splenic rupture, including fatal cases, can occur following administration of human granulocyte colony-stimulating factors. In patients who report upper abdominal or shoulder pain after receiving GRANIX, discontinue GRANIX and evaluate for an enlarged spleen or splenic rupture. 5.2 Acute Respiratory Distress Syndrome (ARDS) Acute respiratory distress syndrome (ARDS) can occur in patients receiving human granulocyte colony-stimulating factors. Evaluate patients who develop fever and lung infiltrates or respiratory distress after receiving GRANIX, for ARDS. Discontinue GRANIX in patients with ARDS. 5.3 Allergic Reactions Serious allergic reactions including anaphylaxis can occur in patients receiving human granulocyte colony-stimulating factors. Reactions can occur on initial exposure. The administration of antihistamines‚ steroids‚ bronchodilators‚ and/or epinephrine may reduce the severity of the reactions. Permanently discontinue GRANIX in patients with serious allergic reactions. Do not administer GRANIX to patients with a history of serious allergic reactions to filgrastim or pegfilgrastim. 5.4 Use in Patients with Sickle Cell Disease Severe and sometimes fatal sickle cell crises can occur in patients with sickle cell disease receiving human granulocyte colony-stimulating factors. Consider the potential risks and benefits prior to the administration of human granulocyte colony-stimulating factors in patients with sickle cell disease. Discontinue GRANIX in patients undergoing a sickle cell crisis. 5.5 Capillary Leak Syndrome Capillary leak syndrome (CLS) can occur in patients receiving human granulocyte colonystimulating factors and is characterized by hypotension, hypoalbuminemia, edema and hemoconcentration. Episodes vary in frequency, severity and may be life-threatening if treatment is delayed. Patients who develop symptoms of capillary leak syndrome should be closely monitored and receive standard symptomatic treatment, which may include a need for intensive care. 5.6 Potential for Tumor Growth Stimulatory Effects on Malignant Cells The granulocyte colony-stimulating factor (G-CSF) receptor through which GRANIX acts has been found on tumor cell lines. The possibility that GRANIX acts as a growth factor for any tumor type, including myeloid malignancies and myelodysplasia, diseases for which GRANIX is not approved, cannot be excluded. 6 ADVERSE REACTIONS The following potential serious adverse reactions are discussed in greater detail in other sections of the labeling: • Splenic Rupture [see Warnings and Precautions (5.1)] • Acute Respiratory Distress Syndrome [see Warnings and Precautions (5.2)] • Serious Allergic Reactions [see Warnings and Precautions (5.3)] • Use in Patients with Sickle Cell Disease [see Warnings and Precautions (5.4)] • Capillary Leak Syndrome [see Warnings and Precautions (5.5)] • Potential for Tumor Growth Stimulatory Effects on Malignant Cells [see Warnings and Precautions (5.6)] The most common treatment-emergent adverse reaction that occurred at an incidence of at least 1% or greater in patients treated with GRANIX at the recommended dose and was numerically two times more frequent than in the placebo group was bone pain. 6.1 Clinical Trials Experience Because clinical trials are conducted under widely varying conditions, adverse reaction rates observed in the clinical trials of a drug cannot be directly compared to rates in the clinical trials of another drug and may not reflect the rates observed in clinical practice. GRANIX clinical trials safety data are based upon the results of three randomized clinical trials in patients receiving myeloablative chemotherapy for breast cancer (N=348), lung cancer (N=240) and non-Hodgkin’s lymphoma (N=92). In the breast cancer study, 99% of patients were female, the median age was 50 years, and 86% of patients were Caucasian. In the lung cancer study, 80% of patients were male, the median age was 58 years, and 95% of patients were Caucasian. In the non-Hodgkin’s lymphoma study, 52% of patients were male, the median age was 55 years, and 88% of patients were Caucasian. In all three studies a placebo (Cycle 1 of the breast cancer study only) or a non-US-approved filgrastim product were used as controls. Both GRANIX and the non-US-approved filgrastim product were administered at 5 mcg/kg subcutaneously once daily beginning one day after chemotherapy for at least five days and continued to a maximum of 14 days or until an ANC of ≥10,000 x 106/L after nadir was reached.

she added. A pharmacokinetic analysis compared tumor biopsies before and after 2 weeks of treatment. Phosphorylated ERK staining intensity (a marker of inhibition of MAP kinase signaling) was consistently reduced by triplet therapy—but not doublet therapy—she re-

Bone pain was the most frequent treatment-emergent adverse reaction that occurred in at least 1% or greater in patients treated with GRANIX at the recommended dose and was numerically two times more frequent than in the placebo group. The overall incidence of bone pain in Cycle 1 of treatment was 3.4% (3.4% GRANIX, 1.4% placebo, 7.5% non-USapproved filgrastim product). Leukocytosis In clinical studies, leukocytosis (WBC counts > 100,000 x 106/L) was observed in less than 1% patients with non-myeloid malignancies receiving GRANIX. No complications attributable to leukocytosis were reported in clinical studies. Additional Adverse Reactions Other adverse reactions known to occur following administration of human granulocyte colony-stimulating factors include myalgia, headache, vomiting, Sweet’s syndrome (acute febrile neutrophilic dermatosis), cutaneous vasculitis and thrombocytopenia. 6.2 Immunogenicity As with all therapeutic proteins, there is a potential for immunogenicity. The incidence of antibody development in patients receiving GRANIX has not been adequately determined. 7 DRUG INTERACTIONS No formal drug interaction studies between GRANIX and other drugs have been performed. Drugs which may potentiate the release of neutrophils‚ such as lithium‚ should be used with caution. Increased hematopoietic activity of the bone marrow in response to growth factor therapy has been associated with transient positive bone imaging changes. This should be considered when interpreting bone-imaging results. 8 USE IN SPECIFIC POPULATIONS 8.1 Pregnancy Pregnancy Category C Risk Summary There are no adequate and well-controlled studies of GRANIX in pregnant women. In animal reproduction studies, treatment of pregnant rabbits with tbo-filgrastim resulted in increased spontaneous abortion and fetal malformations at systemic exposures substantially higher than the human exposure. GRANIX should be used during pregnancy only if the potential benefit justifies the potential risk to the fetus. Animal Data In an embryofetal developmental study, pregnant rabbits were administered subcutaneous doses of tbo-filgrastim during the period of organogenesis at 1, 10 and 100 mcg/kg/day. Increased abortions were evident in rabbits treated with tbo-filgrastim at 100 mcg/kg/day. This dose was maternally toxic as demonstrated by reduced body weight. Other embryofetal findings at this dose level consisted of post-implantation loss‚ decrease in mean live litter size and fetal weight, and fetal malformations such as malformed hindlimbs and cleft palate. The dose of 100 mcg/kg/day corresponds to a systemic exposure (AUC) of approximately 50-90 times the exposures observed in patients treated with the clinical tbo-filgrastim dose of 5 mcg/kg/day. 8.3 Nursing Mothers It is not known whether tbo-filgrastim is secreted in human milk. Because many drugs are excreted in human milk, caution should be exercised when GRANIX is administered to a nursing woman. Other recombinant G-CSF products are poorly secreted in breast milk and G-CSF is not orally absorbed by neonates. 8.4 Pediatric Use The safety and effectiveness of GRANIX in pediatric patients have not been established. 8.5 Geriatric Use Among 677 cancer patients enrolled in clinical trials of GRANIX, a total of 111 patients were 65 years of age and older. No overall differences in safety or effectiveness were observed between patients age 65 and older and younger patients. 8.6 Renal Impairment The safety and efficacy of GRANIX have not been studied in patients with moderate or severe renal impairment. No dose adjustment is recommended for patients with mild renal impairment. 8.7 Hepatic Impairment The safety and efficacy of GRANIX have not been studied in patients with hepatic impairment. 10 OVERDOSAGE No case of overdose has been reported. ©2014 Cephalon, Inc., a wholly-owned subsidiary of Teva Pharmaceutical Industries Ltd. All rights reserved. GRANIX is a registered trademark of Teva Pharmaceutical Industries Ltd. Manufactured by: Distributed by: Sicor Biotech UAB Teva Pharmaceuticals USA, Inc. Vilnius, Lithuania North Wales, PA 19454 U.S. License No. 1803 Product of Israel GRX-40581 January 2015 This brief summary is based on TBO-004 GRANIX full Prescribing Information.

ported, adding that the magnitude of change was less than that observed with dabrafenib alone in BRAF-mutated melanoma.

Adverse Events The most common adverse events across all regimens were dermatitis acneiform and diarrhea. There were no drug-related grade 4-5 events. Dermatologic toxicities were the most significant reason for dose interruptions, delays, and reductions. “Though the numbers are small, we have noted the greatest proportion of serious dermatologic toxicities with the combination of panitumumab and trametinib, without the BRAF inhibitor, suggesting the addition of dabrafenib may lessen these [side effects],” Dr. Atreya said. To define the most effective and tolerable regimen, the researchers will expand the triplet cohort to evaluate two panitumumab doses and first vs later lines of therapy in 60 additional patients. They will also look further at the activity of panitumumab plus trametinib without dabrafenib. n Disclosure: Dr. Atreya has received research funding from GlaxoSmithKline and Novartis and has served as an advisor to Bayer Diagnostics.

Reference 1. Atreya CE, Van Cutsem E, Bendell JC, et al: Updated efficacy of the MEK inhibitor trametinib, BRAF inhibitor dabrafenib, and anti-EGFR antibody panitumumab in patients with BRAFV600E mutated metastatic colorectal cancer. 2015 ASCO Annual Meeting. Abstract 103. Presented May 31, 2015.

The ASCO Post

Like us on

Facebook facebook.com/TheASCOPost

724-43395

DIGITAL

Job Number: 21282 Revision No: 0


ASCOPost.com  |   JULY 25, 2015

PAGE 19

ASCO Annual Meeting Neuro-oncology

New Concept for Brain Tumors: Tumor Treating Fields By Caroline Helwick

A

novel treatment modality that applies electric forces to brain tumors via scalp electrodes improved outcomes in a study reported at the 2015 ASCO Annual Meeting.1 In the first report of the full dataset, tumor treating fields significantly and consistently prolonged both progression-free survival and overall survival in an international randomized, phase III trial of newly diagnosed patients with glioblastoma, according to Roger Stupp, MD, of the University of Zurich in Switzerland, who declared, “A new cancer treatment modality has been born.”

and the cell’s organelle will align during mitosis according to the field forces. If you do this 200.000× per second = 200 KHz, randomly alternating, you will actually perturb the cell to the point that ultimately it will go into apoptosis.” Because glioblastoma is a locoregional disease, it is an ideal target for tumor treating fields, he said, but he suggested its applications could be broader. “We have a new standard of care for patients with glioblastoma, but beyond that, we have a new cancer treatment modality— well beyond neuro-oncology—that we must evaluate in the cancer field,” he said.

We have a new standard of care for patients with glioblastoma, but beyond that, we have a new cancer treatment modality—well beyond neuro-oncology—that we must evaluate in the cancer field. —Roger Stupp, MD

Tumor treating fields are low-amplitude, alternating electrical fields that generate electric forces to disrupt cell division and interfere with the assembly of organelles, either directly through the membrane or by interrupting spindle checkpoints. Showing an image, Dr. Stupp noted, “You see the forces exerted on the cell,

Tumor treating fields are applied by electrodes placed on a patient’s shaved skull and connected to a device that delivers these electrical fields via a battery source. Although the device, carried in a backpack, is currently unwieldy, Dr. Stupp said its size will decrease with further refinements of the prototype.

Tumor Treating Fields for Brain Tumors ■■ A novel therapy for newly diagnosed patients with glioblastoma, known as tumor treating fields, applies electromagnetic forces to disrupt cell division. ■■ The modality is continuously delivered via electrodes on the scalp. ■■ The international phase III EF14 trial randomized 695 patients to receive maintenance temozolomide or maintenance temozolomide plus tumor treating fields after chemotherapy/radiation therapy. ■■ Patients treated with tumor treating fields achieved a median overall survival of 19.4 months compared with 16.6 months in the control arm (hazard ratio = 0.75; P = .0222). ■■ Tumor treating fields require patients to have a fully shaved head and to carry a backpack for the device almost all day. Cost is estimated at $20,000/ month.

EXPERT POINT OF VIEW

C

ommenting on the EF14 study was Martin J. van den Bent, MD, of The Brain Tumor Center at Erasmus MC Cancer Institute in Rotterdam, The Netherlands, who was reticent to predict that tumor treating fields will become a standard of care. He noted that 57% of patients are

definitely evaluate the survival benefit of the treatment, he said. Dr. van den Bent also suggested that the intensive monitoring of the active group may have created a favorable situation for this arm, similar to the way in which early palliative care actually improves survival, according to several studies. In this regard, a sham control arm might have been informative.

Practical Details

Martin J. van den Bent, MD

still alive; therefore, the overall survival data are not mature. The study results were made available prematurely upon the independent data monitoring committee’s recommendation based on an interim analysis, and the control arm was invited to cross over to the intervention. Thus, the study cannot

Longer Survival The international phase III EF14 trial randomized 695 newly diagnosed patients with glioblastoma to receive maintenance temozolomide or maintenance temozolomide plus tumor treating fields after completing chemotherapy/radiation therapy. Patients treated with tumor treating fields achieved a median overall survival of 19.4 months compared with 16.6 months in the control arm (hazard ratio [HR] = 0.75; P = .0222). Those in the tumor treating fields group also saw a 14% increase in 2-year survival compared with patients in the control arm (43% vs 29%). Median progression-free survival was 7.1 months for the tumor treating fields group compared with 4.2 months for the control group (HR = 0.69; P = .0010). “We don’t see any added toxicity

He was also concerned about the “practical details” of tumor treating fields. “There is continuous treatment, which places a burden on patients. It requires a fully shaved head for the electrodes to fit. Patients must wear a backpack all day (with a few breaks allowed), and electrodes must be changed every few days,” he indicated. “The device brings lifestyle issues and hassles, but also, is it worth the cost, which is estimated at $20,000 per month?” he asked. n Disclosure: Dr. van den Bent reported no potential conflicts of interest.

other than toxicity related to the local administration of treatment or carrying of the device,” he added. “What we see is skin reaction to the electrodes or sometimes reaction to the tape where electrodes are attached to the skull. [There was] no increase in seizures, no increase in other neurologic dysfunction, as much as we can tell.” n

Disclosure: Dr. Stupp served as the principal coordinating investigator for the EF14 trial and an uncompensated advisor to Novocure Ltd.

Reference 1. Stupp R Taillibert S, Kanner A, et al: Tumor treating fields (TTFields): A novel treatment modality added to standard chemo- and radiotherapy in newly diagnosed glioblastoma—First report of the full dataset of the EF14 randomized phase III trial. 2015 ASCO Annual Meeting. Abstract 2000. Presented June 2, 2015.

Visit The ASCO Post website at ASCOPost.com


The ASCO Post  |   JULY 25, 2015

PAGE 20

ASCO Annual Meeting Hematology

Advancing Classification and Risk Stratification for Leukemia and Myelodysplastic Syndromes By Alice Goodman

R

efinements in the classification and risk stratification for leukemia and myelodysplastic syndromes were reported by three different investigators at the 2015 ASCO Annual Meeting. The first study showed that leukemia stem cell phenotypes are associated with outcomes in acute myeloid leukemia (AML).1 The second study suggested that the number of activating killer-cell immunoglobulin-like receptor (KIR) genes is associated with the development of myelodysplastic syndromes (MDS), with the potential to be used for risk stratification.2 The third study highlighted an updated international scoring system for chronic lymphocytic leukemia (CLL) that incorporates genetic risk factors.3

FLT3/ITD mutations were uncommon in the CD34-negative phenotype (4 of

22, 18%) and the ALDH-intermediate phenotype (17 of 43, 35%) but were

found in 28 of 33 patients (85%) in the ALDH-high group (P < .001).

ADVERTORIAL

Leukemia Stem Cells in AML Three distinct, mutually exclusive subtypes of leukemia stem cells recently identified are correlated with both specific cytogenetic/molecular risk factors and response and outcomes. These leukemia stem cell phenotypes are based on CD34, CD38, and aldehyde dehydrogenase (ALDH) expression as follows: • CD34-negative • CD34-positive, CD38-negative, ALDH-intermediate (most common subtype) • CD34-positive, CD38-negative, ­A LDH-high In AML, remission does not equal cure. “Current prognostic factors [cytogenetics] are limited, and it can take over a week to get test results, which is not good for such an explosive disease. Grouping AML patients according to leukemia stem cell phenotype may permit rapid risk stratification without identifying molecular/cytogenetic features,” stated lead author Jonathan Gerber, MD, of Levine Cancer Institute, Charlotte, North Carolina. The phase II NCTG0143880 trial was based on tissue samples available for 98 of 165 newly diagnosed patients with poor-risk disease or those who had unfavorable cytogenetic AML. Forty percent of patients harbored adverse cytogenetics. Fluorescent in situ hybridization and/or polymerase chain reaction were used to analyze the cell populations for leukemia-specific abnormalities, and leukemia stem cell phenotypes were found to correlate with specific cytogenetic and molecular risk factors. Poor-risk cytogenetics and/or

CANCER STEM CELLS

SIGNALING PATHWAYS

Cancer Stem Cells and Their Role Despite current advances in cancer therapy, tumor recurrence and metastasis remain a clinical challenge.1 Cancer stem cells are a subset of the total cancer cell population that is highly tumorigenic.2,3 Chemotherapy and radiation have been shown to affect the primary tumor, but not the cancer stem cell.4 Many patients with cancer, even though diagnosed early, succumb to the disease because of recurrence and metastasis.5,6 Cancer stem cells are thought to contribute to this recurrence and metastasis.7 Another characteristic of cancer stem cells is that they possess stemness. Stemness distinguishes cancer stem cells from ordinary cancer cells by their ability to continually self-renew, differentiate into cancer cells, migrate, and regrow the tumor.7,8

Most chemotherapeutic strategies target actively proliferating cancer cells, resulting in bulk tumor shrinkage. Cancer stem cells, however, may be highly resistant to these therapies and may not be eradicated during treatment, resulting in recurrence and metastasis. 4,7 Moreover, chemotherapy and radiation have the potential to induce stemness properties in non-stem cancer cells.2,9 Several signaling pathways are involved in the induction and maintenance of stemness in cancer stem cells, including JAK/STAT, Wnt/ β-catenin, Hedgehog, Notch, and Nanog.10-12 Targeting these aberrant signaling pathways may result in cancer stem cell apoptosis, while reducing the toxicity to normal tissues that is associated with chemotherapy.4


ASCOPost.com  |   JULY 25, 2015

PAGE 21

ASCO Annual Meeting NPM1 mutations, associated with a good prognosis, were found in 14 (64%) of the CD34-negative group vs 6 (14%) of the ALDH-intermediate and 2 (6%) of the ALDH-high group (P < .001). Outcomes differed according to leukemia stem cell phenotype. The rate of complete remission was 86% in the

CD34-negative group; complete remission was 67% in the ALDH-intermediate group and 45% in the ALDHhigh group. Median event-free survival was 13 months, 11.3 months, and 2.2 months for the three phenotypes, respectively. The rate of 2-year event-free survival was 46%, 26%, and 0% for the

REGROWTH

three phenotypes, respectively. Overall survival was not yet reached in the CD34-negative patients; overall survival was 18.7 months in the ALDHintermediate group and 2.2 months in the ALDH-high group. “Risk stratification according to leukemia stem cell phenotype makes

APOPTOSIS

in Recurrence and Metastasis Boston Biomedical is developing the next generation of cancer therapeutics with drugs designed to inhibit cancer stem cell pathways. Clinical trials are underway with the goal of reducing recurrence and metastasis.

5/2015

©2015 Boston Biomedical

Risk stratification according to leukemia stem cell phenotype makes it possible to divert high-risk patients to earlier novel forms of therapy and transplant. These patients don’t do well on conventional chemotherapy. —Jonathan Gerber, MD

cells and minimal residual disease with persistent ALDH expression. The presence of minimal residual disease predicted prognosis; six of seven patients with minimal residual disease relapsed. All patients with no evidence of minimal residual disease remained in clinical remission more than 4 years later.4

Activating KIR Genes in MDS Learn more at www.bostonbiomedical.com

References: 1. Li Y, Rogoff HA, Keates S, et al. Supression of cancer relapse and metastasis by inhibiting cancer stemness. Proc Natl Acad Sci. 2015;112(6):18391844. 2. Hu X, Ghisolfi L, Keates AC, et al. Induction of cancer stemness by chemotherapy. Cell Cycle. 2012;11(14):2691-2698. 3. Clarke MF. Self-renewal and solid-tumor stem cells. Biol Blood Marrow Transplant. 2005:11(2 suppl 2):14-16. 4. Boman BM, Huang E. Human colon cancer stem cells: A new paradigm in gastrointestinal oncology. J Clin Oncol. 2008;26(17):2828-2838. 5. Ahmad A. Pathways for breast cancer recurrence. ISRN Oncol. 2013;2013:290568. doi: 10.1155/2013/290568. 6. Hung JH, Wu YC. Stage I non-small cell lung cancer: recurrence patterns, prognostic factors and survival. In: Cardoso P, ed. Topics inThoracic Surgery. Shanghai, China: InTech; 2012:285-292. http://www.intechopen.com/books/topics-in-thoracic-surgery/stage-i-non-smallcell-lungcancer-recurrence-patterns-prognostic-factors-and-survival. Accessed May 8, 2015. 7. Jordan CT, Guzman ML, Noble M. Cancer stem cells. N Engl J Med. 2006;355(12):1253-1261. 8. Gupta PB, Chaffer CL, Weinberg RA. Cancer stem cells: mirage or reality? Nat Med. 2009;15(9):1010-1012. 9. Ghisolfi L, Keates AC, Hu X, Lee D, Li CJ. Ionizing radiation induces stemness in cancer cells. PLOS ONE. 2012;7(8):1-11. 10. Hoffmeyer K, Raggioli A, Rudloff S, et al. Wnt/βcatenin signaling regulates telomerase in stem cells and cancer cells. Science. 2012;336(6088):1549-1554. 11. Bourguignon LYW, Earle C, Wong G, Spevak CC, Krueger K. Stem cell marker (Nanog) and Stat-3 signaling promote MicroRNA-21 expression and chemoresistance in hyaluronan/CD44-activated head and neck squamous cell carcinoma cells. Oncogene. 2012;31(2):149-160. 12. Espinoza I, Pochampally R, Xing F, Watabe K, Miele L. Notch signaling: targeting cancer stem cells and epithelial-to-mesenchymal transition. Onco Targets Ther. 2013;6:1249-1259.

EDU-NPS-0027

it possible to divert high-risk (ALDHhigh) patients to earlier novel forms of therapy and transplant. These patients don’t do well on conventional chemotherapy,” Dr. Gerber said. Two patterns were evident in patients in complete remission: normal with no ALDH and no leukemia stem

The number of activating KIR genes (killer-cell immunoglobulin-like receptor genes) is related to the risk of developing MDS and categorizing patients with MDS as high risk vs low risk. However, the number of KIR genes is not related to overall survival, according to a preliminary case-control study. The study population included 180 patients with MDS; of them, 120 were at low risk and 60 were at high risk. Patients with MDS were compared with 117 healthy donor controls. The age distribution was similar between both groups; more than 80% were older than age 60. DNA samples were collected in both cases and controls and analyzed for activating KIR genes using polymerase chain reaction. Patients with high-risk MDS had significantly lower numbers of activating continued on page 22


The ASCO Post  |   JULY 25, 2015

PAGE 22

ASCO Annual Meeting Risk Stratification continued from page 21

KIR genes than did patients with low-risk MDS (P = .009) and healthy controls (P = .00001). Patients with low-risk MDS also had significantly lower numbers of activating KIR genes than did controls (P = .04). Moreover, the inheritance of each additional activating KIR gene had a protective effect against the development of high-risk MDS (P < .001). “This study provides novel insights concerning the pathogenesis of MDS in adults. These results suggest that inheritance of a higher number of activating KIR genes is protective against MDS and that once the

tem. The CLL–IPI International Working Group has developed the International Prognostic Index for patients with CLL (CLL–IPI), which combines genetic risk factors with clinical stage, age, and beta-2 microglobulin (B2M) into a clinically applicable prognostic score. Moreover, the CLL–IPI discriminates between prognostic groups and can inform treatment decisions. “The traditional staging systems developed more than 30 years ago for CLL do not discriminate enough anymore. There are newly discovered prognostic markers, but no system integrates all these markers. We report on an international effort to develop a comprehensive score,” said Nadine Kutsch, MD, on behalf of the International CLL–IPI working group. Dr. Kutsch is from the Center for Integrated Oncology Cologne Bonn, University of Cologne, Germany.

May Daher, MD

disease develops, harboring a lower number of KIR genes is associated with higherrisk disease,” said lead author May Daher, MD, of The University of Texas MD Anderson Cancer Center, Houston. The major concern with MDS is the potential to evolve to AML, which is difficult to treat, and occurs in about 20% of patients with MDS. In this study, transformation to AML occurred in 12% of lowrisk patients vs 20% of the high-risk group.

New CLL–IPI Scoring System MDS can be risk stratified using the International Prognostic Scoring Sys-

Nadine Kutsch, MD

The full analysis set relied on data from eight phase III trials in Europe that included 3,472 treatment-naive patients in various stages of CLL. The median age was 61 years, the median observation time was 80 months, and 89% of patients have received treatment so far. The full analysis set was randomly divided into training (n = 2,308) and

Risk Stratification in Hematologic Malignancies ■■ Leukemia stem cell phenotypes are correlated with cytogenetic/molecular risk factors and outcomes in AML. ■■ The number of activating KIR (killer-cell immunoglobulin-like receptor) genes is linked with MDS and can separate high- from low-risk disease. ■■ A new international scoring system for CLL incorporates genetic prognostic factors and can guide management strategy.

internal validation datasets (n = 1,164). The new CLL–IPI was externally validated in a third dataset comprising prospectively collected data from 848 patients treated at the Mayo Clinic, Rochester, Minnesota; the median age of patients was 62 years, and 39% have received treatment so far. The different analysis steps identified five independent predictors for overall survival, and the parameters from the regression model were used to grade the factors according to their individual impact: TP53 (17p) del/mut = 4, IGHV unmutated = 2, B2M > 3.5 mg/L = 2, clinical stage Binet B/C or Rai I–IV = 1, age > 65 years = 1. A prognostic score was developed ranging from 0–10, and four different risk groups were derived: prognostic score of 0–1, low risk; prognostic score of 2–3, intermediate risk; prognostic score of 4–6, high risk; and prognostic score of 7–10, very high risk. These four risk groups, respectively, had different overall survival rates at 5 years: 93%, 79%, 64%, and 23% (P < .001). This model was confirmed on the internal validation dataset and then on the Mayo set, with 5-year overall survival rates of 94%, 91%, 68%, and 21%, for the

low to very-high-risk groups, respectively. The CLL–IPI system is clinically applicable. Low-risk patients should be managed with “watch and wait.” Intermediate-risk patients should not be treated unless they are symptomatic. High-risk patients should be treated, unless they are asymptomatic. Veryhigh-risk patients should be treated with experimental approaches. n

Disclosure: Dr. Kutsch has received honoraria from Janssen and Grifols and travel grants from Mundipharma. Drs. Gerber and Daher reported no potential conflicts of interest.

References 1. Gerber JM, Zeidner JF, Morse S, et al: Correlation of acute myeloid leukemia stem cell phenotype with cytogenetic/molecular features and prognosis. 2015 ASCO Annual Meeting. Abstract 7000. 2. Daher M, Sobieski C, Marin D, et al: Association between KIR genes and risk of MDS. 2015 ASCO Annual Meeting. Abstract 7001. 3. Kutsch N, Bahlo J, Byrd JC, et al: The International Prognostic Index for patients with CLL (CLL-IPI). 2015 ASCO Annual Meeting. Abstract 7002. 4. Gerber JM, Smith BD, Ngwang B, et al: A clinically relevant population of leukemic CD34+CD38– cells in acute myeloid leukemia. Blood 119:3571-3577, 2012.

EXPERT POINT OF VIEW

W

endy Stock, MD, of the University of Chicago, was the formal discussant of these three studies presented at the 2015 ASCO Annual Meeting. Commenting on the first study on acute myeloid leukemia (AML) by Dr. Gerber and colleagues, she said, “This work can improve understanding of the leukemia stem cell and the course of disease. Many investigators are working in this area. Specific mutations lead to preleukemia and additional mutations lead to leukemia.” “We are hopeful that patients with AML will remain in remission, but many relapse after achievement of ‘remission.’ This is likely because we have only succeeded in returning the

leukemic clone to a preleukemic state, which eventually acquires the same or new mutations that will lead to

Wendy Stock, MD

relapse of the disease. The technique described by Dr. Gerber might help us identify what lurks in these residual marrows,” Dr. Stock said. “Moving forward, AML evolu-

tional heterogeneity is a challenge for risk assessment and development of new effective therapies. Aldehyde dehydrogenase (ALDH) expression in remission may be a method to help identify patients at high risk of relapse,” she added. Turning to the second study on myelodysplastic syndromes (MDS) by Dr. Daher and colleagues, Dr. Stock commented, “The novel and major finding of this explorative study is that natural killer cell genotype may be associated with MDS risk.” Dr. Stock added, “This work is interesting and preliminary. We can’t assume the number of KIR genes is associated with prognosis. The findings need to be confirmed in a larger

cohort of MDS patients. We need to determine if activating KIRs do provide immune surveillance and protection for the myeloid progenitor cells.” Finally, Dr. Stock commented on the third study on chronic lymphocytic leukemia (CLL) by Dr. Kutsch and colleagues. “The CLL–IPI [International Prognostic Index] is an important contribution to the field of CLL. This system will help us put a plethora of prognostic factors into perspective,” she said. “The score is easily applicable and can be applied prospectively. Weighted scoring helps clarify the impact,” concluded Dr. Stock. n

Disclosure: Dr. Stock reported no potential conflicts of interest.


ASCOPost.com  |   JULY 25, 2015

PAGE 23

ASCO Annual Meeting Dermatologic Oncology

PET/CT Detects Asymptomatic Melanoma Recurrences By Caroline Helwick

I

n monitoring patients with melanoma at high risk for relapse, surveillance imaging with 18F-fluorodeoxyglucose–positron-emission tomography (FDG-PET)/computed tomography (CT) can detect asymptomatic metastases and thus facilitate early treatment, according to Australian investigators who presented their findings at the 2015 ASCO Annual Meeting.1 Jeremy Lewin, MBBS, of the Peter MacCallum Cancer Centre, East Melbourne, Victoria, Australia, presented the study, which sought to determine if PET/CT can accurately detect early recurrences and thus facilitate earlier treatment.

Benefits and Risks “Many benefits and risks are associated with radiologic surveillance of stage III melanoma,” Dr. Lewin noted. “Among the benefits are the ability to identify treatable locoregional recurrences with the hope of allowing access to early surgical resection or effective new systemic treatments. There is also the reassurance associated with negative scans,” he said.

ing increased asymptomatic relapses,” he said.

Study Details Since 2009, patients at the Peter MacCallum Cancer Centre with fully resected stage III melanoma have undergone such surveillance. This retrospective analysis included 86 patients who had whole-body PET/CT scans, performed according to their estimated substage relapse probabilities. PET/CT scans were performed as follows: • Stage IIIA: 6, 18 months (n = 11) • Stage IIIB: 6, 12, 18, 24, 36, 48, 60 months (n = 50) • Stage IIIC: 6, 12, 18, 24, 36 months (n = 25), including MRI of brain at 6 and 12 months This resulted in a total of 232 PET/CT scans. Relapse was suspected in 35 patients (41%); in this group, 25 relapses were confirmed, and 10 patients were determined to be disease-free. Confirmed relapses, therefore, were observed in 29% of the total scanned group, including 1 of 11 stage IIIA patients (9%), 15 of

The high negative predictive value [of the study’s PET/ CT surveillance protocol] provides reassurance. —Jeremy Lewin, MBBS

Potential risks include false-positives, cumulative radiation exposure, increased cost of care, inconvenience to patients, and psychological distress in anticipation of the scans, he said. Dr. Lewin noted that relapse in stage III melanoma is not homogeneous but is dependent upon substage and may be curable in many cases. The optimal surveillance strategy remains unclear and is often done at the discretion of the treating physician, he indicated. “Given the increasing sensitivity of FDG-PET in identifying metastatic disease, we have been conducting substage-specific radiologic surveillance in stage III patients, in hopes of identify-

The ASCO Post Wants to Hear From You

50 stage IIIB patients (28%), and 10 of 25 stage IIIC patients (40%). “Of the 25 relapses, 22 (88%) were asymptomatic at the time of scanning,” Dr. Lewin reported. “First relapses in stage IIIA/B patients were more likely than stage IIIC to be locoregional.” Among stage IIIA and stage IIIB patients, 40% of relapses were locoregional, whereas 60% were metastatic. Among stage IIIC patients, only 10% were locoregional, whereas 90% were metastatic. Nine relapsed patients (36%) were able to undergo potentially curative continued on page 24

2015 ASPIRE Breast Cancer Research Awards Program Pfizer is proud to announce the Advancing Science through Pfizer – Investigator Research Exchange (ASPIRE) Breast Cancer Research Awards, a competitive, peerreviewed grants program sponsored by Pfizer for investigators in the United States

Call for Research Proposals For complete information on the scope of research, please visit ASPIRE website at www.aspireresearch.org Mission The mission of the ASPIRE Awards is to support clinical research of a Pfizer compound in advanced breast cancer through a competitive grants program to advance knowledge in the treatment and disease management of advanced breast cancer. Awards 2015 ASPIRE Breast Cancer Research Awards program intends to fund three to six clinical studies within scope, for a total of approximately 3 million US dollars. It is open to US investigators. Selection of research proposals will be performed by an independent external review panel of breast cancer experts.

Submissions are due September 8, 2015

We encourage readers to share their opinions and thoughts on issues of interest to the oncology community. Write to The ASCO Post at editor@ASCOPost.com www.aspireresearch.org


The ASCO Post  |   JULY 25, 2015

PAGE 24

ASCO Annual Meeting PET/CT Surveillance continued from page 23

resection, including one stage IIIA patient, six stage IIIB patients, and two stage IIIC patients; five of them (all stage IIIA/B) were free of disease after a median of 32 months’ followup after nodal resection. Most relapsed patients who were unable to have surgery did receive systemic therapy, including ipilimumab (Yervoy), BRAF inhibition, radiotherapy, and chemotherapy. Incidental findings were detected 24 times in 19 patients (22%). This included 6 patients with a new diagnosis of a different malignancy; all received treatment, most with curative intent. A number of patients had other incidental, nonmalignant findings necessitating medical workup

PET/CT in Stage III Melanoma ■■ Australian investigators reported that patients with stage III melanoma who are at high risk for recurrence can be effectively monitored for recurrence with PET/CT surveillance performed at intervals dictated by disease substage. ■■ Recurrences were detected in 29% of 86 patients. ■■ These recurrences were asymptomatic in 88% of patients. ■■ The negative predictive value for the protocol was 94%, and the positive predictive value was 69%.

Sensitivity and Specificity “We found a favorable sensitivity and specificity of substage-specific PET/CT after resected stage III melanoma,” Dr. Lewin reported. By the gold standard of histologically confirmed relapse during the surveillance period, the protocol had

a sensitivity of 88% and a specificity of 84%. Based on a prevalence of 29%, the negative predictive value was 94%, though the positive predictive value was only 69%, he indicated. Negative predictive value was also very high when analyzed perscan and per-patient. Per-scan, the

negative predictive value was 99% for stage IIIB and 97% for stage IIIC patients, and per-patient negative predictive value was 97% and 86%, respectively. The positive predictive value per-scan was 69% and 73%, respectively, and per-patient positive predictive value was 68% and 73%, respectively. “The high [negative predictive value] provides reassurance,” Dr. Lewin commented. n Disclosure: Dr. Lewin reported no potential conflicts of interest.

Reference 1. Lewin J, Sanelli A, Walpole I, et al: Surveillance imaging with FDG-PET in the follow-up of melanoma patients at high risk of relapse. 2015 ASCO Annual Meeting. Abstract 9003. Presented May 30, 2015.

EXPERT POINT OF VIEW

D

aniel G. Coit, MD, of Memorial Sloan Kettering Cancer Center, New York, and Chair of the National Comprehensive Cancer Network (NCCN) Melanoma Guidelines Panel, discussed the findings reported by Lewin et al at the ASCO Annual Meeting. “This is a small retrospective study examining a prospective database of patients with resected stage III melanoma who had PET [positron-emission tomography] scans as a part of a consistent surveillance program, looking at the accuracy of PET. The ‘gold standard’ of true-

Daniel G. Coit, MD

positive was the histologic status of the positive PET finding. There is no consistent working definition of a false-negative—ie, relapse after a negative PET—and, therefore, this biases the results toward a higher

[negative predictive value] (truenegative divided by true-negative plus false-negative),” he suggested. The study does confirm the relatively high false-positive rate of PET, with almost one-third (10 of 35) of the positive scans ultimately found to be unrelated to recurrent melanoma. It does not provide information as to what proportion of all recurrences were detected by patients or physicians without tests. Importantly, the study also lacks a contemporary comparator to PET scans to guide the use of this information, he maintained.

Dr. Coit also questioned whether immediate surgical resection of low-volume asymptomatic systemic recurrence detected by PET is likely to impact long-term outcomes. However, he acknowledged, “while we are not looking for the earliest opportunity to jump in … in the era of effective systemic therapy, we do need to reexamine whether early detection of systemic recurrence is associated with improved survival beyond increased lead time.” n Disclosure: Dr. Coit reported no potential conflicts of interest.

The ASCO Post Wants to Hear From You We encourage readers to share their opinions and thoughts on issues of interest to the oncology community. Write to The ASCO Post at editor@ASCOPost.com

www.ASCOPost.com

Phone: 631.692.0800 Fax: 631.692.0805

Harborside Press 37 Main Street Cold Spring Harbor, NY 11724


ASCOPost.com  |   JULY 25, 2015

PAGE 25

ASCO Annual Meeting Targeted Therapy

Precision Medicine Trials Bring Targeted Treatments to More Patients By Caroline Helwick

A

t the 2015 ASCO Annual Meeting, both ASCO and the National Cancer Institute (NCI) unveiled details of new precision medicine trials that will greatly expand the number of patients with cancer who are benefiting from targeted agents. The trials will match a patient’s tumor molecular profile with an agent that addresses its particular genomic variants. The underlying concept of these efforts is the growing realization that cancer biology is complex and that variants within a tumor are often unexpected.

able drugs now target molecular pathways that are frequently aberrant in tumors, with many more in the pipeline. Tumor/drug pairings will be lim-

ited to currently available agents (used off-label) in the TAPUR trial, whereas several experimental agents will also be available in the NCI-MATCH trial.

Trials Solve a Growing Problem Dr. Schilsky sees the studies as a solution to a dilemma becoming quite continued on page 26

Our commitment to cancer research To deliver on the promise of precision medicine, we will test drugs based on the biology of the cancer, not the anatomic site. —Clifford A. Hudis, MD, FACP

TAPUR and NCI-MATCH Trials “Almost every cancer could be seen as rare with unique molecular characteristics,” said Clifford A. Hudis, MD, FACP, Past President of ASCO. With that premise in mind, he added, “We are announcing the expansion of a project we have been pursuing.” ASCO’s Targeted Agent and Profiling Utilization Registry (TAPUR) was described at a press briefing by Dr. Hudis and Richard L. Schilsky, MD, FACP, FASCO, Chief Medical Officer of ASCO, while the NCI’s Molecular Analysis for Therapy Choice (MATCH) trial was outlined by James H. Doroshow, MD, FACP, Deputy Director of the NCI. “To deliver on the promise of precision medicine, we will test drugs based on the biology of the cancer, not the anatomic site,” Dr. Hudis continued. This approach to clinical trials, he added, represents “a dramatic shift toward research based on biology and a paradigm move in the field.” More than 30 commercially avail-

Merck is passionate about improving health and is committed to helping people with cancer. Keynote is a series of investigational trials studying pembrolizumab (MK-3475)—a type of investigational immunotherapy that works by targeting the PD-1 pathway. Merck has investigational clinical trials underway or planned in multiple cancer types.

If you have patients who may be eligible for a trial, visit

www.keynoteclinicaltrials.com

for more information about this ongoing research.


The ASCO Post  |   JULY 25, 2015

PAGE 26

ASCO Annual Meeting Precision Medicine Trials continued from page 25

common in the clinic: what to do with the results of molecular testing. “Some 40% to 70% of the time, a genomic profile on a patient’s tumor is likely to turn up something the physician can act on. The question is, how to get the drug suggested by the patient’s tumor profile,” he said. Gaining access to targeted drugs off-label can be difficult for a variety of reasons, he noted. “And even if a patient can receive the drug, we have no mechanism for learning from the experience of that patient,” he said. Accelerated Approval and Breakthrough Therapy status are increasingly being granted in the U.S. Food and Drug Administration’s drug approval process. “Many drugs are being introduced into practice now with far less information than we have had before,” Dr. Schilsky noted. “That’s what we designed TAPUR for—so we can learn from the realworld practice of prescribing targeted treatments.” When patient access is facilitated, he added, “the angst of getting the drug is taken off the table.” If TAPUR succeeds, he further predicted, “benefits will be accrued to all stakeholders.”

TAPUR Launching Early 2016 Early next year, TAPUR will start enrolling patients from three healthcare systems—the Michigan Cancer Research Consortium, Cancer Research Consortium of West Michigan, and Carolinas HealthCare System. The

Table 1: Initial 10 Substudies of the NCI-MATCH Clinical Trial

Agent

Molecular Target

Estimated Prevalence

Crizotinib

ALK rearrangement

4%

Crizotinib

ROS1 translocations

5%

Dabrafenib and trametinib

BRAF V600E or V600K mutations

7%

Trametinib

BRAF fusions, or non-V600E, non-V600K BRAF mutations

2.8%

Agents Pending Final Regulatory Review Ado-trastuzumab emtansine

HER2 amplification

Afatinib

EGFR-activating mutations

1%–4%

Afatinib

HER2-activating mutations

2%–5%

AZD9291

EGFR T790M mutations and rare EGFR-activating mutations

1%–2%

Sunitinib

cKIT mutations

4%

VS6063

NF2 loss

2%

study will be open to patients with advanced solid tumors, B-cell non-Hodgkin lymphoma, and multiple myeloma who are no longer responding to available drugs and who demonstrate a genomic variant known to be a drug target or to predict sensitivity to a drug. At the time of the announcement, 13 different targeted agents will be supplied free of charge by five participating manufacturers—AstraZeneca, Bristol-Myers Squibb, Eli Lilly and Company, Genentech, and Pfizer. Other companies are likely to join, Dr. Schilsky indicated.

Precision Medicine Trials ■■ ASCO and the NCI are launching precision medicine trials that will match patients’ tumor variants with targeted agents. ■■ ASCO’s TAPUR trial will soon enroll patients from three health-care systems, with drugs provided free of charge by five manufacturers. ■■ NCI’s MATCH trial will encompass the 2,400 centers affiliated with their National Clinical Trials Network. ■■ The aim is to facilitate patient access to targeted agents and to evaluate the efficacy of these agents in tumor-variant groups, in real-world settings.

A molecular tumor board will review the proposed drug-target match and guide the clinician on potential treatments, on or off the study. For patients enrolled and treated through TAPUR, safety and efficacy outcomes will be recorded. TAPUR’s primary endpoint is objective response rate, which will give a signal of the drug’s potential activity. It is ASCO’s intent that patients who respond to a therapy would be able to continue to receive that drug at no cost for as long as the patient benefits. For more on the TAPUR trial, see the June 10, 2015, issue of The ASCO Post, page 95, or visit www.asco.org/ TAPUR.

NCI-MATCH in 2,400 Sites The prospective NCI-MATCH trial is larger and available to more clinicians than TAPUR, which is a registry. NCI-MATCH, in fact, is the “largest, most rigorous precision oncology trial in history” and “unprecedented in its scope,” Dr. Doroshow announced, adding, “The pain and

5%

gain of getting this going has been worth it.” The study is being conducted throughout the 2,400 sites of the NCIsupported National Clinical Trials Network. Dr. Doroshow predicted that 3,000 patients will be screened to find 1,000 with mutations to which a drug can be matched. With objective response as the primary endpoint, the study will initially focus on 10 variants and treatment arms (Table 1). The number of targets will increase soon to 20 or more. “If we receive the resources we have requested, we hope to leverage this trial and other precision medicine trials and develop further MATCH-like studies where we go deeper into a particular disease, based on early signals from this investigation,” Dr. Doroshow said. For more on the NCI-MATCH trial, visit www.cancer.gov/about-cancer/ treatment/clinical-trials/nci-supported/nci-match#1. n Disclosure: Drs. Hudis, Doroshow, and Schilsky reported no potential conflicts of interest.

Visit The ASCO Post website at ASCOPost.com


Change your perspective on what’s possible for your patients with multiple myeloma


3

21

153

High-dose dex

6

43

2

15

6 9 Months (ITT population)

0

1

12

0

15

Hazard ratio (2-sided 95% CI) 0.45 (0.35, 0.59) Log-rank P-value=<0.001 (2-sided) Data cutoff: September 7, 2012

POMALYST + low-dose dexamethasone (dex) High-dose dex

Progression-free survival

Progression-free survival (PFS) based on the assessment by the Independent Review Adjudication Committee (IRAC) review at the final PFS analysis.

107

302

Number of patients at risk:

0

POMALYST + low-dose dex

0.0

0.2

0.4

0.6

0.8

1.0

Kaplan-Meier median: POMALYST + low-dose dex=3.6 [3.0, 4.6] Kaplan-Meier median: high-dose dex=1.8 [1.6, 2.1] Events: POMALYST + low-dose dex=164/302; high-dose dex=103/153

See where survival may lead

POMALYST + low-dose dex is the only approved therapy that delivered a survival benefit in patients who failed lenalidomide and bortezomib

POMALYST is only available through a restricted distribution program, POMALYST REMS速.

POMALYST速 (pomalidomide) is a thalidomide analogue indicated, in combination with dexamethasone, for patients with multiple myeloma who have received at least two prior therapies including lenalidomide and a proteasome inhibitor and have demonstrated disease progression on or within 60 days of completion of the last therapy.

Proportion of patients


Hazard ratio 0.45 (95% CI: 0.35, 0.59; P <0.001)

Patients >75 years of age started treatment with 20 mg dex using the same schedule.

Learn more about the survival benefit of POMALYST at www.mmperspective.com

a

Study Design: Phase 3 multicenter, randomized, open-label study, where POMALYST + low-dose dex was compared with high-dose dex in patients with relapsed and refractory multiple myeloma, who had received at least 2 prior treatment regimens, including lenalidomide and bortezomib, and demonstrated disease progression on or within 60 days of the last therapy (N=455). Patients with creatinine clearance ≥45 mL/min qualified for the trial. Patients in the POMALYST + low-dose dex arm (n=302) were administered 4 mg POMALYST orally on Days 1-21 of each 28-day cycle. Low-dose dex (40 mga) was administered once per day on Days 1, 8, 15, and 22 of a 28-day cycle. For the high-dose dex arm (n=153), dex (40 mga) was administered once per day on Days 1-4, 9-12, and 17-20 of a 28-day cycle. Treatment continued until patients had disease progression.

VENOUS AND ARTERIAL THROMBOEMBOLISM • Deep venous thrombosis (DVT), pulmonary embolism (PE), myocardial infarction, and stroke occur in patients with multiple myeloma treated with POMALYST. Antithrombotic prophylaxis is recommended.

POMALYST is available only through a restricted program called POMALYST REMS®.

Please see brief summary of full Prescribing Information, including Boxed WARNINGS, and Important Safety Information on the following pages.

• POMALYST can cause fetal harm and is contraindicated in females who are pregnant. If POMALYST is used during pregnancy or if the patient becomes pregnant while taking this drug, the patient should be apprised of the potential hazard to a fetus

CONTRAINDICATIONS: Pregnancy

• For females of reproductive potential: Exclude pregnancy before start of treatment. Prevent pregnancy during treatment by the use of 2 reliable methods of contraception.

EMBRYO-FETAL TOXICITY • POMALYST is contraindicated in pregnancy. POMALYST is a thalidomide analogue. Thalidomide is a known human teratogen that causes severe life-threatening birth defects.

See full prescribing information for complete boxed warning

WARNING: EMBRYO-FETAL TOXICITY and VENOUS AND ARTERIAL THROMBOEMBOLISM

Median OS for POMALYST + low-dose dex was 12.4 months (95% CI: 10.4, 15.3) vs 8.0 months (95% CI: 6.9, 9.0) for high-dose dex

Hazard ratio 0.70 (95% CI: 0.54, 0.92; P =0.009)

The difference in OS between the POMALYST + low-dose dex vs high-dose dex was statistically significant, with a 30% reduced risk of death

POMALYST + low-dose dex significantly prolonged overall survival (OS)

reduced risk of progression or death

55%

POMALYST + low-dose dex delivered significantly longer progression-free survival vs high-dose dex


POMALYST® (pomalidomide) is a thalidomide analogue indicated, in combination with dexamethasone, for patients with multiple myeloma who have received at least two prior therapies including lenalidomide and a proteasome inhibitor and have demonstrated disease progression on or within 60 days of completion of the last therapy.

Important Safety Information WARNING: EMBRYO-FETAL TOXICITY and VENOUS AND ARTERIAL THROMBOEMBOLISM Embryo-Fetal Toxicity • POMALYST is contraindicated in pregnancy. POMALYST is a thalidomide analogue. Thalidomide is a known human teratogen that causes severe birth defects or embryo-fetal death. In females of reproductive potential, obtain 2 negative pregnancy tests before starting POMALYST treatment. • Females of reproductive potential must use 2 forms of contraception or continuously abstain from heterosexual sex during and for 4 weeks after stopping POMALYST treatment.

POMALYST is only available through a restricted distribution program called POMALYST REMS®.

CONTRAINDICATIONS: Pregnancy • POMALYST can cause fetal harm and is contraindicated in females who are pregnant. If POMALYST is used during pregnancy or if the patient becomes pregnant while taking this drug, the patient should be apprised of the potential hazard to a fetus WARNINGS AND PRECAUTIONS Embryo-Fetal Toxicity • Females of Reproductive Potential: Must avoid pregnancy while taking POMALYST and for at least 4 weeks after completing therapy. Must commit either to abstain continuously from heterosexual sexual intercourse or to use 2 methods of reliable birth control, beginning 4 weeks prior to initiating treatment with POMALYST, during therapy, during dose interruptions, and continuing for 4 weeks following discontinuation of POMALYST therapy. Must obtain 2 negative pregnancy tests prior to initiating therapy • Males: Pomalidomide is present in the semen of patients receiving the drug. Males must always use a latex or synthetic condom during any sexual contact with females of reproductive potential while taking POMALYST and for up to 28 days after discontinuing POMALYST, even if they have undergone a successful vasectomy. Males must not donate sperm • Blood Donation: Patients must not donate blood during treatment with POMALYST and for 1 month following discontinuation of POMALYST therapy because the blood might be given to a pregnant female patient whose fetus must not be exposed to POMALYST POMALYST REMS® Program Because of the embryo-fetal risk, POMALYST is available only through a restricted program under a Risk Evaluation and Mitigation Strategy (REMS) called “POMALYST REMS®.” Prescribers and pharmacies must be certified with the program; patients must sign an agreement form and

comply with the requirements. Further information about the POMALYST REMS® program is available at www.CelgeneRiskManagement.com or by telephone at 1-888-423-5436. Venous and Arterial Thromboembolism: Venous thromboembolic events (DVT and PE) and arterial thromboembolic events (ATE) (myocardial infarction and stroke) have been observed in patients treated with POMALYST. In Trial 2, where anticoagulant therapies were mandated, thromboembolic events occurred in 8.0% of patients treated with POMALYST and low-dose dexamethasone (Low-dose Dex) vs 3.3% treated with high-dose dexamethasone. Venous thromboembolic events (VTE) occurred in 4.7% of patients treated with POMALYST and Low-dose Dex vs 1.3% treated with highdose dexamethasone. Arterial thromboembolic events include terms for arterial thromboembolic events, ischemic cerebrovascular conditions, and ischemic heart disease. Arterial thromboembolic events occurred in 3.0% of patients treated with POMALYST and Low-dose Dex vs 1.3% treated with high-dose dexamethasone. Patients with known risk factors, including prior thrombosis, may be at greater risk, and actions should be taken to try to minimize all modifiable factors (e.g., hyperlipidemia, hypertension, smoking). Hematologic Toxicity: In Trials 1 and 2 in patients who received POMALYST + Low-dose Dex, neutropenia (46%) was the most frequently reported Grade 3/4 adverse reaction, followed by anemia and thrombocytopenia. Monitor patients for hematologic toxicities, especially neutropenia. Monitor complete blood counts weekly for the first 8 weeks and monthly thereafter. Patients may require dose interruption and/or modification. Hepatotoxicity: Hepatic failure, including fatal cases, has occurred in patients treated with POMALYST. Elevated levels of alanine aminotransferase and bilirubin have also been observed in patients treated with POMALYST. Monitor

Venous and Arterial Thromboembolism • Deep venous thrombosis (DVT), pulmonary embolism (PE), myocardial infarction, and stroke occur in patients with multiple myeloma treated with POMALYST. Prophylactic antithrombotic measures were employed in clinical trials. Thromboprophylaxis is recommended, and the choice of regimen should be based on assessment of the patient’s underlying risk factors.


WARNINGS AND PRECAUTIONS (continued) liver function tests monthly. Stop POMALYST upon elevation of liver enzymes. After return to baseline values, treatment at a lower dose may be considered. Hypersensitivity Reactions: Angioedema and severe dermatologic reactions have been reported. Discontinue POMALYST for angioedema, skin exfoliation, bullae, or any other severe dermatologic reactions, and do not resume therapy. Dizziness and Confusional State: In Trials 1 and 2 in patients who received POMALYST + Low-dose Dex, 14% experienced dizziness and 7% a confusional state; 1% of patients experienced Grade 3 or 4 dizziness and 3% experienced a Grade 3 or 4 confusional state. Instruct patients to avoid situations where dizziness or confusional state may be a problem and not to take other medications that may cause dizziness or confusional state without adequate medical advice. Neuropathy: In Trials 1 and 2, patients who received POMALYST + Low-dose Dex experienced neuropathy (18%) and peripheral neuropathy (~12%). In Trial 2, 2% of patients experienced Grade 3 neuropathy. Risk of Second Primary Malignancies: Cases of acute myelogenous leukemia have been reported in patients receiving POMALYST as an investigational therapy outside of multiple myeloma. Tumor Lysis Syndrome: Tumor lysis syndrome (TLS) may occur in patients treated with POMALYST. Patients at risk are those with high tumor burden prior to treatment. These patients should be monitored closely and appropriate precautions taken.

Avoid the use of strong CYP1A2 inhibitors. If medically necessary to co-administer strong inhibitors of CYP1A2 in the presence of strong inhibitors of CYP3A4 and P-gp, reduce POMALYST dose by 50%. Cigarette smoking may reduce pomalidomide exposure due to CYP1A2 induction. Patients should be advised that smoking may reduce the efficacy of pomalidomide. USE IN SPECIFIC POPULATIONS Pregnancy: If pregnancy does occur during treatment, immediately discontinue the drug and refer patient to an obstetrician/gynecologist experienced in reproductive toxicity for further evaluation and counseling. Report any suspected fetal exposure to POMALYST to the FDA via the MedWatch program at 1-800-332-1088 and also to Celgene Corporation at 1-888-423-5436. Nursing Mothers: It is not known if pomalidomide is excreted in human milk. Pomalidomide was excreted in the milk of lactating rats. Because many drugs are excreted in human milk and because of the potential for adverse reactions in nursing infants from POMALYST, a decision should be made whether to discontinue nursing or to discontinue the drug, taking into account the importance of the drug to the mother. Pediatric Use: Safety and effectiveness of POMALYST in patients under the age of 18 have not been established. Geriatric Use: No dosage adjustment is required for POMALYST based on age. Patients >65 years of age were more likely than patients ≤65 years of age to experience pneumonia. Renal and Hepatic Impairment: Pomalidomide is metabolized in the liver. Pomalidomide and its metabolites are primarily excreted by the kidneys. The influence of renal and hepatic impairment on the safety, efficacy, and pharmacokinetics of pomalidomide has not been evaluated. Avoid POMALYST in patients with a serum creatinine >3.0 mg/dL. Avoid POMALYST in patients with serum bilirubin >2.0 mg/dL and AST/ALT >3.0 x ULN.

ADVERSE REACTIONS Nearly all patients treated with POMALYST + Low-dose Dex experienced at least one adverse reaction (99%). In Trial 2, the most common adverse reactions included neutropenia (51.3%), fatigue and asthenia (46.7%), upper respiratory tract infection (31%), thrombocytopenia (29.7%), pyrexia (26.7%), dyspnea (25.3%), diarrhea (22%), constipation (21.7%), Please see brief summary of full Prescribing back pain (19.7%), cough (20%), pneumonia (19.3%), Information, including Boxed WARNINGS, edema peripheral (17.3%), peripheral neuropathy (17.3%), on following pages. bone pain (18%), nausea (15%), and muscle spasms (15.3%). Grade 3 or 4 adverse reactions included neutropenia (48.3%), thrombocytopenia (22%), and pneumonia (15.7%). DRUG INTERACTIONS Pomalidomide is primarily metabolized by CYP1A2 and CYP3A. Pomalidomide is also a substrate for P-glycoprotein (P-gp). POMALYST is only available through a restricted distribution program, POMALYST REMS®.

POMALYST® and POMALYST REMS® are registered trademarks of Celgene Corporation. © 2015 Celgene Corporation 05/15 US-POM150008a


The ASCO Post  |   JULY 25, 2015

PAGE 32

Announcements

New Appointments Announced at Taussig Cancer Institute

T

he following new appointments were recently announced at Cleveland Clinic’s Taussig Cancer Institute.

James Stevenson, MD Dr. Stevenson has been appointed

Vice Chairman of the Department of Hematology and Medical Oncology. As Vice Chairman, Dr. Stevenson will assist the Chairman and Department Administrator in operations, which include managing and coordinating

disease-specific programs and duties of advanced practice providers within the department. He will also focus on the clinical, academic, and administrative development of junior faculty members.

James Stevenson, MD

T:7”

POMALYST® (pomalidomide) capsules, for oral use

2.2 Dose Adjustments for Toxicities

The following is a Brief Summary; refer to full Prescribing Information for complete product information.

Table 1: Dose Modification Instructions for POMALYST for Hematologic Toxicities

WARNING: EMBRYO-FETAL TOXICITY and VENOUS AND ARTERIAL THROMBOEMBOLISM Embryo-Fetal Toxicity • POMALYST is contraindicated in pregnancy. POMALYST is a thalidomide analogue. Thalidomide is a known human teratogen that causes severe birth defects or embryo-fetal death. In females of reproductive potential, obtain 2 negative pregnancy tests before starting POMALYST treatment. • Females of reproductive potential must use 2 forms of contraception or continuously abstain from heterosexual sex during and for 4 weeks after stopping POMALYST treatment [see Contraindications (4), Warnings and Precautions (5.1), and Use in Specific Populations (8.1, 8.6)]. POMALYST is only available through a restricted distribution program called POMALYST REMS® [see Warnings and Precautions (5.2)]. Venous and Arterial Thromboembolism • Deep venous thrombosis (DVT), pulmonary embolism (PE), myocardial infarction, and stroke occur in patients with multiple myeloma treated with POMALYST. Prophylactic antithrombotic measures were employed in clinical trials. Thromboprophylaxis is recommended, and the choice of regimen should be based on assessment of the patient’s underlying risk factors [see Warnings and Precautions (5.3)].

2 DOSAGE AND ADMINISTRATION 2.1 Multiple Myeloma Females of reproductive potential must have negative pregnancy testing and use contraception methods before initiating POMALYST [see Warnings and Precautions (5.1) and Use in Specific Populations (8.6)]. The recommended starting dose of POMALYST is 4 mg once daily orally on Days 1-21 of repeated 28-day cycles until disease progression. POMALYST should be given in combination with dexamethasone. POMALYST may be taken with water. Inform patients not to break, chew, or open the capsules. POMALYST should be taken without food (at least 2 hours before or 2 hours after a meal).

Dose Modification • Interrupt POMALYST treatment, follow CBC weekly • Resume POMALYST treatment at 3 mg daily

• For each subsequent • Interrupt POMALYST drop <500 per mcL treatment • Return to more than or • Resume POMALYST equal to 500 per mcL treatment at 1 mg less than the previous dose Thrombocytopenia • Platelets <25,000 per mcL • Platelets return to >50,000 per mcL

• Interrupt POMALYST treatment, follow CBC weekly • Resume POMALYST treatment at 3 mg daily

• For each subsequent • Interrupt POMALYST drop <25,000 per mcL treatment • Resume POMALYST • Return to more than or equal to 50,000 per treatment at 1 mg less than previous dose mcL ANC, absolute neutrophil count To initiate a new cycle of POMALYST, the neutrophil count must be at least 500 per mcL and the platelet count must be at least 50,000 per mcL. If toxicities occur after dose reductions to 1 mg, then discontinue POMALYST. Permanently discontinue POMALYST for angioedema, skin exfoliation, bullae, or any other severe dermatologic reaction [see Warnings and Precautions (5.6)]. For other Grade 3 or 4 toxicities, hold treatment and restart treatment at 1 mg less than the previous dose when toxicity has resolved to less than or equal to Grade 2 at the physician’s discretion. 2.3 Dose Adjustment for Strong CYP1A2 Inhibitors in the Presence of Strong CYP3A4 and P-gp Inhibitors Avoid co-administration of strong inhibitors of CYP1A2. If necessary to co-administer strong inhibitors of CYP1A2 in the presence of strong inhibitors of CYP3A4 and P-gp, reduce POMALYST dose by 50%. No clinical efficacy or safety data exist [see Drug Interactions (7.1))]. 4 CONTRAINDICATIONS Pregnancy POMALYST can cause fetal harm when administered to a pregnant female [see Warnings and Precautions (5.1) and Use in Specific Populations (8.1)]. POMALYST is contraindicated in females who are pregnant. Pomalidomide is a thalidomide analogue and is teratogenic in both rats and rabbits when administered during the period of organogenesis. If this drug is used during pregnancy or if the patient becomes pregnant while taking this drug, the patient should be apprised of the potential hazard to a fetus. 5 WARNINGS AND PRECAUTIONS 5.1 Embryo-Fetal Toxicity POMALYST is a thalidomide analogue and is contraindicated for use during pregnancy. Thalidomide is a known human teratogen that causes severe birth defects or embryo-fetal death [see Use in Specific Populations (8.1)]. POMALYST is only available through the POMALYST REMS program [see Warnings and Precautions (5.2)].

5.2 POMALYST REMS Program Because of the embryo-fetal risk [see Warnings and Precautions (5.1)], POMALYST is available only through a restricted program under a Risk Evaluation and Mitigation Strategy (REMS) called “POMALYST REMS.” Required components of the POMALYST REMS program include the following: • Prescribers must be certified with the POMALYST REMS program by enrolling and complying with the REMS requirements. • Patients must sign a Patient-Physician Agreement Form and comply with the REMS requirements. In particular, female patients of reproductive potential who are not pregnant must comply with the pregnancy testing and contraception requirements [see Use in Specific Populations (8.6)] and males must comply with contraception requirements [see Use in Specific Populations (8.6)]. • Pharmacies must be certified with the POMALYST REMS program, must only dispense to patients who are authorized to receive POMALYST, and comply with REMS requirements. Further information about the POMALYST REMS program is available at www.CelgeneRiskManagement.com or by telephone at 1-888-423-5436. 5.3 Venous and Arterial Thromboembolism Venous thromboembolic events (deep venous thrombosis and pulmonary embolism) and arterial thromboembolic events (myocardial infarction and stroke) have been observed in patients treated with POMALYST. In Trial 2, where anticoagulant therapies were mandated, thromboembolic events occurred in 8.0% of patients treated with POMALYST and low dose-dexamethasone (Low-dose Dex), and 3.3% of patients treated with high-dose dexamethasone. Venous thromboembolic events (VTE) occurred in 4.7% of patients treated with POMALYST and Low-dose Dex, and 1.3% of patients treated with high-dose dexamethasone. Arterial thromboembolic events include terms for arterial thromboembolic events, ischemic cerebrovascular conditions, and ischemic heart disease. Arterial thromboembolic events occurred in 3.0% of patients treated with POMALYST and Low-dose Dex, and 1.3% of patients treated with high-dose dexamethasone.

T:10”

1 INDICATIONS AND USAGE 1.1 Multiple Myeloma POMALYST, in combination with dexamethasone, is indicated for patients with multiple myeloma who have received at least two prior therapies including lenalidomide and a proteasome inhibitor and have demonstrated disease progression on or within 60 days of completion of the last therapy.

Toxicity Neutropenia • ANC <500 per mcL or febrile neutropenia (fever more than or equal to 38.5°C and ANC <1,000 per mcL) • ANC return to more than or equal to 500 per mcL

Females of Reproductive Potential Females of reproductive potential must avoid pregnancy while taking POMALYST and for at least 4 weeks after completing therapy. Females must commit either to abstain continuously from heterosexual sexual intercourse or to use 2 methods of reliable birth control, beginning 4 weeks prior to initiating treatment with POMALYST, during therapy, during dose interruptions, and continuing for 4 weeks following discontinuation of POMALYST therapy. Two negative pregnancy tests must be obtained prior to initiating therapy. The first test should be performed within 10-14 days and the second test within 24 hours prior to prescribing POMALYST therapy and then weekly during the first month, then monthly thereafter in women with regular menstrual cycles, or every 2 weeks in women with irregular menstrual cycles [see Use in Specific Populations (8.6)]. Males Pomalidomide is present in the semen of patients receiving the drug. Therefore, males must always use a latex or synthetic condom during any sexual contact with females of reproductive potential while taking POMALYST and for up to 28 days after discontinuing POMALYST, even if they have undergone a successful vasectomy. Male patients taking POMALYST must not donate sperm [see Use in Specific Populations (8.6)]. Blood Donation Patients must not donate blood during treatment with POMALYST and for 1 month following discontinuation of the drug because the blood might be given to a pregnant female patient whose fetus must not be exposed to POMALYST.


ASCOPost.com  |   JULY 25, 2015

PAGE 33

Announcements

Dr. Stevenson joined Cleveland Clinic’s hematology and medical oncology staff in 2012.

Timothy Gilligan, MD Dr. Gilligan has been appointed Vice Chairman of Education for the Institute. In this role, he will oversee and

Timothy Gilligan, MD

support the Institute’s education activities, which include fellowship and residency programs, medical student and resident rotations, continuing education programs, and faculty development activities. He will assess the education portfolio and determine opportunities for growth.

Currently, Dr. Gilligan is a medical oncologist at Cleveland Clinic and Associate Professor of Medicine at the Cleveland Clinic Lerner College of Medicine. He specializes in genitourinary cancers in general and has appointments in the departments of urology and bioethics. n

T:7”

Patients with known risk factors, including prior thrombosis, may be at greater risk, and actions should be taken to try to minimize all modifiable factors (e.g., hyperlipidemia, hypertension, smoking). Thromboprophylaxis is recommended, and the choice of regimen should be based on assessment of the patient’s underlying risk factors. 5.4 Hematologic Toxicity In trials 1 and 2 in patients who received POMALYST + Low-dose Dex, neutropenia was the most frequently reported Grade 3/4 adverse reaction, followed by anemia and thrombocytopenia. Neutropenia of any grade was reported in 51% of patients in both trials. The rate of Grade 3/4 neutropenia was 46%. The rate of febrile neutropenia was 8%. Monitor patients for hematologic toxicities, especially neutropenia. Monitor complete blood counts weekly for the first 8 weeks and monthly thereafter. Patients may require dose interruption and/or modification [see Dosage and Administration (2.2)]. 5.5 Hepatotoxicity Hepatic failure, including fatal cases, has occurred in patients treated with POMALYST. Elevated levels of alanine aminotransferase and bilirubin have also been observed in patients treated with POMALYST. Monitor liver function tests monthly. Stop POMALYST upon elevation of liver enzymes and evaluate. After return to baseline values, treatment at a lower dose may be considered. 5.6 Hypersensitivity Reactions Angioedema and severe dermatologic reactions have been reported. Discontinue POMALYST for angioedema, skin exfoliation, bullae, or any other severe dermatologic reactions, and do not resume therapy [see Dosage and Administration (2.2)].

5.8 Neuropathy In trials 1 and 2 in patients who received POMALYST + Low-dose Dex, 18% of patients experienced neuropathy, with approximately 12% of the patients experiencing peripheral neuropathy. Two percent of patients experienced Grade 3 neuropathy in trial 2. There were no cases of Grade 4 neuropathy adverse reactions reported in either trial. 5.9 Risk of Second Primary Malignancies Cases of acute myelogenous leukemia have been reported in patients receiving POMALYST as an investigational therapy outside of multiple myeloma. 5.10 Tumor Lysis Syndrome Tumor lysis syndrome (TLS) may occur in patients treated with pomalidomide. Patients at risk for TLS are those with high tumor burden prior to treatment. These patients should be monitored closely and appropriate precautions taken. 6 ADVERSE REACTIONS The following adverse reactions are described in detail in other labeling sections: • Fetal Risk [see Boxed Warnings, Warnings and Precautions (5.1, 5.2)] • Venous and Arterial Thromboembolism [see Boxed Warnings, Warnings and Precautions (5.3)] • Hematologic Toxicity [see Warnings and Precautions (5.4)] • Hepatotoxicity [see Warnings and Precautions (5.5)] • Hypersensitivity Reactions [see Warnings and Precautions (5.6)] • Dizziness and Confusional State [see Warnings and Precautions (5.7)] • Neuropathy [see Warnings and Precautions (5.8)]

In Trial 1, Grade 3/4 at least one adverse reaction reported in 92% of patients treated with POMALYSTa alone (N=107) and 91% with POMALYST + Low-dose Dex (N=112).* Grade 3/4 Adverse Reactions ≥ 5% in either arm, respectively, included: Blood and lymphatic system disorders: Neutropeniab (48%, 41%), Anemiab (23%, 21%), Thrombocytopeniab (22%, 19%), Leukopenia (7%, 10%), Febrile neutropeniab (6%, 3%), Lymphopenia (2%, 7%); General disorders and administration site conditions: Fatigue and astheniab (12%, 17%), Edema peripheral (0%, 0%), Pyrexiab (<5%, <5%), Chills (0%, 0%); Gastrointestinal disorders: Nauseab (<5%, <5%), Constipationb (<5%, <5%), Diarrhea (<5%, <5%), Vomitingb (<5%, 0%); Musculoskeletal and connective tissue disorders: Back painb (14%, 10%), Musculoskeletal chest pain (<5%, 0%), Muscle spasms (<5%, <5%), Arthralgia (<5%, <5%), Muscular weakness (6%, 4%), Bone pain (<5%, <5%), Musculoskeletal pain (<5%, <5%), Pain in extremity (0%, <5%); Infections and infestations: Upper respiratory tract infection (<5%, <5%), Pneumoniab (20%, 29%), Urinary tract infectionb (2%, 9%), Sepsisb (6%, 5%); Metabolism and nutrition disorders: Decreased appetite (<5%, 0%), Hypercalcemiab (10%, 1%), Hypokalemia (<5%, <5%), Hyperglycemia (<5%, <5%), Hyponatremia (<5%, <5%) Dehydrationb (5%, 5%), Hypocalcemia (0%, <5%); Respiratory, thoracic and mediastinal disorders: Dyspneab (8%, 13%), Cough (0%, 0%), Epistaxis (<5%, 0%), Productive cough (0%, 0%), Oropharyngeal pain (0%, 0%); Nervous system disorders: Dizziness (<5%, <5%), Peripheral neuropathy (0%, 0%), Headache (0%, <5%), Tremor (0%, 0%); Skin and subcutaneous tissue disorders: Rash (0%, <5%), Pruritus (0%, 0%), Dry skin (0%, 0%), Hyperhidrosis (0%, 0%), Night sweats (0%, 0%); Investigations: Blood creatinine increasedb (6%, 3%), Weight decreased (0%, 0%), Weight increased (0%, 0%); Psychiatric disorders: Anxiety (0%, 0%), Confusional stateb (6%, 3%), Insomnia (0%, 0%); Renal and urinary disorders: Renal failureb (8%, 7%). * Regardless of attribution of relatedness to POMALYST. a POMALYST alone arm includes all patients randomized to the POMALYST alone arm who took study drug; 61 of the 107 patients had dexamethasone added during the treatment period. b Serious adverse reactions were reported in at least 2 patients in any POMALYST treatment arm. Data cutoff: 01 March 2013 In Trial 2 of 450 patients who received POMALYST + Low-dose Dex (N=300) or High-dose-Dex (N=150), at least one adverse reaction was reported in 99% of patients. All Adverse Reactions ≥5% in POMALYST + Lowdose Dex arm, and at least 2% point higher than the High-dose-Dex arm included: Blood and lymphatic system disorders: Neutropeniab (51%, 21%), Thrombocytopenia (30%, 29%)a, Leukopenia (13%, 5%), Febrile neutropeniab (9%, 0%); General disorders and administration site conditions: Fatigue and asthenia (47%, 43%), Pyrexiab (27%, 23%), Edema peripheral (17%, 11%), Pain (4%, 2%)a; Infections and infestations: Upper respiratory tract infectionb (31%, 13%), Pneumoniab (19%, 13%), Neutropenic sepsisb (1%, 0%)a; Gastrointestinal disorders: Diarrhea (22%, 19%), Constipation (22%, 15%), Nausea (15%, 11%), Vomiting (8%, 4%); Musculoskeletal and connective tissue disorders: Back painb (20%, 16%), Bone Painb (18%, 14%), Muscle spasms (15%, 7%), Arthralgia (9%, 5%), Pain in extremity (7%, 6%)a; Respiratory, thoracic and mediastinal disorders: Dyspneab (25%, 17%), Cough (20%, 10%), Chronic obstructive pulmonary diseaseb (2%, 0%)a; Nervous system disorders: Peripheral neuropathy (17%, 12%), Dizziness (12%, 9%), Headache (8%, 5%), Tremor (6%, 1%), Depressed level of consciousness (2%, 0%)a; Metabolism and nutrition disorders: Decreased appetite (13%, 8%), Hypokalemia (9%, 8%)a, Hypocalcemia (4%, 6%)a;

T:10”

5.7 Dizziness and Confusional State In trials 1 and 2 in patients who received POMALYST + Low-dose Dex, 14% of patients experienced dizziness and 7% of patients experienced a confusional state; 1% of patients experienced Grade 3 or 4 dizziness, and 3% of patients experienced Grade 3 or 4 confusional state. Instruct patients to avoid situations where dizziness or confusional state may be a problem and not to take other medications that may cause dizziness or confusional state without adequate medical advice.

• Risk of Second Primary Malignancies [see Warnings and Precautions (5.9)] • Tumor Lysis Syndrome [see Warnings and Precautions (5.10)] 6.1 Clinical Trials Experience Multiple Myeloma Because clinical trials are conducted under widely varying conditions, adverse reaction rates observed in the clinical trials of a drug cannot be directly compared with rates in the clinical trials of another drug and may not reflect the rates observed in practice. In Trial 1, data were evaluated from 219 patients (safety population) who received treatment with POMALYST + Low-dose Dex (112 patients) or POMALYST alone (107 patients). Median number of treatment cycles was 5. Sixty-seven percent of patients in the study had a dose interruption of either drug due to adverse reactions. Forty-two percent of patients in the study had a dose reduction of either drug due to adverse reactions. The discontinuation rate due to adverse reactions was 11%. In Trial 2, data were evaluated from 450 patients (safety population) who received treatment with POMALYST + Low-dose Dex (300 patients) or Highdose Dexamethasone (High-dose Dex) (150 patients). The median number of treatment cycles for the POMALYST + Low-dose Dex arm was 5. In the POMALYST + Low-dose Dex arm, 67% of patients had a dose interruption of POMALYST, the median time to the first dose interruption of POMALYST was 4.1 weeks. Twenty-seven percent of patients had a dose reduction of POMALYST, the median time to the first dose reduction of POMALYST was 4.5 weeks. Eight percent of patients discontinued POMALYST due to adverse reactions. Tables 2 and 3 summarize the adverse reactions reported in Trials 1 and 2, respectively. In Trial 1 of 219 patients who received POMALYST alonea (N=107) or POMALYST + Low-dose Dex (N=112), all patients had at least one adverse reaction.* Adverse reactions ≥10% in either arm, respectively, included: Blood and lymphatic system disorders: Neutropeniab (53%, 49%), Anemiab (38%, 42%), Thrombocytopeniab (26%, 23%), Leukopenia (13%, 20%), Febrile neutropeniab (<10%, <10%), Lymphopenia (4%, 15%); General disorders and administration site conditions: Fatigue and astheniab (58%, 63%), Edema peripheral (25%, 17%), Pyrexiab (23%, 32%), Chills (10%, 13%); Gastrointestinal disorders: Nauseab (36%, 24%), Constipationb (36%, 37%), Diarrhea (35%, 36%), Vomitingb (14%, 14%); Musculoskeletal and connective tissue disorders: Back painb (35%, 32%), Musculoskeletal chest pain (23%, 20%), Muscle spasms (22%, 20%), Arthralgia (17%, 15%), Muscular weakness (14%, 13%), Bone pain (12%, 7%), Musculoskeletal pain (12%, 17%), Pain in extremity (8%, 14%); Infections and infestations: Upper respiratory tract infection (37%, 29%), Pneumoniab (28%, 34%), Urinary tract infectionb (10%, 17%), Sepsisb (<10%, <10%); Metabolism and nutrition disorders: Decreased appetite (23%, 19%), Hypercalcemiab (22%, 12%), Hypokalemia (12%, 12%), Hyperglycemia (11%, 15%), Hyponatremia (11%, 13%), Dehydrationb (<10%, <10%), Hypocalcemia (6%, 12%); Respiratory, thoracic and mediastinal disorders: Dyspneab (36%, 45%), Cough (17%, 22%), Epistaxis (17%, 11%), Productive cough (9%, 13%), Oropharyngeal pain (6%, 11%); Nervous system disorders: Dizziness (22%, 18%), Peripheral neuropathy (22%, 18%), Headache (15%, 13%), Tremor (10%, 13%); Skin and subcutaneous tissue disorders: Rash (21%, 16%), Pruritus (15%, 9%), Dry skin (9%, 11%), Hyperhidrosis (8%, 16%), Night sweats (5%, 13%); Investigations: Blood creatinine increasedb (19%, 10%), Weight decreased (15%, 9%), Weight increased (1%, 11%); Psychiatric disorders: Anxiety (13%, 7%), Confusional stateb (12%, 13%), Insomnia (7%, 16%); Renal and urinary disorders: Renal failureb (15%, 10%).


The ASCO Post  |   JULY 25, 2015

PAGE 34

Announcements

Cyrus Ghajar, PhD, Receives $4.1 Million From Department of Defense

C

yrus Ghajar, PhD, a metastatic breast cancer researcher at Fred Hutchinson Cancer Research Center, has received a $4.1 million Department of Defense Breast Cancer Research Program (BCRP) “Era of Hope”

Scholar Award. The Department of Defense’s BCRP is the second biggest funder of breast cancer research in the United States. Its Era of Hope award encourages high-impact, collaborative research, particularly

among young, innovative researchers. Dr. Ghajar is the Director of the Laboratory for the Study of Metastatic Microenvironments (LSM2), which is housed within the Translational Research Program in Fred Hutchinson’s Public Health

Sciences Division. The LSM2 studies how microenvironments within distant tissues influence dormancy, drug resistance, and the reemergence of disseminated tumor cells. He will use the funds to research ways to prevent breast cancer

T:7”

Other Adverse Reactions Other adverse reactions of POMALYST in patients with multiple myeloma, not described above, and considered important: Cardiac disorders: Myocardial infarction, Atrial fibrillation, Angina pectoris, Cardiac failure congestive Ear and labyrinth disorders: Vertigo Gastrointestinal disorders: Abdominal pain General disorders and administration site conditions: General physical health deterioration, Non-cardiac chest pain, Multi-organ failure Hepatobiliary disorders: Hyperbilirubinemia Infections and infestations: Pneumocystis jiroveci pneumonia, Respiratory syncytial virus infection, Neutropenic sepsis, Bacteremia, Pneumonia respiratory syncytial viral, Cellulitis, Urosepsis, Septic shock, Clostridium difficile colitis, Pneumonia streptococcal, Lobar pneumonia, Viral infection, Lung infection Investigations: Alanine aminotransferase increased, Hemoglobin decreased

Injury, poisoning and procedural complications: Fall, Compression fracture, Spinal compression fracture Metabolism and nutritional disorders: Hyperkalemia, Failure to thrive Nervous System disorders: Depressed level of consciousness, Syncope Psychiatric disorders: Mental status change Renal and urinary disorders: Urinary retention, Hyponatremia Reproductive system and breast disorders: Pelvic pain Respiratory, thoracic, and mediastinal disorders: Interstitial lung disease, Pulmonary embolism, Respiratory failure, Bronchospasm Vascular disorders: Hypotension 6.2 Postmarketing Experience The following adverse drug reactions have been identified from the worldwide postmarketing experience with POMALYST: Pancytopenia, tumor lysis syndrome, allergic reactions (e.g., angioedema, urticaria), elevated liver enzymes, hepatic failure (including fatal cases). Because these reactions are reported voluntarily from a population of uncertain size, it is not always possible to reliably estimate their frequency or establish a causal relationship to drug exposure. 7 DRUG INTERACTIONS Pomalidomide is primarily metabolized by CYP1A2 and CYP3A. Pomalidomide is also a substrate for P-glycoprotein (P-gp). 7.1 Drugs That May Increase Pomalidomide Plasma Concentrations CYP1A2 inhibitors: Pomalidomide exposure is increased when POMALYST is co-administered with a strong CYP1A2 inhibitor (fluvoxamine) in the presence of a strong CYP3A4/5 and P-gp inhibitor (ketoconazole). Ketoconazole in the absence of a CYP1A2 inhibitor does not increase pomalidomide exposure. Avoid co-administration of strong CYP1A2 inhibitors (e.g. ciprofloxacin and fluvoxamine) [see Dosage and Administration (2.3)]. If it is medically necessary to co-administer strong inhibitors of CYP1A2 in the presence of strong inhibitors of CYP3A4 and P-gp, POMALYST dose should be reduced by 50%. The effect of a CYP1A2 inhibitor in the absence of a co-administered CYP3A4 and P-gp inhibitor has not been studied. Monitor for toxicities if CYP1A2 inhibitors are to be co-administered in the absence of a co-administered CYP3A4 and P-gp inhibitor, and reduce dose if needed. 7.2 Drugs That May Decrease Pomalidomide Plasma Concentrations Smoking: Cigarette smoking may reduce pomalidomide exposure due to CYP1A2 induction. Patients should be advised that smoking may reduce the efficacy of pomalidomide. CYP1A2 inducers: Co-administration of POMALYST with drugs that are CYP1A2 inducers has not been studied and may reduce pomalidomide exposure. 8 USE IN SPECIFIC POPULATIONS 8.1 Pregnancy Pregnancy Category X [see Boxed Warnings and Contraindications (4)] Risk Summary POMALYST can cause embryo-fetal harm when administered to a pregnant female and is contraindicated during pregnancy. POMALYST is a thalidomide analogue. Thalidomide is a human teratogen, inducing a high frequency of severe and life-threatening birth defects such as amelia (absence of limbs), phocomelia (short limbs), hypoplasticity of the bones, absence of bones, external ear abnormalities (including anotia, micropinna, small or absent external auditory canals), facial palsy, eye abnormalities (anophthalmos, microphthalmos), and congenital heart defects. Alimentary tract, urinary tract, and genital malformations have also been documented, and mortality at or shortly after birth has been reported in about 40% of infants.

Pomalidomide was teratogenic in both rats and rabbits when administered during the period of organogenesis. If this drug is used during pregnancy or if the patient becomes pregnant while taking this drug, the patient should be apprised of the potential hazard to a fetus. If pregnancy does occur during treatment, immediately discontinue the drug. Under these conditions, refer patient to an obstetrician/ gynecologist experienced in reproductive toxicity for further evaluation and counseling. Report any suspected fetal exposure to POMALYST to the FDA via the MedWatch program at 1-800-FDA-1088 and also to Celgene Corporation at 1-888-423-5436. Animal Data Pomalidomide was teratogenic in both rats and rabbits in the embryo-fetal developmental studies when administered during the period of organogenesis. In rats, pomalidomide was administered orally to pregnant animals at doses of 25 to 1000 mg/kg/day. Malformations or absence of urinary bladder, absence of thyroid gland, and fusion and misalignment of lumbar and thoracic vertebral elements (vertebral, central, and/or neural arches) were observed at all dose levels. There was no maternal toxicity observed in this study. The lowest dose in rats resulted in an exposure (AUC) approximately 85-fold of the human exposure at the recommended dose of 4 mg/day. Other embryo-fetal toxicities included increased resorptions leading to decreased number of viable fetuses. In rabbits, pomalidomide was administered orally to pregnant animals at doses of 10 to 250 mg/kg/day. Increased cardiac malformations such as interventricular septal defect were seen at all doses with significant increases at 250 mg/kg/day. Additional malformations observed at 250 mg/kg/day included anomalies in limbs (flexed and/or rotated fore- and/ or hindlimbs, unattached or absent digit) and associated skeletal malformations (not ossified metacarpal, misaligned phalanx and metacarpal, absent digit, not ossified phalanx, and short not ossified or bent tibia), moderate dilation of the lateral ventricle in the brain, abnormal placement of the right subclavian artery, absent intermediate lobe in the lungs, low-set kidney, altered liver morphology, incompletely or not ossified pelvis, an increased average for supernumerary thoracic ribs, and a reduced average for ossified tarsals. No maternal toxicity was observed at the low dose (10 mg/kg/day) that resulted in cardiac anomalies in fetuses; this dose resulted in an exposure (AUC) approximately equal to that reported in humans at the recommended dose of 4 mg/day. Additional embryo-fetal toxicity included increased resorption. 8.3 Nursing Mothers It is not known if pomalidomide is excreted in human milk. Pomalidomide was excreted in the milk of lactating rats. Because many drugs are excreted in human milk and because of the potential for adverse reactions in nursing infants from POMALYST, a decision should be made whether to discontinue nursing or to discontinue the drug, taking into account the importance of the drug to the mother. 8.4 Pediatric Use Safety and effectiveness of POMALYST in patients below the age of 18 years have not been established. 8.5 Geriatric Use No dosage adjustment is required for POMALYST based on age. Of the total number of patients in clinical studies of POMALYST, 44% were aged older than 65 years, while 10% were aged older than 75 years. No overall differences in effectiveness were observed between these patients and younger patients. In these studies, patients older than 65 years were more likely than patients less than or equal to 65 years of age to experience pneumonia. 8.6 Females of Reproductive Potential and Males POMALYST can cause fetal harm when administered during pregnancy [see Use in Specific Populations (8.1)]. Females of reproductive potential must avoid pregnancy while taking POMALYST and for at least 4 weeks after completing therapy.

T:10”

Skin and subcutaneous tissue disorders: Rash (8%, 1%), Pruritus (7%, 3%), Hyperhidrosis (5%, 1%); Investigations: Neutrophil count decreased (5%, 1%), Platelet count decreased (3%, 2%)a, White blood cell count decreased (3%, 1%)a, Alanine aminotransferase increased (2%, 1%)a, Aspartate aminotransferase increased (1%, 1%)a, Lymphocyte count decreased (1%, 1%)a; Renal and urinary disorders: Renal failure (10%, 12%)a; Injury, poisoning and procedural complications: Femur fractureb (2%, 1%)a; Reproductive system and breast disorders: Pelvic pain (2%, 2%)a. In Trial 2, Grade 3/4 at least one adverse reaction was reported in 86% of patients treated with POMALYST + Low-dose Dex (N=300) and 85% with High-dose Dex (N=150). Grade 3/4 Adverse Reactions ≥1% in POMALYST + Low-dose Dex arm, and at least 1% point higher than the High-dose-Dex arm, respectively, included: Blood and lymphatic system disorders: Neutropeniab (48%, 16%), Thrombocytopenia (22%, 26%)a, Leukopenia (9%, 3%), Febrile neutropeniab (9%, 0%); General disorders and administration site conditions: Fatigue and asthenia (9%, 12%)a, Pyrexiab (3%, 5%)a, Edema peripheral (1%, 2%)a, Pain (2%, 1%); Infections and infestations: Upper respiratory tract infectionb (3%, 1%), Pneumoniab (16%, 10%), Neutropenic sepsisb (1%, 0%); Gastrointestinal disorders: Diarrhea (1%, 1%)a, Constipation (2%, 0%), Nausea (1%, 1%)a, Vomiting (1%, 0%); Musculoskeletal and connective tissue disorders: Back painb (5%, 4%), Bone painb (7%, 5%), Muscle spasms (0%, 1%)a, Arthralgia (1%, 1%)a, Pain in extremity (2%, 0%); Respiratory, thoracic and mediastinal disorders: Dyspneab (6%, 5%), Cough (1%, 1%)a, Chronic obstructive pulmonary diseaseb (1%, 0%); Nervous system disorders: Peripheral neuropathy (2%, 1%)a, Dizziness (1%, 1%)a, Headache (0%, 0%)a, Tremor (1%, 0%)a, Depressed level of consciousness (1%, 0%); Metabolism and nutrition disorders: Decreased appetite (1%, 1%)a, Hypokalemia (4%, 3%), Hypocalcemia (2%, 1%); Skin and subcutaneous tissue disorders: Rash (1%, 0%), Pruritus (0%, 0%)a, Hyperhidrosis (0%, 0%)a; Investigations: Neutrophil count decreased (5%, 1%), Platelet count decreased (3%, 1%), White blood cell count decreased (3%, 0%), Alanine aminotransferase increased (2%, 0%), Aspartate aminotransferase increased (1%, 0%), Lymphocyte count decreased (1%, 0%); Renal and urinary disorders: Renal failure (6%, 5%); Injury, poisoning and procedural complications: Femur fractureb (2%, 1%); Reproductive system and breast disorders: Pelvic pain (1%, 0%). a Percentage did not meet the criteria to be considered as an adverse reaction for POMALYST for that category of event (i.e., all adverse events or Grade 3 or 4 adverse events). b Serious adverse reactions were reported in at least 3 patients in the POM + Low-dose Dex arm, AND at least 1% higher than the High-dose-Dex arm percentage. Data cutoff: 01 March 2013


ASCOPost.com  |   JULY 25, 2015

PAGE 35

Announcements

metastasis by treating dormant disseminated tumor cells.

Dormant Tumor Cells as the Root of Recurrence Metastatic breast cancer claims 40,000 lives a year; it’s estimated that 30% of all breast cancer cases will be-

come metastatic. Interestingly, 20% of these cases will not emerge until a decade following therapy. “The hypothesis is that these cells— which have left the breast and are in other organs, basically sleeping—eventually wake up,” he said. “When women relapse 7 or 10 years after treatment,

these dormant cells are likely the root of recurrence.” For this work, Dr. Ghajar has teamed with other Fred Hutchinson researchers, as well as investigators at Harvard Medical School and at the University of Colorado, Denver. Dr. Ghajar also has involved two local

T:7”

8.7 Renal Impairment Pomalidomide and its metabolites are primarily excreted by the kidneys. The influence of renal impairment on the safety, efficacy, and pharmacokinetics of pomalidomide has not been evaluated. Patients with serum creatinine greater than 3.0 mg/dL were excluded in clinical studies. Avoid POMALYST in patients with a serum creatinine greater than 3.0 mg/dL. 8.8 Hepatic Impairment Pomalidomide is metabolized in the liver. The influence of hepatic impairment on the safety, efficacy, and pharmacokinetics of pomalidomide has not been evaluated. Patients with serum bilirubin greater than 2.0 mg/dL and AST/ALT greater than 3.0 x upper limit normal (ULN) were excluded in clinical studies. Avoid POMALYST in patients with serum bilirubin greater than 2.0 mg/dL and AST/ALT greater than 3.0 x ULN. 10 OVERDOSAGE No specific information is available on the treatment of overdose with pomalidomide, and it is unknown whether pomalidomide or its metabolites are dialyzable. 13 NONCLINICAL TOXICOLOGY 13.1 Carcinogenesis, Mutagenesis, Impairment of Fertility Studies examining the carcinogenic potential of pomalidomide have not been conducted. One of 12 monkeys dosed with 1 mg/kg of pomalidomide (an exposure approximately 15-fold of the exposure in patients at the recommended dose of 4 mg/day) developed acute myeloid leukemia in a 9-month repeat-dose toxicology study. Pomalidomide was not mutagenic or clastogenic in a battery of tests, including the bacteria reverse mutation assay (Ames test), the in vitro assay using human peripheral blood lymphocytes, and the micronucleus test in orally treated rats administered doses up to 2000 mg/kg/day.

In a fertility and early embryonic development study in rats, drug-treated males were mated with untreated or treated females. Pomalidomide was administered to males and females at doses of 25 to 1000 mg/kg/day. When treated males were mated with treated females, there was an increase in post-implantation loss and a decrease in mean number of viable embryos at all dose levels. There were no other effects on reproductive functions or the number of pregnancies. The lowest dose tested in animals resulted in an exposure (AUC) approximately 100-fold of the exposure in patients at the recommended dose of 4 mg/day. When treated males in this study were mated with untreated females, all uterine parameters were comparable to the controls. Based on these results, the observed effects were attributed to the treatment of females. 17 PATIENT COUNSELING INFORMATION See FDA-approved Patient Labeling (Medication Guide) Embryo-Fetal Toxicity Advise patients that POMALYST is contraindicated in pregnancy [see Contraindications (4)]. POMALYST is a thalidomide analogue and may cause serious birth defects or death to a developing baby [see Warnings and Precautions (5.1) and Use in Specific Populations (8.1)]. • Advise females of reproductive potential that they must avoid pregnancy while taking POMALYST and for at least 4 weeks after completing therapy. • Initiate POMALYST treatment in females of reproductive potential only following a negative pregnancy test. • Advise females of reproductive potential of the importance of monthly pregnancy tests and the need to use 2 different forms of contraception, including at least 1 highly effective form, simultaneously during POMALYST therapy, during therapy interruption, and for 4 weeks after she has completely finished taking POMALYST. Highly effective forms of contraception other than tubal ligation include IUD and hormonal (birth control pills, injections, patch, or implants) and a partner’s vasectomy. Additional effective contraceptive methods include latex or synthetic condom, diaphragm, and cervical cap. • Instruct patient to immediately stop taking POMALYST and contact her doctor if she becomes pregnant while taking this drug, if she misses her menstrual period or experiences unusual menstrual bleeding, if she stops taking birth control, or if she thinks FOR ANY REASON that she may be pregnant. • Advise patient that if her doctor is not available, she can call 1-888-668-2528 for information on emergency contraception [see Warnings and Precautions (5.1) and Use in Specific Populations (8.6)]. • Advise males to always use a latex or synthetic condom during any sexual contact with females of reproductive potential while taking POMALYST and for up to 28 days after discontinuing POMALYST, even if they have undergone a successful vasectomy. • Advise male patients taking POMALYST that they must not donate sperm [see Warnings and Precautions (5.1) and Use in Specific Populations (8.6)]. • All patients must be instructed to not donate blood while taking POMALYST and for 1 month following discontinuation of POMALYST [see Warnings and Precautions (5.1) and Use in Specific Populations (8.6)]. POMALYST REMS Program Because of the risk of embryo-fetal toxicity, POMALYST is only available through a restricted program called POMALYST REMS [see Warnings and Precautions (5.2)]. • Patients must sign a Patient-Physician Agreement Form and comply with the requirements to receive POMALYST. In particular, females of reproductive potential must comply with the pregnancy testing, contraception requirements, and participate in monthly telephone surveys. Males must comply

with the contraception requirements [see Use in Specific Populations (8.6)]. • POMALYST is available only from pharmacies that are certified in POMALYST REMS. Provide patients with the telephone number and Web site for information on how to obtain the product. Venous and Arterial Thromboembolism Inform patients of the risk of developing DVT, PE, MI, and stroke and to report immediately any signs and symptoms suggestive of these events for evaluation [see Boxed Warnings and Warnings and Precautions (5.3)]. Hematologic Toxicities Inform patients on the risks of developing neutropenia, thrombocytopenia, and anemia and the need to report signs and symptoms associated with these events to their healthcare provider for further evaluation [see Warnings and Precautions (5.4)]. Hepatotoxicity Inform patients on the risks of developing hepatotoxicity, including hepatic failure and death, and to report signs and symptoms associated with these events to their healthcare provider for evaluation [see Warnings and Precautions (5.5)]. Hypersensitivity Inform patients of the risk for angioedema and severe skin reactions and to report any signs and symptoms associated with these events to their healthcare provider for evaluation [see Warnings and Precautions (5.6)]. Dizziness and Confusional State Inform patients of the potential risk of dizziness and confusional state with the drug, to avoid situations where dizziness or confusional state may be a problem, and not to take other medications that may cause dizziness or confusional state without adequate medical advice [see Warnings and Precautions (5.7)]. Neuropathy Inform patients of the risk of neuropathy and to report the signs and symptoms associated with these events to their healthcare provider for further evaluation [see Warnings and Precautions (5.8)]. Second Primary Malignancies Inform the patient that the potential risk of developing acute myelogenous leukemia during treatment with POMALYST is unknown [see Warnings and Precautions (5.9)]. Tumor Lysis Syndrome Inform patients of the potential risk of tumor lysis syndrome and to report any signs and symptoms associated with this event to their healthcare provider for evaluation [see Warnings and Precautions (5.10)]. Dosing Instructions Inform patients on how to take POMALYST [see Dosage and Administration (2.1)] • POMALYST should be taken once daily at about the same time each day. • POMALYST should be taken without food (at least 2 hours before or 2 hours after a meal). • The capsules should not be opened, broken, or chewed. POMALYST should be swallowed whole with water. • Instruct patients that if they miss a dose of POMALYST, they may still take it up to 12 hours after the time they would normally take it. If more than 12 hours have elapsed, they should be instructed to skip the dose for that day. The next day, they should take POMALYST at the usual time. Warn patients not to take 2 doses to make up for the one that they missed. Other Information Advise patients who smoke to stop because smoking may reduce the efficacy of pomalidomide [see Drug Interactions (7.2)]. Manufactured for: Celgene Corporation Summit, NJ 07901 POMALYST®, REVLIMID®, THALOMID®, and POMALYST REMS® are registered trademarks of Celgene Corporation. Pat. http://www.celgene.com/therapies © 2005-2015 Celgene Corporation All rights reserved. POM HCP Bsv.004 04/15

Cyrus Ghajar, PhD

According to Dr. Ghajar, the phenomenon of tumor dormancy has not been the subject of much study over the years. As a result, “we don’t really know a whole lot about these cells in terms of what puts them to sleep and what wakes them up,” he said. “You can have a cell with oncogenic mutations, but if the microenvironment around the cell is telling it to behave, it will,” he said. Dr. Ghajar found that single breast cancer tumor cells can exist peacefully on the blood vessels of various organs for months. However, if the blood vessel is disrupted—which inflammation and other processes can bring on—the cells will wake up and begin to form tumors. “This grant is leveraging that finding,” he said. “We want to identify factors that keep the cells asleep, and we’re going to try to systematically reinforce those cues in an attempt to prevent metastasis. We’re also going to profile the outside of the blood vessel and figure out if there are molecules that uniquely mediate chemo resistance. We want to see if there are interactions we can disrupt that would make dormant cells die when patients are given chemotherapy.” Finally, Dr. Ghajar has created a three-dimensional, laboratory-based breast tumor model that will allow his team to target sleeping tumor cells in hopes of destroying them—and provide a lasting cure for patients with cancer. n T:10”

Females Females of reproductive potential must commit either to abstain continuously from heterosexual sexual intercourse or to use 2 methods of reliable birth control simultaneously: one highly effective form of contraception – tubal ligation, IUD, hormonal (birth control pills, injections, hormonal patches, vaginal rings, or implants), or partner’s vasectomy, and 1 additional effective contraceptive method – male latex or synthetic condom, diaphragm, or cervical cap. Contraception must begin 4 weeks prior to initiating treatment with POMALYST, during therapy, during dose interruptions, and continuing for 4 weeks following discontinuation of POMALYST therapy. Reliable contraception is indicated even where there has been a history of infertility, unless due to hysterectomy. Females of reproductive potential should be referred to a qualified provider of contraceptive methods, if needed. Females of reproductive potential must have 2 negative pregnancy tests before initiating POMALYST. The first test should be performed within 10-14 days, and the second test within 24 hours prior to prescribing POMALYST. Once treatment has started and during dose interruptions, pregnancy testing for females of reproductive potential should occur weekly during the first 4 weeks of use, then pregnancy testing should be repeated every 4 weeks in females with regular menstrual cycles. If menstrual cycles are irregular, the pregnancy testing should occur every 2 weeks. Pregnancy testing and counseling should be performed if a patient misses her period or if there is any abnormality in her menstrual bleeding. POMALYST treatment must be discontinued during this evaluation. Males Pomalidomide is present in the semen of males who take POMALYST. Therefore, males must always use a latex or synthetic condom during any sexual contact with females of reproductive potential while taking POMALYST and for up to 28 days after discontinuing POMALYST, even if they have undergone a successful vasectomy. Male patients taking POMALYST must not donate sperm.

breast cancer patient advocates on this project. Dr. Ghajar is investigating two different paths for dealing with these dormant disseminated tumor cells: keeping them asleep and inactive, or destroying them altogether. “Basically, we are hedging our bets. You might be able to keep these cells asleep forever, but this carries an inherent risk, because you are leaving these ticking time bombs in your body. Perhaps we can mitigate this with a strong enough ‘sedative,’” he said. “But just in case, we also have ways we think will allow us to specifically get rid of them.”


The ASCO Post  |   JULY 25, 2015

PAGE 36

Advocates in Oncology Survivorship

NCCS Advocates for Legislation to Establish Medicare Service for Cancer Survivorship Care Planning By Kirsten Boyd Goldberg

S

ome years after successful treatment of a childhood cancer, a 16-year-old survivor required surgery to replace both hip joints, which were damaged from therapy containing steroids. An x-ray of the teenager’s destroyed joints is a stark reminder of the serious health challenges faced by cancer survivors for the rest of their lives. “Cure comes at a cost,” said Smita Bhatia, MD, MPH, Director of the Institute for Cancer Outcomes and Survivorship at the University of Alabama Birmingham School of Medicine. Dr. Bhatia spoke about the teenager with osteonecrosis, as well as the range of other health issues faced by survivors and how health-care providers can best serve this population, in a June 3 webinar sponsored by the National Coalition for Cancer Survivorship (NCCS). “Approximately 75% of [childhood cancer survivors] have at least one chronic condition by the time they are 30 years out from diagnosis, and 30% will have severe or life-threatening chronic conditions,” Dr. Bhatia said. “The burden of morbidity is substantial. There is a need for long-term follow-up.” In 1971, there were 3 million cancer survivors in the United States, representing about 1.5% of the population, Dr. Bhatia noted. By 2014, there were 14.5 million survivors, about 4% of the population. At the current rate, the number of U.S. cancer survivors is expected to increase to almost 19 million by 2024.1

Slipping Through the Cracks Survivors face not only the late effects of cancer treatment and the possibility of recurrence or second cancers, but also difficulties with insurance coverage and the coordination of health care between oncologists and primary care physicians, Dr. Bhatia said. When cancer survivors transition

to primary care, they often slip through the cracks, and needed care to prevent second cancers or other conditions is missed, Dr. Bhatia noted. Survivors often will have stacks of paper medical records but won’t clearly remember

ment summary and follow-up care plan. ASCO currently offers a generic SCP template as well as templates for breast, colorectal, prostate, small cell, and non–small cell lung cancers and diffuse large B-cell lymphoma.

Approximately 75% of [childhood cancer survivors] have at least one chronic condition by the time they are 30 years out from diagnosis. The burden of morbidity is substantial. There is a need for long-term follow-up. —Smita Bhatia, MD, MPH

exactly what treatments they received. This information is important and can help prevent a primary care physician from ordering unnecessary or even harmful tests. It could also encourage a patient’s future health-care providers to be vigilant about preventing or catching problems early.

Survivorship Care Plans The Commission on Cancer has mandated that by 2019, every patient with cancer who completes cancer treatment should receive a treatment summary. Nearly 10 years ago, the Institute of Medicine released the report “From Cancer Patient to Cancer Survivor: Lost in Transition,” calling for survivorship care plans for every patient completing cancer treatment. To help oncology providers develop, implement, or expand their survivorship care programs, ASCO has issued a suite of care-planning resources, including a new treatment plan template and disease-specific survivorship care plan (SCP) templates. The SCP templates consist of a treat-

The templates are part of ASCO’s Cancer Survivorship Compendium, an online repository of tools and resources to help oncologists implement or expand survivorship care programs. The SCP templates and the Compendium are available at asco.org/survivorship. More cancer centers and oncology practices are likely to develop survivorship care programs in the coming years, Dr. Bhatia said. For cancer survivors especially, it’s important to detect health problems early, but often, insurance companies block preventive tests, Dr. Bhatia said. “We spend a large amount of time arguing with insurance companies on this,” she said. “We need to educate our insurance companies. They are being penny wise and pound foolish in not allowing coverage for these tests. We are saving them many thousands of dollars by detecting and preventing these complications early.”

Needed Legislation The NCCS is advocating support for legislation that would establish a

new Medicare service for cancer care planning. Congresswoman Lois Capps (D-CA24) and Congressman Charles Boustany (R-LA3) intend to introduce the Planning Actively for Cancer Treatment (PACT) Act in the near future, said NCCS Chief Executive Officer Shelley Fuld Nasso. The proposed planning service could be provided to patients at the time of cancer diagnosis, at the end of active treatment, at the beginning of long-term survivorship, and when there is a significant change in treatment, according to an NCCS statement. The PACT Act would encourage health-care providers to create a survivorship care plan, including a treatment summary and information about monitoring and follow-up post treatment. The NCCS webinar, “Strategies for Designing a System for Survivorship Care,” including Dr. Bhatia’s slides, may be viewed on the NCCS website at http://www.canceradvocacy.org/aboutus/events/strategies-for-designing-survivorship-care-system-webinar/. The event was part of a new NCCS program titled, “Cancer Policy Advocate Training (CPAT),” designed to prepare advocates to be fully engaged in cancer care policy initiatives. NCCS held in-person training in Washington DC, June 25–26, to address the needs of survivors from diagnosis through treatment and long-term survivorship care. n Disclosure: Dr. Bhatia reported no potential conflicts of interest.

Reference 1. American Cancer Society: Cancer Treatment and Survivorship Facts and Figures 2014-2015. Available at http:// www.cancer.org/acs/groups/content/@ research/documents/document/acspc-042801.pdf. Accessed July 2, 2015.

Delivered to your inbox every weekday evening. Visit ASCOPost.com to learn more.


ASCOPost.com  |   JULY 25, 2015

PAGE 37

Journal Spotlight Thoracic Cancer

Early-Phase Studies Show Activity of Novel EGFR Inhibitors Rociletinib and AZD9291 in T790M-Positive NSCLC By Matthew Stenger

T

wo early-phase studies have shown that the novel epidermal growth factor receptor (EGFR) inhibitors rociletinib and AZD9291 exhibit high activity in non–small cell lung cancer (NSCLC) patients with the EGFR T790M mutation who had progressed on prior EGFR inhibitor therapy.1,2 The T790M mutation confers resistance to current EGFR inhibitors. The novel inhibitors are also active in disease with sensitizing EGFR mutations, and both also produced responses in T790M-negative patients who had previously received EGFR inhibitor therapy. Both rociletinib and AZD9291 have been granted Breakthrough Therapy status by the U.S. Food and Drug Administration, and both agents continue to be studied in ongoing clinical trials.

Rociletinib Study In a phase I/II study1 reported in The New England Journal of Medicine by Lecia V. Sequist, MD, MPH, of Massachusetts General Hospital and Harvard Medical School, and colleagues, 92 evaluable patients with advanced EGFR-mutant NSCLC that had progressed on treatment with an EGFR inhibitor were treated with a free-base form of rociletinib (first group of enrolled patients) at a dose of 900 mg twice daily (10 with centrally confirmed presence or absence of T790M mutation) or a hydrogen bromide salt form at doses of 500 mg twice daily to 1,000 mg twice daily (all remaining patients; 53 with centrally confirmed presence or absence of T790M mutation).

Responses A total of 83 patients were evaluable for response. Among 46 patients with centrally confirmed T790M-positive

tumors, 59% (95% confidence interval [CI] = 45%–73%) had a partial response, and 35% had stable disease (disease control rate = 93%). Response rates were similar in patients with deletion 19 or L858R EGFR mutations. At the time of analysis, estimated median progression-free survival was 13.1 months (95% CI = 5.4–13.1 months), with data censored for 82% of patients. Among 17 patients with T790M-negative tumors on central testing, the response rate was 29% (95% CI = 8%–51%), and 29% had stable disease (disease control rate = 59%). Estimated median progression-free survival in these patients was 5.6 months (95% CI = 1.3 months to not reached). Among 20 patients whose tumors were not assessable for T790M mutation, the response rate was 15%.

The investigators concluded: “Rociletinib was active in patients with ­EGFR-mutated NSCLC associated with the T790M resistance mutation.”

AZD9291 Study In a phase I study2 of predominantly Asian patients reported in The New England Journal of Medicine by Pasi A. Jänne, MD, of Dana-Farber Cancer Institute, and colleagues, 253 evaluable

Safety No maximum tolerated dose was identified. Among 253 patients in the safety analysis, the most common adverse events of any grade were diarrhea (47%), rash (40%), nausea (22%), and decreased appetite (21%); diarrhea and rash appeared to be dose-dependent. Adverse events of grade ≥ 3 occurred in 32% of patients. Adverse events led to dose reduction in 7% and treatment

AZD9291 was highly active in patients with lung cancer with the EGFR T790M mutation who had had disease progression during prior therapy with EGFR tyrosine kinase inhibitors. —Pasi A. Jänne, MD, and colleagues

Safety No maximum tolerated dose of rociletinib was identified. The most common dose-limiting adverse event was hyperglycemia. Among 92 patients receiving therapeutic doses of rociletinib, the most common treatment-related adverse events of any grade were hyperglycemia (47%), nausea (35%), fatigue (24%), and diarrhea (22%); QTc prolongation occurred in 12%. The most common grade 3 adverse events were hyperglycemia (22%) and QTc prolongation (5%). Glucose-lowering treatment was required in 38% of patients. Patients with grade 3 QTc prolongation were asymptomatic and managed by dose reduction. No acneiform rash was observed; one patient had maculopapular rash. Overall, 48% of patients had dose reduction.

Novel EGFR Inhibitors in T790M-Positive NSCLC ■■ In patients with T790M-positive disease receiving rociletinib, the response rate was 59%, and the disease control rate was 93%. ■■ In patients with T790M-positive disease receiving AZD9291, the response rate was 61%, and the disease control rate was 95%.

patients with advanced disease who had disease progression after prior EGFR inhibitor therapy received AZD9291 at daily dose levels of 20 mg to 240 mg.

Responses Among 239 patients evaluable for response, 51% (95% CI = 45%–58%) responded (one complete response), and 33% had stable disease (disease control rate = 84%). Response rates were 50% among 150 Asian patients and 54% among 89 non-Asian patients. Response rates were similar at each of the AZD9291 dose levels. Among 127 evaluable patients with centrally confirmed T790M mutation, the response rate was 61% (95% CI = 52%–70%), and the disease control rate was 95%. Among 61 evaluable patients with no T790M mutation on central testing, the response rate was 21% (95% CI = 12%–34%), and the disease control rate was 61%. Median progression-free survival was 9.6 months (95% CI = 8.3 months to not reached) in T790M-positive patients and 2.8 months (95% CI = 2.1–4.3 months) in T790M-negative patients.

discontinuation in 6%. Six patients (2%) had pneumonitis-like events, with all of them discontinuing treatment. QTc prolongation occurred in 4% of patients, and hyperglycemia occurred in 2%, with no dose reduction or discontinuation in any of these patients. There were no significant differences in the severity or frequency of adverse events between Asian and non-Asian patients. The investigators concluded: “AZD9291 was highly active in patients with lung cancer with the EGFR T790M mutation who had had disease progression during prior therapy with EGFR tyrosine kinase inhibitors.” n Disclosure: The rociletinib study was funded by Clovis Oncology. The AZD9291 study was funded by AstraZeneca. For full disclosures of the study authors, visit www.nejm.org.

References 1. Sequist LV, et al: Rociletinib in ­EGFR-mutated non-small-cell lung cancer. N Engl J Med 372:1700-1709, 2015. 2. Jänne PA, et al: AZD9291 in EGFR inhibitor-resistant non-small-cell lung cancer. N Engl J Med 372:1689-1699, 2015.

For more on EGFR-mutated NSCLC, see the commentary by Ramaswamy Govindan, MD, on page 41.


For patients with bone metastases from solid tumors

Prevent bone complications longer In a prespecified integrated analysis of 3 pivotal trials (N = 5,723),

8.2

XGEVA® was proven to delay the median time to first bone complication by

months longer vs zoledronic acid1

XGEVA® is a convenient 120 mg subcutaneous injection administered once every 4 weeks.2

Bone complications, or skeletal-related events (SREs), are defined as radiation to bone, pathologic fracture, surgery to bone, and spinal cord compression.2,3 Median Time to First Bone Complication1 27.7

months

Data from a prespecified integrated analysis of three international, phase 3, double-blind, double-dummy, active-controlled trials comparing XGEVA® with zoledronic acid for the prevention of bone complications in patients with bone metastases from solid tumors or multiple myeloma.1

XGEVA VA® 120 mg Q4W (n = 2,862) VA

19.5

months

zoledronic acid 4 mg Q4W (n = 2,861) 1 YEAR

HR* = 0.83 (95% CI: 0.76-0.90)

2 YEARS

P < 0.001

IMPORTANT SAFETY INFORMATION Hypocalcemia • Pre-existing hypocalcemia must be corrected prior to initiating therapy with XGEVA®. XGEVA® can cause severe symptomatic hypocalcemia, and fatal cases have been reported. Monitor calcium levels and administer calcium, magnesium, and vitamin D as necessary. Monitor levels more frequently when XGEVA® is administered with other drugs that can also lower calcium levels. Advise patients to contact a healthcare professional for symptoms of hypocalcemia. • An increased risk of hypocalcemia has been observed in clinical trials of patients with increasing renal dysfunction, most commonly with severe dysfunction (creatinine clearance less than 30 mL/minute and/or on dialysis), and with inadequate/no calcium supplementation. Monitor calcium levels and calcium and vitamin D intake. Hypersensitivity • XGEVA® is contraindicated in patients with known clinically significant hypersensitivity to XGEVA®, including anaphylaxis that has been reported with use of XGEVA®. Reactions may include hypotension, dyspnea, upper airway edema, lip swelling, rash, pruritus, and urticaria. If an anaphylactic or other clinically significant allergic reaction occurs, initiate appropriate therapy and discontinue XGEVA® therapy permanently.

*Hazard ratio (HR) is defined as the increase or decrease in likelihood of an event of interest (in this case a bone complication) for one group relative to that in a comparator group. P value for superiority.

Drug Products with Same Active Ingredient • Patients receiving XGEVA® should not take Prolia® (denosumab). Osteonecrosis of the Jaw • Osteonecrosis of the jaw (ONJ) can occur in patients receiving XGEVA®, manifesting as jaw pain, osteomyelitis, osteitis, bone erosion, tooth or periodontal infection, toothache, gingival ulceration, or gingival erosion. Persistent pain or slow healing of the mouth or jaw after dental surgery may also be manifestations of ONJ. In clinical trials in patients with osseous metastasis, the incidence of ONJ was higher with longer duration of exposure. • Perform an oral examination and appropriate preventive dentistry prior to the initiation of XGEVA® and periodically during XGEVA® therapy. Advise patients regarding oral hygiene practices. Avoid invasive dental procedures during treatment with XGEVA®. • Patients who are suspected of having or who develop ONJ while on XGEVA® should receive care by a dentist or an oral surgeon. In these patients, extensive dental surgery to treat ONJ may exacerbate the condition. Atypical Subtrochanteric and Diaphyseal Femoral Fracture • Atypical femoral fracture has been reported with XGEVA®. These fractures can occur anywhere in the femoral shaft from just below the lesser trochanter to above the supracondylar flare and are transverse or short oblique in orientation without evidence of comminution.


XGEVA® is indicated for the prevention of skeletal-related events in patients with bone metastases from solid tumors. XGEVA® is not indicated for the prevention of skeletal-related events in patients with multiple myeloma. • RANK Ligand (RANKL) is produced by bone cells in the skeleton and is a key mediator of bone resorption4 • RANKL production is increased at sites of bone metastases, and stimulates osteoclasts to destroy bone4 • XGEVA® acts precisely to bind RANKL and inhibits osteoclast formation, function, and survival2 • Pre-existing hypocalcemia must be corrected prior to initiating therapy with XGEVA®2

Learn more at XGEVA.com

• Atypical femoral fractures most commonly occur with minimal or no trauma to the affected area. They may be bilateral and many patients report prodromal pain in the affected area, usually presenting as dull, aching thigh pain, weeks to months before a complete fracture occurs. A number of reports note that patients were also receiving treatment with glucocorticoids (e.g. prednisone) at the time of fracture. During XGEVA® treatment, patients should be advised to report new or unusual thigh, hip, or groin pain. Any patient who presents with thigh or groin pain should be suspected of having an atypical fracture and should be evaluated to rule out an incomplete femur fracture. Patient presenting with an atypical femur fracture should also be assessed for symptoms and signs of fracture in the contralateral limb. Interruption of XGEVA® therapy should be considered, pending a risk/benefit assessment, on an individual basis.

Embryo-Fetal Toxicity • XGEVA® can cause fetal harm when administered to a pregnant woman. Based on findings in animals, XGEVA® is expected to result in adverse reproductive effects.

©2014 Amgen Inc. All rights reserved. 04/14 80218-R1-V1

• Advise females of reproductive potential to use highly effective contraception during therapy, and for at least 5 months after the last dose of XGEVA®. Apprise the patient of the potential hazard to a fetus if XGEVA® is used during pregnancy or if the patient becomes pregnant while patients are exposed to XGEVA®.

Adverse Reactions • The most common adverse reactions in patients receiving XGEVA® with bone metastasis from solid tumors were fatigue/asthenia, hypophosphatemia, and nausea. The most common serious adverse reaction was dyspnea. The most common adverse reactions resulting in discontinuation were osteonecrosis and hypocalcemia. Please see brief summary of Prescribing Information on the following page. REFERENCES: 1. Lipton A, Fizazi K, Stopeck AT, et al. Superiority of denosumab to zoledronic acid for prevention of skeletal-related events: a combined analysis of 3 pivotal, randomised, phase 3 trials. Eur J Cancer. 2012;48:3082-3092. 2. XGEVA® (denosumab) prescribing information, Amgen. 3. Brodowicz T, O’Byrne K, Manegold C. Bone matters in lung cancer. Ann Oncol. 2012;23:2215-2222. 4. Roodman GD. Mechanisms of bone metastasis. N Engl J Med. 2004;350:1655-1664.

www.XGEVA.com


S:9.5”

Brief Summary: Consult package insert for complete Prescribing Information

Body System GASTROINTESTINAL Nausea Diarrhea GENERAL Fatigue/ Asthenia IN VESTIGATIONS Hypocalcemiab Hypophosphatemiab NEUROLOGICAL Headache RESPIRATORY Dyspnea Cough

Xgeva n = 2841 %

Zoledronic Acid n = 2836 %

31 20

32 19

45

46

18 32

9 20

13

14

21 15

18 15

Adverse reactions reported in at least 10% of patients receiving Xgeva in Trials 1, 2, and 3, and meeting one of the following criteria: • At least 1% greater incidence in Xgeva-treated patients, or • Between-group difference (either direction) of less than 1% and more than 5% greater incidence in patients treated with zoledronic acid compared to placebo (US Prescribing Information for zoledronic acid) b Laboratory-derived and below the central laboratory lower limit of normal [8.3 – 8.5 mg/dL (2.075 – 2.125 mmol/L) for calcium and 2.2 – 2.8 mg/dL (0.71 – 0.9 mmol/L) for phosphorus] Severe Mineral/Electrolyte Abnormalities • Severe hypocalcemia (corrected serum calcium less than 7 mg/dL or less than 1.75 mmol/L) occurred in 3.1% of patients treated with Xgeva and 1.3% of patients treated with zoledronic acid. Of patients who experienced severe hypocalcemia, 33% experienced 2 or more episodes of severe hypocalcemia and 16% experienced 3 or more episodes. • Severe hypophosphatemia (serum phosphorus less than 2 mg/dL or less than 0.6 mmol/L) occurred in 15.4% of patients treated with Xgeva and 7.4% of patients treated with zoledronic acid. Osteonecrosis of the Jaw In the primary treatment phases of Trials 1, 2, and 3, ONJ was confirmed in 1.8% of patients in the Xgeva group (median exposure of 12.0 months; range 0.1 – 40.5) and 1.3% of patients in the zoledronic acid group. The trials in patients with breast (Trial 1) or prostate (Trial 3) cancer included an Xgeva open label extension treatment phase where patients were offered Xgeva 120 mg once every 4 weeks (median overall exposure of 14.9 months; range 0.1 – 67.2). The patient-year adjusted incidence of confirmed ONJ was 1.1% during the first year of treatment and 4.1% thereafter. The median time to ONJ was 20.6 months (range: 4 – 53). Atypical Subtrochanteric and Diaphyseal Fracture Atypical femoral fracture has been reported with Xgeva. Postmarketing Experience. Because postmarketing reactions are reported voluntarily from a population of uncertain size, it is not always possible to reliably estimate their frequency or establish a causal relationship to drug exposure. The following adverse reactions have been identified during post approval use of Xgeva: • Hypocalcemia: Severe symptomatic hypocalcemia, including fatal cases. • Hypersensitivity, including anaphylactic reactions. • Musculoskeletal pain, including severe musculoskeletal pain. Positive rechallenge has been reported. Immunogenicity. As with all therapeutic proteins, there is potential for immunogenicity. Using an electrochemiluminescent bridging immunoassay, less than 1% (7/2758) of patients with osseous metastases treated with denosumab doses ranging from 30 – 180 mg every 4 weeks or every 12 weeks for up to 3 years tested positive for binding antibodies. No patient with positive binding antibodies tested positive for neutralizing antibodies as assessed using a chemiluminescent cell-based in vitro biological assay. There was no evidence of altered pharmacokinetic profile, toxicity profile, or clinical response associated with binding antibody development. The incidence of antibody formation is highly dependent on the sensitivity and specificity of the assay. Additionally, the observed incidence of a positive antibody (including neutralizing antibody) test result may be influenced by several factors, including assay methodology, sample handling, timing of sample collection, concomitant medications, and underlying disease. For these reasons, comparison of antibodies to denosumab with the incidence of antibodies to other products may be misleading. DRUG INTERACTIONS: No formal drug-drug interaction trials have been conducted with Xgeva. In clinical trials in patients with breast cancer metastatic to bone, Xgeva was administered in combination with standard anticancer treatment. Serum denosumab concentrations at 1 and 3 months and reductions in the bone turnover marker uNTx/Cr (urinary N-terminal telopeptide corrected for creatinine) at 3 months were similar in patients with and without prior intravenous bisphosphonate therapy. There was no evidence that various anticancer treatments affected denosumab systemic exposure and pharmacodynamic effect. Serum denosumab concentrations at 1 and 3 months were not altered by concomitant chemotherapy and/or hormone therapy. The median reduction in uNTx/Cr from baseline to month 3 was similar between patients receiving concomitant chemotherapy and/or hormone therapy. USE IN SPECIFIC POPULATIONS: Pregnancy: Category D. Risk Summary: Xgeva can cause fetal harm when administered to a pregnant woman based on findings in animals. In utero denosumab exposure in cynomolgus monkeys resulted in increased fetal loss, stillbirths, and postnatal mortality, along with evidence of absent lymph nodes, abnormal bone growth and decreased neonatal growth. There are no adequate and well-controlled studies with Xgeva in pregnant women. Women should be advised not to become pregnant when taking Xgeva. If this drug is used during pregnancy, or if the patient becomes pregnant while taking this drug, the patient should be apprised of the potential hazard to the fetus. Women who become pregnant during Xgeva treatment are encouraged to enroll in Amgen’s Pregnancy Surveillance Program. Patients or their physicians should call 1-800-77-AMGEN (1-800-772-6436) to enroll. Clinical Considerations: The effects of Xgeva are likely to be greater during the second and third trimesters of pregnancy. Monoclonal antibodies are transported across the placenta in a linear fashion as pregnancy progresses, with the largest amount transferred during the third trimester. If the patient becomes pregnant during Xgeva therapy, consider the risks and benefits in continuing or discontinuing treatment with Xgeva. a

Animal Data: The effects of denosumab on prenatal development have been studied in both cynomolgus monkeys and genetically engineered mice in which RANK ligand (RANKL) expression was turned off by gene removal (a “knockout mouse”). In cynomolgus monkeys dosed subcutaneously with denosumab throughout pregnancy at a pharmacologically active dose, there was increased fetal loss during gestation, stillbirths, and postnatal mortality. Other findings in offspring included absence of axillary, inguinal, mandibular, and mesenteric lymph nodes; abnormal bone growth, reduced bone strength, reduced hematopoiesis, dental dysplasia and tooth malalignment; and decreased neonatal growth. At birth out to one month of age, infants had measurable blood levels of denosumab (22-621% of maternal levels). Following a recovery period from birth out to 6 months of age, the effects on bone quality and strength returned to normal; there were no adverse effects on tooth eruption, though dental dysplasia was still apparent; axillary and inguinal lymph nodes remained absent, while mandibular and mesenteric lymph nodes were present, though small; and minimal to moderate mineralization in multiple tissues was seen in one recovery animal. There was no evidence of maternal harm prior to labor; adverse maternal effects occurred infrequently during labor. Maternal mammary gland development was normal. There was no fetal NOAEL (no observable adverse effect level) established for this study because only one dose of 50 mg/kg was evaluated. In RANKL knockout mice, absence of RANKL (the target of denosumab) also caused fetal lymph node agenesis and led to postnatal impairment of dentition and bone growth. Pregnant RANKL knockout mice showed altered maturation of the maternal mammary gland, leading to impaired lactation. Nursing Mothers. It is not known whether Xgeva is excreted into human milk. Measurable concentrations of denosumab were present in the maternal milk of cynomolgus monkeys up to 1 month after the last dose of denosumab (≤ 0.5% milk:serum ratio). Because many drugs are excreted in human milk and because of the potential for serious adverse reactions in nursing infants from Xgeva, a decision should be made whether to discontinue nursing or discontinue the drug, taking into account the importance of the drug to the mother. Maternal exposure to Xgeva during pregnancy may impair mammary gland development and lactation based on animal studies in pregnant mice lacking the RANK/RANKL signaling pathway that have shown altered maturation of the maternal mammary gland, leading to impaired lactation postpartum. However, in cynomolgus monkeys treated with denosumab throughout pregnancy, maternal mammary gland development was normal, with no impaired lactation. Mammary gland histopathology at 6 months of age was normal in female offspring exposed to denosumab in utero; however, development and lactation have not been fully evaluated. Pediatric Use. Xgeva is not recommended in pediatric patients. The safety and effectiveness of Xgeva in pediatric patients have not been established. Treatment with Xgeva may impair bone growth in children with open growth plates and may inhibit eruption of dentition. In neonatal rats, inhibition of RANKL (the target of Xgeva therapy) with a construct of osteoprotegerin bound to Fc (OPG-Fc) at doses ≤ 10 mg/kg was associated with inhibition of bone growth and tooth eruption. Adolescent primates treated with denosumab at doses 5 and 25 times (10 and 50 mg/kg dose) higher than the recommended human dose of 120 mg administered once every 4 weeks, based on body weight (mg/kg), had abnormal growth plates, considered to be consistent with the pharmacological activity of denosumab. Cynomolgus monkeys exposed in utero to denosumab exhibited bone abnormalities, reduced hematopoiesis, tooth malalignment, decreased neonatal growth, and an absence of axillary, inguinal, mandibular, and mesenteric lymph nodes. Some bone abnormalities recovered once exposure was ceased following birth; however, axillary and inguinal lymph nodes remained absent 6 months post-birth. Geriatric Use. Of patients who received Xgeva in Trials 1, 2, and 3, 1260 (44%) were 65 years of age or older. No overall differences in safety or efficacy were observed between these patients and younger patients. Renal Impairment. Two clinical trials were conducted in patients without cancer and with varying degrees of renal function. In one study, patients (N=55) with varying degrees of renal function (ranging from normal through end-stage renal disease requiring dialysis) received a single 60 mg subcutaneous dose of denosumab. In a second study, patients (N=32) with severe renal dysfunction (creatinine clearance less than 30 mL/minute and/or on dialysis) were given two 120 mg subcutaneous doses of denosumab. In both studies, greater risk of developing hypocalcemia was observed with increasing renal impairment, and with inadequate/no calcium supplementation. Hypocalcemia was mild to moderate in severity in 96% of patients. Monitor calcium levels and, calcium and vitamin D intake. Females and Males of Reproductive Potential. Contraception Females: Counsel patients on pregnancy planning and prevention. Advise females of reproductive potential to use highly effective contraception during therapy, and for at least 5 months after the last dose of Xgeva. Advise patients to contact their healthcare provider if they become pregnant, or a pregnancy is suspected, during treatment or within 5 months after the last dose of Xgeva. Males: The extent to which denosumab is present in seminal fluid is unknown. There is potential for fetal exposure to denosumab when a male treated with Xgeva has unprotected sexual intercourse with a pregnant partner. Advise males of this potential risk. OVERDOSAGE: There is no experience with overdosage of Xgeva. HOW SUPPLIED/STORAGE AND HANDLING: Xgeva is supplied in a singleuse vial. Store Xgeva in a refrigerator at 2°C to 8°C (36°F to 46°F) in the original carton. Do not freeze. Once removed from the refrigerator, Xgeva must not be exposed to temperatures above 25°C/77°F or direct light and must be used within 14 days. Discard Xgeva if not used within the 14 days. Do not use Xgeva after the expiry date printed on the label. Protect Xgeva from direct light and heat. Avoid vigorous shaking of Xgeva. PATIENT COUNSELING INFORMATION: Advise patients to contact a healthcare professional for any of the following: • Symptoms of a hypersensitivity reaction, including rash, urticaria, pruritus, lip swelling, shortness of breath, hypotension and respiratory tract edema • Symptoms of hypocalcemia, including paresthesias or muscle stiffness, twitching, spasms, or cramps • Symptoms of ONJ, including pain, numbness, swelling of or drainage from the jaw, mouth, or teeth • Persistent pain or slow healing of the mouth or jaw after dental surgery • Symptoms of atypical femoral fracture, including new or unusual thigh, hip, or groin pain • Pregnancy or nursing Advise patients of the need for: • Avoiding therapy with Xgeva if a serious allergic reaction occurred with prior Xgeva or Prolia therapy • Proper oral hygiene and routine dental care • Informing their dentist that they are receiving Xgeva • Avoiding invasive dental procedures during treatment with Xgeva • The use of highly effective contraception during and for at least 5 months after treatment with Xgeva for females of reproductive potential Advise patients that denosumab is also marketed as Prolia®. Patients should inform their healthcare provider if they are taking Prolia. Amgen Manufacturing Limited, a subsidiary of Amgen Inc. One Amgen Center Drive Thousand Oaks, California 91320-1799 ©2010-2014 Amgen Inc. All rights reserved. Printed in USA.

DOUS14CDNY4736_XGEVA_Tabloid_BS_V10_8pt_r13.indd 1

8/11/14 11:56 AM

DOUS14CDNY3469_A_ONCAd_Tabloid_BS_r11.indd 1

9/19/14 11:45 AM

S:13”

INDICATIONS AND USAGE: Bone Metastasis from Solid Tumors. Xgeva is indicated for the prevention of skeletal-related events in patients with bone metastases from solid tumors. Important Limitation of Use. Xgeva is not indicated for the prevention of skeletal-related events in patients with multiple myeloma. DOSAGE AND ADMINISTRATION: Recommended Dosage. The recommended dose of Xgeva is 120 mg administered as a subcutaneous injection every 4 weeks in the upper arm, upper thigh, or abdomen. Administer calcium and vitamin D as necessary to treat or prevent hypocalcemia. Preparation and Administration. Visually inspect Xgeva for particulate matter and discoloration prior to administration. Xgeva is a clear, colorless to pale yellow solution that may contain trace amounts of translucent to white proteinaceous particles. Do not use if the solution is discolored or cloudy or if the solution contains many particles or foreign particulate matter. Prior to administration, Xgeva may be removed from the refrigerator and brought to room temperature (up to 25°C/77°F) by standing in the original container. This generally takes 15 to 30 minutes. Do not warm Xgeva in any other way. Use a 27-gauge needle to withdraw and inject the entire contents of the vial. Do not re-enter the vial. Discard vial after single-use or entry. CONTRAINDICATIONS: Hypocalcemia. Pre-existing hypocalcemia must be corrected prior to initiating therapy with Xgeva. Hypersensitivity. Xgeva is contraindicated in patients with known clinically significant hypersensitivity to Xgeva. WARNINGS AND PRECAUTIONS: Drug Products with Same Active Ingredient. Xgeva includes the same active ingredient (denosumab) found in Prolia. Patients receiving Xgeva should not take Prolia. Hypersensitivity. Clinically significant hypersensitivity including anaphylaxis has been reported with use of Xgeva. Reactions may include hypotension, dyspnea, upper airway edema, lip swelling, rash, pruritus, and urticaria. If an anaphylactic or other clinically significant allergic reaction occurs, initiate appropriate therapy and discontinue Xgeva therapy permanently. Hypocalcemia. Xgeva can cause severe symptomatic hypocalcemia, and fatal cases have been reported. Correct pre-existing hypocalcemia prior to Xgeva treatment. Monitor calcium levels and administer calcium, magnesium, and vitamin D as necessary. Monitor levels more frequently when Xgeva is administered with other drugs that can also lower calcium levels. In the postmarketing setting, severe symptomatic hypocalcemia has been reported. Advise patients to contact a healthcare professional for symptoms of hypocalcemia. An increased risk of hypocalcemia has been observed in clinical trials of patients with increasing renal dysfunction, most commonly with severe dysfunction (creatinine clearance less than 30 mL/minute and/or on dialysis), and with inadequate/no calcium supplementation. Monitor calcium levels and calcium and vitamin D intake. Osteonecrosis of the Jaw. Osteonecrosis of the jaw (ONJ) can occur in patients receiving Xgeva, manifesting as jaw pain, osteomyelitis, osteitis, bone erosion, tooth or periodontal infection, toothache, gingival ulceration, or gingival erosion. Persistent pain or slow healing of the mouth or jaw after dental surgery may also be manifestations of ONJ. In clinical trials, in patients with osseous metastasis, the incidence of ONJ was higher with longer duration of exposure (see Adverse Reactions). Seventy-nine percent of patients with ONJ had a history of tooth extraction, poor oral hygiene, or use of a dental appliance as a predisposing factor. Perform an oral examination and appropriate preventive dentistry prior to the initiation of Xgeva and periodically during Xgeva therapy. Advise patients regarding oral hygiene practices. Avoid invasive dental procedures during treatment with Xgeva. Patients who are suspected of having or who develop ONJ while on Xgeva should receive care by a dentist or an oral surgeon. In these patients, extensive dental surgery to treat ONJ may exacerbate the condition. Atypical Subtrochanteric and Diaphyseal Femoral Fracture. Atypical femoral fracture has been reported with Xgeva. These fractures can occur anywhere in the femoral shaft from just below the lesser trochanter to above the supracondylar flare and are transverse or short oblique in orientation without evidence of comminution. Atypical femoral fractures most commonly occur with minimal or no trauma to the affected area. They may be bilateral and many patients report prodromal pain in the affected area, usually presenting as dull, aching thigh pain, weeks to months before a complete fracture occurs. A number of reports note that patients were also receiving treatment with glucocorticoids (e.g. prednisone) at the time of fracture. During Xgeva treatment, patients should be advised to report new or unusual thigh, hip, or groin pain. Any patient who presents with thigh or groin pain should be suspected of having an atypical fracture and should be evaluated to rule out an incomplete femur fracture. Patient presenting with an atypical femur fracture should also be assessed for symptoms and signs of fracture in the contralateral limb. Interruption of Xgeva therapy should be considered, pending a risk/benefit assessment, on an individual basis. EMBRYO-FETAL TOXICITY: Xgeva can cause fetal harm when administered to a pregnant woman. Based on findings in animals, Xgeva is expected to result in adverse reproductive effects. In utero denosumab exposure in cynomolgus monkeys resulted in increased fetal loss, stillbirths, and postnatal mortality, along with evidence of absent peripheral lymph nodes, abnormal bone growth, and decreased neonatal growth (see Use in Specific Populations). Advise females of reproductive potential to use highly effective contraception during therapy, and for at least 5 months after with the last dose of Xgeva. Apprise the patient of the potential hazard to a fetus if Xgeva is used during pregnancy or if the patient becomes pregnant while patients are exposed to Xgeva. Advise patients to contact their healthcare provider if they become pregnant or a pregnancy is suspected during this time. ADVERSE REACTIONS: The following adverse reactions are discussed below and elsewhere in the labeling: • Hypocalcemia • Osteonecrosis of the Jaw The most common adverse reactions in patients receiving Xgeva (per-patient incidence greater than or equal to 25%) were fatigue/asthenia, hypophosphatemia, and nausea (see Table 1). The most common serious adverse reaction in patients receiving Xgeva was dyspnea. The most common adverse reactions resulting in discontinuation of Xgeva were osteonecrosis and hypocalcemia. Clinical Trials Experience. Because clinical trials are conducted under widely varying conditions, adverse reaction rates observed in the clinical trials of a drug cannot be directly compared to rates in other clinical trials and may not reflect the rates observed in practice. The safety of Xgeva was evaluated in three randomized, double-blind, double-dummy trials in which a total of 2841 patients with bone metastasis from prostate cancer, breast cancer, or other solid tumors, or lytic bony lesions from multiple myeloma received at least one dose of Xgeva. In Trials 1, 2, and 3, patients were randomized to receive either 120 mg of Xgeva every 4 weeks as a subcutaneous injection or 4 mg (dose adjusted for reduced renal function) of zoledronic acid every 4 weeks by intravenous (IV) infusion. Entry criteria included serum calcium (corrected) from 8 to 11.5 mg/dL (2 to 2.9 mmol/L) and creatinine clearance 30 mL/min or greater. Patients who had received IV bisphosphonates were excluded, as were patients with prior history of ONJ or osteomyelitis of the

jaw, an active dental or jaw condition requiring oral surgery, non-healed dental/ oral surgery, or any planned invasive dental procedure. During the study, serum chemistries including calcium and phosphorus were monitored every 4 weeks. Calcium and vitamin D supplementation was recommended but not required. The median duration of exposure to Xgeva was 12 months (range: 0.1 – 41) and median duration on-study was 13 months (range: 0.1 – 41). Of patients who received Xgeva, 46% were female. Eighty-five percent were White, 5% Hispanic/Latino, 6% Asian, and 3% Black. The median age was 63 years (range: 18 – 93). Seventy-five percent of patients who received Xgeva received concomitant chemotherapy. Table 1. Per-patient Incidence of Selecteda Adverse Reactions of Any Severity (Trials 1, 2, and 3)


ASCOPost.com  |   JULY 25, 2015

PAGE 41

Perspective

Progress in the Treatment of Patients With EGFR-Mutated NSCLC By Ramaswamy Govindan, MD

A

pproximately 10% to 15% of patients with advanced non–small cell lung (NSCLC) cancer have mutations in the epidermal growth factor receptor (EGFR) in tumor cells. Specific therapies to inhibit the activity of EGFR-mutated NSCLC have now been clearly shown to improve response rate and progression-free survival compared with cytotoxic chemotherapy in this subset of patients. There is some evidence to suggest that even overall survival might be improved with upfront use of afatinib (Gilotrif, vs platinum-based doublet chemotherapy) in ­EGFR-mutated NSCLC involving

nately, cytotoxic chemotherapy drugs and second-generation EGFR tyrosine kinase inhibitors, such as neratinib, afatinib, and dacomitinib, are not very active in this setting.

Novel Agents Offer New Hope Two papers recently published in The New England Journal of Medicine and reviewed in this issue of The ASCO Post have given new hope for patients with the EGFR T790M mutation. Jänne and colleagues1 conducted a phase I study with an expansion cohort for the T790M population with the EGFR inhibitor AZD9291. A total

Progress in the area of immune checkpoint inhibitors and the development of novel thirdgeneration EGFR tyrosine kinase inhibitors represents significant hope for patients with EGFR-mutated NSCLC. —Ramaswamy Govindan, MD

exon 19 deletions. However, nearly all patients with EGFR-mutated NSCLC eventually develop progressive disease and succumb to widespread metastatic disease. Nearly 50% of patients with EGFRmutated NSCLC have the gatekeeper mutation in which threonine is replaced with methionine (T790M), resulting in decreased affinity for the first-generation inhibitors. UnfortuDr. Govindan is Co-Director, Section of Medical Oncology, and Professor of Medicine, Washington University School of Medicine, St. Louis.

of 253 patients were treated, including 222 patients in 5 extension cohorts. The overall objective tumor response rate was 51% (95% confidence interval [CI] = 45%–58%). Among 127 patients with centrally confirmed EGFR T790M, the response rate was 61% (95% CI = 52%–70%). The response rate among 61 patients without centrally detectable EGFR T790M was only 21% (95% CI = 12%–34%). Median progression-free survival was longer in the T790M-positive patients (9.6 months, 95% CI = 8.3 months to not reached) than in those without T790M (2.8 months, 95% CI = 2.1–

4.3 months). Most notably, AZD9291 was extremely well tolerated. Since AZD9291 is specific for the mutated EGFR, the typical side effects related to the inhibition of wild-type EGFR were seen less frequently. The most common adverse events (all grades) were diarrhea, rash, nausea, and decreased appetite. Rociletinib (CO-1686) is active against EGFR-mutated NSCLC with or without T790M mutation. In the phase I/II study reported by Sequist et al,2 rociletinib was administered to 130 patients with EGFR-mutated NSCLC who had disease progression following treatment with an EGFR inhibitor. In the expansion cohort, patients with EGFR T790M-positive disease received rociletinib at a dose of 500 mg twice a day to 750 mg twice a day. The objective response rate among 46 patients with T790M-positive disease who could be evaluated was 59% (95% CI = 45%–73%), and the response rate among the T790Mnegative patients was 29% (95% CI = 8%–51%). As was observed with AZD9291, the estimated median progression-free survival was markedly longer in the T790M-positive patients than in the T790M-negative patients (13.1 months, 95% CI = 5.4–13.1 months, vs 5.6 months, 95% CI = 1.3 months to not reached).

Reasonable Future Options Just like AZD9291, rociletinib was extremely well tolerated, given its selectivity for the mutant form of EGFR. Unlike what was observed with AZD9291, hyperglycemia was observed with rociletinib treatment but was well tolerated and easily managed with oral hypoglycemic drugs.

Clearly, these two drugs, when they are approved, will represent a reasonable option for patients with EGFR-mutated NSCLC with the T790M mutation. It is important to biopsy tumors at the time of disease progression in patients with EGFRmutated NSCLC, since these agents are available for now as a part of some ongoing trials or through compassionate use programs. It is important to understand the molecular mechanisms underlying the treatment resistance in patients with EGFR-mutated NSCLC who do not have T790M mutations so that appropriate therapies can be developed in this population. Without question, patients who receive rociletinib or AZD9291 are likely to develop resistance to therapy. Molecular mechanisms underlying treatment resistance to these third-generation EGFR tyrosine kinase inhibitors will need to be studied. Currently, efforts are ongoing to optimize plasma-based screening for the emergence of EGFR T790M clones. Progress in the area of immune checkpoint inhibitors and the development of novel third-generation EGFR tyrosine kinase inhibitors represents significant hope for patients with ­EGFR-mutated NSCLC. n Disclosure: Dr. Govindan reported no potential conflicts of interest.

References 1. Jänne PA, Yang JC, Kim DW, et al: AZD9291 in EGFR inhibitor-resistant non-small-cell lung cancer. N Engl J Med 372:1689-1699, 2015. 2. Sequist LV, Soria JC, Goldman JW, et al: Rociletinib in EGFR-mutated non-small-cell lung cancer. N Engl J Med 372:1700-1709, 2015.

Save the Date November 5-8, 2015 JW Marriott Phoenix Desert Ridge

Learn more at jadprolive.com A CE/CME Conference for Advanced Practitioners in Oncology

For conference exhibitor or sponsorship opportunities, contact David Horowitz at 631-935-7658 or email at david@harborsidepress.com


The ASCO Post  |   JULY 25, 2015

PAGE 42

JCO Spotlight Guidelines

ASCO Endorses ASTRO Guideline on Definitive and Adjuvant Radiotherapy in Locally Advanced NSCLC By Matthew Stenger

A

s reported in the Journal of Clinical Oncology by Andrea Bezjak, MD, of Princess Margaret Cancer Center, Toronto, and colleagues,1 ASCO has

Andrea Bezjak, MD

recently endorsed the American Society for Radiation Oncology (ASTRO) evidence-based guideline on externalbeam radiotherapy for patients with locally advanced non–small cell lung cancer (NSCLC).2,3 The ASTRO guideline recommendations were considered by ASCO experts to be clear, thorough, and based on the most relevant scientific evidence. An ASCO endorsement panel endorsed the guideline and added qualifying statements. The endorsement panel was co-chaired by Dr. Bezjak and Christopher G. Azzoli, MD, of Massachusetts General Hospital, Boston. The primary ASTRO guideline question is: What is the role of externalbeam radiotherapy in the management of locally advanced NSCLC? The target population for the guideline consists of patients with stage II or III locally advanced NSCLC whose disease is unresectable and patients with stage II or III disease who are eligible for surgery.

Key Recommendations ASCO’s key recommendations extracted from the ASTRO guideline are summarized here, with ASCO qualify-

ing language shown in italics. • For curative-intent treatment of locally advanced NSCLC, concurrent chemoradiation is recommended because it improves local control and overall survival compared with sequential chemotherapy followed by radiation or radiation therapy alone. • The standard dose fractionation of radiation with concurrent chemotherapy is 60 Gy given in fractions of 2 Gy once per day over 6 weeks. Dose escalation beyond 60 Gy with conventional fractionation has not been demonstrated to be of benefit. • There is no role for the routine use of induction chemotherapy before chemoradiotherapy. • There is no role for the routine use of consolidation chemotherapy after

because it improves overall survival when compared with radiotherapy alone. • Radiotherapy alone may be used for patients ineligible for combinedmodality treatment; it may offer better tolerability but poorer survival. • Postoperative radiotherapy may be recommended for patients with complete resection of N2 disease to improve local control, but it should be delivered sequentially after adjuvant chemotherapy. • Postoperative radiotherapy is recommended for patients with incomplete resection (microscopic or gross positive margin or gross residual disease), to be given either concurrently or sequentially with chemotherapy. • Patients with resectable stage III

The target population for the guideline consists of patients with stage II or III locally advanced NSCLC whose disease is unresectable and patients with stage II or III disease who are eligible for surgery. chemoradiotherapy. Current data fail to support the routine use of consolidation chemotherapy after chemoradiotherapy, but this remains an option for patients who did not receive full systemic chemotherapy doses during radiotherapy. • The ideal concurrent chemotherapy regimen has not been determined. The two most common regimens are cisplatin/etoposide and carboplatin/ paclitaxel. • For patients who cannot tolerate concurrent chemoradiotherapy, sequential chemotherapy followed by radical (definitive) radiation is recommended

NSCLC should be managed by a multidisciplinary team that uses best surgical judgment. The best candidates for preoperative chemoradiotherapy have preoperatively planned lobectomy (as opposed to pneumonectomy), no weight loss, are female, and have only one involved nodal station. n

Disclosure: Drs. Bezjak and Azzoli reported no potential conflicts of interest.

References 1. Bezjak A, Temin S, Franklin G, et al: Definitive and adjuvant radiotherapy in locally advanced non-small-cell lung cancer: American Society of Clinical Oncology

The ASCO Post

Like us on Facebook facebook.com/TheASCOPost

clinical practice guideline endorsement of the American Society for Radiation Oncology evidence-based clinical practice guideline. J Clin Oncol. May 5, 2015 (early release online). 2. Rodrigues G, Choy H, Bradley J, et al: Definitive radiotherapy in locally advanced non-small cell lung cancer: Executive summary of an American Society for Radiation Oncology (ASTRO) evidence-based clinical practice guideline. Pract Radiat Oncol 5:141-148, 2015. 3. Rodrigues G, Choy H, Bradley J, et al: Adjuvant radiotherapy in locally advanced non-small cell lung cancer: Executive summary of an American Society for Radiation Oncology (ASTRO) evidence-based clinical practice guideline. Pract Radiat Oncol 5:149-155, 2015.

For More Information

M

ore information on ­A SCO’s endorsement of the ASTRO clinical practice guideline for radiotherapy in locally advanced NSCLC is available from ASCO at www.asco. org/endorsements/NSCLCradiotherapy. Information for patients about NSCLC is available at www.cancer.net/cancer-types/ lung-cancer. The ASTRO guideline is available at http://www.practicalradonc.org/article/S18798 5 0 0 ( 1 5 ) 0 0 0 8 2 -X / f u l l te x t and http://www.practicalradonc.org/article/%20S18798500(15)00083-1/fulltext.


DEEPER RESPONSES? LONGER DURATION OF THERAPY? It’s time to take a deeper look at the critical debates in multiple myeloma today. SEE WHAT YOUR PEERS SAY AND SHARE YOUR OPINION AT DEBATINGMYELOMA.COM

Takeda Oncology and are registered trademarks of Takeda Pharmaceutical Company Limited. Other trademarks are the property of their respective owners. Copyright © 2015, Millennium Pharmaceuticals, Inc.

All rights reserved.

Printed in the USA

USO/NON/15/0122

4/15


The ASCO Post  |   JULY 25, 2015

PAGE 44

Perspective ASTRO Guidelines continued from page 1

treatment planning and execution, which have a significant impact on the quality of the radiation treatment delivered. Guidelines cannot take into account the expertise of specific disciplines at a given institution, which may have a significant impact on the quality of care. Lastly, they may not fully explore areas of uncertainty where evidence is lacking, and decisions must be made based on best oncologic principles and judgment. Thus, guidelines have to be applied to clinical practice in a thoughtful manner.

Key Points of the ASCO Endorsement Patients with stage II or III NSCLC can be managed with curative-intent radiotherapy or surgical resection, depending on the technical resectability of the tumor and the medical comorbidities of the patients. Chemotherapy should be added to the treatment paradigm for all eligible patients, either concurrently/sequentially with curativeintent radiotherapy or neoadjuvantly/ adjuvantly with surgical resection. Postoperative radiotherapy is recommended for patients with incomplete resection and may be recommended for patients with pathologic N2 nodal involvement. The standard dose for curative-intent radiotherapy is 60 Gy in 2-Gy daily fractions, and ideally this should be delivered with concurrent chemotherapy in all patients who are candidates for this approach.

Areas of Uncertainty Curative-Intent Radiotherapy Concurrent chemoradiotherapy has resulted in the best outcomes for inoperable patients with locally advanced NSCLC. However, determining who can tolerate concurrent vs sequential chemoradiotherapy is more challenging and less well defined. Factors that should be considered are the patient’s performance status, weight loss, age, esophageal or pulmonary comorbidities, oxygen desaturation on exertion, and patient preference regarding the aggressiveness of treatment. The recent RTOG 0617 trial demonDr. Rimner is a radiation oncologist in the Department of Radiation Oncology, Memorial Sloan Kettering Cancer Center, New York.

strated that a dose of 74 Gy is inferior to 60 Gy when treating the entire tumor volume with concurrent chemoradiotherapy.3 However, it remains unknown whether the optimal radiotherapy dose may lie between 60 Gy and 74 Gy. The challenge of identifying the optimal radiotherapy dose in this setting is that locally advanced NSCLC can vary greatly in tumor volume, disease extent, and proximity to organs at risk, and these factors all play a role in determining what radiotherapy dose is safe and most effective. RTOG 0617 newly identified a radiation dose to the heart as predictive of survival. The pathophysiologic mechanism for this finding remains to be elucidated. The results of RTOG 0617 do not settle the question of the optimal radiotherapy dose in patients who receive sequential chemoradiotherapy or radiotherapy alone (for patients who are not candidates for chemotherapy at all). Thus, it remains unclear what the radiotherapy dose in this patient population should be and whether doses above 60 Gy in the absence of concurrent chemotherapy are beneficial or not. Although RTOG 0617 demonstrated higher-than-expected rates of survival, even with standard-dose radiotherapy, rates of intrathoracic tumor control with 60 Gy are suboptimal.4 Thus, other strategies to improve local tumor control by escalating the radiotherapy dose have been pursued. They include increasing the dose to subportions of the tumor volume based on mid-treatment positron emission tomography scans (RTOG 1106/ACRIN 6697; http://clinicaltrials.gov, NCT01507428) or escalating the radiotherapy dose to the point that the expected risk of toxicity is the same across patients—ie, isotoxic treatment (IDEAL-CRT; http://public.ukcrn.org. uk, UKCRN ID 6961). Proton therapy can often minimize the radiotherapy dose to organs at risk, in particular the heart, lungs, and esophagus, and whether this translates to a clinical benefit over conventional photon therapy is currently being investigated in the RTOG 1308 randomized trial.

Pre- and Postoperative Radiotherapy The guidelines recommend management by a multidisciplinary team, evaluating each individual patient for resectability and taking into account technical feasibility and medical co-

morbidities. Whether to utilize radiotherapy in the preoperative vs postoperative setting remains an unanswered question. The optimal approach for an individual patient will depend on the expertise of the treatment team as well as the tumor location, size, and involvement of adjacent structures. For superior sulcus tumors, preoperative chemoradiotherapy is recommended, since complete resection and long-term local tumor control are likely even more critical given the proximity to the brachial plexus and severe morbidity of a potential local recurrence.5 There is substantial evidence suggesting a benefit of postoperative radiotherapy in patients with completely resected N2 disease, but not with N0-1 disease. Whether postoperative radiotherapy is indicated in patients with N2 involvement at presentation, but not after response to induction chemotherapy, remains controversial.6 Currently, there is no good evidence for the optimal design of the target area for postoperative radiotherapy. We typically treat the involved lymph node stations as well as the ipsilateral hilum and bronchial stump. The contouring guidelines of an ongoing European randomized trial of postoperative radiotherapy (Lung ART; http://clinicaltrials.gov, NCT00410683) recommend treating one level above and below the involved nodal levels.7 It is hoped that this trial will provide level 1 evidence on the benefit of postoperative radiotherapy in N2 disease and further insight into which factors determine the outcome of patients undergoing postoperative radiotherapy.

Future Outlook The future management of locally advanced NSCLC with radiotherapy will undoubtedly have to take into account the advances in our understanding of the genomic changes in NSCLC. Tyrosine kinase inhibitors against sensitizing EGFR mutations and ALK rearrangement have demonstrated the effectiveness of targeted therapies in extending progression-free and overall survival. RTOG 1306 is one of the first studies to incorporate these therapies in combination with concurrent chemoradiotherapy. More studies will be needed to determine how to maximize the benefit of targeted therapies and radiotherapy in selected patient populations. Lastly, immunotherapy has resulted

in impressive and prolonged responses in advanced NSCLC. There is increasing evidence that radiotherapy may serve as an in situ vaccine to expose tumor antigens and thus may have synergistic effects in combination with immunotherapy. Studies investigating immunotherapy in the neoadjuvant or adjuvant setting in locally advanced NSCLC have been initiated and raise hopes for significant improvement in the outcomes of patients with this diagnosis. n Disclosure: Dr. Rimner potential conflicts of interest.

reported

no

References 1. Bezjak A, Temin S, Franklin G, et al: Definitive and adjuvant radiotherapy in locally advanced non-small-cell lung cancer: American Society of Clinical Oncology Clinical Practice guideline endorsement of the American Society for Radiation Oncology evidence-based clinical practice guideline. J Clin Oncol. 33:2100-2105, 2015. 2. Woolf SH, Grol R, Hutchinson A, et al: Clinical guidelines: Potential benefits, limitations, and harms of clinical guidelines. BMJ 318:527-530, 1999. 3. Bradley JD, Paulus R, Komaki R, et al: Standard-dose versus high-dose conformal radiotherapy with concurrent and consolidation carboplatin plus paclitaxel with or without cetuximab for patients with stage IIIA or IIIB nonsmall-cell lung cancer (RTOG 0617): A randomised, two-by-two factorial phase 3 study. Lancet Oncol 16:187-199, 2015. 4. Machtay M, Paulus R, Moughan J, et al: Defining local-regional control and its importance in locally advanced non-small cell lung carcinoma. J Thorac Oncol 7:716-722, 2012. 5. Rusch VW, Giroux DJ, Kraut MJ, et al: Induction chemoradiation and surgical resection for superior sulcus nonsmall-cell lung carcinomas: Long-term results of Southwest Oncology Group Trial 9416 (Intergroup Trial 0160). J Clin Oncol 25:313-318, 2007. 6. Amini A, Lou F, Correa AM, et al: Predictors for locoregional recurrence for clinical stage III-N2 non-small cell lung cancer with nodal downstaging after induction chemotherapy and surgery. Ann Surg Oncol 20:1934-1940, 2013. 7. Spoelstra FO, Senan S, Le Péchoux C, et al: Variations in target volume definition for postoperative radiotherapy in stage III non-small-cell lung cancer: Analysis of an international contouring study. Int J Radiat Oncol Biol Phys 76:1106-1113, 2010.


For reliability and quality,

our roots go deep

At Amgen, we pour commitment, passion, and a drive for perfection into every biologic medicine we make. From innovative biotechnology to extensive experience in biologic manufacturing, see how Amgen strives to deliver on its commitment to your patients.

Take a deeper look at our reliability and quality visit biotechnologybyamgen.com Download the LAYAR app on your smartphone and scan this page. Š2014 Amgen Inc. All rights reserved. 80012-R2-V1

Our Roots Go Deep


The ASCO Post  |   JULY 25, 2015

PAGE 46

Direct From ASCO

2015 Palliative Care in Oncology Symposium to Highlight the Science of Symptom Management

F

or the past few decades, ASCO has led efforts to integrate palliative care into all phases of cancer treatment. Through numerous educational programs, advocacy efforts, and most recently, the first annual Palliative Care in Oncology Symposium, ASCO has championed the idea that palliative care, which includes symptom management, psychosocial counseling, and discussion about treatment goals and expectations, must be a service available for individuals with serious illness not only at the end of life, but from the moment of diagnosis. Following the successes of the inaugural Palliative Care in Oncology Symposium last year (see infographic), the 2015

Jennifer S. Temel, MD

Symposium will be held on October 9–10, 2015, in Boston, and will strongly emphasize the science underlying palliative care. The Symposium will continue to serve as a platform for research on the social, spiritual, and communications-oriented aspects of palliative care and is again cosponsored by ASCO, the American Academy of Hospice and Palliative Medicine, the American Society for Radiation Oncology, and the Multinational Association of Supportive Care in Cancer. Jennifer S. Temel, MD, Chair of the 2015 Palliative Care in Oncology Steering Committee, as well as Clinical Director of Thoracic Oncology at Massachusetts General Hospital, described the research that will be presented at the 2015 Symposium. “One of our primary goals for this year’s meeting is to focus on the science of symptoms in patients with cancer. We have added two tracks covering both the science and treatment of two common and distressing symptoms in patients with cancer: pain and cachexia,” she said. “We will also have ample opportunity to discuss and review novel research in the field, ranging from survivorship to end-of-life care.” Sessions will more fully integrate both the latest scientific research and valuable best practices regarding difficult conversations with patients about priorities

throughout treatment and into the end of life. In keeping with last year, symptom management will be discussed, but this year there will also be discussion of the biologic mechanisms of symptoms. Several sessions will follow a topic from presentation of the latest laboratory research, through the effects of those data on clinical care, to information about psychological drivers for patients and their families. A new case report series, “Putting the Person in Personalized Medicine,” has been added at the end of some sessions to better illustrate the session information in the context of the reality of patient care and the emotions that often accompany these issues. In addition, a session on nonpharmacologic interventions for pain, such as massage and music therapy, is being offered concurrently with a session focusing on the more translational mechanisms and management of pain. New programmatic features like these demonstrate mindfulness of the whole-patient perspective and allow the diverse audience members to better tailor their meeting experience. The 2015 Symposium will also feature expanded opportunities for networking through such programs as the Fellows and Junior Faculty Lunch and expert-led Poster Walks. Meet the Faculty Networking Roundtables will provide an opportunity for attendees to network with colleagues across specialties, as well as with faculty and other experts in the field in a casual setting. In addition, a new, innovative approach to mentorship—Expert Office Hours—will allow for one-on-one mentoring conversations with faculty. The Symposium will reprise one of the most popular programs at the 2014 Symposium—Death Cafés—forums that began in Europe and that allow people to engage in open, informal discussions about an often difficult-to-discuss topic: the end of life. Some events require advance registration, and ticketed events will allow seating for non–ticket holders on a first-come, first-served basis. The early registration and housing deadline for the 2015 Palliative Care in Oncology Symposium is Wednesday, September 2, at 11:59 PM (EDT). Visit www.pallonc.org register, reserve housing, and view program information. n Selected portions reprinted from ASCO Daily News. © American Society of Clinical Oncology. “ASCO Leads the Way in Innovative Palliative Care Education.” https:// am.asco.org/asco-leads-way-innovative-palliative-care-education. All rights reserved.

INAUGURAL

Palliative Care in Oncology Symposium pallonc.org

This live educational event emphasized the integration of palliative care across the cancer care continuum. Attendance at the 2014 meeting was nearly double what was expected for this first offering.

An impressive

97%

of attendees

who provided evaluations said they would recommend this meeting to their colleagues.

Because of the Symposium, attendees will make the most changes to their practice and/or research in the following areas: Patient Care Communication Strategies Clinical Trial Participation Therapeutics

16% 14%

63% 62%

ATTENDEE FEEDBACK “This was one of the most valuable and enriching conferences of my career.” “I learned a great deal about how palliative care can be successfully integrated into the dayto-day operations of a large academic cancer center. I have so much to share with my social work, nursing, and physician colleagues.”

TWITTER ENGAGEMENT: #PALLONC 497 users sent 3,703 tweets, which garnered 7,182,424 potential impressions. During the meeting,

2014 SCIENTIFIC HIGHLIGHTS Early findings from a study of 319 families suggest that the use of a telephone-based system that monitors symptoms and provides health coaching to caregivers leads to decreased symptoms among patients and their caregivers in the final weeks of life (Abstract 85). An email trigger in the outpatient oncology clinic is highly effective in promoting advance care planning conversations for trained clinicians (Abstract 84). 79% of the conversations occurred after one trigger, and 92% occurred by the second trigger. A fully-integrated, inpatient co-rounding partnership between palliative care specialists and oncologists at Duke University Hospital (Abstract 3) resulted in: A statistically significant decrease in mean length of stay (p = 0.02) from 4.51 days to 4.17 days, Readmission rates decreased by 15% (for 7 days, p = 0.03) and 23% (for 30 days, p = 0.05), ICU transfers decreased by 15%.


ASCOPost.com  |   JULY 25, 2015

PAGE 47

Direct From ASCO

ASCO, Conquer Cancer Foundation Congratulate 2015 Grant and Award Recipients

T

he Conquer Cancer Foundation of ASCO presented more than $6.7 million in grants and awards to more than 200 promising oncology researchers at the 2015 ASCO Annual Meeting. The Conquer Cancer Foundation and ASCO congratulate the recipients on their contributions to the field of oncology and offer profound thanks to the generous organizations that supported these awards. To see the complete list of grant and award recipients, visit conquercancerfoundation.org/grantrecipients and conquercancerfoundation.org/awardrecipients.

Comparative Effectiveness Research Professorship in Breast Cancer The Comparative Effectiveness Research Professorship in Breast Cancer provides flexible funding to outstanding researchers who have made, and are continuing to make, significant contributions that have changed the direction of cancer research and who provide mentorship to junior researchers. The professorship is supported by the Breast Cancer Research Foundation. • Dawn L. Hershman, MD, MS, Columbia University Medical Center

supported by the Breast Cancer Research Foundation. • Priyanka Sharma, MD, University of Kansas Medical Center

Career Development Awards The Career Development Award provides funding to clinical investigators who have received their initial

Trim: 7.625 x 10.5

Coming soon…

Dawn L. Hershman, MD, MS, 2015 Comparative Effectiveness Research Professorship in Breast Cancer Recipient

Advanced Clinical Research Award in Breast Cancer The Advanced Clinical Research Award funds investigators in their fourth to ninth year of faculty appointment, who are committed to clinical cancer research in an area not currently funded. The award is

faculty appointment to establish an independent clinical cancer research program. This year’s recipients will each re-

Learn about currently available AstraZeneca products at astrazeneca-us.com. ©2015 AstraZeneca. All rights reserved. 3121800 4/15

continued on page 48


The ASCO Post  |   JULY 25, 2015

PAGE 48

Direct From ASCO 2015 Grant Recipients continued from page 47

ceive a 3-year grant totaling $200,000: • Sandra P. D’Angelo, MD, Memorial Sloan Kettering Cancer Center • Ayca Gucalp, MD, Memorial Sloan Kettering Cancer Center • Siwen Hu-Lieskovan, MD, PhD, David Geffen School of Medicine at UCLA • Douglas B. Johnson, MD, Vanderbilt University Medical Center • David Margel, MD, PhD, Rabin Medical Center • Sean Matthew McBride, MD, MPH, Memorial Sloan Kettering Cancer Center • Katherine Reeder-Hayes, MD, MBA, University of North Carolina at Chapel Hill • Rizwan Romee, MD, Washington University • Florian R. Schroeck, MD, MS, White River Junction VA Medical Center/The Dartmouth Institute • Vamsidhar Velcheti, MD, The Cleveland Clinic • Jason R. Westin, MD, The University of Texas MD Anderson Cancer Center Career Development Awards are supported by Astellas, AstraZeneca, the Breast Cancer Research Foundation®, the Conquer Cancer Foundation Mission Endowment, Genentech BioOncology, Lilly, and Takeda Oncology.

Young Investigator Awards The Young Investigator Award provides funding to promising investigators to encourage and promote quality research in clinical oncology. This year’s 58 awardees will each receive a 1-year grant of $50,000. Young Investigator Awards are supported by AbbVie, Inc.; the American Association for Cancer Research; Amgen; Anonymous; the ASCO and Conquer Cancer Foundation Boards of Directors; the ASCO State Affiliate Council and ASCO Clinical Practice Committee; AstraZeneca; Boehringer Ingelheim Pharmaceuticals, Inc.; the Breast Cancer Research Foundation; Celgene Corporation; The Cholangiocarcinoma Foundation; the Conquer Cancer Foundation; Conquer Cancer Foundation Women Who Conquer Cancer; Eisai Inc.; Ethicon; Genentech BioOncology; Gilead Sciences, Inc.; Incyte Corporation; Janssen Biotech, Inc.; the John and Elizabeth Leonard Family Foundation; Journal of Clinical Oncology; the Kidney Cancer Association; Lilly; Merck & Co., Inc.; Novartis Oncology; Onyx Pharmaceuticals; Susan K. Parsons, MD, and Walter Armstrong; Pfizer Oncology; the Reid R. Sacco Adolescent and

Young Adult Alliance; Roche; the Scopus Foundation; Young A. Sohn and Mark Armenante; the Strike 3 Foundation; Takeda Oncology; The WWWW Foundation Inc. (QuadW); and The Sarcoma Fund of the QuadW Foundation of Communities Foundation of Texas.

International Development and Education Award The International Development and Education Award provides support for early-career oncologists in low- and middle-income countries to further their knowledge and careers and establish strong long-term relationships with leading ASCO members who serve as scientific mentors. This year’s 20 awardees attended the 2015 ASCO Annual Meeting and visited their mentor’s institution, where they gained knowledge and skills to share with their peers back home. Notably, this is the first year IDEAs supported oncologists from Ghana, Mauritius, and South Africa. International Development and Education Awards are supported by AstraZeneca, Bayer HealthCare Pharmaceuticals Inc., the Conquer Cancer Foundation, Roche, and Takeda Oncology.

Medical Student Rotation for Underrepresented Populations The Medical Student Rotation for Underrepresented Populations provides 8- to 10-week clinical or clinical research oncology rotations for U.S. medical students from populations underrepresented in medicine who are interested in pursuing oncology as a career. This year, seven medical students were selected to participate in the program: • Oladapo Adeniran, University of Illinois at Chicago • Cecil Benitez, Stanford University • Mario Martinez, University of Illinois at Chicago • Angel Moran, University of California, Davis • Dionisia Quiroga, Michigan State University • Elisa Quiroz, Ponce Health Sciences University • Jasmine Smith, University of South Carolina School of Medicine Greenville The Medical Student Rotation for Underrepresented Populations is supported by the Conquer Cancer Foundation Mission Endowment, Eisai Inc., Genentech BioOncology, and Lilly.

Resident Travel Award for Underrepresented Populations The Resident Travel Award for Underrepresented Populations provides

financial support for residents from underrepresented populations to attend the ASCO Annual Meeting. This year eight residents were selected to participate in the program: • Olufunke Akinbobuyi, MD, Morehouse School of Medicine • Idoroenyi Amanam, MD, St. Mary Medical Center • Ebenezer Appah, MD, Meharry Medical College • Frederick Doamekpor, MD, Morehouse School of Medicine • Nancy Osuji-Oduh, MD, Morehouse School of Medicine • Linnea Perkins, MD, Ochsner Clinic • Sonya Reid-Lawrence, MD, Meharry Medical College • Oluchi Ukaegbu, MD, Vanderbilt University Medical Center The Resdient Travel Award is supported by Janssen Biotech, Inc. and Novartis Oncology.

Clinical Trials Participation Awards The Clinical Trials Participation Award recognizes and promotes high-quality clinical research sites with the intent to increase the awareness of, and participation in, clinical trials among physicians. The awards are presented to community-based practices participating in clinical trials at an exceptional level. The Coalition of Cancer Cooperative Groups has supported these awards for 13 consecutive years. • Cancer Care Specialists of Central Illinois–Decatur, Illinois • Blue Ridge Cancer Care–Roanoke, Virginia • Oncology Hematology Care, Inc.– Cincinnati, Ohio • Scottsdale Healthcare–Scottsdale, Arizona • The Division of Gynecologic Oncology at St. Joseph’s Hospital and Medical Center– Phoenix, Arizona • Cone Health Cancer Center– Greensboro, North Carolina

Merit Awards for High-Quality Research The Conquer Cancer Foundation distributed 99 Merit Awards to oncology fellows who submitted high-quality research for presentation at the 2015 ASCO Annual Meeting, four of which were Special Merit Awards for the highest-ranking abstracts in select categories. The 2015 Special Merit Award recipients are: • Mark Applebaum, MD, University of Chicago, Brigid Leventhal Special Merit Award

• Maria-Jose de Miguel-Luken, MD, The Institute of Cancer Research and The Royal Marsden Hospital, Bradley Stuart Beller Special Merit Award, (supported by the Bradley Stuart Beller Endowment Fund) • Sébastien Héritier, MD, Versailles University & APHP, Trousseau Hospital, James B. Nachman ASCO Junior Faculty Award in Pediatric Oncology • Ryan Nipp, MD, Dana-Farber Cancer Institute/Harvard Cancer Center, Pain and Symptom Management Research Merit Award

Oncology Trainee Travel Awards The Oncology Trainee Travel Award supports the continuing education and professional development of trainee oncologists by providing individual travel grants to defray travel expenses for attending the ASCO Annual Meeting. In 2015, the Conquer Cancer Foundation awarded 64 Trainee Travel Awards, supported by Takeda Oncology.

2015 Long-Term International Fellowship The Long-Term International Fellowship provides early-career oncologists in low- to middle-income countries the support and resources needed to advance their training by deepening their relationship with a mentor from the United States, Canada, or Europe and his or her institution. This year’s recipients are: • Malek Baassiri, MD HOME INSTITUTION: American University of Beirut Medical Center (Lebanon) U.S. SPONSORING INSTITUTION:

St. Jude Children’s Research Hospital MENTOR: Melissa M. Hudson, MD • Kibrom Hagos, MD HOME INSTITUTION: Black Lion Hospital, Addis Ababa University (Ethiopia) U.S. SPONSORING INSTITUTION:

Oregon Health and Science University MENTOR: Charles R. Thomas, MD, FASCO The Long-Term International Fellowship is supported by the Conquer Cancer Foundation, Gilead Sciences, Inc., and Roche. n Selected portions reprinted from ASCO Connection. © American Society of Clinical Oncology. “ASCO, Conquer Cancer Foundation Congratulate 2015 Grant and Award Recipients.” ASCO Connection, July 2015: 30-33. All rights reserved.


ASCOPost.com  |   JULY 25, 2015

PAGE 49

Direct From ASCO

ASCO Urges Evidence-Based, Flexible Drug Handling Requirements in Comments on USP 800

A

SCO has responded to changing health-care standards by submitting comments to the U.S. Pharmacopeial Convention (USP) on the revised version of its proposed General Chapter 800, “Hazardous Drugs— Handling in Healthcare Settings.” While ASCO notes some positive changes to the proposal, the Society urges further revisions to avoid hardships for community-based oncology practices. In a letter to USP, Robin Zon, MD, FACP, FASCO, Chair of the ASCO Task Force on Safe Handling of Che-

Robin Zon, MD, FACP, FASCO

motherapy, urged the organization to provide flexibility to communitybased oncology practices in enhancing workplace safety. The proposed General Chapter 800 contains poten-

tially burdensome requirements for oncology practices when handling toxic materials. Moreover, there is no scientific or clinical evidence demonstrating that such requirements would have a positive impact on patients or practice staff. ASCO has offered to work with USP on standards that are practical for ensuring the safe handling of chemotherapy drugs. ASCO has previously worked with organizations to improve worker safety around dangerous drugs and issued a joint statement with the Hematology/Oncology Pharmacy Associa-

tion and the Oncology Nursing Society. If and when USP releases a final General Chapter 800, ASCO will work with the State Affiliates to closely monitor any state-level action to adopt the recommendations. Please follow ASCO in Action (www.asco.org/advocacy) as this issue unfolds. To read ASCO’s letter, please go to: http://www.asco.org/sites/ w w w.asco.org/f i les/may_2015_ asco_usp_800_comments.pdf. n © 2015. American Society of Clinical Oncology. All rights reserved.

Order Cancer.Net Fact Sheets for Your Practice

Comprehensive Oncology Practice Business Guide Now Available

B

A

e sure to check out Cancer.Net’s ASCO Answers fact sheets on a variety of topics, including diagnosis and treatment options, side effects, and more than 25 cancer types. These one-page (front and back) introduc-

tions include an overview, illustrations, terms to know, and questions to ask the doctor. Find these and a library of titles at www.cancer.net/factsheets, or order copies for your practice at www.cancer. net/estore. Shipping is free, and ASCO members save 20%. n © 2015. American Society of Clinical Oncology. All rights reserved.

Together, we’re taking it down.

While nearly 1 in 2 people will get cancer in their lifetime, 2 in 3 will survive it.* And that’s a profound testament to the progress you’ve helped forge as we work collectively to make it harder for cancer to survive.

Join The Campaign to Conquer Cancer at conquer.org/progress. #ConquerCancer * CancerProgress.Net. Progress & Timeline. Timeline. Major Milestones Against Cancer. Available at: http://cancerprogress.net/timeline/major-milestones-against-cancer. Accessed April 7, 2015.

SCO has released Practical Tips for the Oncology Practice 6th Edition, a comprehensive business and management resource for oncology practices. Recognizing the increasing burdens that oncology practices face in today’s healthcare delivery system, ASCO is releasing this guide with new information on Medicare reimbursements, ICD9 and ICD-10 codes, and safe drug handling guidelines and requirements. This guide spans 26 chapters that provide information that every oncology practice needs: new insight into the coverage and billing of oncology-related care; recent updates to Medicare regulations and an overview of the Medicare appeals process; and the latest information on Physician Quality Reporting Programs. The 6th edition will only be available to download as an eBook from the ASCO University for $275. This

6 T H

E D I T I O N

Coding, Medicare Coverage, and Regulatory Policies: A Guide for Oncology Practices

new electronic format allows for enhanced search functions to easily find and access key information. ASCO members will receive a 20% discount when they purchase the download. n © 2015. American Society of Clinical Oncology. All rights reserved.

Visit The ASCO Post website at ASCOPost.com


ARE YOU TESTING FOR RAS ? Amgen is committed to the integration of RAS testing into routine clinical practice, which may help oncologists select an appropriate first-line treatment plan for their patients with mCRC.

KRAS and NRAS are part of the RAS family of oncogenes1

KRAS

EXON 2

NRAS

EXON 2

12

12

13

13

EXON 3 59

61

EXON 3 59

61

EXON 4 117 146

EXON 4 117 146

• A mutation on the RAS gene can continuously activate intracellular signaling, leading to increased cell growth and proliferation. The EGFR pathway is always “turned on,” and the cell keeps growing and dividing regardless of anti-EGFR therapy2 • Patients with mutant-type RAS tumors should not be treated with Vectibix®2 • In addition, in an exploratory subgroup analysis of Study 3,* overall survival was shorter in patients with RAS-mutant mCRC who received Vectibix® and FOLFOX versus FOLFOX alone2

Indication Vectibix® is indicated for the treatment of patients with wild-type KRAS (exon 2 in codons 12 or 13) metastatic colorectal cancer (mCRC) as determined by an FDA-approved test for this use: • As first-line therapy in combination with FOLFOX • As monotherapy following disease progression after prior treatment with fluoropyrimidine-, oxaliplatin-, and irinotecan-containing chemotherapy Limitation of Use Vectibix® is not indicated for the treatment of patients with RAS-mutant mCRC or for whom RAS mutation status is unknown.

Important Safety Information WARNING: DERMATOLOGIC TOXICITY Dermatologic Toxicity: Dermatologic toxicities occurred in 90% of patients and were severe (NCI-CTC grade 3 and higher) in 15% of patients receiving Vectibix® monotherapy [see Dosage and Administration (2.3), Warnings and Precautions (5.1), and Adverse Reactions (6.1)]. • In Study 1, dermatologic toxicities occurred in 90% of patients and

were severe (NCI-CTC grade 3 and higher) in 15% of patients with mCRC receiving Vectibix®. The clinical manifestations included, but were not limited to, acneiform dermatitis, pruritus, erythema, rash, skin exfoliation, paronychia, dry skin, and skin fissures. • Monitor patients who develop dermatologic or soft tissue toxicities while receiving Vectibix® for the development of inflammatory or infectious sequelae. Life-threatening and fatal infectious complications including necrotizing fasciitis, abscesses, and sepsis have been observed in patients treated with Vectibix®. Life-threatening and fatal bullous mucocutaneous disease with blisters, erosions, and skin sloughing has also been observed in patients treated with Vectibix®. It could not be determined whether these mucocutaneous adverse reactions were directly related to EGFR inhibition or to idiosyncratic immune-related effects (eg, Stevens-Johnson syndrome or toxic epidermal necrolysis). Withhold or discontinue Vectibix® for dermatologic or soft tissue toxicity associated with severe or life-threatening inflammatory or infectious complications. Dose modifications for Vectibix® concerning dermatologic toxicity are provided in the product labeling.

• Vectibix® is not indicated for the treatment of patients with colorectal

cancer that harbor somatic mutations in exon 2 (codons 12 and 13), exon 3 (codons 59 and 61), and exon 4 (codons 117 and 146) of either KRAS or NRAS and hereafter is referred to as“RAS”.

• Retrospective subset analyses across several randomized clinical trials

were conducted to investigate the role of RAS mutations on the clinical effects of anti-EGFR-directed monoclonal antibodies (panitumumab or cetuximab). Anti-EGFR antibodies in patients with tumors containing RAS mutations resulted in exposing those patients to anti-EGFR- related adverse reactions without clinical benefit from these agents.

• Additionally, in Study 3, 272 patients with RAS-mutant mCRC tumors

received Vectibix® in combination with FOLFOX and 276 patients received FOLFOX alone. In an exploratory subgroup analysis, OS was shorter (HR = 1.21, 95% CI: 1.01–1.45) in patients with RAS-mutant mCRC who received Vectibix® and FOLFOX versus FOLFOX alone.


Important Safety Information (continued) • Progressively decreasing serum magnesium levels leading to severe

(grade 3–4) hypomagnesemia occurred in up to 7% of patients in Study 2. Monitor patients for hypomagnesemia and hypocalcemia prior to initiating Vectibix® treatment, periodically during Vectibix® treatment, and for up to 8 weeks after the completion of treatment. Other electrolyte disturbances, including hypokalemia, have also been observed. Replete magnesium and other electrolytes as appropriate.

• In Study 1, 4% of patients experienced infusion reactions and 1% of

patients experienced severe infusion reactions (NCI-CTC grades 3–4). Infusion reactions, manifesting as fever, chills, dyspnea, bronchospasm, and hypotension, can occur following Vectibix® administration. Fatal infusion reactions occurred in postmarketing experience. Terminate the infusion for severe infusion reactions.

• Severe

diarrhea and dehydration, leading to acute renal failure and other complications, have been observed in patients treated with Vectibix® in combination with chemotherapy.

• Fatal

and non-fatal cases of interstitial lung disease (ILD) (1%) and pulmonary fibrosis have been observed in patients treated with Vectibix®. Pulmonary fibrosis occurred in less than 1% (2/1467) of patients enrolled in clinical studies of Vectibix®. In the event of acute onset or worsening of pulmonary symptoms, interrupt Vectibix® therapy. Discontinue Vectibix® therapy if ILD is confirmed.

• In

patients with a history of interstitial pneumonitis or pulmonary fibrosis, or evidence of interstitial pneumonitis or pulmonary fibrosis, the benefits of therapy with Vectibix® versus the risk of pulmonary complications must be carefully considered.

• Exposure

to sunlight can exacerbate dermatologic toxicity. Advise patients to wear sunscreen and hats and limit sun exposure while receiving Vectibix®.

• Keratitis

and ulcerative keratitis, known risk factors for corneal perforation, have been reported with Vectibix® use. Monitor for evidence of keratitis or ulcerative keratitis. Interrupt or discontinue Vectibix® for acute or worsening keratitis.

• In

an interim analysis of an open-label, multicenter, randomized clinical trial in the first-line setting in patients with mCRC, the addition of Vectibix® to the combination of bevacizumab and chemotherapy resulted in decreased OS and increased incidence of NCI-CTC grade 3–5 (87% vs 72%) adverse reactions. NCI-CTC grade 3–4 adverse reactions occurring at a higher rate in Vectibix®-treated patients included rash/acneiform dermatitis (26% vs 1%), diarrhea (23% vs 12%), dehydration (16% vs 5%; primarily occurring in patients

with diarrhea), hypokalemia (10% vs 4%), stomatitis/mucositis (4% vs < 1%), and hypomagnesemia (4% vs 0). • NCI-CTC grade 3–5 pulmonary embolism occurred at a higher rate

in Vectibix®-treated patients (7% vs 3%) and included fatal events in three (< 1%) Vectibix®-treated patients.

• As

a result of the toxicities experienced, patients randomized to Vectibix®, bevacizumab, and chemotherapy received a lower mean relative dose intensity of each chemotherapeutic agent (oxaliplatin, irinotecan, bolus 5-FU, and/or infusional 5-FU) over the first 24 weeks on study, compared with those randomized to bevacizumab and chemotherapy.

• Advise patients of the need for adequate contraception in both males

and females while receiving Vectibix® and for 6 months after the last dose of Vectibix® therapy. Vectibix® may be transmitted from the mother to the developing fetus, and has the potential to cause fetal harm when administered to pregnant women.

• Because

many drugs are excreted into human milk and because of the potential for serious adverse reactions in nursing infants from Vectibix®, a decision should be made whether to discontinue nursing or to discontinue the drug, taking into account the importance of the drug to the mother. If nursing is interrupted, it should not be resumed earlier than 2 months following the last dose of Vectibix®.

• Women

who become pregnant during Vectibix® treatment are encouraged to enroll in Amgen’s Pregnancy Surveillance Program. Women who are nursing during Vectibix® treatment are encouraged to enroll in Amgen’s Lactation Surveillance Program. Patients or their physicians should call 1-800-77-AMGEN (1-800-772-6436) to enroll.

• In Study 1, the most common adverse reactions (≥ 20%) with Vectibix®

were skin rash with variable presentations, paronychia, fatigue, nausea, and diarrhea. The most common (> 5%) serious adverse reactions in the Vectibix® arm were general physical health deterioration and intestinal obstruction.

• In

Study 3, the most commonly reported adverse reactions (≥ 20%) in patients with wild-type KRAS mCRC receiving Vectibix® (6 mg/kg every 2 weeks) and FOLFOX therapy (N = 322) were diarrhea, stomatitis, mucosal inflammation, asthenia, paronychia, anorexia, hypomagnesemia, hypokalemia, rash, acneiform dermatitis, pruritus, and dry skin. Serious adverse reactions (≥ 2% difference between treatment arms) in Vectibix® -treated patients with wild-type KRAS mCRC were diarrhea and dehydration.

Visit Vectibix.com to learn more

*A phase 3, open-label, randomized, multicenter study of 1,183 previously untreated patients with mCRC who were treated with Vectibix® Q2W + FOLFOX or FOLFOX Q2W alone.2 EGFR = epidermal growth factor receptor; mCRC = metastatic colorectal cancer; Q2W = every two weeks. References: 1. Douillard J-Y, Oliner KS, Siena S, et al. Panitumumab-FOLFOX4 treatment and RAS mutations in colorectal cancer. N Engl J Med. 2013;369:1023–1034. 2. Vectibix® (panitumumab) prescribing information, Amgen. Please see Brief Summary of full Prescribing Information on adjacent pages.

©2015 Amgen Inc. All rights reserved. 05/15 USA-945-105509


KING SIZE (6 pt condensed type) S:9.25”

Vectibix® (panitumumab) BRIEF SUMMARY OF FULL PRESCRIBING INFORMATION WARNING: DERMATOLOGIC TOXICITY Dermatologic Toxicity: Dermatologic toxicities occurred in 90% of patients and were severe (NCI-CTC grade 3 and higher) in 15% of patients receiving Vectibix® monotherapy [see Dosage and Administration (2.3), Warnings and Precautions (5.1), and Adverse Reactions (6.1)].

VECT15CDLA0437_B_Vectibix_BS_9.25x13_v23_Mar2015_r11_MBS.indd 1

Study 1

SYSTEM ORGAN CLASS Preferred Term

Vectibix® Plus Best Supportive Care (N = 229)

Best Supportive Care (N = 234)

Any Grade n (%)

Grade 3-4 n (%)

Any Grade n (%)

Grade 3-4 n (%)

1 (< 1)

EYE DISORDERS Growth of eyelashes

13 (6)

GASTROINTESTINAL DISORDERS Nausea

52 (23)

2 (< 1)

37 (16)

Diarrhea

49 (21)

4 (2)

26 (11)

Vomiting

43 (19)

6 (3)

28 (12)

Stomatitis

15 (7)

2 (< 1)

2 (< 1)

GENERAL DISORDERS AND ADMINISTRATION SITE CONDITIONS Fatigue

60 (26)

10 (4)

34 (15)

Mucosal inflammation

15 (7)

1 (< 1)

2 (< 1)

57 (25)

4 (2)

7 (3)

INFECTIONS AND INFESTATIONS Paronychia RESPIRATORY, THORACIC, AND MEDIASTINAL DISORDERS Dyspnea

41 (18)

12 (5)

30 (13)

Cough

34 (15)

1 (< 1)

17 (7)

Erythema

150 (66)

13 (6)

2 (< 1)

Pruritus

132 (58)

6 (3)

4 (2)

Acneiform dermatitis

131 (57)

17 (7)

2 (< 1)

Rash

51 (22)

3 (1)

2 (< 1)

Skin fissures

45 (20)

3 (1)

1 (< 1)

Exfoliative rash

41 (18)

4 (2)

Acne

31 (14)

3 (1)

Dry skin

23 (10)

Nail disorder

22 (10)

Skin exfoliation

21 (9)

2 (< 1)

Skin ulcer

13 (6)

1 (< 1)

8 (3)

SKIN AND SUBCUTANEOUS TISSUE DISORDERS

Adverse reactions in Study 1 that did not meet the threshold criteria for inclusion in Table 1 were conjunctivitis (4.8% vs < 1%), dry mouth (4.8% vs 0%), pyrexia (16.6% vs 13.2%), chills (3.1% vs < 1%), pustular rash (4.4% vs 0%), papular rash (1.7% vs 0%), dehydration (2.6% vs 1.7%), epistaxis (3.9% vs 0%), and pulmonary embolism (1.3% vs 0%). In Study 1, dermatologic toxicities occurred in 90% of patients receiving Vectibix®. Skin toxicity was severe (NCI-CTC grade 3 and higher) in 15% of patients. Ocular toxicities occurred in 16% of patients and included, but were not limited to, conjunctivitis (5%). One patient experienced an NCI-CTC grade 3 event of mucosal inflammation. The incidence of paronychia was 25% and was severe in 2% of patients [see Warnings and Precautions (5.1)]. In Study 1 (N = 229), median time to the development of dermatologic, nail, or ocular toxicity was 12 days after the first dose of Vectibix®; the median time to most severe skin/ocular toxicity was 15 days after the first dose of Vectibix®; and the median time to resolution after the last dose of Vectibix® was 98 days. Severe toxicity necessitated dose interruption in 11% of Vectibix ®-treated patients [see Dosage and Administration (2.3)]. Subsequent to the development of severe dermatologic toxicities, infectious complications, including sepsis, septic death, necrotizing fasciitis, and abscesses requiring incisions and drainage were reported. Vectibix® in Combination with FOLFOX Chemotherapy The most commonly reported adverse reactions (≥ 20%) in patients with wild-type KRAS mCRC receiving Vectibix® (6 mg/kg every 2 weeks) and FOLFOX therapy (N = 322) in Study 3 were diarrhea, stomatitis, mucosal inflammation, asthenia, paronychia, anorexia, hypomagnesemia, hypokalemia, rash, acneiform dermatitis, pruritus, and dry skin (Table 2). Serious adverse reactions (≥ 2% difference between treatment arms) in Vectibix®-treated patients with wild-type KRAS mCRC were diarrhea and dehydration. The commonly reported adverse reactions (≥ 1%) leading to discontinuation in patients with wild-type KRAS mCRC receiving Vectibix® were rash, paresthesia, fatigue, diarrhea, acneiform dermatitis, and hypersensitivity. One grade 5 adverse reaction, hypokalemia, occurred in a patient who received Vectibix®. Table 2: Adverse Reactions (≥ 5% Difference) Observed in Patients with Wild-type (WT) KRAS Tumors Treated with Vectibix® and FOLFOX Chemotherapy Compared to FOLFOX Chemotherapy Alone (Study 3) Vectibix® Plus FOLFOX (n = 322) SYSTEM ORGAN CLASS Preferred Term

FOLFOX Alone (n = 327)

Any Grade n (%)

Grade 3-4 n (%)

Any Grade n (%)

Grade 3-4 n (%)

58 (18)

5 (2)

10 (3)

Diarrhea

201 (62)

59 (18)

169 (52)

29 (9)

Stomatitis

87 (27)

15 (5)

42 (13)

1 (< 1)

EYE DISORDERS Conjunctivitis GASTROINTESTINAL DISORDERS

GENERAL DISORDERS AND ADMINISTRATION SITE CONDITIONS Mucosal inflammation

82 (25)

14 (4)

53 (16)

1 (< 1)

Asthenia

79 (25)

16 (5)

62 (19)

11 (3)

68 (21)

11 (3)

58 (18)

3 (< 1)

22 (7)

Anorexia

116 (36)

14 (4)

85 (26)

6 (2)

INFECTIONS AND INFESTATIONS Paronychia

Vectibix® Plus FOLFOX (n = 322) SYSTEM ORGAN CLASS Preferred Term

FOLFOX Alone (n = 327)

Any Grade n (%)

Grade 3-4 n (%)

Any Grade n (%)

Grade 3-4 n (%)

179 (56) 104 (32) 75 (23) 68 (21) 50 (16) 50 (16) 47 (15) 44 (14) 32 (10)

55 (17) 33 (10) 3 (< 1) 5 (2) 7 (2) 1 (< 1)

24 (7)

1 (< 1)

10 (3) 4 (1)

14 (4) 13 (4) 14 (4) 1 (< 1) 30 (9) 1 (< 1) 4 (1)

30 (9)

4 (1)

9 (3)

SKIN AND SUBCUTANEOUS TISSUE DISORDERS Rash Acneiform dermatitis Pruritus Dry skin Erythema Skin fissures Alopecia Acne Nail disorder Palmar-plantar erythrodysesthesia syndrome

2 (< 1)

Adverse reactions that did not meet the threshold criteria for inclusion in Table 2 were abdominal pain (28% vs 23%), localized infection (3.7% vs < 1%), cellulitis (2.5% vs 0%), hypocalcemia (5.6% vs 2.1%), and deep vein thrombosis (5.3% vs 3.1%). Infusion Reactions Infusional toxicity manifesting as fever, chills, dyspnea, bronchospasm or hypotension was assessed within 24 hours of an infusion during the clinical study. Vital signs and temperature were measured within 30 minutes prior to initiation and upon completion of the Vectibix ® infusion. The use of premedication was not standardized in the clinical trials. Thus, the utility of premedication in preventing the first or subsequent episodes of infusional toxicity is unknown. Across clinical trials of Vectibix® monotherapy, 3% (24/725) experienced infusion reactions of which < 1% (3/725) were severe (NCI-CTC grade 3-4). In one patient, Vectibix® was permanently discontinued for a serious infusion reaction [see Dosage and Administration (2.2, 2.3)]. Immunogenicity As with all therapeutic proteins, there is potential for immunogenicity. The immunogenicity of Vectibix® has been evaluated using two different screening immunoassays for the detection of binding anti-panitumumab antibodies: an acid dissociation bridging enzyme-linked immunosorbent assay (ELISA) detecting high-affinity antibodies and a Biacore ® biosensor immunoassay detecting both high- and low-affinity antibodies. For patients whose sera tested positive in screening immunoassays, an in vitro biological assay was performed to detect neutralizing antibodies. Monotherapy: The incidence of binding anti-panitumumab antibodies (excluding preexisting and transient positive patients) was 0.4% (5/1123) as detected by the acid dissociation ELISA and 3.2% (36/1123) as detected by the Biacore® assay. The incidence of neutralizing anti-panitumumab antibodies (excluding preexisting and transient positive patients) was 0.8% (9/1123). There was no evidence of altered pharmacokinetic or safety profiles in patients who developed antibodies to Vectibix®. In combination with chemotherapy: The incidence of binding anti-panitumumab antibodies (excluding preexisting positive patients) was 0.9% (12/1297) as detected by the acid dissociation ELISA and 0.7% (9/1296) as detected by the Biacore® assay. The incidence of neutralizing antipanitumumab antibodies (excluding preexisting positive patients) was 0.2% (2/1297). No evidence of an altered safety profile was found in patients who developed antibodies to Vectibix®. The detection of antibody formation is highly dependent on the sensitivity and specificity of the assay. Additionally, the observed incidence of antibody (including neutralizing antibody) positivity in an assay may be influenced by several factors, including assay methodology, sample handling, timing of sample collection, concomitant medications, and underlying disease. For these reasons, comparison of the incidence of antibodies to panitumumab with the incidence of antibodies to other products may be misleading. Postmarketing Experience The following adverse reactions have been identified during post-approval use of Vectibix ®. Because these reactions are reported in a population of uncertain size, it is not always possible to reliably estimate their frequency or establish a causal relationship to drug exposure. • Skin and subcutaneous tissue disorders: Skin necrosis, angioedema, life-threatening and fatal bullous mucocutaneous disease [see Boxed Warning, Dosage and Administration (2.3), and Warnings and Precautions (5.1)] • Immune system disorders: Infusion reaction [see Dosage and Administration (2.3) and Warnings and Precautions (5.4)] • Eye disorders: Keratitis/ulcerative keratitis [see Warnings and Precautions (5.8)] DRUG INTERACTIONS No formal drug-drug interaction studies have been conducted between Vectibix® and oxaliplatin or fluoropyrimidine. USE IN SPECIFIC POPULATIONS Pregnancy Pregnancy Category C There are no studies of Vectibix® in pregnant women. Reproduction studies in cynomolgus monkeys treated with 1.25 to 5 times the recommended human dose of panitumumab resulted in significant embryolethality and abortions; however, no other evidence of teratogenesis was noted in offspring [see Nonclinical Toxicology (13.3)]. Vectibix® should be used during pregnancy only if the potential benefit justifies the potential risk to the fetus. Based on animal models, EGFR is involved in prenatal development and may be essential for normal organogenesis, proliferation, and differentiation in the developing embryo. Human IgG is known to cross the placental barrier; therefore, panitumumab may be transmitted from the mother to the developing fetus, and has the potential to cause fetal harm when administered to pregnant women. Women who become pregnant during Vectibix® treatment are encouraged to enroll in Amgen’s Pregnancy Surveillance Program. Patients or their physicians should call 1-800-77-AMGEN (1-800-772-6436) to enroll. Nursing Mothers It is not known whether panitumumab is excreted into human milk; however, human IgG is excreted into human milk. Published data suggest that breast milk antibodies do not enter the neonatal and infant circulation in substantial amounts. Because many drugs are excreted into human milk and because of the potential for serious adverse reactions in nursing infants from Vectibix®, a decision should be made whether to discontinue nursing or to discontinue the drug, taking into account the importance of the drug to the mother. If nursing is interrupted, based on the mean half-life of panitumumab, nursing should not be resumed earlier than 2 months following the last dose of Vectibix® [see Clinical Pharmacology (12.3)]. Women who are nursing during Vectibix® treatment are encouraged to enroll in Amgen’s Lactation Surveillance Program. Patients or their physicians should call 1-800-77-AMGEN (1-800-772-6436) to enroll. Pediatric Use The safety and effectiveness of Vectibix® have not been established in pediatric patients. The pharmacokinetic profile of Vectibix® has not been studied in pediatric patients. Geriatric Use Of the 737 patients who received Vectibix® monotherapy in Study 1 and 2, 36% were 65 and over while 8% were 75 and over. No overall differences in safety or efficacy were observed in elderly patients (≥ 65 years of age) treated with Vectibix® monotherapy. Of the 322 patients in Study 3 who received Vectibix® plus FOLFOX, 128 (40%) were 65 and over while 8% were 75 and over. Patients older than 65 years of age experienced an increased incidence of serious adverse events (52% vs 36%) and an increased incidence of serious diarrhea (15% vs 5%) as compared to younger patients. OVERDOSAGE Doses up to approximately twice the recommended therapeutic dose (12 mg/kg) resulted in adverse reactions of skin toxicity, diarrhea, dehydration, and fatigue. Patient Counseling Information Advise patients to contact a healthcare professional for any of the following: • Skin and ocular/visual changes [see Boxed Warning, Dosage and Administration (2.3), Warnings and Precautions (5.1, 5.8), and Adverse Reactions (6.1, 6.3)] • Signs and symptoms of infusion reactions, including fever, chills, or breathing problems [see Dosage and Administration (2.3), Warnings and Precautions (5.4), and Adverse Reactions (6.1, 6.3)] • Diarrhea and dehydration [see Warnings and Precautions (5.5)] • Persistent or recurrent coughing, wheezing, dyspnea, or new-onset facial swelling [see Warnings and Precautions (5.6) and Adverse Reactions (6.1)] • Pregnancy or nursing [see Use in Specific Populations (8.1, 8.3)] Advise patients of the need for: • Periodic monitoring of electrolytes [see Warnings and Precautions (5.3)] • Limitation of sun exposure (use of sunscreen, wear hats) while receiving Vectibix® and for 2 months after the last dose of Vectibix® therapy [see Warnings and Precautions (5.7)] • Adequate contraception in both males and females while receiving Vectibix ® and for 6 months after the last dose of Vectibix® therapy [see Use in Specific Populations (8.1, 8.3)]

INVESTIGATIONS Weight decreased METABOLISM AND NUTRITION DISORDERS Hypomagnesemia

96 (30)

21 (7)

26 (8)

1 (< 1)

Hypokalemia

68 (21)

32 (10)

42 (13)

15 (5)

Dehydration

26 (8)

8 (2)

10 (3)

5 (2)

RESPIRATORY, THORACIC, AND MEDIASTINAL DISORDERS Epistaxis

46 (14)

30 (9)

This brief summary is based on the Vectibix ® Prescribing Information v23, 03/15. Vectibix® (panitumumab) Manufactured by: Amgen Inc. One Amgen Center Drive Thousand Oaks, CA 91320-1799 USA Patent: http://pat.amgen.com/vectibix/ © 2006-2015 Amgen Inc. All rights reserved. v23 03/15

4/27/15 6:12 PM

S:13”

INDICATIONS AND USAGE Metastatic Colorectal Cancer Vectibix® is indicated for the treatment of patients with wild-type KRAS (exon 2 in codons 12 or 13) metastatic colorectal cancer (mCRC) as determined by an FDA-approved test for this use: • As first-line therapy in combination with FOLFOX [see Clinical Studies (14.2)]. • As monotherapy following disease progression after prior treatment with fluoropyrimidine-, oxaliplatin-, and irinotecan-containing chemotherapy [see Clinical Studies (14.1)]. Limitation of Use Vectibix® is not indicated for the treatment of patients with RAS-mutant mCRC or for whom RAS mutation status is unknown [see Dosage and Administration (2.1), Warnings and Precautions (5.2), and Clinical Pharmacology (12.1)]. DOSAGE AND ADMINISTRATION Patient Selection Prior to initiation of treatment with Vectibix®, assess RAS mutational status in colorectal tumors and confirm the absence of a RAS mutation. Information on FDA-approved tests for the detection of KRAS mutations in patients with metastatic colorectal cancer is available at: http://www.fda.gov/CompanionDiagnostics. Recommended Dose The recommended dose of Vectibix® is 6 mg/kg, administered as an intravenous infusion over 60 minutes, every 14 days. If the first infusion is tolerated, administer subsequent infusions over 30 to 60 minutes. Administer doses higher than 1000 mg over 90 minutes [see Dosage and Administration (2.4)]. Appropriate medical resources for the treatment of severe infusion reactions should be available during Vectibix® infusions [see Warnings and Precautions (5.4)]. Dose Modifications Dose Modifications for Infusion Reactions [see Warnings and Precautions (5.4) and Adverse Reactions (6.1, 6.3)] • Reduce infusion rate by 50% in patients experiencing a mild or moderate (grade 1 or 2) infusion reaction for the duration of that infusion. • Terminate the infusion in patients experiencing severe infusion reactions. Depending on the severity and/or persistence of the reaction, permanently discontinue Vectibix®. Dose Modifications for Dermatologic Toxicity [see Boxed Warning, Warnings and Precautions (5.1), and Adverse Reactions (6.1, 6.3)] • Upon first occurrence of a grade 3 (NCI-CTC/CTCAE) dermatologic reaction, withhold 1 to 2 doses of Vectibix®. If the reaction improves to < grade 3, reinitiate Vectibix ® at the original dose. • Upon the second occurrence of a grade 3 (NCI-CTC/CTCAE) dermatologic reaction, withhold 1 to 2 doses of Vectibix®. If the reaction improves to < grade 3, reinitiate Vectibix® at 80% of the original dose. • Upon the third occurrence of a grade 3 (NCI-CTC/CTCAE) dermatologic reaction, withhold 1 to 2 doses of Vectibix®. If the reaction improves to < grade 3, reinitiate Vectibix® at 60% of the original dose. • Upon the fourth occurrence of a grade 3 (NCI-CTC/CTCAE) dermatologic reaction, permanently discontinue Vectibix®. Permanently discontinue Vectibix® following the occurrence of a grade 4 dermatologic reaction or for a grade 3 (NCI-CTC/CTCAE) dermatologic reaction that does not recover after withholding 1 or 2 doses. Preparation and Administration Do not administer Vectibix® as an intravenous push or bolus. CONTRAINDICATIONS None. WARNINGS AND PRECAUTIONS Dermatologic and Soft Tissue Toxicity In Study 1, dermatologic toxicities occurred in 90% of patients and were severe (NCI-CTC grade 3 and higher) in 15% of patients with mCRC receiving Vectibix ®. The clinical manifestations included, but were not limited to, acneiform dermatitis, pruritus, erythema, rash, skin exfoliation, paronychia, dry skin, and skin fissures. Monitor patients who develop dermatologic or soft tissue toxicities while receiving Vectibix® for the development of inflammatory or infectious sequelae. Life-threatening and fatal infectious complications including necrotizing fasciitis, abscesses, and sepsis have been observed in patients treated with Vectibix®. Life-threatening and fatal bullous mucocutaneous disease with blisters, erosions, and skin sloughing has also been observed in patients treated with Vectibix®. It could not be determined whether these mucocutaneous adverse reactions were directly related to EGFR inhibition or to idiosyncratic immunerelated effects (eg, Stevens-Johnson syndrome or toxic epidermal necrolysis). Withhold or discontinue Vectibix® for dermatologic or soft tissue toxicity associated with severe or life-threatening inflammatory or infectious complications [see Boxed Warning and Adverse Reactions (6.1, 6.3)]. Dose modifications for Vectibix® concerning dermatologic toxicity are provided [see Dosage and Administration (2.3)]. Increased Tumor Progression, Increased Mortality, or Lack of Benefit in Patients with RAS Vectibix® is not indicated for the treatment of patients with colorectal cancer that harbor somatic mutations in exon 2 (codons 12 and 13), exon 3 (codons 59 and 61), and exon 4 (codons 117 and 146) of either KRAS or NRAS and hereafter is referred to as “RAS ” [see Indications and Usage (1.1), Dosage and Administration (2.1), Clinical Pharmacology (12.1) and Clinical Studies (14)]. Retrospective subset analyses across several randomized clinical trials were conducted to investigate the role of RAS mutations on the clinical effects of anti-EGFR-directed monoclonal antibodies (panitumumab or cetuximab). Anti-EGFR antibodies in patients with tumors containing RAS mutations resulted in exposing those patients to anti-EGFR related adverse reactions without clinical benefit from these agents [see Indications and Usage (1.1), and Clinical Pharmacology (12.1)]. Additionally, in Study 3, 272 patients with RAS-mutant mCRC tumors received Vectibix® in combination with FOLFOX and 276 patients received FOLFOX alone. In an exploratory subgroup analysis, OS was shorter (HR = 1.21, 95% CI: 1.01-1.45) in patients with RAS-mutant mCRC who received Vectibix® and FOLFOX versus FOLFOX alone [see Indications and Usage (1.1)]. Electrolyte Depletion/Monitoring Progressively decreasing serum magnesium levels leading to severe (grade 3-4) hypomagnesemia occurred in up to 7% (in Study 2) of patients across clinical trials. Monitor patients for hypomagnesemia and hypocalcemia prior to initiating Vectibix® treatment, periodically during Vectibix® treatment, and for up to 8 weeks after the completion of treatment. Other electrolyte disturbances, including hypokalemia, have also been observed. Replete magnesium and other electrolytes as appropriate. Infusion Reactions In Study 1, 4% of patients experienced infusion reactions and 1% of patients experienced severe infusion reactions (NCI-CTC grade 3-4). Infusion reactions, manifesting as fever, chills, dyspnea, bronchospasm, and hypotension, can occur following Vectibix® administration [see Adverse Reactions (6.1, 6.3)]. Fatal infusion reactions occurred in postmarketing experience. Terminate the infusion for severe infusion reactions [see Dosage and Administration (2.3)]. Acute Renal Failure in Combination with Chemotherapy Severe diarrhea and dehydration, leading to acute renal failure and other complications, have been observed in patients treated with Vectibix® in combination with chemotherapy. Pulmonary Fibrosis/Interstitial Lung Disease (ILD) Fatal and nonfatal cases of interstitial lung disease (ILD) (1%) and pulmonary fibrosis have been observed in patients treated with Vectibix®. Pulmonary fibrosis occurred in less than 1% (2/1467) of patients enrolled in clinical studies of Vectibix®. In the event of acute onset or worsening of pulmonary symptoms, interrupt Vectibix® therapy. Discontinue Vectibix® therapy if ILD is confirmed. In patients with a history of interstitial pneumonitis or pulmonary fibrosis, or evidence of interstitial pneumonitis or pulmonary fibrosis, the benefits of therapy with Vectibix® versus the risk of pulmonary complications must be carefully considered. Photosensitivity Exposure to sunlight can exacerbate dermatologic toxicity. Advise patients to wear sunscreen and hats and limit sun exposure while receiving Vectibix®. Ocular Toxicities Keratitis and ulcerative keratitis, known risk factors for corneal perforation, have been reported with Vectibix® use. Monitor for evidence of keratitis or ulcerative keratitis. Interrupt or discontinue Vectibix® therapy for acute or worsening keratitis. Increased Mortality and Toxicity with Vectibix® in Combination with Bevacizumab and Chemotherapy In an interim analysis of an open-label, multicenter, randomized clinical trial in the first-line setting in patients with mCRC, the addition of Vectibix® to the combination of bevacizumab and chemotherapy resulted in decreased OS and increased incidence of NCI-CTC grade 3-5 (87% vs 72%) adverse reactions. NCI-CTC grade 3-4 adverse reactions occurring at a higher rate in Vectibix®-treated patients included rash/acneiform dermatitis (26% vs 1%), diarrhea (23% vs 12%), dehydration (16% vs 5%), primarily occurring in patients with diarrhea, hypokalemia (10% vs 4%), stomatitis/mucositis (4% vs < 1%), and hypomagnesemia (4% vs 0). NCI-CTC grade 3-5 pulmonary embolism occurred at a higher rate in Vectibix®-treated patients (7% vs 3%) and included fatal events in three (< 1%) Vectibix®-treated patients. As a result of the toxicities experienced, patients randomized to Vectibix®, bevacizumab, and chemotherapy received a lower mean relative dose intensity of each chemotherapeutic agent (oxaliplatin, irinotecan, bolus 5-FU, and/or infusional 5-FU) over the first 24 weeks on study compared with those randomized to bevacizumab and chemotherapy. ADVERSE REACTIONS The following adverse reactions are discussed in greater detail in other sections of the label: • Dermatologic and Soft Tissue Toxicity [see Boxed Warning, Dosage and Administration (2.3), and Warnings and Precautions (5.1)] • Increased Tumor Progression, Increased Mortality, or Lack of Benefit in RAS- and KRAS-Mutant mCRC [see Indications and Usage (1.1) and Warnings and Precautions (5.2)] • Electrolyte Depletion/Monitoring [see Warnings and Precautions (5.3)] • Infusion Reactions [see Dosage and Administration (2.3), and Warnings and Precautions (5.4)] • Acute Renal Failure in Combination with Chemotherapy [see Warnings and Precautions (5.5)] • Pulmonary Fibrosis/Interstitial Lung Disease (ILD) [see Warnings and Precautions (5.6)] • Photosensitivity [see Warnings and Precautions (5.7)] • Ocular Toxicities [see Warnings and Precautions (5.8)]

• Increased Mortality and Toxicity with Vectibix® in combination with Bevacizumab and Chemotherapy [see Warnings and Precautions (5.9)] Clinical Trials Experience Because clinical trials are conducted under widely varying conditions, adverse reaction rates in the clinical trials of a drug cannot be directly compared to rates in clinical trials of another drug and may not reflect the rates observed in practice. The adverse reaction information from clinical studies does, however, provide a basis for identifying the adverse events that appear to be related to drug use and for approximating rates. Safety data are presented from two clinical trials in which patients received Vectibix®: Study 1, an openlabel, multinational, randomized, controlled, monotherapy clinical trial (N = 463) evaluating Vectibix® with best supportive care (BSC) versus BSC alone in patients with EGFR-expressing mCRC and Study 3, a randomized, controlled trial (N = 1183) in patients with mCRC that evaluated Vectibix® in combination with FOLFOX chemotherapy versus FOLFOX chemotherapy alone. Safety data for Study 3 are limited to 656 patients with wild-type KRAS mCRC. Vectibix® Monotherapy In Study 1, the most common adverse reactions (≥ 20%) with Vectibix® were skin rash with variable presentations, paronychia, fatigue, nausea, and diarrhea. The most common (> 5%) serious adverse reactions in the Vectibix® arm were general physical health deterioration and intestinal obstruction. The most frequently reported adverse reactions for Vectibix® leading to withdrawal were general physical health deterioration (n = 2) and intestinal obstruction (n = 2). For Study 1, the data described in Table 1 and in other sections below, except where noted, reflect exposure to Vectibix® administered to patients with mCRC as a single agent at the recommended dose and schedule (6 mg/kg every 2 weeks). Table 1: Adverse Reactions (≥ 5% Difference) Observed in Patients Treated with Vectibix® Monotherapy and Best Supportive Care Compared to Best Supportive Care Alone (Study 1)


ASCOPost.com  |   JULY 25, 2015

PAGE 53

Geriatrics for the Oncologist

Gynecologic Cancer in the Older Patient: The Activities of the Elderly Working Group of NRG Oncology By Amina Ahmed, MD

Geriatrics for the Oncologist is guest edited by Stuart Lichtman, MD, and developed in collaboration with the International Society of Geriatric Oncology (SIOG). Visit SIOG.org for more on geriatric oncology.

T

he elderly population in the United States is growing, and by the year 2050, persons older than 65 years of age will comprise 20% of the population. Not only will there be an increase in the number of persons aged 65 years and older, the life expectancy of those older than 65 years of age has increased; so the number of those older than 85

older than 65 years of age accounting for 60% of newly diagnosed malignancies and 70% of all cancer deaths.1 Rates of gynecologic cancers increase with age; the median age at diagnosis for ovarian cancer is 63 years, with 40.8% of diagnosed cases in those 65 years and older. Of the cases diagnosed in the elderly, half were diagnosed after age 75 years.2 A total of 40% of patients with endometrial carcinoma are diagnosed after age 65 years, and 43% of patients older than age 75 years will have an advanced stage at presentation.3

Underrepresented in Clinical Trials Despite the high cancer incidence in the elderly population, older patients

GOG 273 is the first study directed toward elderly women with ovarian carcinoma receiving first-line chemotherapy to assess tolerance and predict the ability to complete therapy. —Amina Ahmed, MD

years of age is expected to double to 8.9 million by 2050. As the population ages, diseases that affect the elderly will become more prevalent. Increasing age is associated with increasing rates of cancer, with persons Dr. Ahmed is a gynecologic oncologist, ­Advocate Health Care, Section of Gynecologic Oncology, Lutheran General Hospital, Park Ridge, Illinois.

are still underrepresented in clinical trials. There is a general paucity of data on treatment efficacy and toxicity in elderly women with gynecologic malignancies. The lack of information from elderly patients may contribute to the increased risk of toxicity from treatment and poor outcomes observed in this group. Many unanswered questions remain: Under what situations should a full dose be maintained? What can be

done to ameliorate toxicities? When are we likely to have a negative functional outcome, such as causing a formerly independent patient to have to move to a nursing home? In 1995, the Cancer and Leukemia Group B (CALGB) organized a Cancer in the Elderly Committee, which was led by Hyman Muss, MD, and Harvey Cohen, MD, with vice chairs Arti Hurria, MD, and Aminah Jatoi, MD. The newly formed Alliance for Clinical Trials in Oncology (CALGB, the North Central Cancer Treatment Group, and the American College of Surgeons Oncology Group) maintained this strengthened committee. The Gynecologic Oncology Group (GOG) formed an Elderly Working Group in 2010, led by Gini Fleming, MD, to try to improve outcomes for older patients. The activities of the GOG, now the NRG Elderly Working Group, are summarized here.

NRG Elderly Working Group Studies Elderly patients with ovarian carcinoma may have significant comorbidities, which can affect treatment choices or produce worsened toxicities and impact survivorship and quality of life. They are also more prone to certain toxicities, particularly neuropathy. GOG 273 is the first study directed toward elderly women with ovarian carcinoma receiving first-line chemotherapy to assess tolerance and predict the ability to complete therapy. GOG 273 was opened in August 2011. Vivian von Gruenigen, MD, the study chair, presented a preliminary report of GOG 273 at the 2014 Society of Gynecologic Oncology Annual Meeting. Phy-

GUEST EDITOR

Stuart M. Lichtman, MD

D

r. Lichtman is an Attending Physician at Memorial Sloan Kettering Cancer Center, Commack, New York, and Professor of Medicine, Weill Cornell Medical College, New York. He is also President Elect of the International Society of Geriatric Oncology (www.siog.org). sician-selected treatment arm 1 consisted of carboplatin, paclitaxel, and granulocyte colony-stimulating factor every 3 weeks and arm 2 consisted of carboplatin only every 3 weeks. Patients treated on arm 1 were younger, more fit, and had higher chemotherapy completion rates and required fewer dose reductions or delays than patients treated on arm 2. Multivariate analysis revealed that assignment to arm 2, neoadjuvant chemotherapy, and limited social activities (but not instrumental activities of daily living) were associated with an inability to complete four cycles of chemotherapy. Impressively, quality of life improved in patients on both arms, likely reflecting the efficacy of chemotherapy in this disease.

For more information, visit http://www.siog.org

continued on page 54


The ASCO Post  |   JULY 25, 2015

PAGE 54

Geriatrics for the Oncologist NRG Elderly Working Group continued from page 53

A third arm was added to GOG 273 in 2013, as there was a desire to assess tolerance of the popular regimen of dose-dense chemotherapy with weekly paclitaxel and every 3-week carboplatin. Analysis of data from this third arm is underway. In addition, a parallel study is being conducted by NRG Oncology Japan, which is an exciting opportunity to see whether cross-cultural use of tools such as a Geriatric Assessment Score has similar predictive power. As accrual goals have been met, GOG 273 is now closed. More recently, NRG CC-002, the second trial through the NRG Elderly Working Group for which I served as study chair, was opened in February 2015. This prospective study is evaluating whether a preoperative Geriatric Assessment Score will be associated with major postoperative complications in elderly women with suspected ovarian, fallopian tube, primary peritoneal, or advanced-stage papillary serous uterine carcinoma undergoing open cytoreductive surgery. The study is accruing well, and as of June 2015, approximately 69 patients have been enrolled. An interim analysis is underway to see how many patients are receiving primary surgical cytoreduction vs interval cytoreduction vs no surgery. In the future, the hope is to incorporate a standardized Geriatric Assessment Score into all trials, allowing a platform for comparison of outcomes for older patients. This way it will be possible to determine whether a study allowed only the healthiest elders or also included a more vulnerable population.

Other Concepts and Expanded Disease Sites Numerous other concepts are currently being developed by the NRG Elderly Working Group. A few of them are listed here: • An exercise intervention concept for elderly women with advanced ovarian carcinoma. • A palliative care integration study of elderly women with advanced ovarian carcinoma to assess whether integrated palliative care at diagnosis is cost-effective and improves quality of life. • Trials to test specific recently approved regimens, including singleagent olaparib (Lynparza) and the combination of bevacizumab (Avastin) with chemotherapy, in elderly patients with platinum-resistant ovarian carcinoma. The NRG Elderly Working Group

is excited to expand its involvement to other disease sites. A retrospective review of outcomes in legacy Radiation Therapy Oncology Group (RTOG) head and neck cancer trials was presented at the 2015 ASCO Annual Meeting by Julie Ann Kish, MD, FACP.4 It demonstrated that patients aged 70 years and older were un-

derrepresented in RTOG trials evaluating treatment for locally advanced head and neck cancer relative to the overall population and had worse survival than patients younger than age 70 years. A trial that would randomize patients aged 70 years or older with head and neck cancer to receive either cetuximab (Erbitux) or anti–

PD-1 (programmed cell death protein 1) therapy with radiation is being proposed. Work continues across tumor types to refine our ability to predict which older patients will not tolerate standard therapy well, to find regimens that improve cancer outcomes in older patients, and to decrease toxicity in older patients. n

Advertisement not displayed in digital edition at advertiser’s request


ASCOPost.com  |   JULY 25, 2015

PAGE 55

Geriatrics for the Oncologist Disclosure: Dr. Ahmed reported no potential conflicts of interest.

References 1. Berger NA, Savvides P, Koroukian SM, et al: Cancer in the elderly. Trans Am Clin Climatol Assoc 117:147-156, 2006. 2. Howlader N, Noone AM, Krapcho M, et al (eds): SEER Cancer Statistics Re-

view, 1975-2008, National Cancer Institute. Bethesda, MD. Available at http://seer. cancer.gov/csr/1975_2008/, based on November 2010 SEER data submission, posted to the SEER web site, 2011. Accessed July 2, 2015. 3. Lee NK, Fleming GF: Treatment of advanced stage and recurrent endometrial cancer in elderly women, in Lichtman SM, Audi-

sio RA (eds): Management of Gynecological Cancers in Older Women, pp 257-275. London, Springer-Verlag London, 2013. 4. Kish JA, Zhang Q, Langer CJ, et al: The effect of age on outcome in prospective, phase III NRG Oncology/RTOG trials of radiotherapy ± chemotherapy in locally advanced head and neck cancer. 2015 ASCO Annual Meeting. Abstract 6003.

Advertisement not displayed in digital edition at advertiser’s request

Watch future issues of The ASCO Post for continued discussion on geriatrics for the oncologist and practical tools to use with your older patients.


The ASCO Post  |   JULY 25, 2015

PAGE 56

Announcements

Matthew Trunnell Named Vice President and Chief Information Officer at Fred Hutchinson Cancer Research Center

F

red Hutchinson Cancer Research Center announced the appointment of Matthew Trunnell as its Vice President and Chief Infor-

mation Officer (CIO). Mr. Trunnell has worked at the intersection of information technology and life sciences research in both academic and com-

mercial environments in support of large-scale genomics research for more than 15 years. He will be responsible for many key areas at Fred Hutchinson, in-

cluding research informatics and enterprise information technology. He will assume his new role on August 31. continued on page 57

Advertisement not displayed in digital edition at advertiser’s request


ASCOPost.com  |   JULY 25, 2015

PAGE 57

Announcements

John A. Zaia, MD, Named Director of Center for Gene Therapy at City of Hope

G

ene therapy pioneer John A. Zaia, MD, has been named Director of the Center for Gene Therapy within City of Hope’s Hematologic Malignancies and Stem Cell Transplantation Institute. Internationally known and re-

spected for his groundbreaking research into potential gene therapy treatments for HIV, Dr. Zaia will maximize the potential of gene therapy not just for HIV, but also for cancer and other diseases. Dr. Zaia, the Aaron D. and Edith

Miller Chair in Gene Therapy, and past Chair of the Department of Virology, is also the Principal Investigator of the new Alpha Clinic for Cell Therapy and Innovation (ACT-I) at City of Hope. The clinic is dedicated to identifying new stem cell

cures for currently incurable diseases, and to helping those cures become a standard option for patients who need them. Among the treatments tested in the clinic will be immunotherapy approaches developed in the hematologic institute using gene therapy. Dr. Zaia will bridge the current gap between the promise and the reality of stem cell treatments, speeding lifesaving treatments to the patients who need them.

John A. Zaia, MD

“When it comes to gene therapy, John has been both a visionary and a teacher,” said Stephen J. Forman, MD, the Director of the Hematologic Institute. “His leadership will be vital in helping doctors and researchers maximize the potential of gene therapy as a source of treatments.” n

Matthew Trunnell continued from page 56

Advertisement not displayed in digital edition at advertiser’s request

Mr. Trunnell comes to Fred Hutchinson from Broad Institute of MIT and Harvard, where, as CIO, he managed a 50-fold scale-up of research information technology (IT) data infrastructure in support of nextgeneration sequencing and initiated a new organization comprising computational biologists and software engineers focused on developing advanced informatics capabilities.

Matthew Trunnell

As CIO, Mr. Trunnell will be responsible for data, informatics, and information analytics across the organization. He will play a significant role in leading efforts to employ IT in acquiring research funding, in research to advance science, and in supporting administrative systems and processes. n


The ASCO Post  |   JULY 25, 2015

PAGE 58

Announcements

David Gius, MD, PhD, Named Co-leader of Northwestern’s Women’s Cancer Research Program

D

avid Gius, MD, PhD, has been named Co-leader of the Women’s Cancer Research Program at the Robert H. Lurie Comprehensive Cancer Center of Northwestern University, a position he has served in interim since February 2014. Dr. Gius, a nationally recognized radiation oncologist and researcher, is a Professor in the Department of Radiation Oncology at Northwestern University Feinberg School of Medicine.

family plays in regulating a cell’s ability to recover from stress. In other laboratory studies, Dr. Gius is testing chemical agents that could yield promising new

drug treatments for women with hormone receptor–positive breast cancer. Dr. Gius joined Northwestern in 2012 from Vanderbilt University’s In-

gram Cancer Center. He is a member of the National Institutes of Health Radiation and Therapeutics and Biology Study Section. n

David Gius, MD, PhD

The Women’s Cancer Research Program embraces a multidisciplinary approach to integrate its basic, translational, and clinical components. The program seeks to define the common mechanisms responsible for the risk and progression of tumors arising from the reproductive tissues of women, including the breast, ovary, endometrium, myometrium, uterine cervix, and vagina. In doing so, the goals of the Women’s Cancer Research Program are to identify molecular markers that contribute to the disease process and outcome, to develop targeted molecular approaches, and to translate these agents from preclinical models to the bedside. In collaboration with Coleader Seema Khan, MD, Dr. Gius will continue to increase the Women’s Cancer Research Program’s translational research activity. Dr. Gius’ laboratory research concentrates on the connection between aging, cellular metabolism, and cancer, focusing on the role that the Sirtuin gene

The ASCO Post

Like us on

Facebook facebook.com/TheASCOPost

Learn more about the unmet needs in EGFRm+ NSCLC at targetEGFR.com


ASCOPost.com  |   JULY 25, 2015

PAGE 59

Announcements

Laura van’t Veer, PhD, Receives European Inventor Award 2015

L

aura van’t Veer, PhD, Co-leader of the University of California San Francisco Helen Diller Family Comprehensive Cancer Center (HDFCCC) Breast Oncology Program, and her team at the Netherlands Cancer Institute

were awarded the European Inventor Award for the invention of a gene-based tissue test, which makes it possible to offer targeted treatment for breast cancer. The European Patent Office presented the award on June 11 in Paris.

It provides women in the early stage of breast cancer with a reliable prognosis as to whether chemotherapy is actually necessary. Patients who have a lower risk of recurrence are able to recover and stay tumor-free without che-

motherapy. The technology has already helped over 40,000 women with treatment for cancer and means that 20% to 30% fewer women may have to undergo lengthy chemotherapy. Speaking at the ceremony, she thanked her team and described the satisfaction of turning “research findings into real products for patients.” She said: “I appreciate being able to provide something that provides the most optimal treatment.”

We want to change the face of EGFR-targeted therapy Cutaneous toxicities are caused by inhibition of wild-type epidermal growth factor receptor (EGFR) and can be debilitating1,2 Inhibition of wild-type and mutant EGFR in non–small cell lung cancer (NSCLC)

Cutaneous toxicities can be dose-limiting

Normal, or wild-type, EGFR is highly expressed on epithelial cells in the skin, liver, and gastrointestinal tract.3-5 Current EGFR tyrosine kinase inhibitors (TKIs) target not only the oncogenic mutant forms of EGFR, but also wild-type EGFR, which may lead to cutaneous toxicities including rash, stomatitis, and paronychia.1,2,6-8

The symptoms and psychosocial impact of cutaneous toxicities can negatively affect both patient quality of life and patient compliance.11,12 In some studies, rash and paronychia were among the most frequent causes of dose modification, combining to cause dose reductions in as many as 33% of patients.7,8

90% of patients treated with approved EGFR TKIs experience rash7,8

The future of EGFR inhibition

The skin is dependent on wild-type EGFR signaling for normal growth and differentiation.1,9,10 Druginduced inhibition of wild-type EGFR disrupts its normal function and can cause cutaneous inflammation and injury. This accounts for the high incidence of cutaneous toxicities associated with EGFR TKIs.1,9

Strategies that eliminate inhibition of wild-type EGFR may be most effective at mitigating cutaneous toxicities and maintaining optimal dosing.9 At Clovis Oncology, we’re committed to exploring new approaches in EGFR therapy to advance the fight against NSCLC.

Clovis Oncology is leading the fight

REFERENCES: 1. Lynch TJ Jr et al. Epidermal growth factor receptor inhibitor–associated cutaneous toxicities: an evolving paradigm in clinical management. Oncologist. 2007;12(5):610-621. 2. Pérez-Soler R et al. HER1/EGFR inhibitor-associated rash: future directions for management and investigation outcomes from the HER1/EGFR Inhibitor Rash Management Forum. Oncologist. 2005;10(5):345-356. 3. Harandi A et al. Clinical efficacy and toxicity of anti-EGFR therapy in common cancers. J Clin Oncol. 2009;2009:567486. doi:10.1155/2009/567486. 4. Natarajan A et al. The EGF receptor is required for efficient liver regeneration. Proc Natl Acad Sci U S A. 2007;104(43):17081-17086. 5. Tissue atlas: EGFR. The Human Protein Atlas website. http://www.proteinatlas.org /ENSG00000146648-EGFR/tissue. Accessed February 17, 2015. 6. Antonicelli A et al. EGFR-targeted therapy for non-small cell lung cancer: focus on EGFR oncogenic mutation. Int J Med Sci. 2013;10(3):320-330. 7. Tarceva [package insert]. Northbrook, IL: Astellas Pharma US Inc; 2014. 8. Gilotrif [package insert]. Ridgefield, CT: Boehringer Ingelheim Pharmaceuticals Inc; 2014. 9. Lacouture ME. Mechanisms of cutaneous toxicities to EGFR inhibitors. Nat Rev Cancer. 2006;6(10):803-812. 10. Melosky B et al. Management of common toxicities in metastatic NSCLC related to anti-lung cancer therapies with EGFR–TKIs. Front Oncol. 2014;4:238. doi:10.3389/fonc.2014.00238. 11. White KJ et al. Psychosocial impact of cutaneous toxicities associated with epidermal growth factor receptor–inhibitor treatment. Clin J Oncol Nurs. 2011;15(1):88-96. 12. National Cancer Institute. Common Terminology Criteria for Adverse Events (CTCAE), Version 4.0. http://evs.nci .nih.gov/ftp1/CTCAE/CTCAE_4.03_2010-06-14_QuickReference_8.5x11.pdf. Published May 28, 2009. Updated June 14, 2010. Accessed February 2, 2015.

Copyright © 2015 Clovis Oncology. DARO-101(1) 3/15

Laura van’t Veer, PhD (Netherlands), winner of the European Inventor Award 2015 in the smalland medium-sized enterprises category, at the ceremony at the Palais Brongniart in Paris on June 11. Photo credit: European Patent Office.

How the Test Works In 2001, the biologist and her team at the Netherlands Cancer Institute discovered that a specific DNA signature made up of 70 genes is what determines the risk factor for the recurrence of breast cancer in women. Before this, doctors relied solely on the pathologic findings, age of the patient, and number of divided cancer cells when deciding upon treatment. “We discovered that we could identify a gene activity pattern in the tumors that would allow us to distinguish high-risk cases from low-risk ones,” said Dr. van’t Veer. The test measures the activity of tumor-specific DNA with the help of a microchip. Using just a small tissue sample from the patient, the test can deliver results within 10 days and determine if the patient is at high risk or not. “You can compare the activity of genes that are either switched on or off. If the gene is switched on, the tumor is very aggressive. If those 70 genes are switched off, then the tumor will remain in its place, and there is not much risk of recurrence,” she explained. n


The ASCO Post  |   JULY 25, 2015

PAGE 60

Hematology Expert Review

Microangiopathic Hemolytic Anemia and Thrombocytopenia Part 1: Diagnosis

By Syed A. Abutalib, MD, and Jean M. Connors, MD

T

he ASCO Post is pleased to present Hematology Expert Review, an occasional feature that includes a case report detailing a particular hematologic condition followed by questions. Answers to each question appear on page 61 with expert commentary. In this installment, we present the case of a 26-year old woman who presented postpartum day 8 with heavy vaginal bleeding, dizziness, and other symptoms. “Part 2: Management” will appear in the ­August 10 issue of The ASCO Post.

Case Study: A 26-year-old woman presented to labor and delivery triage with fatigue, heavy vaginal bleeding, and gastrointestinal (GI) upset on postpartum day 8 after vacuumassisted vaginal delivery. Prior to presentation, she reported diarrhea and nausea with nonbilious, nonbloody emesis, which since has resolved. She had left upper quadrant pain, which worsened with deep inspiration. On review of systems, she experienced dizziness, with no syncopal episodes. She denied fever or chills, bleeding gums or easy bruising, dys-

uria, or vaginal discharge other than blood. Her medical history was notable for sickle cell trait, scoliosis, and chronic borderline hypertension. Her hypertension worsened during her most recent pregnancy, but her blood pressure normalized during the postpartum period. Her family history was notable for a mother with diabetes and hypertension and no known history of bleeding disorders. On physical examination, her temperature was 99.4˚ F. Her heart rate was 122 beats/min, and her blood pressure ranged from 90 to 130/50 to 60 mm Hg. She was breathing at a rate of 30 to 33 breaths/min, with an oxygen saturation of 100% in ambient air. She had conjunctival pallor, and her mucous membranes were pale. Her neck was supple without lymphadenopathy, and her abdomen was soft and nondistended, with diffuse mild tenderness to deep palpation. Her lungs were clear to auscultation bilaterally. Her neurologic exam was within normal limits. Her laboratory results revealed sodium 137 mmol/L, potassium 3.7 mmol/L, chloride 103 mmol/L, bicarbonate 20 mmol/L, calcium 8.4

Fig. 1: Fragmentation of erythrocytes (schistocytes), anemia, and thrombocytopenia.

GUEST EDITORS

Syed A. Abutalib, MD

Syed A. Abutalib, MD, Assistant Director, Hematology & Bone Marrow Transplantation Service, Cancer Treatment Centers of America, Zion, Illinois mg/dL, urea nitrogen 33 mg/dL, creatinine 1.61 mg/dL (baseline, 0.6 mg/dL), and glucose 123 mg/dL. Alanine aminotransferase (ALT) was 20 U/L, aspartate aminotransferase (AST) 51 U/L, total bilirubin 3.1 mg/dL, direct bilirubin 0.4 mg/dL, albumin 3.4 g/dL, total protein 6.8 g/ dL, and alkaline phosphatase 79 U/L. White blood cell count was 10,000/μL, with 67% neutrophils, 18% lymphocytes, 7% monocytes, 0% eosinophils, and 0% basophils. Hemoglobin was 5.3 g/dL, hematocrit 15.3%, and platelets 2,000/μL. Repeat laboratory studies confirmed severe anemia and thrombocytopenia. Reticulocyte count was 9.06%. Prothrombin time was 15.1 seconds, activated partial thromboplastin time was 37.7 seconds, and international normalized ratio was 1.2. Fibrinogen was 445 mg/dL, and haptoglobin was < 8 mg/dL. Urinalysis showed 3+ blood, 2+ protein, negative leukocyte esterase, negative nitrites, 25 to 50 red bloods/high-powered field, 0 to 4 white blood cells/high-powered field, and 0 to 2 granular casts/highpowered field.

Jean M. Connors, MD

Jean M. Connors, MD, Assistant Professor, Harvard Medical School, Hematology Division, Brigham and Women’s Hospital/Dana-Farber Cancer Institute, Boston

Question 1 In this case, what is the most appropriate next best test? A. Ferritin measurement B. Peripheral blood smear examination C. Cyanocobalamin measurement

Question 2 Based on the patient’s clinical presentation and the peripheral blood smear film, what is the most likely diagnosis? A. HELLP syndrome (hemolysis with a microangiopathic blood smear, elevated liver enzymes, low platelet count) B. Disseminated intravascular coagulation C. Thrombotic thrombocytopenic purpura or hemolytic uremic syndrome

Question 3 What test would aid in making the correct diagnosis prior to treatment? A. Assess mutation in complement proteins B. von Willebrand factor multimer analysis C. ADAMTS13 analysis

See page 61 for answers and expert discussion.


ASCOPost.com  |   JULY 25, 2015

PAGE 61

Hematology Expert Review

Microangiopathic Hemolytic Anemia and Thrombocytopenia Part 1: Diagnosis Answers From Hematology Expert Review Questions on Page 60 Question 1: In this case, what is the most appropriate next best test? Correct Answer: B. Peripheral blood smear examination.

upper quadrant pain, transaminitis, microangiopathic hemolytic anemia, and low platelet count prior to delivery.

Although disseminated intravascular coagulation from retained products of conception, endometritis, or other in-

fection is possible, the fibrinogen level is significantly higher than one would continued on page 62

Expert Perspective In the appropriate clinical setting, information obtained from a carefully examined peripheral blood smear film is indispensable. The peripheral blood smear, shown in Fig. 1, reveals schistocytes (“fragmented red blood cells” noted by arrows) consistent with microangiopathic hemolytic anemia and confirms “true” thrombocytopenia. These findings on peripheral blood smear can be observed in variety of conditions.1,2 Therefore, it is equally important to interpret them in the context of the patient’s clinical condition. This clinical exercise will further narrow the differential diagnosis, which includes thrombotic thrombocytopenic purpura; Shiga toxin–associated hemolytic uremic syndrome (StxHUS or “typical” hemolytic uremic syndrome); atypical hemolytic uremic syndrome; disseminated intravascular coagulation; pregnancy-associated conditions such as preeclampsia, eclampsia, and HELLP syndrome; and severe “malignant” hypertension. Although fragmented red blood cells with thrombocytopenia may be observed in the setting of severe cyanocobalamin deficiency,3,4 a normal white blood cell count, mean corpuscular volume, and a high reticulocyte count do not favor this diagnosis, and measurement of cyanocobalamin levels will not be useful. Ferritin measurement is not useful at this time, as current findings are not consistent with iron-deficiency anemia. Question 2: Based on the patient’s clinical presentation and the peripheral blood smear film, what is the most likely diagnosis? Correct Answer: C. Thrombotic thrombocytopenic purpura or hemolytic uremic syndrome.

Introducing the Institute for Clinical Immuno-Oncology

A NEW ERA OF ONCOLOGY STARTS NOW. Only ICLIO has everything the multidisciplinary team needs to put immunotherapy into practice.

Expert Perspective It is late to expect preeclampsia, eclampsia, or development of HELLP syndrome in this postpartum patient, as she had no signs of these disorders prior to delivery except mild worsening of hypertension without proteinuria. Patients with HELLP syndrome develop right

Visit us at accc-iclio.org

The Association of Community Cancer Centers (ACCC) has launched the Institute for Clinical Immuno-Oncology (ICLIO), a unique initiative that will accelerate the adoption of immunotherapy (I-O) in the community. ICLIO brings together leaders in the field to educate multidisciplinary teams to go beyond a clinical understanding of I-O and tackle real-world implementation issues. Visit us at accc-iclio.org for a wealth of resources and get involved in this transformative community.

ICLIO is made possible by a charitable donation from Bristol-Myers Squibb.


The ASCO Post  |   JULY 25, 2015

PAGE 62

Hematology Expert Review Hemolytic Anemia continued from page 61

expect in the setting of acute disseminated intravascular coagulation. Catastrophic antiphospholipid antibody syndrome and connective-tissue disorders (especially scleroderma renal crisis) may cause microangiopathic hemolytic anemia and thrombocytopenia.1,2,5 However, our patient’s clinical history and laboratory data do not support these diagnoses. Our patient’s clinical presentation and laboratory and peripheral blood smear findings are most consistent with the diagnosis of thrombotic thrombocytopenic purpura or hemolytic uremic syndrome. Distinguishing between thrombotic thrombocytopenic purpura (acquired or congenital6) and hemolytic uremic syndrome (typical or atypical7,8) based on presenting clinical findings and initial laboratory tests is difficult, as there is a significant overlap in clinical symptoms and laboratory findings. Patients with thrombotic thrombocytopenic purpura often do not have the classic pentad of clinical findings that include microangiopathic hemolytic anemia, thrombocytopenia, fever, neurologic symptoms, and renal dysfunction, whereas 30% of patients with a diagnosis of hemolytic uremic syndrome will have fever and neurologic symptoms not commonly considered for the diagnosis of hemolytic uremic syndrome.1,2,5 Question 3: What test would aid in making the correct diagnosis prior to treatment? Correct Answer: C. ADAMTS13 analysis.

Expert Perspective ADAMTS13 is a metalloproteinase present in the blood responsible for cleaving ultralarge von Willebrand factor multimers into smaller-sized multimers. The ultralarge multimers are prothrombotic and can lead to the development of microvascular thrombosis and end-organ damage. Deficiency in ADAMTS13 results in thrombotic thrombocytopenic purpura and can be acquired or congenital. Most cases of adult thrombotic thrombocytopenic purpura are acquired, due to the formation of immunoglobulin G autoantibody, which binds ADAMTS13 and clears it from the circulation. Congenital cases may be due to mutations resulting in low plasma levels of this protein, as in Upshaw-Schulman syndrome, which can manifest as thrombotic thrombocyto-

penic purpura at different ages.9-11 Tests for ADAMTS13 (activity levels and antibody detection) and Shiga toxin (produced by a variety of bacteria [see Table 1 in the online version of this article at www.ascopost.com]) should be sent to support the diagnosis of thrombotic thrombocytope-

nic purpura vs Shiga toxin–associated hemolytic uremic syndrome, respectively. In our patient, the ADAMTS13 level was reported as less than 10% of normal, strongly supporting the diagnosis of thrombotic thrombocytopenic purpura. Activity levels less than 10% are con-

sidered severely low and will confirm the diagnosis of thrombotic thrombocytopenic purpura in this patient. Activity levels in the normal range (greater than 50% in most laboratories) exclude the diagnosis of thrombotic thrombocytopenic purpura, despite the presence of microangiopathic hemolytic

Advertisement not displayed in digital edition at advertiser’s request


ASCOPost.com  |   JULY 25, 2015

PAGE 63

Hematology Expert Review

anemia, thrombocytopenia, and a normal coagulation profile. Activity levels between 10% and 50% are concerning but cannot be relied upon to make the diagnosis of thrombotic thrombocytopenic purpura with certainty, as decreases of these levels can be observed in a variety of inflammatory conditions.

Hemolytic uremic syndrome also results in the clinical and laboratory findings of microangiopathic hemolytic anemia and thrombocytopenia, but often the renal failure is a more dominant feature than the other “pentad” findings seen in thrombotic thrombocytopenic purpura. Hemolytic uremic syndrome

can be classified as either typical or atypical, depending on the etiology.12 The most common form of hemolytic uremic syndrome occurs following infection (“typical hemolytic uremic syndrome”) with a Shiga-like toxin– producing bacteria with a prodrome of bloody diarrhea symptoms, as our

Advertisement not displayed in digital edition at advertiser’s request

patient experienced, but the tests were negative for Shiga toxin. Shiga-like toxin–producing Escherichia coli (STEC) strain O157:H7 is the most common pathogen resulting in typical STEC hemolytic uremic syndrome, but other bacteria, including Shigella strains, continued on page 64


The ASCO Post  |   JULY 25, 2015

PAGE 64

Hematology Expert Review Hemolytic Anemia continued from page 63

Campylobacter, and less frequently Streptococcus pneumoniae, can result in the development of typical hemolytic uremic syndrome. In our patient, stool culture for organisms and Shiga toxin antigen detec-

tion assays (usually ELISA [enzymelinked immunosorbent assay]–type assays) were negative. Polymerase chain reaction tests can be used to detect the presence of Shiga-toxin (Stx1 and Stx2) genes but are not widely available.12 Atypical hemolytic uremic syndrome is due to inherited mutations in the genes

of complement regulatory proteins, resulting in excessive activation of the complement system. At least six mutated genes (CFH, CFI, MCP, CD46, C3, CFB, and THBD) and anti-CFH antibodies that can result in atypical hemolytic uremic syndrome have been identified,7,8,13 with the most common being CFH.

Measurement of complement levels in the setting of acute manifestations, however, is not useful in establishing the diagnosis, as levels can be low in a variety of acute and severe illnesses. Evaluation of genetic mutations is laborious, can take many months to report, and is costly. The tests can be done once

Advertisement not displayed in digital edition at advertiser’s request


ASCOPost.com  |   JULY 25, 2015

PAGE 65

Hematology Expert Review

common causes of microangiopathic hemolytic anemia and thrombocytopenia are excluded and are most useful in patients with relapsing courses of the disease. Although a von Willebrand factor multimer gel analysis may be abnormal in thrombotic thrombocytopenic purpura, results cannot be relied on to

confirm the diagnosis of thrombotic thrombocytopenic purpura. n

Disclosure: Drs. Abutalib and Connors reported no potential conflicts of interest.

References 1. George JN, Nester CM: Syndromes of thrombotic microangiopathy. N Engl J

Med 371:654-666, 2014. 2. Tsai HM: Advances in the pathogenesis, diagnosis, and treatment of thrombotic thrombocytopenic purpura. J Am Soc Nephrol 14:1072-1081, 2003. 3. Tadakamalla AK, Talluri SK, Besur S: Pseudo-thrombotic thrombocytopenic purpura: A rare presentation of pernicious

Advertisement not displayed in digital edition at advertiser’s request

anemia. N Am J Med Sci 3:472-474, 2011. 4. Dalsania CJ, Khemka V, Shum M, et al: A sheep in wolf ’s clothing. Am J Med 121:107-109, 2008. 5. Sadler JE: Von Willebrand factor, ADAMTS13, and thrombotic thrombocytopenic purpura. Blood 112:11-18, 2008. 6. Levy GG, Nichols WC, Lian EC, et al: Mutations in a member of the ADAMTS gene family cause thrombotic thrombocytopenic purpura. Nature 413:488-494, 2001. 7. Fremeaux-Bacchi V, Fakhouri F, Garnier A, et al: Genetics and outcome of atypical hemolytic uremic syndrome: A nationwide French series comparing children and adults. Clin J Am Soc Nephrol 8:554562, 2013. 8. Franchini M: Atypical hemolytic uremic syndrome: From diagnosis to treatment. Clin Chem Lab Med. March 18, 2015 (early release online). 9. Furlan M, Robles R, Lämmle B: Partial purification and characterization of a protease from human plasma cleaving von Willebrand factor to fragments produced by in vivo proteolysis. Blood 87:4223-4234, 1996. 10. Tsai HM: Physiologic cleavage of von Willebrand factor by a plasma protease is dependent on its conformation and requires calcium ion. Blood 87:42354244, 1996. 11. Moatti-Cohen M, Garrec C, Wolf M, et al: Unexpected frequency of UpshawSchulman syndrome in pregnancy-onset thrombotic thrombocytopenic purpura. French Reference Center for Thrombotic Microangiopathies. Blood 119:5888-5897, 2012. 12. Trachtman H, Austin C, Lewinski M, Stahl RA: Renal and neurological involvement in typical Shiga toxin-associated HUS. Nat Rev Nephrol 8:658-669, 2012. 13. Gruppo RA, Rother RP: Eculizumab for congenital atypical hemolytic-uremic syndrome. N Engl J Med 360:544-546, 2009.

Send Us Your NEWS Write to editor@ASCOPost.com. All submissions will be considered for publication


The ASCO Post  |   JULY 25, 2015

PAGE 66

Clinical Trials Resource Guide

Clinical Trials Actively Recruiting Patients With Melanoma Compiled by Liz Janetschek

T

he information contained in this Clinical Trials Resource Guide includes actively recruiting clinical studies for patients with melanoma. The trials are investigating immune response; vaccine therapy; ipilimumab in conjunction with numerous therapies; and chemotherapy regimens. All of the studies are listed on the National Institutes of Health website at ClinicalTrials.gov.

PHASE I Study Type: Phase I pilot/interventional/parallel assignment Study Title: A Pilot Study of Ipilimumab and Radiation in Poor Prognosis Melanoma Study Sponsor and Collaborators: Duke University Purpose: To assess the safety of the combination of ipilimumab administered concurrently with radiotherapy for patients with locally advanced or unresectable melanoma and patients at high risk for recurrence after resection Primary Outcome Measures: Compare the off-line spectral analyses of saline and alcohol skin swabs of normal control, melanoma-negative and melanoma-positive skin biopsies [time frame: 1 day] Principal Investigator: Nikolaus Gravenstein, MD, University of Florida; 352-494-4938, ngravenstein@ anest.ufl.edu ClinicalTrials.gov Identifier: NCT02393001 Study Type: Phase I/interventional/ single-group assignment Study Title: A Phase I Trial of a Recombinant Human hsp110-gp100 Chaperone Complex Vaccine for Advanced Stage IIIB/C or IV Melanoma Study Sponsor and Collaborators: Roswell Park Cancer Institute, National Cancer Institute (NCI) Purpose: To study the side effects and best dose of vaccine therapy in treating patients with stage III-IV advanced melanoma Primary Outcome Measures: Maximum tolerated dose of recombinant human hsp110-gp100 chaperone complex melanoma vaccine based on the probability of dose-limiting toxicity (DLT), graded according to National

Cancer Institute Common Terminology Criteria for Adverse Events version 4 [time frame: up to 30 days after the last vaccine dose] Principal Investigator: John Kane, MD, FACS, Roswell Park Cancer Institute; 877-275-7724, ASKRPCI@roswellpark.org ClinicalTrials.gov Identifier: NCT01744171 Study Type: Phase I/interventional/ single-group assignment Study Title: A Phase I Trial With a Personalized Neoantigen Cancer Vaccine in Melanoma Study Sponsor and Collaborators: Dana-Farber Cancer Institute Purpose: To determine if it is possible to make and administer safely a vaccine against melanoma by using information gained from specific characteristics of the participant’s own melanoma Primary Outcome Measures: Number of participants experiencing clinical and laboratory adverse events [time frame: 7 weeks from first study drug administration] Number of participants for whom sequencing and analysis leads to identification of at least 10 actionable peptides to initiate vaccine production [time frame: 12 weeks] Principal Investigator: Patrick Ott, MD, PhD, Brigham and Women’s Hospital/Dana-Farber Cancer Institute; 617-582-5030, Patrick_Ott@dfci.harvard.edu ClinicalTrials.gov Identifier: NCT01744171

PHASE I/II Study Type: Phase I/II/interventional/single-group assignment Study Title: A Phase I/II Trial to Evaluate the Safety, Immunogenicity, and Clinical Activity of a Helper Peptide Vaccine Plus CTLA-4 Blockade in Advanced Melanoma (Mel62; 6PAC) Study Sponsor and Collaborators: Craig L. Slingluff Jr, MD, University of Virginia Purpose: To evaluate whether it is safe to administer a peptide vaccine with ipilimumab, and to evaluate the effects of the combination of the peptide vaccine and ipilimumab on the immune system Primary Outcome Measures: Safety (Adverse event profile) [time frame: 30 days after the last vaccination]

Principal Investigator: Elizabeth Gaughan, MD, University of Virginia; contact Emily Allred, PhD; 434-9821902, eh4m@virginia.edu ClinicalTrials.gov Identifier: NCT02385669 Study Type: Phase I/II/interventional/single-group assignment Study Title: A Phase I/II Trial of Vemurafenib and Metformin to Unresectable Stage IIIC and Stage IV BRAF V600E+ Melanoma Patients Study Sponsor and Collaborators: James Graham Brown Cancer Center, University of Louisville Purpose: To evaluate the safety of Vemurafenib in combination with Metformin in melanoma patients Primary Outcome Measures: Observation of CTCAE grade 4 or higher adverse events in six patients [time frame: Duration of phase I portion, approximately 6 months] Principal Investigator: Jason A. Chesney, MD, PhD, James Graham Brown Cancer Center-University of Louisville; 502-562-3429, jason. chesney@louisville.edu ClinicalTrials.gov Identifier: NCT01638676

PHASE II Study Type: Phase II/interventional/ parallel assignment Study Title: A Phase II, Open-Label, Multicenter, Randomized Study of CDX-1401, a Dendritic Cell Targeting NY-ESO-1 Vaccine, in Patients With Malignant Melanoma Pre-treated With Recombinant CDX-301, a Recombinant Human Flt3 Ligand Study Sponsor and Collaborators: National Cancer Institute (NCI) Purpose: To study how well DEC205/NY-ESO-1 fusion protein CDX1401 (CDX-1401) and neoantigenbased melanoma-poly-ICLC vaccine (poly-ICLC) vaccine therapy work when given with or without recombinant flt3 ligand (CDX-301) in treating patients with stage IIB-IV melanoma Primary Outcome Measures: Immune T-cell response to NY-ESO-1 [time frame: Up to 12 weeks after final vaccination] Principal Investigator: Nina Bhardwaj, MD, PhD, Cancer Immunotherapy Trials Network; contact desired location (multiple trial sites listed on cancer.gov)

ClinicalTrials.gov NCT02129075

Identifier:

Study Type: Phase II/interventional/ single-group assignment Study Title: A Pilot Study of the Administration of Young Tumor Infiltrating Lymphocytes Following a NonMyelo­ablative Lymphocyte Depleting Chemotherapy Regimen in Metastatic Melanoma Study Sponsor and Collaborators: National Cancer Institute (NCI), National Institutes of Health Clinical Center Purpose: To study if chemotherapy and white blood cell therapy without aldesleukin is a safe and effective treatment for metastatic melanoma Primary Outcome Measures: Determine whether objective responses can be mediated in patients with metastatic melanoma [time frame: Approximately 2 years] Principal Investigator: Steven A. Rosenberg, MD, NCI; 301-496-4164, sar@mail.nih.gov ClinicalTrials.gov Identifier: NCT0146881

PHASE III Study Type: Phase III/interventional/parallel assignment Study Title: A Phase III Randomized Study of Adjuvant Ipilimumab Anti-­ CTLA4 Therapy Versus HighDose Interferon a-2b for Resected High Risk Melanoma Study Sponsor and Collaborators: National Cancer Institute (NCI) Purpose: To study ipilimumab to see how well it works compared to highdose interferon alfa-2b in treating patients with high-risk stage III-IV melanoma that has been removed by surgery Primary Outcome Measures: Overall survival and recurrence-free survival [time frame: From randomization to the time of disease recurrence or death from any cause, assessed up to 20 years] Principal Investigator: Ahmad Tarhini, MD, PhD, ECOG-ARIN Cancer Research Group; contact desired location (multiple trial sites listed on cancer.gov) ClinicalTrials.gov Identifier: NCT01274338 n Editor’s Note: The clinical trials presented here do not represent all the trials listed on ClinicalTrials.gov. For the complete list, go to ClinicalTrials.gov.


MANY PEOPLE DIAGNOSED WITH SQUAMOUS NSCLC:

STILL WAITING FOR NEW TREATMENT OPTIONS TO EVOLVE1

EVERY ADVANCE MATTERS In 2012, there were an estimated 450,000 cases of squamous NSCLC worldwide.*2,3 Unfortunately, despite decades of research, the median overall survival for squamous NSCLC remains at approximately 8 to 10 months.4,5 Lilly Oncology is committed to finding treatment advances for people living with lung cancer.

VISIT SQUAMOUSNSCLC.COM TO LEARN MORE.

S:13 in

T:14 in

B:14.25 in

MANY PEOPLE DIAGNOSED WITH SQUAMOUS NSCLC: * Worldwide prevalence of squamous NSCLC based on World Health Organization/GLOBOCAN estimates of worldwide lung cancer prevalence and distribution of histology as reported by the American Cancer Society.

STILL WAITING

References: 1. Referenced with permission from The NCCN Clinical Practice Guidelines in Oncology (NCCN Guidelines®) for Non-Small Cell Lung Cancer V.4.2015. © National Comprehensive Cancer Network, Inc 2015. All rights reserved. Accessed February 16, 2015. To view the most recent and complete version of the guidelines, go online to http://www.nccn.org. NATIONAL COMPREHENSIVE CANCER NETWORK®, NCCN®, NCCN GUIDELINES®, and all other NCCN content are trademarks owned by the National Comprehensive Cancer Network, Inc. 2. From Ferlay J., Soerjomataram I, Ervik M., Dikshit R., Eser S., Mathers C., Rebelo M., Parkin D.M., Forman D., Bray, F. GLOBOCAN 2012 v1.0, Cancer Incidence and Mortality Worldwide: IARC CancerBase No. 11 [Internet]. Lyon, France: International Agency for Research on Cancer; 2013. Available from: http://globocan.iarc.fr, accessed on 23/02/2015. 3. American Cancer Society. What is non-small cell lung cancer? http://www.cancer.org/cancer/lungcancer-non-smallcell/detailedguide/ non-small-cell-lung-cancer-what-is-non-small-cell-lung-cancer. Accessed February 16, 2015. 4. Hoang T, Dahlberg SE, Schiller JH, et al. Does histology predict survival of advanced non-small cell lung cancer patients treated with platin-based chemotherapy? An analysis of the Eastern Cooperative Oncology Group Study E1594. Lung Cancer. 2013;81(1):47-52. 5. Ellis LM, Bernstein DS, Voest EE, et al. American Society of Clinical Oncology perspective: raising the bar for clinical trials by defining clinically meaningful outcomes. J Clin Oncol. 2014;32(12):1277-1280.

MARCH 2015

GLOONC00009a

CLIENT: Lilly 30725_ELNECI_Journal_ad_Tab_M8.indd 1

FINISH SIZE: 10.5” wide x 14” high

JOB#: 30725

COLLECT DATE:

R|O|U|T|I|N|G| |S|T|A|G|E ED

3/4/15 1:31 PM

ARTIST:

LOCATION: Mechanicals

CW

AD

M8

CPM

ACD

AE

inser v

FOR NEW TREATMENT OPTIONS TO EVOLVE1

EVERY ADVANCE MATTERS In 2012, there were an estimated 450,000 cases of squamous NSCLC worldwide.*2,3 Unfortunately, despite decades of research, the median overall survival for squamous NSCLC remains at approximately 8 to 10 months.4,5 Lilly Oncology is committed to finding treatment advances for people living with lung cancer.

VISIT SQUAMOUSNSCLC.COM TO LEARN MORE.

* Worldwide prevalence of squamous NSCLC based on World Health Organization/GLOBOCAN estimates of worldwide lung cancer prevalence and distribution of histology as reported by the American Cancer Society.

References: 1. Referenced with permission from The NCCN Clinical Practice Guidelines in Oncology (NCCN Guidelines®) for Non-Small Cell Lung Cancer V.4.2015. © National Comprehensive Cancer Network, Inc 2015. All rights reserved. Accessed February 16, 2015. To view the most recent and complete version of the guidelines, go online to http://www.nccn.org. NATIONAL COMPREHENSIVE CANCER NETWORK®, NCCN®, NCCN GUIDELINES®, and all other NCCN content are trademarks owned by the National Comprehensive Cancer Network, Inc. 2. From Ferlay J., Soerjomataram I, Ervik M., Dikshit R., Eser S., Mathers C., Rebelo M., Parkin D.M., Forman D., Bray, F. GLOBOCAN 2012 v1.0, Cancer Incidence and Mortality Worldwide: IARC CancerBase No. 11 [Internet]. Lyon, France: International Agency for Research on Cancer; 2013. Available from: http://globocan.iarc.fr, accessed on 23/02/2015. 3. American Cancer Society. What is non-small cell lung cancer? http://www.cancer.org/cancer/lungcancer-non-smallcell/detailedguide/ non-small-cell-lung-cancer-what-is-non-small-cell-lung-cancer. Accessed February 16, 2015. 4. Hoang T, Dahlberg SE, Schiller JH, et al. Does histology predict survival of advanced non-small cell lung cancer patients treated with platin-based chemotherapy? An analysis of the Eastern Cooperative Oncology Group Study E1594. Lung Cancer. 2013;81(1):47-52. 5. Ellis LM, Bernstein DS, Voest EE, et al. American Society of Clinical Oncology perspective: raising the bar for clinical trials by defining clinically meaningful outcomes. J Clin Oncol. 2014;32(12):1277-1280.

MARCH 2015

GLOONC00009a


The ASCO Post  |   JULY 25, 2015

PAGE 68

Inside the Black Box

Inside the Black Box

FDA’s Pregnancy Category Labeling By Todd Palmby, PhD, and Eias Zahalka, PhD, MBA

INSIDE THE BLACK BOX is an occasional column providing insight into the U.S. Food and Drug Administration (FDA) and its policies and procedures. In this installment, FDA supervisory toxicologist Todd Palmby, PhD, and pharmacologist Eias Zahalka, PhD, MBA, discuss the approach taken in the Office of Hematology and Oncology Products (OHOP) to assess and convey the risk of potential embryo-fetal harm when treating pregnant patients with cancer. Drs. Palmby and Zahalka are members of the Division of Hematology Oncology Toxicology (DHOT) within the OHOP. The DHOT is responsible for reviewing and evaluating nonclinical (animal) studies submitted to the OHOP.

W

ith an increasing number of products approved to treat cancer patients in the United States, oncologists sometimes struggle with making informed decisions about treatment options for pregnant patients and patients who are thinking about becoming pregnant. Finding and interpreting information about a product’s risk to embryo-fetal development can be challenging. The ASCO Post spoke with Drs. Palmby and Zahalka about the basis for determining the risk associated with a product to cause embryo-fetal harm if used to treat a pregnant patient with cancer and how that risk is conveyed to health-care professionals and patients.

Balancing Risk and Benefit How can a health-care professional or patient determine the level of risk of fetal harm if a product is used to treat a pregnant patient? Dr. Palmby: The best place for a health-care professional to learn about a product’s level of risk for causing embryo-fetal harm is the drug labeling, also referred to as the package insert. What is known about the embryo-fetal risk is found in the Pregnancy section of the labeling. Depending on the severity of the risk, information may also be found in the Warnings and Precautions section, the Contraindications section, and sometimes within a Boxed Warning. Information on embryo-fetal risk may also be included in other communications associated with the product, such as a Medication Guide or other FDA-approved patient labeling.

When a patient with cancer is pregnant, discussions of treatment options between the patient and the health-care professional can be guided by information on the level of risk to the fetus and balanced against the benefits of treatment. The Patient Counseling Information section of the package insert includes advice for health-care professionals to convey to women that a product may have the potential to cause embryo-fetal harm and thus to recommend avoiding pregnancy.

Quantitative Assessment of Risk How is a product’s risk of causing fetal harm conveyed in a label, and what information is used to determine the level of risk to the fetus if the product is administered to a pregnant patient? Dr. Zahalka: Prior to product approval, clinical trials do not generally include pregnant patients. Therefore, when a new drug is approved, animal data are usually used to establish such risk. The source of the animal data can be from peer-reviewed published scientific data or original nonclinical studies (ie, nonclinical studies investigating effects during organogenesis or the peri-/postnatal period). The nonclinical studies can be conducted in rodent and/or other species (usually rabbits for small molecules and monkeys for biotechnology products). A quantitative assessment of risk is presented in the Pregnancy section of the labeling in the form of calcu-

lated exposure- or dose-multiples of the human dose. These multiples are provided to inform the health-care professional how close the clinical exposure or dose is to the observed adverse effect level in animal models. Once a drug has been studied or is on the market for some time, human data may be available and can be added to the labeling. The source of clinical data can be obtained from observational studies conducted during the postmarketing period or from published case reports.

FDA Clinical Reviewers

Todd Palmby, PhD

Pregnancy and Lactation Labeling Rule What is the Pregnancy and Lactation Labeling Rule, and how does it change the labeling of oncology products to convey the risk of use during pregnancy? Dr. Zahalka: The wheels of change have been in motion since the late 1990s and will affect the labeling of all pharmaceuticals, including oncology products. In 2008, a proposed Pregnancy and Lactation Labeling Rule (PLLR) was issued. The PLLR issued on December 4, 2014, will change the way product labeling organizes the information about embryo-fetal risk. The letter categories will be discontinued and replaced by a narrative risk summary, including the available human or animal data. The final PLLR, which goes into effect on June 30, 2015, incorporated comments provided by experts in the field and important stakeholders who are responsible for making the changes in their product labeling including the pharmaceutical industry. The new label format will be applied to any drug or biologic approved after implementation of the PLLR as well as to previously approved drugs that are subject to label revision by way of FDA’s 2006 Physician Labeling Rule. Revision of the labels of previously approved drugs to conform to the PLLR format is expected to be implemented over 3 to 5 years.

More Detailed Information What was the purpose for changing the pregnancy category approach in the package insert? Dr. Zahalka: The main concern with labeling practices using pregnancy categories is that complete risk information is not conveyed by a single let-

Eias Zahalka, PhD, MBA

ter category rating system. For instance, the spectrum of animal data for a Category C labeled drug on which the risk was based could range from minor fetal body weight changes to severe malformations, which gave the reader no indication where along that spectrum this drug fell. Additional criticisms are that the animal data descriptions in the pregnancy section cannot be understood by the average person and that the letter system utilized gave the reader an incorrect impression that reproductive risk increases from Category A to X. The goal of the PLLR is to give health-care professionals and patients more detailed information in the labeling regarding the potential risk of drug administration during pregnancy and lactation. The labeling will replace the letter category with a risk summary paragraph, which provides an overview of clinical and nonclinical effects, along with a reader-friendly estimate of risk. It is hoped that the additional information will enable a more informed approach toward managing risk.

Changes in Labeling Section How will the package inserts differ under the Pregnancy and Lactation Lacontinued on page 70


A CE Event for Advanced Practitioners in Oncology

NOVEMBER 5-8, 2015 JW Marriott Desert Ridge • Phoenix, Arizona

Call for Abstracts SUBMISSION DEADLINE

September 1, 2015 Submit your research for presentation at this exciting conference for advanced practitioners in oncology. Topics can include research, educational initiatives, professional development, clinical projects, and evidence-based practice improvements. Abstracts accepted for presentation at JADPRO LIVE will also be published in the November/December issue of the Journal of the Advanced Practitioner in Oncology indexed in PubMed Central®. Deadline for abstract submission is September 1, 2015. For more information, visit jadprolive.com

This CE/CME/CEU accredited conference is jointly provided by:


The ASCO Post  |   JULY 25, 2015

PAGE 70

Inside the Black Box Pregnancy Category Labeling continued from page 68

beling Rule compared with the former pregnancy category approach? Dr. Zahalka: There will be no major change in content as compared with the former approach; however, the labeling section will be organized somewhat differently. Under the PLLR, the labeling will contain three subsections: one for pregnancy, one for lactation, and one for females and males of reproductive age. In the Pregnancy section, the labeling change eliminates the pregnancy categories A, B, C, D, and X. The previous Sections 8.1 “Pregnancy” and 8.2 “Labor and Delivery” are merged into the single “Pregnancy” subsection under the PLLR. Additionally, section 8.3 “Nursing Mothers” is renamed the “Lactation” subsection and is now section 8.2. A new 8.3 section is named Females and Males of Reproductive Potential. (For a comparison of the current prescription drug labeling and the new PLLR labeling requirements, visit http://www.fda.gov/drugs/developmentapprovalprocess/developmentresources/labeling/ucm093307.htm.) The Pregnancy and Lactation subsections will have three principal components: a risk summary, a clinical considerations section, and a data section. The inclusion of a narrative risk summary in these subsections is a key change under the PLLR. It is intended to provide the reader with enough detail to inform risk but not extensive technical details. In this manner, it is hoped that the Pregnancy/Lactation sections of the label will be more informative than the Pregnancy category labeling system and will be presented in a more reader-friendly format. The subsection Females and Males of Reproductive Potential includes information on pregnancy testing, recommendations on the use of contraception, and the risks of infertility. Finally, when there is an open pregnancy registry for the product, contact information will be included in the Pregnancy section.

Oncology Products vs Other Products Are the pregnancy sections of the package insert handled differently for oncology products than for products to treat other diseases? Dr. Palmby: In many cases, products approved for the treatment of cancer pose a risk to embryo-fetal development if administered to a pregnant patient. Although some anticancer agents were previously a Pregnancy Category C or X, the majority were Cat-

egory D. Under the PLLR, products will include a risk statement indicating the potential to cause embryo-fetal toxicity. Most oncology specialists are aware of the potential for embryo-fetal harm if an anticancer product is administered to a pregnant patient, which must be balanced against the potential benefit of treatment. Products approved to treat diseases other than cancer that are teratogenic in animals may be contraindicated for use in pregnancy, especially if there are also data of adverse effects in pregnant patients or a mechanism of action indicating an increased risk of embryo-fetal toxicity. In contrast, there are a limited number of anticancer products that are contraindicated for use in pregnancy. This is due to a number of factors, including the relatively controlled setting in which anticancer agents are used, the need to limit barriers to obtaining necessary treatments for cancer patients, the serious and life-threatening nature of the disease, the often limited therapeutic options available, and the desire to leave treatment choices in the hands of patients and their health-care professionals.

of a product if it is administered to a pregnant patient while not contraindicating its use in pregnancy. Historically, this was accomplished by assigning a pregnancy Category D, whereas under the PLLR, a risk summary will be used. Some patients are diagnosed with cancer while pregnant, others become pregnant while being treated for their disease, and still other patients wish to become pregnant. Labeling a product as having the potential to cause embryo-fetal toxicity lets patients and their health-care professional choose the appropriate treatment plan given the seriousness of their disease and the need for treatment.

Risk Evaluation and Mitigation Strategies What strategies are used to mitigate the potential embryo-fetal risk a product may have? Dr. Palmby: For oncology products, these strategies range from product labeling to Risk Evaluation and Mitigation Strategies (REMS). For many products approved to treat cancer, including available information in the package insert is sufficient to convey the appropriate level

The Pregnancy and Lactation subsections will have three principal components: a risk summary, a clinical considerations section, and a data section. The inclusion of a narrative risk summary in the subsection is a key change. —Todd Palmby, PhD

Intent of Pregnancy Labeling Why do labels often state that a product can cause embryo-fetal toxicity when administered to a pregnant woman? Dr. Palmby: For many oncology products, the risk of embryo-fetal toxicity is significant, based on a combination of data from pregnant animals and the product’s mechanism of action. In addition, many products used to treat cancer are dosed near a maximal tolerated dose in patients, which means that often there is little to no difference between the drug exposures achieved in patients compared with those in pregnant animals during embryo-fetal development studies. The toxicity to the developing fetus can be expected, since many of the drugs used are often designed to damage DNA or inhibit a pathway or process critical for cell proliferation or survival. The intent of pregnancy labeling for oncology products is to convey the significant level of embryo-fetal risk

of risk to health-care professionals. It is recognized that oncology specialists treating female patients with cancer are aware of the potential risk to a developing fetus that many anticancer products pose and have discussions with their patients about avoiding pregnancy while being treated. In a few cases, oncology products have been contraindicated for use in pregnant patients and have associated REMS, such as thalidomide (Thalomid), lenalidominde (Revlimid), and pomalidomide (Pomalyst). Boxed warnings in the package insert are sometimes warranted for particular products. The labels for trastuzumab (Herceptin), pertuzumab (Perjeta), and ado-trastuzumab emtansine (Kadcyla) include a boxed warning due to postmarketing data-reporting effects in offspring of pregnant patients treated with trastuzumab and related mechanisms of action between these products. In addition, a large number of women of reproductive potential are likely to be treated with these products.

GUEST EDITOR

Richard Pazdur, MD

Inside the Black Box is Guest Edited by Richard Pazdur, MD, Director of the FDA’s Office of Hematology and Oncology Products. Does the indicated patient population matter when determining the level of FDA’s concern about a product that can cause embryo-fetal harm? Dr. Palmby: Yes, the patient population is a large part of how FDA determines the level of risk management that is necessary for a product. Many components are considered, including animal data (eg, severity of effects, relative exposure) and mechanism of action, age range and reproductive potential of the population to be treated, the intended prescribers, whether patients have been previously treated, and the stage of the disease. For example, the active ingredient in vismodegib (Erivedge) is a hedgehog pathway inhibitor, which caused severe malformations in offspring of pregnant rats at exposures significantly lower than those observed in patients treated with the recommended dose. Similar findings were first observed with cyclopamine, a plant alkaloid that also inhibits the hedgehog pathway and causes craniofacial deformities in the offspring of sheep grazing in fields with weeds containing this chemical. In addition, there is a large body of literature describing the critical role of the hedgehog pathway in embryonic development. Following an extensive discussion among FDA reviewers and the applicant, it was decided that REMS were needed for embryo-fetal toxicity, but certain elements should not be included due to the specific indicated population and the impact those elements may have on patient access to the product. The REMS put into place when vismodegib was approved were specific for that indication and would have to be reevaluated for any subsequent indications considered for approval. continued on page 71


ASCOPost.com  |   JULY 25, 2015

PAGE 71

Announcements

Memorial Sloan Kettering Cancer Center Receives Gift of $150 Million From David H. Koch

M

emorial Sloan Kettering Cancer Center (MSK) announced that it has received the largest gift in its history, a commitment of $150 million, from long-time MSK board member David H. Koch. This unprecedented contribution will fund a state-of-the-art outpatient medical facility to be known as The David H. Koch Center for Cancer Care. The gift is the largest single donation ever made by Mr. Koch, whose overall contributions to a wide range of organizations and causes nationwide total nearly $1.3 billion. At MSK, where he has served as a member of the Boards of Overseers and Managers since 1990, he has now made $225 million in gifts and pledges, including funds to establish the David H. Koch Center for the Immunologic Control of Cancer, endow two David H. Koch Chairs, and provide ongoing support for prostate cancer research. “We are deeply grateful to Mr. Koch for his visionary generosity, which will help ensure that Memorial Sloan Kettering continues to set the pace for advances in patient care,” said Douglas A. Warner III, Chairman of the Boards of Overseers and Managers. “It is my ardent hope now that the new Center for Cancer Care will transform cancer treatment worldwide, with the utmost emphasis on the needs of patients and their families,” said Mr. Koch. “The novel clinical trials that will take place within its walls, and the other vital services including interventional radiology, outpatient bone marrow transplants, and programs for patients with lung, head and neck, and hematologic cancers, promise to be medically revolutionary, and I am

honored to support this important work.” “The creation of this outstanding facility will be a real game changer for Memorial Sloan Kettering,” said Craig B.

Thompson, MD, MSK’s President and Chief Executive Officer. “Thanks to Mr. Koch’s landmark commitment, we will be able to offer cutting-edge therapies

and an expanded roster of innovative clinical trials to more patients than ever.” The David H. Koch Center for Cancer Care is scheduled to open in 2019. n

When one didn’t exist, When one didn’t exist, Dana-Farber Dana-Farber created a pathway a pathway tocreated treat children with to treat children with rare brain cancers. rare brain cancers.

Pregnancy Category Labeling continued from page 70

Resources for Patients How can a patient learn more about taking medications during pregnancy? Dr. Zahalka: There is additional information in the following links to the FDA’s website that may be useful: http://www.fda.gov/forconsumers/byaudience/forwomen/womenshealthtopics/ucm117976.htm h tt p : / / w w w. f d a . g o v / f o rc o n sumers/byaudience/for women/ ucm118567.htm http://www.fda.gov/drugs/developmentapprovalprocess/developmentresources/labeling/ucm093307.htm n Disclosure: Drs. Palmby and Zahalka reported no potential conflicts of interest.

Researchers at Dana-Farber have been studying some of the rarest and most dangerous childhood brain cancers, hoping to develop new, more effective approaches treatment. Ourofwork with cancers likedangerous DIPG, a brain stem glioma that affects Researchers at Dana-Farber have beentostudying some the rarest and most childhood brain cancers, hoping onlyto200 children each year and AT/RT, a lethaltobrain tumor that 100cancers annually, creating a better develop new, more effective approaches treatment. Our affects work with likeisDIPG, a brain stemunderstanding glioma that affects of how attack theseeach diseases. Taking on rare childhood cancers helped open new pathwaysa in the study of onlyto200 children year and AT/RT, a lethal brain tumor thathas affects 100usannually, is creating better understanding many cancers well,diseases. includingTaking ovarian, colon and possibly pancreatic While the biology of study cancerof of adult how to attackas these onbreast, rare childhood cancers has helped uscancer. open new pathways in the remains we believe even small steps can breast, lead to colon giant leaps forwardpancreatic toward a brighter children, manycomplex, adult cancers as well, including ovarian, and possibly cancer. future While for theour biology of cancer and remains for everyone. complex, we believe even small steps can lead to giant leaps forward toward a brighter future for our children, and for everyone. See videos, whitepapers and more at DiscoverCareBelieve.org/DIPG. See videos, whitepapers and more at DiscoverCareBelieve.org/DIPG.

© 2015 Dana-Farber Cancer Institute

© 2015 Dana-Farber Cancer Institute


The ASCO Post  |   JULY 25, 2015

PAGE 72

Integrative Oncology American Ginseng By Jyothirmai Gubili, MS Editor, Integrative Medicine Service, Memorial Sloan Kettering Cancer Center

T

he use of dietary supplements by patients with cancer has increased significantly over the past 20 years despite insufficient evidence of safety and effectiveness. Finding reliable sources of information about dietary supplements can be daunting. Patients typically rely on family, friends, and the Internet, often receiving misleading information. The ASCO Post’s Integrative Oncology series is intended to facilitate the availability of evidence-based information on integrative and complementary therapies commonly used by patients with cancer. We chose American ginseng for this issue because of its increasing use by patients with cancer. Compiled by Barrie R. Cassileth, PhD, and Jyothirmai Gubili, MS, of Memorial Sloan Kettering Cancer Center. The free About Herbs website is managed by K. Simon Yeung, PharmD, MBA, LAc, of Memorial Sloan Kettering Cancer Center.

American Ginseng Scientific name: Panax quinquefolius Common names: Xi yang shen, Tienchi ginseng, western ginseng, five fingers

Overview

A perennial herb with a fleshy root, American ginseng is a native of eastern North America. It was used by Native Americans to treat headaches, fever, indigestion, and infertility. The plant was introduced to China over 300 years ago, where it was prized as a health tonic and continues to be popular today. Both the fresh and dried forms of the root are used in herbal remedies for a variety of ailments. The dried roots of American ginseng are also

Jyothirmai Gubili, MS

used in supplemental form to improve athletic performance, strength, and stamina and to treat diabetes and cancer. Extracts of American ginseng are available in the form of powders, capsules, or liquid tinctures. The whole roots are used to prepare tea. However, quality control is a major issue. Because of the heavy demand for and high price of this herb, adulteration with morphologically similar plants is not uncommon. American ginseng should not be confused with Panax ginseng or Siberian ginseng, which have different medicinal properties.

patients with diabetes5 and that it is safe for long-term use6; has a modest effect in reducing the number and severity of colds7; and enhances working memory in healthy adults8 and in patients with schizophrenia.9 Studies conducted to explore the anticancer potential of American ginseng revealed that its activity was enhanced when combined with antioxidants.10 It had synergistic effects with fluorouracil against colorectal cancer cells11 and conferred protection against oxidative stress in irradiated human lymphocytes.12 Data from an epidemiologic study indicate that American ginseng improved survival and quality of life in patients with breast cancer patients.13 Recent findings from a randomized controlled trial support its benefits in improving cancer-related fatigue.14

The Science

Ginsenosides, the saponin glycosides present in the root and other parts of the plant, have been identified as the active compounds. They have been shown to exert both stimulatory and inhibitory effects on the central nervous system,1 alter cardiovascular tone, enhance humoral and cellular-dependent immunity, and exert anticancer effects in vitro.2 The ginsenoside Rg1 is associated with improvements in humoral and cell-mediated immune response and increases in T helper cells, T lymphocytes, and natural killer cells in mice.3 A metabolite of the ginsenoside Rb1, compound K, was shown to have antiproliferative effects.4 Clinical data suggest that American ginseng may improve glucose control in

Herb-Drug Interactions

Warfarin: American ginseng has been shown to antagonize warfarin’s effects.15 Cytochrome P450 (CYP) 3A4 substrates: Certain ginsenosides can induce CYP3A4 and may affect the metabolism of some drugs that are substrates of this enzyme.16,17

Contraindications

Patients with breast cancer should avoid American ginseng, as it may stimulate the growth of breast cancer cells.18 n

Disclosure: Ms. Gubili reported no potential conflicts of interest.

References 1. Attele AS, Wu JA, Yuan CS: Ginseng pharmacology: Multiple constituents and multiple actions. Biochem Pharmacol 58:1685-1693, 1999. 2. Xie JT, Du GJ, McEntee E, et al: Effects of triterpenoid glycosides from fresh ginseng berry on SW480 human colorectal cancer cell line. Cancer Res Treat 43:49-55, 2011. 3. Chen SE, Sawchuk RJ, Staba EJ: American ginseng. III: Pharmacokinetics of ginsenosides in the rabbit. Eur J Drug Metab Pharmacokinet 5:161-168, 1980. 4. Wang CZ, Du GJ, Zhang Z, et al: Ginsenoside compound K, not Rb1, possesses potential chemopreventive activities in human colorectal cancer. Int J Oncol 40:19701976, 2012. 6. Mucalo I, Jovanovski E, Vuksan V, et al: American ginseng extract (Panax quinquefolius L.) is safe in long-term use in type 2 diabetic patients. Evid Based Complement Alternat Med 2014:969168, 2014. 7. Predy GN, Goel V, Lovlin R, et al: Efficacy of an extract of North American ginseng containing poly-furanosyl-pyranosyl-saccharides for preventing upper respiratory tract infections. CMAJ 173:1043-1048, 2005. 8. Scholey A, Ossoukhova A, Owen L, et al: Effects of American ginseng (Panax quinquefolius) on neurocognitive function: An acute, randomised, double-blind, placebo-controlled, crossover study. Psychopharmacology (Berl) 212:345-356, 2010. 9. Chen EY, Hui CL: HT1001, a proprietary North American ginseng extract, improves working memory in schizophrenia: A double-blind, placebo-controlled study. Phytother Res 26:1166-1172, 2012.

Learn More About

Herbs, Botanicals, & Other Products Visit the free About Herbs website at

www.mskcc.org/aboutherbs


ASCOPost.com  |   JULY 25, 2015

PAGE 73

Integrative Oncology 10. Li B, Wang CZ, He TC, et al: Antioxidants potentiate American ginsenginduced killing of colorectal cancer cells. Cancer Lett 289:62-70, 2010. 11. Li XL, Wang CZ, Sun S, et al: American ginseng berry enhances chemopreventive effect of 5-FU on human colorectal cancer cells. Oncol Rep 22:943-952, 2009. 12. Lee TK, O’Brien KF, Wang W, et al: Radioprotective effect of American ginseng on human lymphocytes at 90 minutes postirradiation: A study of 40 cases. J Altern Complement Med 16:561-567, 2010. 13. Cui Y, Shu XO, Gao YT, et al: Association of ginseng use with survival and quality of life among breast cancer patients. Am J Epidemiol 163:645-653, 2006. 14. Barton DL, Liu H, Dakhil SR, et al: Wisconsin ginseng (Panax quinquefolius) to improve cancer-related fatigue: A randomized, double-blind trial, N07C2. J Natl

Cancer Inst 105:1230-1238, 2013. 15. Yuan CS, Wei G, Dey L, et al: Brief communication: American ginseng reduces warfarin’s effect in healthy patients: A randomized, controlled trial. Ann Intern Med 141:23-27, 2004. 16. Hao M, Ba Q, Yin J, et al: Deglyco-

sylated ginsenosides are more potent inducers of CYP1A1, CYP1A2 and CYP3A4 expression in HepG2 cells than glycosylated ginsenosides. Drug Metab Pharmacokinet 26:201-205, 2011. 17. Hao M, Zhao Y, Chen P, et al: Structure-activity relationship and substrate-

dependent phenomena in effects of ginsenosides on activities of drug-metabolizing P450 enzymes. PLoS One 3:e2697, 2008. 18. Amato P, Christophe S, Mellon PL: Estrogenic activity of herbs commonly used as remedies for menopausal symptoms. Menopause 9:145-150, 2002.

NOW ENROLLING

REFLECTIONS–

a clinical trial program created by Pfizer to investigate the potential biosimilarity of several products trastuzumab

rituximab

infliximab

adalimumab

CLINICAL TRIALS B327-02

GUEST EDITOR

bevacizumab

STUDY DESIGN METASTATIC BREAST CANCER Patients with HER2+ mBC who have not received prior therapy in the first-line setting with the exception of endocrine therapy

PF-05280014 + paclitaxel vs Herceptin® (trastuzumab) + paclitaxel in the first-line treatment of patients with HER2+ metastatic breast cancer (mBC)

Target enrollment: 690 patients

A global, phase 3, comparative clinical trial evaluating the efficacy, safety, pharmacokinetics (PK), and immunogenicity of PF-05280014, an investigational compound being studied as a potential biosimilar to Herceptin. trastuzumab

• Primary endpoint: objective response rate (ORR) • NCT01989676

rituximab

infliximab

1:1 randomization double-blind

adalimumab

bevacizumab PF-05280014 + paclitaxel

trastuzumab + paclitaxel

Barrie R. Cassileth, MS, PhD

I

ntegrative Oncology is guest edited by Barrie R. Cassileth, MS, PhD, Chief of the Integrative Medicine Service and Laurance S. Rockefeller Chair in Integrative Medicine at Memorial Sloan Kettering Cancer Center, New York. The Integrative Medicine Service at Memorial rituximab infliximabSloan Ketteradalimumab ing Cancer Center developed and maintains a free website—About Herbs (www.mskcc.org/aboutherbs)—that provides objective and unbiased information about herbs, vitamins, minerals, and other dietary supplements, and unproved anticancer treatments. Each of the close to 300 and growing number of entries offers health-care professional and patient versions, and entries are regularly updated with the latest research findings. In addition, the About Herbs app, Memorial Sloan Kettering Cancer Center’s very first mobile application, can be downloaded at http://itunes.apple.com/us/app/ about-herbs/id554267162?mt=8. The app is compatible with iPad, iPhone, and iPod Touch devices.

B328-06

FOLLICULAR LYMPHOMA

PF-05280586 vs Rituxan®/MabThera (rituximab) for the first-line treatment of patients with CD20-positive, low tumor burden follicular lymphoma (LTB-FL)

Patients with LTB-FL who have not received prior therapy in the first-line setting Target enrollment: 394 patients

A global, phase 3, comparative clinical trial evaluating the efficacy, safety, PK, and immunogenicity of PF-05280586, an investigational compound being studied as a potential biosimilar to Rituxan/MabThera.

1:1 randomization double-blind

• Primary endpoint: ORR • NCT02213263

PF-05280586

bevacizumab

rituximab

ADVANCED NON-SQUAMOUS NSCLC

B73910-03

Eligible patients with advanced non-squamous NSCLC who have not received prior therapy in the first-line treatment setting and who are eligible to receive carboplatin/paclitaxel/bevacizumab

PF-06439535 + paclitaxel and carboplatin vs Avastin® (bevacizumab) + paclitaxel and carboplatin in the firstline treatment of patients with advanced non-squamous NSCLC

Target enrollment: 798 patients

A global, phase 3, comparative clinical trial evaluating the efficacy, safety, PK, and immunogenicity of PF-06439535, an investigational compound being studied as a potential biosimilar to Avastin.

1:1 randomization double-blind

• Primary endpoint: ORR • NCT02364999

PF-06439535 + paclitaxel and carboplatin

bevacizumab + paclitaxel and carboplatin

PF-05280014, PF-05280586, and PF-06439535 are investigational compounds. For more information about these trials, including secondary endpoints and eligibility criteria, please visit: www.pfizercancertrials.com and www.clinicaltrials.gov or call the Pfizer clinical trial call center at 1-800-718-1021.

BIO752800-01

© 2015 Pfizer Inc.

This information is current as of April 2015. HER2=human epidermal growth factor receptor 2; NSCLC=non-small cell lung cancer. Herceptin® is a registered US trademark of Genentech Inc. Rituxan® is a registered US trademark of Biogen Idec Inc. MabThera is a trademark of F. Hoffmann-La Roche AG. Avastin® is a registered US trademark of Genentech, Inc.

All rights reserved.

April 2015


The ASCO Post  |   JULY 25, 2015

PAGE 74

Journal Spotlight Dermatologic Oncology

Adjuvant Ipilimumab Increases Recurrence-Free Survival After Complete Resection of High-Risk Stage III Melanoma By Matthew Stenger

I

n the phase III European Organisation for Research and Treatment of Cancer (EORTC) 18071 trial reported in The Lancet Oncology, Alexander M.M. Eggermont, MD, PhD, Director General of the Gustave Roussy Cancer Center in Villejuif, France, and colleagues found that adjuvant therapy with the immune checkpoint inhibitor ipilimumab (Yervoy) significantly improved recurrence-free survival vs placebo in patients with completely resected stage III melanoma (ie, with regional lymph node involvement) at high risk of recurrence.1 Ipilimumab was associated with a high rate of immune-related adverse events.

Study Details In this double-blind trial, 951 patients with stage III cutaneous melanoma (excluding lymph node metastasis ≤ 1 mm or in-transit metastasis) and adequate resection of lymph nodes who had not received previous systemic therapy were enrolled at 91 hospitals in 19 countries and randomly assigned between July 2008 and August 2011 to receive ipilimumab (n = 475) or placebo (n = 476). The study drug was given as an intravenous infusion of 10 mg/kg every 3 weeks for four doses and then

every 3 months for up to 3 years. The primary endpoint was recurrence-free survival on independent review in the intention-to-treat population. The ipilimumab and placebo groups were generally balanced for age (median, 51 and 52 years, 17% and 18% ≥ 65 years), sex (62% male in both), stage provided at randomization (IIIA in 21%

ceration (no in 54% and 51%, yes in 41% and 43%, unknown in 4% and 6%).

Recurrence-Free Survival The median number of study medication doses was four in the ipilimumab group and eight in the placebo group, with 42% vs 70% receiving at least one maintenance dose and 29% vs 57% re-

The risk–benefit ratio of adjuvant ipilimumab at this dose and schedule requires additional assessment based on distant metastasis-free survival and overall survival endpoints to define its definitive value. —Alexander M.M. Eggermont, MD, PhD, and colleagues

in both, IIIB in 38% in both, IIIC with 1–3 positive nodes in 15% and 17%, IIIC with ≥ 4 positive nodes in 15% and 16%), American Joint Committee on Cancer 2002 stage (IIIA in 21% and 18%), microscopic (44% and 41%) and macroscopic (56% and 59%) nodal involvement, number of nodes positive on pathology (1 in 46% in both, 2–3 in 34% and 33%, ≥ 4 in 20% and 21%), and ul-

Cell Communication in Cancer

ceiving at least seven doses (ie, approximately 1 year of treatment). After a median follow-up of 2.74 years, median recurrence-free survival was 26.1 months (95% confidence interval [CI] = 19.3–39.3 months) in the ipilimumab group vs 17.1 months (95% CI = 13.4–21.6 months) in the placebo group (hazard ratio [HR] stratified by stage indicated at randomization = 0.75, 95% CI = 0.64–0.90, P = .0013). The effect of ipilimumab was consistent across subgroups, including benefit irrespective of number of positive nodes. The 1-, 2-, and 3-year recurrence-free survival rates were 63.5% vs 56.1%, 51.5% vs 43.8%, and 46.5% vs 34.8%.

Outcomes by Nodal Involvement and Ulceration

By Matthew J. Ware and Biana Godin Vilentchouk, Houston Methodist Research Institute, Houston, Texas This scanning electron microscopy image shows a tunneling nanotube (TNT) between two pancreatic cancer cells. TNTs are thin protrusions of cellular membrane that bridge two neighboring cells and aid in the transfer of molecules and information, including drug resistance. The cells shown in this image are resistant to a common chemotherapeutic drug, gemcitabine. Understanding the formation and role of TNTs in cancer cell communication within the tumor microenvironment will provide insight into the progression of this disease. The NIH National Cancer Institute provides support for this research program.

Three-year recurrence-free survival was 57.6% vs 39.2% among patients with microscopic stage III disease (HR = 0.65, 99% CI = 0.45–0.96, P = .004) and 37.8% vs 31.7% among patients with palpable nodes (HR = 0.81, 99% CI = 0.61–1.08, P = .06). A post hoc analysis of outcome by nodal involvement and ulceration status showed hazard ratios of 0.64 (99% CI = 0.46–0.90) among all patients with ulcerated primary tumor and 0.84 (99% CI = 0.60–1.17) among all with nonulcerated primaries, 0.58 (99% CI = 0.34–0.97) and 0.75 (99% CI = 0.41– 1.37) among patients with microscopic nodal involvement with ulcerated and nonulcerated primaries, and 0.70 (99%

CI = 0.45–1.08) and 0.86 (99% CI = 0.57–1.27) among patients with macroscopic nodal involvement with ulcerated and nonulcerated primaries.

Adverse Events Grade 3 or 4 adverse events occurred in 54% of ipilimumab recipients vs 25% of placebo recipients, and grade 3 or 4 immune-related adverse events occurred in 43% vs 2%. The most common grade 3 or 4 immune-related adverse events in the ipilimumab group were gastrointestinal (16% vs < 1% in the placebo group), hepatic (11% vs < 1%), and endocrine (8% vs 0%). Median time to onset of grade ≥ 2 immune-related adverse events ranged from 4.3 weeks for skin events to 13.1 weeks for neurologic events. Grade 2 to 4 immune-related adverse events other than endocrine events resolved to baseline or grade 1 with use of established management protocols in 82% to 95% of cases, with median time to resolution of 4 to 8 weeks. Endocrine events resolved in 56% of patients, with median time to resolution of 31 weeks. Adverse events led to discontinuation of treatment in 52% of ipilimumab patients, including in 39% during the initial treatment period, vs 4% of placebo patients. Five patients (1%) died from drugrelated adverse events in the ipilimumab group. No treatment-related deaths were reported in the placebo group. The investigators concluded: Adjuvant ipilimumab significantly improved recurrence-free survival for patients with completely resected highrisk stage III melanoma. The adverse event profile was consistent with that observed in advanced melanoma, but at higher incidences in particular for endocrinopathies. The risk-benefit ratio of adjuvant ipilimumab at this dose and schedule requires additional assessment based on distant metastasis-free survival and overall survival endpoints to define its definitive value. n

Disclosure: The trial was funded by BristolMyers Squibb. For full disclosures of the study authors, visit www.thelancet.com.

Reference 1. Eggermont AMM, Chiarion-Sileni V, Grob JJ, et al: Adjuvant ipilimumab versus placebo after complete resection of highrisk stage III melanoma (EORTC 18071): A randomised, double-blind, phase 3 trial. Lancet Oncol 16:522-530, 2015.


ASCOPost.com  |   JULY 25, 2015

PAGE 75

Perspective

Adjuvant Ipilimumab in High-Risk Stage III Melanoma: Encouraging Study Results Yet Questions Remain By Michael A. Postow, MD

I

pilimumab (Yervoy) is a fully human monoclonal antibody that blocks the negative T-cell regulator cytotoxic T-lymphocyte antigen 4 (CTLA-4) and has improved overall survival for patients with unresectable or metastatic melanoma in two phase III studies.1,2 Based upon these results, ipilimumab was tested in the adjuvant setting for patients with resected, stage III melanoma.

A Closer Look at EORTC 18071 The European Organisation for Research and Treatment of Cancer (EORTC) 18071 trial—reported in The Lancet Oncology by Alexander ­Eggermont, MD, and colleagues3 and reviewed in this issue of The ASCO Post—was a double-blinded, randomized-controlled trial where patients with resected stage III cutaneous melanoma (lymph node involvement > 1 mm) received ipilimumab (10 mg/kg every 3

vival information has yet been presented. The investigators should be congratulated for the conduct of this large study involving 951 patients treated in 19 countries worldwide and demonstrating, for the first time, a benefit for ipilimumab in the adjuvant setting. Nonetheless, several important questions remain about adjuvant ipilimumab, particularly when considering this trial’s results against other standard melanoma adjuvant approaches such as interferon alfa2b (Intron A) and observation.

Longer Follow-Up and Further Study Needed As demonstrated by the primary endpoint results of EORTC 18071, adjuvant ipilimumab improves recurrence-free survival, but whether ipilimumab improves overall survival in this context remains unknown. Many treating physicians consider demon-

The positive recurrence-free survival data for ipilimumab in the EORTC 18071 study are encouraging, yet additional follow-up, including overall survival data, is necessary to assess the true value of ipilimumab in the adjuvant setting. —Michael A. Postow, MD

weeks for four doses and then every 3 months for up to 3 years) or placebo. The study’s primary endpoint was recurrencefree survival as assessed by an independent review committee. Dr. E ­ ggermont first reported the trial’s results at the 2014 ASCO Annual Meeting. The study met its primary recurrencefree survival endpoint. Ipilimumab conferred a recurrence-free survival benefit (median recurrence-free survival of 26.1 months, 95% confidence interval [CI] = 19.3–39.3) for ipilimumab vs 17.1 months (95% CI = 13.4–21.6) for placebo (hazard ratio = 0.75, 95% CI = 0.64–0.90, P = .0013). No overall surDr. Postow is Assistant Attending Physician, Melanoma and Immunotherapeutics Oncology Service, Memorial Sloan Kettering Cancer Center, New York.

stration of an overall survival benefit to be a particularly critical endpoint in an adjuvant trial testing any agent that could cause side effects. Pegylated and high-dose interferon, standard adjuvant melanoma treatment options, have improved recurrence-free survival compared with observation.4-6 Yet controversies over interferon’s long-term overall survival benefit and side-effect profile have limited its unanimous use. With longer follow-up, all are hopeful that an overall survival benefit will emerge for ipilimumab administered as an adjuvant therapy. However, since ipilimumab is a standard treatment for patients with recurrent melanoma, many patients in the placebo group who experience disease recurrence will subsequently receive ipilimumab, possibly affecting interpretations of overall survival differ-

ences between the study arms. If there is ultimately no overall survival difference, it would likely be reasonable to reserve ipilimumab treatment for patients upon relapse. In subsequent analyses of this study, it will be important to consider the number of relapsing patients who receive subsequent ipilimumab. In EORTC 18071, adjuvant ipilimumab was administered at a high dose of 10 mg/kg, approximately three times the current U.S. Food and Drug Administration–approved dose of 3 mg/ kg. It remains unclear whether the dose of ipilimumab is related to efficacy. A phase III trial is testing ipilimumab at 10 mg/ kg vs 3 mg/kg in patients with unresectable or metastatic melanoma. Another phase III trial, Eastern Cooperative Oncology Group (ECOG) 1609, is testing ipilimumab 10 mg/kg vs ipilimumab 3 mg/kg vs high-dose interferon in the adjuvant setting and will provide additional information on the relevance of the dose of ipilimumab to therapeutic outcome. The ECOG 1609 trial will also be the first to report the efficacy of ipilimumab vs high-dose interferon. In the EORTC 18071 trial, adverse events led to discontinuation of treatment in 52% of patients receiving ipilimumab, with 40% discontinuing ipilimumab before the end of the initial 12-week dosing period (before maintenance dosing). Although this may seem to be a high number, the clinical significance of this discontinuation rate remains completely unknown, since the likelihood of benefit from ipilimumab is not clearly related to the amount of treatment received. The specific side effects related to ipilimumab in this study (diarrhea, rash, hepatitis, endocrinopathy) were generally similar to those seen in prior trials of patients with unresectable or metastatic melanoma, and most side effects resolved within 4 to 8 weeks with immunosuppressive agents such as steroids. Of note were five ipilimumab treatment-related deaths (1%). Four of these five patients received steroid immunosuppression, and one received antitumor necrosis-alpha therapy. Whether these deaths could have been prevented with alternative immunosuppressive management remains unknown, but they serve as a reminder

that patients receiving ipilimumab require significant clinical attention. As newer immunotherapeutic agents such as those targeting the programmed cell death 1 (PD-1) receptor (nivolumab [Opdivo] and pembrolizumab [Keytruda]) demonstrate success in the metastatic setting,7,8 their potential efficacy as adjuvant therapies remains of great interest. Clinical trials examining adjuvant pembrolizumab vs placebo (NCT02362594) and nivolumab vs ipilimumab (NCT02388906) have been or will soon be initiated for patients with resected melanoma. One potential pitfall to our ultimate understanding of the relative values of specific adjuvant treatments in melanoma will be the varied approaches used in the control arms of the ongoing large adjuvant trials. Some studies use control arms containing regimens such as interferon or ipilimumab, and in other trials, observation remains the control.

Closing Thoughts The positive recurrence-free survival data for ipilimumab in the EORTC 18071 study are encouraging, yet additional follow-up, including overall survival data, is necessary to assess the true value of ipilimumab in the adjuvant setting. In the meantime, enrollment of patients into ongoing clinical trials testing new approaches in the adjuvant treatment of melanoma, such as anti–PD-1 therapy, is encouraged. n

Disclosure: Dr. Postow is a consultant for Bristol-Myers-Squibb and Amgen and has received a research grant from Bristol-Myers Squibb.

References 1. Hodi FS, et al: N Engl J Med 363:711-723, 2010. 2. Robert C, et al: N Engl J Med 364:2517-2526, 2011. 3. Eggermont AM, et al: Lancet Oncol 16:522-530, 2015. 4. Eggermont AM, et al: J Clin Oncol 30:3810-3818, 2012. 5. Kirkwood JM, et al: J Clin Oncol 18:2444-2458, 2000. 6. Kirkwood JM, et al: Clin Cancer Res 10:1670-1677, 2004. 7. Robert C, et al: N Engl J Med 372:2521-2532, 2015. 8. Larkin J, et al: N Engl J Med. May 31, 2015 (early release online).


When faced with overactive signaling . . .

Indications and Usage Jakafi is indicated for treatment of patients with polycythemia vera who have had an inadequate response to or are intolerant of hydroxyurea.

Important Safety Information Treatment with Jakafi can cause thrombocytopenia, anemia and neutropenia, which are each dose-related effects. Perform a pre-treatment complete blood count (CBC) and monitor CBCs every 2 to 4 weeks until doses are stabilized, and then as clinically indicated Manage thrombocytopenia by reducing the dose or temporarily interrupting Jakafi. Platelet transfusions may be necessary Patients developing anemia may require blood transfusions and/or dose modifications of Jakafi

Jakafi is a registered trademark of Incyte Corporation. Š 2015, Incyte Corporation. All rights reserved. RUX-1452a 03/15

Severe neutropenia (ANC <0.5 X 109/L) was generally reversible by withholding Jakafi until recovery Serious bacterial, mycobacterial, fungal and viral infections have occurred. Delay starting Jakafi until active serious infections have resolved. Observe patients receiving Jakafi for signs and symptoms of infection and manage promptly Tuberculosis (TB) infection has been reported. Observe patients taking Jakafi for signs and symptoms of active TB and manage promptly. Prior to initiating Jakafi, evaluate patients for TB risk factors and test those at higher risk for latent infection. Consult a physician with expertise in the treatment of TB before starting Jakafi in patients with evidence of active or latent TB. Continuation of Jakafi during treatment of active TB should be based on the overall risk-benefit determination Progressive multifocal leukoencephalopathy (PML) has occurred with ruxolitinib treatment for myelofibrosis. If PML is suspected, stop Jakafi and evaluate Advise patients about early signs and symptoms of herpes zoster and to seek early treatment


Inhibit the JAK pathway* in polycythemia vera not controlled with hydroxyurea 1-3

Jakafi® (ruxolitinib) is the first and only FDA-approved treatment for patients who have had an inadequate response to or are intolerant of hydroxyurea3 Jakafi demonstrated superior results in a phase 3 trial vs best available therapy3,4†

Primary Response at Week 323,4 80

* Ruxolitinib, a kinase inhibitor, inhibits JAK1 and JAK2 (Janus-associated kinases 1 and 2), which mediate the signaling of cytokines and growth factors important for hematopoiesis and immune function.3 A randomized, open-label, active-controlled phase 3 trial comparing Jakafi with best available therapy (BAT) in 222 patients. Best available therapy included hydroxyurea (60%), interferon/pegylated interferon (12%), anagrelide (7%), pipobroman (2%), lenalidomide/thalidomide (5%), and observation (15%). Patients had been diagnosed with polycythemia vera for at least 24 weeks, had an inadequate response to or were intolerant of hydroxyurea, required phlebotomy, and exhibited splenomegaly. The primary end point was the proportion of subjects achieving a response at week 32, with response defined as having achieved both hematocrit (Hct) control (the absence of phlebotomy eligibility beginning at the week 8 visit and continuing through week 32) and spleen volume reduction (a ≥35% reduction from baseline in spleen volume at week 32). Phlebotomy eligibility was defined as Hct >45% that is ≥3 percentage points higher than baseline or Hct >48% (lower value).3,4

P < 0.0001

Jakafi (n = 110) BAT (n = 112)

(n = 66)

38%

40

21%

a

20 0

a

Individual Components of Primary End Point

60%

60

Patients (%)

Composite Primary End Point

(n = 23)

1%b

(n = 22)

(n = 1)

Hct Control + Spleen Volume Reduction

95% CI, 14%-30%

When discontinuing Jakafi, myeloproliferative neoplasmrelated symptoms may return within one week. After discontinuation, some patients with myelofibrosis have experienced fever, respiratory distress, hypotension, DIC, or multi‐organ failure. If any of these occur after discontinuation or while tapering Jakafi, evaluate and treat any intercurrent illness and consider restarting or increasing the dose of Jakafi. Instruct patients not to interrupt or discontinue Jakafi without consulting their physician. When discontinuing or interrupting Jakafi for reasons other than thrombocytopenia or neutropenia, consider gradual tapering rather than abrupt discontinuation Non‐melanoma skin cancers including basal cell, squamous cell, and Merkel cell carcinoma have occurred. Perform periodic skin examinations The three most frequent non-hematologic adverse reactions (incidence >10%) were bruising, dizziness and headache A dose modification is recommended when administering Jakafi with strong CYP3A4 inhibitors or fluconazole or in patients with renal or hepatic impairment. Patients should be closely monitored and the dose titrated based on safety and efficacy

20%

b

(n = 42)

1%

(n = 1)

Hct Control Without Phlebotomy

≥35% Spleen Volume Reduction

95% CI, 0%-5%

Use of Jakafi during pregnancy is not recommended and should only be used if the potential benefit justifies the potential risk to the fetus. Women taking Jakafi should not breast-feed

Please see Brief Summary of Full Prescribing Information for Jakafi on the following page. References: 1. Rampal R et al. Blood. 2014;123(22):e123-e133. 2. Keohane C et al. Biologics. 2013;7:189-198. 3. Jakafi Prescribing Information. Wilmington, DE: Incyte Corporation. 4. Vannucchi AM et al. N Engl J Med. 2015;372(5):426-435.

Review the clinical trial data at

www.jakafidata.com


Table 3: Polycythemia Vera: Treatment Emergent Adverse Events Occurring in ≥ 6% of Patients on Jakafi in the Open-Label, Active-controlled Study up to Week 32 of Randomized Treatment Jakafi (N=110) BRIEF SUMMARY: For Full Prescribing Information, see package insert. CONTRAINDICATIONS None. WARNINGS AND PRECAUTIONS Thrombocytopenia, Anemia and Neutropenia Treatment with Jakafi can cause thrombocytopenia, anemia and neutropenia. [see Dosage and Administration (2.1) in Full Prescribing Information]. Manage thrombocytopenia by reducing the dose or temporarily interrupting Jakafi. Platelet transfusions may be necessary [see Dosage and Administration (2.1.1) and Adverse Reactions (6.1) in Full Prescribing Information]. Patients developing anemia may require blood transfusions and/or dose modifications of Jakafi. Severe neutropenia (ANC less than 0.5 X 109/L) was generally reversible by withholding Jakafi until recovery [see Adverse Reactions (6.1)]. Perform a pre-treatment complete blood count (CBC) and monitor CBCs every 2 to 4 weeks until doses are stabilized, and then as clinically indicated. [see Dosage and Administration (2.1.1) and Adverse Reactions (6.1) in Full Prescribing Information]. Risk of Infection Serious bacterial, mycobacterial, fungal and viral infections have occurred. Delay starting therapy with Jakafi until active serious infections have resolved. Observe patients receiving Jakafi for signs and symptoms of infection and manage promptly. Tuberculosis Tuberculosis infection has been reported in patients receiving Jakafi. Observe patients receiving Jakafi for signs and symptoms of active tuberculosis and manage promptly. Prior to initiating Jakafi, patients should be evaluated for tuberculosis risk factors, and those at higher risk should be tested for latent infection. Risk factors include, but are not limited to, prior residence in or travel to countries with a high prevalence of tuberculosis, close contact with a person with active tuberculosis, and a history of active or latent tuberculosis where an adequate course of treatment cannot be confirmed. For patients with evidence of active or latent tuberculosis, consult a physician with expertise in the treatment of tuberculosis before starting Jakafi. The decision to continue Jakafi during treatment of active tuberculosis should be based on the overall risk-benefit determination. PML Progressive multifocal leukoencephalopathy (PML) has occurred with ruxolitinib treatment for myelofibrosis. If PML is suspected, stop Jakafi and evaluate. Herpes Zoster Advise patients about early signs and symptoms of herpes zoster and to seek treatment as early as possible if suspected [see Adverse Reactions (6.1)]. Symptom Exacerbation Following Interruption or Discontinuation of Treatment with Jakafi Following discontinuation of Jakafi, symptoms from myeloproliferative neoplasms may return to pretreatment levels over a period of approximately one week. Some patients with myelofibrosis have experienced one or more of the following adverse events after discontinuing Jakafi: fever, respiratory distress, hypotension, DIC, or multi-organ failure. If one or more of these occur after discontinuation of, or while tapering the dose of Jakafi, evaluate for and treat any intercurrent illness and consider restarting or increasing the dose of Jakafi. Instruct patients not to interrupt or discontinue Jakafi therapy without consulting their physician. When discontinuing or interrupting therapy with Jakafi for reasons other than thrombocytopenia or neutropenia [see Dosage and Administration (2.5) in Full Prescribing Information], consider tapering the dose of Jakafi gradually rather than discontinuing abruptly. Non-Melanoma Skin Cancer Non-melanoma skin cancers including basal cell, squamous cell, and Merkel cell carcinoma have occurred in patients treated with Jakafi. Perform periodic skin examinations. ADVERSE REACTIONS The following serious adverse reactions are discussed in greater detail in other sections of the labeling: • Thrombocytopenia, Anemia and Neutropenia [see Warnings and Precautions (5.1)] • Risk of Infection [see Warnings and Precautions (5.2)] • Symptom Exacerbation Following Interruption or Discontinuation of Treatment with Jakafi [see Warnings and Precautions (5.3)] • Non-Melanoma Skin Cancer [see Warnings and Precautions (5.4)]. Because clinical trials are conducted under widely varying conditions, adverse reaction rates observed in the clinical trials of a drug cannot be directly compared to rates in the clinical trials of another drug and may not reflect the rates observed in practice. Clinical Trials Experience in Myelofibrosis The safety of Jakafi was assessed in 617 patients in six clinical studies with a median duration of follow-up of 10.9 months, including 301 patients with myelofibrosis in two Phase 3 studies. In these two Phase 3 studies, patients had a median duration of exposure to Jakafi of 9.5 months (range 0.5 to 17 months), with 89% of patients treated for more than 6 months and 25% treated for more than 12 months. One hundred and eleven (111) patients started treatment at 15 mg twice daily and 190 patients started at 20 mg twice daily. In patients starting treatment with 15 mg twice daily (pretreatment platelet counts of 100 to 200 X 109/L) and 20 mg twice daily (pretreatment platelet counts greater than 200 X 109/L), 65% and 25% of patients, respectively, required a dose reduction below the starting dose within the first 8 weeks of therapy. In a double-blind, randomized, placebo-controlled study of Jakafi, among the 155 patients treated with Jakafi, the most frequent adverse drug reactions were thrombocytopenia and anemia [see Table 2 ]. Thrombocytopenia, anemia and neutropenia are dose related effects. The three most frequent non-hematologic adverse reactions were bruising, dizziness and headache [see Table 1]. Discontinuation for adverse events, regardless of causality, was observed in 11% of patients treated with Jakafi and 11% of patients treated with placebo. Table 1 presents the most common adverse reactions occurring in patients who received Jakafi in the double-blind, placebo-controlled study during randomized treatment. Table 1: Myelofibrosis: Adverse Reactions Occurring in Patients on Jakafi in the Double-blind, Placebo-controlled Study During Randomized Treatment Jakafi (N=155) Adverse Reactions

Adverse Events

All Gradesa (%)

Grade 3-4 (%)

All Grades (%)

Headache

16

<1

19

<1

Abdominal Painb

15

<1

15

<1

Diarrhea

15

0

7

<1

Dizzinessc

15

0

13

0

Fatigue

15

0

15

3

Pruritus

14

<1

23

4

Dyspnead

13

3

4

0

Muscle Spasms

12

<1

5

0

Nasopharyngitis

9

0

8

0

Constipation

8

0

3

0

Cough

8

0

5

0

Edemae

8

0

7

0

Arthralgia

7

0

6

<1

Asthenia

7

0

11

2

Epistaxis

6

0

3

0

Herpes Zosterf

6

<1

0

0

Nausea

6

0

4

0

a b c d e f

Laboratory Parameter

Grade 3 (%)

Grade 4 (%)

Anemia

72

<1

Thrombocytopenia

27

5

Placebo (N=151)

Neutropenia

3

0

<1

0

8

0

0

16

0

0

0

Elevated AST

23

0

0

23

<1

0

0

Hypertriglyceridemia

15

0

0

13

0

0

15

0

0

7

0

Headache

15

0

0

5

0

0

a

Urinary Tract Infectionsd

9

0

0

5

<1

<1

b

Weight Gaine

7

<1

0

1

<1

0

Flatulence

5

0

0

<1

0

0

Herpes Zosterf

2

0

0

<1

0

0

National Cancer Institute Common Terminology Criteria for Adverse Events (CTCAE), version 3.0 includes contusion, ecchymosis, hematoma, injection site hematoma, periorbital hematoma, vessel puncture site hematoma, increased tendency to bruise, petechiae, purpura includes dizziness, postural dizziness, vertigo, balance disorder, Meniere’s Disease, labyrinthitis includes urinary tract infection, cystitis, urosepsis, urinary tract infection bacterial, kidney infection, pyuria, bacteria urine, bacteria urine identified, nitrite urine present includes weight increased, abnormal weight gain includes herpes zoster and post-herpetic neuralgia

Description of Selected Adverse Drug Reactions Anemia In the two Phase 3 clinical studies, median time to onset of first CTCAE Grade 2 or higher anemia was approximately 6 weeks. One patient (<1%) discontinued treatment because of anemia. In patients receiving Jakafi, mean decreases in hemoglobin reached a nadir of approximately 1.5 to 2.0 g/dL below baseline after 8 to 12 weeks of therapy and then gradually recovered to reach a new steady state that was approximately 1.0 g/dL below baseline. This pattern was observed in patients regardless of whether they had received transfusions during therapy. In the randomized, placebo-controlled study, 60% of patients treated with Jakafi and 38% of patients receiving placebo received red blood cell transfusions during randomized treatment. Among transfused patients, the median number of units transfused per month was 1.2 in patients treated with Jakafi and 1.7 in placebo treated patients. Thrombocytopenia In the two Phase 3 clinical studies, in patients who developed Grade 3 or 4 thrombocytopenia, the median time to onset was approximately 8 weeks. Thrombocytopenia was generally reversible with dose reduction or dose interruption. The median time to recovery of platelet counts above 50 X 109/L was 14 days. Platelet transfusions were administered to 5% of patients receiving Jakafi and to 4% of patients receiving control regimens. Discontinuation of treatment because of thrombocytopenia occurred in <1% of patients receiving Jakafi and <1% of patients receiving control regimens. Patients with a platelet count of 100 X 109/L to 200 X 109/L before starting Jakafi had a higher frequency of Grade 3 or 4 thrombocytopenia compared to patients with a platelet count greater than 200 X 109/L (17% versus 7%). Neutropenia In the two Phase 3 clinical studies, 1% of patients reduced or stopped Jakafi because of neutropenia. Table 2 provides the frequency and severity of clinical hematology abnormalities reported for patients receiving treatment with Jakafi or placebo in the placebo-controlled study. Table 2: Myelofibrosis: Worst Hematology Laboratory Abnormalities in the Placebo-Controlled Studya Placebo (N=151) Grade 3 (%)

Grade 4 (%)

Thrombocytopenia

70

9

4

31

1

0

Anemia

96

34

11

87

16

3

Neutropenia

19

5

2

4

<1

1

b

10

0

0

a

<1

0

<1

All Grades (%)

<1

0

<1

Grade 4 (%)

0

3

<1

18

Grade 3 (%)

0

24

35

23

All Gradesb (%)

58

<1

25

Dizzinessc

Laboratory Parameter

<1

Hypercholesterolemia

Bruisingb

Jakafi (N=155)

Grade 4 (%)

Elevated ALT

Grade 4 (%)

f

Grade 3 (%)

Chemistry

Grade 3 (%)

e

All Grades (%)

Hematology

All Grades (%)

d

Best Available Therapy (N=111)

All Gradesb (%)

Grade 4 (%)

c

National Cancer Institute Common Terminology Criteria for Adverse Events (CTCAE), version 3.0 includes abdominal pain, abdominal pain lower, and abdominal pain upper includes dizziness and vertigo includes dyspnea and dyspnea exertional includes edema and peripheral edema includes herpes zoster and post-herpetic neuralgia

Jakafi (N=110)

Grade 3 (%)

b

Grade 3-4 (%)

Other clinically important treatment emergent adverse events observed in less than 6% of patients treated with Jakafi were: Weight gain, hypertension, and urinary tract infections Clinically relevant laboratory abnormalities are shown in Table 4. Table 4: Polycythemia Vera: Selected Laboratory Abnormalities in the Open-Label, Active-controlled Study up to Week 32 of Randomized Treatmenta

All Gradesa (%)

a

Best Available Therapy (N=111)

Presented values are worst Grade values regardless of baseline National Cancer Institute Common Terminology Criteria for Adverse Events, version 3.0

Additional Data from the Placebo-controlled Study 25% of patients treated with Jakafi and 7% of patients treated with placebo developed newly occurring or worsening Grade 1 abnormalities in alanine transaminase (ALT). The incidence of greater than or equal to Grade 2 elevations was 2% for Jakafi with 1% Grade 3 and no Grade 4 ALT elevations. 17% of patients treated with Jakafi and 6% of patients treated with placebo developed newly occurring or worsening Grade 1 abnormalities in aspartate transaminase (AST). The incidence of Grade 2 AST elevations was <1% for Jakafi with no Grade 3 or 4 AST elevations. 17% of patients treated with Jakafi and <1% of patients treated with placebo developed newly occurring or worsening Grade 1 elevations in cholesterol. The incidence of Grade 2 cholesterol elevations was <1% for Jakafi with no Grade 3 or 4 cholesterol elevations. Clinical Trial Experience in Polycythemia Vera In a randomized, open-label, active-controlled study, 110 patients with polycythemia vera resistant to or intolerant of hydroxyurea received Jakafi and 111 patients received best available therapy [see Clinical Studies (14.2) in Full Prescribing Information]. The most frequent adverse drug reaction was anemia. Table 3 presents the most frequent non-hematologic treatment emergent adverse events occurring up to Week 32. Discontinuation for adverse events, regardless of causality, was observed in 4% of patients treated with Jakafi.

Presented values are worst Grade values regardless of baseline National Cancer Institute Common Terminology Criteria for Adverse Events, version 3.0

DRUG INTERACTIONS Drugs That Inhibit or Induce Cytochrome P450 Enzymes Ruxolitinib is metabolized by CYP3A4 and to a lesser extent by CYP2C9. CYP3A4 inhibitors: The Cmax and AUC of ruxolitinib increased 33% and 91%, respectively following concomitant administration with the strong CYP3A4 inhibitor ketoconazole in healthy subjects. Concomitant administration with mild or moderate CYP3A4 inhibitors did not result in an exposure change requiring intervention [see Pharmacokinetics (12.3) in Full Prescribing Information]. When administering Jakafi with strong CYP3A4 inhibitors, consider dose reduction [see Dosage and Administration (2.3) in Full Prescribing Information]. Fluconazole: The AUC of ruxolitinib is predicted to increase by approximately 100% to 300% following concomitant administration with the combined CYP3A4 and CYP2C9 inhibitor fluconazole at doses of 100 mg to 400 mg once daily, respectively [see Pharmacokinetics (12.3) in Full Prescribing Information]. Avoid the concomitant use of Jakafi with fluconazole doses of greater than 200 mg daily [see Dosage and Administration (2.3) in Full Prescribing Information]. CYP3A4 inducers: The Cmax and AUC of ruxolitinib decreased 32% and 61%, respectively, following concomitant administration with the strong CYP3A4 inducer rifampin in healthy subjects. No dose adjustment is recommended; however, monitor patients frequently and adjust the Jakafi dose based on safety and efficacy [see Pharmacokinetics (12.3) in Full Prescribing Information]. USE IN SPECIFIC POPULATIONS Pregnancy Pregnancy Category C: Risk Summary There are no adequate and well-controlled studies of Jakafi in pregnant women. In embryofetal toxicity studies, treatment with ruxolitinib resulted in an increase in late resorptions and reduced fetal weights at maternally toxic doses. Jakafi should be used during pregnancy only if the potential benefit justifies the potential risk to the fetus. Animal Data Ruxolitinib was administered orally to pregnant rats or rabbits during the period of organogenesis, at doses of 15, 30 or 60 mg/kg/day in rats and 10, 30 or 60 mg/kg/day in rabbits. There was no evidence of teratogenicity. However, decreases of approximately 9% in fetal weights were noted in rats at the highest and maternally toxic dose of 60 mg/kg/day. This dose results in an exposure (AUC) that is approximately 2 times the clinical exposure at the maximum recommended dose of 25 mg twice daily. In rabbits, lower fetal weights of approximately 8% and increased late resorptions were noted at the highest and maternally toxic dose of 60 mg/kg/day. This dose is approximately 7% the clinical exposure at the maximum recommended dose. In a pre- and post-natal development study in rats, pregnant animals were dosed with ruxolitinib from implantation through lactation at doses up to 30 mg/kg/day. There were no drug-related adverse findings in pups for fertility indices or for maternal or embryofetal survival, growth and development parameters at the highest dose evaluated (34% the clinical exposure at the maximum recommended dose of 25 mg twice daily). Nursing Mothers It is not known whether ruxolitinib is excreted in human milk. Ruxolitinib and/or its metabolites were excreted in the milk of lactating rats with a concentration that was 13-fold the maternal plasma. Because many drugs are excreted in human milk and because of the potential for serious adverse reactions in nursing infants from Jakafi, a decision should be made to discontinue nursing or to discontinue the drug, taking into account the importance of the drug to the mother. Pediatric Use The safety and effectiveness of Jakafi in pediatric patients have not been established. Geriatric Use Of the total number of myelofibrosis patients in clinical studies with Jakafi, 52% were 65 years of age and older. No overall differences in safety or effectiveness of Jakafi were observed between these patients and younger patients. Renal Impairment The safety and pharmacokinetics of single dose Jakafi (25 mg) were evaluated in a study in healthy subjects [CrCl 72-164 mL/min (N=8)] and in subjects with mild [CrCl 53-83 mL/min (N=8)], moderate [CrCl 38-57 mL/min (N=8)], or severe renal impairment [CrCl 15-51 mL/min (N=8)]. Eight (8) additional subjects with end stage renal disease requiring hemodialysis were also enrolled. The pharmacokinetics of ruxolitinib was similar in subjects with various degrees of renal impairment and in those with normal renal function. However, plasma AUC values of ruxolitinib metabolites increased with increasing severity of renal impairment. This was most marked in the subjects with end stage renal disease requiring hemodialysis. The change in the pharmacodynamic marker, pSTAT3 inhibition, was consistent with the corresponding increase in metabolite exposure. Ruxolitinib is not removed by dialysis; however, the removal of some active metabolites by dialysis cannot be ruled out. When administering Jakafi to patients with myelofibrosis and moderate (CrCl 30-59 mL/min) or severe renal impairment (CrCl 15-29 mL/min) with a platelet count between 50 X 109/L and 150 X 109/L, a dose reduction is recommended. A dose reduction is also recommended for patients with polycythemia vera and moderate (CrCl 30-59 mL/min) or severe renal impairment (CrCl 15-29 mL/min). In all patients with end stage renal disease on dialysis, a dose reduction is recommended [see Dosage and Administration (2.4) in Full Prescribing Information]. Hepatic Impairment The safety and pharmacokinetics of single dose Jakafi (25 mg) were evaluated in a study in healthy subjects (N=8) and in subjects with mild [Child-Pugh A (N=8)], moderate [Child-Pugh B (N=8)], or severe hepatic impairment [Child-Pugh C (N=8)]. The mean AUC for ruxolitinib was increased by 87%, 28% and 65%, respectively, in patients with mild, moderate and severe hepatic impairment compared to patients with normal hepatic function. The terminal elimination half-life was prolonged in patients with hepatic impairment compared to healthy controls (4.1-5.0 hours versus 2.8 hours). The change in the pharmacodynamic marker, pSTAT3 inhibition, was consistent with the corresponding increase in ruxolitinib exposure except in the severe (Child-Pugh C) hepatic impairment cohort where the pharmacodynamic activity was more prolonged in some subjects than expected based on plasma concentrations of ruxolitinib. When administering Jakafi to patients with myelofibrosis and any degree of hepatic impairment and with a platelet count between 50 X 109/L and 150 X 109/L, a dose reduction is recommended. A dose reduction is also recommended for patients with polycythemia vera and hepatic impairment [see Dosage and Administration (2.4) in Full Prescribing Information]. OVERDOSAGE There is no known antidote for overdoses with Jakafi. Single doses up to 200 mg have been given with acceptable acute tolerability. Higher than recommended repeat doses are associated with increased myelosuppression including leukopenia, anemia and thrombocytopenia. Appropriate supportive treatment should be given. Hemodialysis is not expected to enhance the elimination of ruxolitinib. Jakafi is a registered trademark of Incyte. All rights reserved. U.S. Patent Nos. 7598257; 8415362; 8722693; 8822481; 8829013 © 2011-2014 Incyte Corporation. All rights reserved. Issued: December 2014 RUX-1428a


ASCOPost.com  |   JULY 25, 2015

PAGE 79

In the News Dermatologic Oncology

Capitalizing on Increased Interest in Skin Cancer During Summer to Reeducate People About Sunscreens and ‘Smart Sun Strategies’ By Charlotte Bath

A

mid the encouraging studies reported at the 2015 ASCO Annual Meeting about advances in the treatment of melanoma was a troubling finding about the incidence of melanoma increasing. An analysis of data from nine Surveillance, Epidemiology, and End Results (SEER) registries found that the incidence of melanoma in children, adolescents, and young adults increased 253% between 1973 and 2011.1 Just as the Annual Meeting was ending, the Centers for Disease Control and Prevention (CDC) released a Vital Signs report noting that the rate of new cases of melanoma had doubled, from 11.2 to 22.7 per 100,000 between 1982 and 2011. Without additional prevention efforts, the CDC predicted, that rate will continue to increase, with 112,000 new cases projected in 2030.2,3 Among the efforts urged by the CDC to prevent melanoma is education about protecting the skin from exposure to ultraviolet radiation. And it seems the public is particularly interested in information on skin cancer in the summer months. Indeed, a Research Letter in JAMA Dermatology reported that Google searches for information about skin cancer “increased during each summer yearly but have remained stable for 5 years (2010–2104). Searches for melanoma mirrored the search volume for skin cancer.”4 The authors concluded: “Because the U.S. population seeks information regarding skin cancer at a greater level during the summer months, this might be the most efficient time for educational and public health initiatives.”

Time of Year to Raise Awareness In an interview with The ASCO Post, Lynn M. Schuchter, MD, said, “We definitely take advantage of this time of year to raise awareness about skin cancer and melanoma.” Dr. Schuchter is the University of Pennsylvania Willard Robinson Professor of Hematology/ Oncology, Chief of the Hematology/ Oncology Division, and Program Leader for the Abramson Cancer Center’s National Cancer Institute approved and funded Melanoma Research Program. She is also Scientfic Program Leader for the ASCO Annual Meeting in 2016 and served as an ASCO expert at this year’s Annual Meeting.

Dr. Schuchter noted that May is Melanoma Awareness Month, and skin cancer education efforts continue through the summer months. “We do try to capitalize on that,” Dr. Schuchter said. Dermatologists as well as other physicians and health educators work to promote the use of sunscreen products and get people “reeducated on ‘smart sun strategies,’” she added. “We’re working on communication strategies so that especially young people take in and understand the warning that ultraviolet radiation is a carcinogen,” Dr. Schuchter continued. “People sometimes don’t understand that melanoma is serious. They hear the words ‘skin cancer’ and don’t get too alarmed, but obviously melanoma is a serious skin cancer.”

advertising for tanning salons to make sure that indoor tanning is not being promoted as a safe way to tan, Dr. Schuchter added. Indoor tanning, she stressed, is not a safe alternative to sun exposure.

Trend Toward Earlier Diagnosis The SEER data analysis also found a trend in diagnosis at an earlier disease stage.1 Skin cancer screening, however, is not recommended by the U.S. Preventive Services Task Force, which concluded: “the current evidence is insufficient to assess the benefits and harms of using a whole-body skin examination by a primary care clinician or patient skin self-examination for the early detection of cutaneous melanoma, basal cell cancer, or squamous cell skin can-

We definitely take advantage of this time of year to raise awareness about skin cancer and melanoma. —Lynn M. Schuchter, MD

Restricting Access to Indoor Tanning The SEER data analysis showing a 253% increased incidence of melanoma among children, adolescents, and young adults found that white female young adults are at particular high risk.1 “There is a lot of concern that the rising increase in young women is partially related to tanning salon use,” Dr. Schuchter said. The CDC Vital Signs report called on policymakers and members of the community to “restrict the availability and use of indoor tanning by minors.”2 Legislation that would do that—mainly by requiring parental consent for indoor tanning by those under 18 years old—“has been moving more through state legislatures” and is “very difficult to enforce,” Dr. Schuchter noted. Other legislation being promoted would require that the U.S. Food and Drug Administration label tanning salons as carcinogenic, Dr. Schuchter said. On the regulatory front, efforts include looking very carefully at the

cer in the adult general population.”5 That statement is in the process of being updated. There is no recommendation because there hasn’t been a study showing mortality data related to screening, Dr. Schuchter said. “But there are definitely groups of people who would benefit from skin examinations. Those who have had a personal history of melanoma, family history of melanoma, or other nonmelanoma skin cancers and individuals with dysplastic nevi should be screened,” she explained. There are tools to calculate melanoma risk, Dr. Schuchter noted. “There is the Fears model that we have developed, where you can quickly—just by looking at someone’s back and asking a few questions—identify, from a primary care perspective, whether a patient should be referred to a dermatologist,” Dr. Schuchter reported. These questions are aimed at determining a patient’s age, race, and gender and whether the patient lives in the northern, southern, or central United States.

This melanoma risk calculation tool was developed using data from a case control study that included 718 nonHispanic white patients with invasive cutaneous melanoma and 945 matched controls.6 The tool is not recommended for use with other races or ethnicities. When screening is recommended because of a high risk for melanoma, it is usually on an annual basis, although sometimes more frequent exams are called for. “There is a group of patients who have atypical nevi or dysplastic nevi who may have skin exams twice a year,” Dr. Schuchter noted.

Other ASCO 2015 Melanoma Studies The melanoma study presented at ASCO 2015 probably receiving the most media attention was the CheckMate 067 trial finding that for patients with previously untreated advanced melanoma, initial treatment with nivolumab (Opdivo) improved progression-free survival compared to ipilimumab (Yervoy), but the benefit was greater when the two agents were used together.7 “That study combined with other recent data shows the importance of checkpoint blockade in treating melanoma. There is no question about that. This approach is highly effective,” Dr. Schuchter stated. “But I don’t think it is clear that we should be using combination ipilimumab and nivolumab as the new standard of care if you are using immunotherapy,” she continued. “As single agents, nivolumab and pembrolizumab (Keytruda) are very active, and [CheckMate 067] wasn’t powered to tell the difference between single-agent nivolumab and the combination. I think we need to wait for longer follow-up and survival data before saying that combination therapy is the first choice, because of the toxicity issue. There is no question that when you combine the two, there is much more significant toxicity.” Dr Schuchter pointed out that there are other strategies being tested combining targeted therapies with immunotherapies. “It is a rapidly evolving field, and now it is going to be important to look not just at response rate and progression-free survival but at overall survival as well—and to look closely at toxicity,” she stressed. continued on page 80


The ASCO Post  |   JULY 25, 2015

PAGE 80

In the News Skin Cancer Interest continued from page 79

The authors of a study showing that complete lymph node dissection did not significantly improve survival for patients with melanoma and micrometastases said the results raise questions about general recommendations for complete lymph node dissection in patients with positive nodes, and the authors expect that surgical practice will change.8 “I think that studies like this are potentially practice-changing, saying that maybe we don’t need to do complete lymph node dissection,” Dr. Schuchter said. She noted, however, that there is “a much larger international study being done ­[Multicentre Selective Lymphadenectomy Trial 2, or MSLT-2)], looking at complete lymph node dissection vs observation with monitoring. I think that is going to be the definitive study. Meanwhile, these are the data we have, and the overall survival was identical, with nearly 500 patients enrolled.” The results of MSLT-2 are not expected until 2022. “In my own practice, I recommend complete lymph node dissection for most of my patients,” Dr. Schuchter

said. “In selected patients, one could consider not proceeding because of the concern about lymphedema.” She pointed out that adjuvant clinical trials currently require complete surgical staging. Thus, if patients don’t have complete lymph node dissection, they are not eligible for adjuvant clinical trials. She predicted that “eventually we will see clinical trials in the adjuvant setting also change” so that complete lymph node dissection is not a requirement for eligibility. Updates were also presented at ASCO 2015 on combined BRAF and MEK inhibitors for patients with BRAF-mutant melanoma. “Going forward, most studies are focused on how best to sequence the therapies,” Dr. Schuchter said. “Do you do immunotherapy first or targeted therapy? Should you be doing combination strategies? There is a lot of new research in the area of resistance, both for targeted therapies and immunotherapies—understanding why a patient responds to immunotherapy and then stops, and what are the mechanisms of resistance.” While the majority of patients with stage IV melanoma “are either not go-

ing to respond or will respond and then [have disease progression],” Dr. Schuchter said, “a minority may be cured.” She noted that in 26 years of treating patients with melanoma, “it is the first time I am saying that for stage IV melanoma.” n

Disclosure: Dr. Schuchter reported no potential conflicts of interest.

References 1. Mitsis DKL, Groman A, Beaupin LM, et al: Trends in demographics, incidence, and survival in children, adolescents, and young adults (AYA) with melanoma: A Surveillance, Epidemiology, and End Results (SEER) population-based analysis. 2015 ASCO Annual Meeting. Abstract 9058. Presented June 1, 2015. 2. Centers for Disease Control and Prevention: Preventing melanoma. Vital Signs, June 2015. Available at cdc.gov/vitalsigns/ melanoma. Accessed June 17, 2015. 3. Centers for Disease Control and Prevention: Rates of new melanoma—deadly skin cancers—have doubled over last three decades. Available at cdc.gov/media/releases/2015. Accessed June 17, 2015. 4. Bloom R, Amber KT, Hu S, et al:

Google search trends and skin cancer: Evaluating the US population’s interest in skin cancer and its association with melanoma outcomes. JAMA Dermatology. June 10, 2015 (early release online). 5. U.S. Preventive Services Task Force: Skin cancer screening (recommendation statement). Released February 2009. Available at uspreventiveservicestaskforce.org. Accessed June 22, 2015. 6. Fears TR, Guerry D 4th, Pfeiffer RM, et al: Identifying individuals at high risk of melanoma: A practical predictor of absolute risk. J Clin Oncol 24:3590-3596, 2006. 7. Wolchok JD, Chiarion-Sileni V, Gonzales R, et al: Efficacy and safety results from a phase III trial of nivolumab alone or combined with ipilimumab versus ipilimumab alone in treatment-naive patients with advanced melanoma (Checkmate 067). 2015 ASCO Annual Meeting. Abstract LBA1. Presented May 31, 2015. 8. Leiter U, Stadler R, Mauch C, et al: Survival of SLNB-positive melanoma patients with and without complete lymph node dissection: A multicenter, randomized DECOG trial. 2015 ASCO Annual Meeting. Abstract LBA9002. Presented May 30, 2015.

Expect Questions About Preventing Melanoma By Charlotte Bath

A

recent report from the Centers for Disease Control and Prevention (CDC) stated that the incidence of melanoma “doubled between 1982 and 2011, but comprehensive skin cancer prevention programs could prevent 20% of new cases between 2020 and 2030.”1,2 That report should serve to increase interest in strategies to reduce exposure to ultraviolet radiation. Augmenting that interest is the strong summer sun and national media coverage of the CDC findings, including reports by CBS News and the Los Angeles Times. The CDC Vital Signs report included standard advice on protecting against ultraviolet radiation exposure by wearing a hat and clothes that cover the skin, applying broad-spectrum sunscreen, and seeking shade. In addition, the report called on policymakers and members of the community

to “restrict the availability and use of indoor tanning by minors.”2 Lynn M. Schuchter, MD, told The ASCO Post, “There is a lot of concern that the rising increase in young females is related to tanning salon use.” Dr. Schuchter is the University of Pennsylvania Willard Robinson Professor of Hematology/Oncology, Chief of the Hematology/Oncology Division, and Program Leader for the Abramson Cancer Center’s National Cancer Institute approved and funded Melanoma Research Program. She is also Chair of ASCO’s Cancer Research Committee and served as an ASCO expert at this year’s Annual Meeting.

Nicotinamide Does Not Work for Melanoma A study presented at the 2015 ASCO Annual Meeting found that two daily doses of nicotinamide, a

form of vitamin B3, reduced the incidence of skin cancers by 23% in highrisk persons but only for those with nonmelanoma skin cancers.3 “That prevention strategy is not for melanoma,” Dr. Schuchter confirmed. While press materials concerning that study noted that nicotinamide is safe, inexpensive, and “ready to go straight to the clinic,” reports of its efficacy seem to have curtailed its availability. “Locally, patients have tried to get it and said that the drugstores and vitamin stores are sold out,” Dr. Schuchter noted. “I would not say that I am recommending it in my practice,” she added. The research does not indicate that everyone should be taking nicotinamide, “but I think one could certainly discuss it with some patients who are at a very high risk and have had multiple nonmelanoma skin cancers—for these patients, it could be considered

in addition to sunscreen and seeing a dermatologist.” n Disclosure: Dr. Schuchter reported no potential conflicts of interest.

References

1. Centers for Disease Control and Prevention: Preventing melanoma. CDC Vital Signs, June 2015. Available at cdc.gov/vitalsigns/melanoma. Accessed June 17, 2015.

2. Centers for Disease Control and Prevention: Rates of new melanoma —deadly skin cancers—have doubled over last three decades. Available at cdc.gov/media/releases/2015. Accessed June 17, 2015. 3. Martin AJ, Chen A, Choy B, et al: Oral nicotinamide to reduce actinic cancer: A phase 3 double-blind randomized controlled trial. 2015 ASCO Annual Meeting. Abstract 9000. Presented May 30, 2015.

Visit The ASCO Post website at ASCOPost.com


If she has ovarian cancer

TEST FOR BRCA

If indicated* TREAT WITH LYNPARZA

Help her continue the fight with the first approved PARP inhibitor1

* INDICATION LYNPARZA is indicated as monotherapy in patients with deleterious or suspected deleterious germline BRCA mutated (as detected by an FDA-approved test) advanced ovarian cancer who have been treated with three or more prior lines of chemotherapy. The indication is approved under accelerated approval based on objective response rate and duration of response. Continued approval for this indication may be contingent upon verification and description of clinical benefit in confirmatory trials.

SELECT SAFETY INFORMATION Myelodysplastic syndrome/Acute Myeloid Leukemia (MDS/AML) have been confirmed in 2% of patients enrolled in both a single arm monotherapy trial (6 out of 298) and a randomized placebo controlled trial (3 out of 136). Overall, MDS/AML were reported in <1% of patients (22 out of 2,618) treated with LYNPARZA. The majority of MDS/AML cases were fatal (17 out of 22) and the duration of therapy in patients who developed secondary MDS/AML varied from <6 months to >2 years. Monitor complete blood count testing at baseline and monthly thereafter. Do not start LYNPARZA until patients have recovered from hematological toxicity caused by previous chemotherapy (≤CTCAE Grade 1). For prolonged hematological toxicities, interrupt LYNPARZA and monitor blood counts weekly until recovery. If the levels have not recovered to CTCAE Grade 1 or less after 4 weeks, refer the patient to a hematologist for further investigations, including bone marrow analysis and blood sample for cytogenetics. If MDS/AML is confirmed, discontinue LYNPARZA.

Please see the following pages for additional Safety Information and Brief Summary of the full Prescribing Information.


LYNPARZA demonstrated an objective response rate of 34% in patients with BRCA-mutated advanced ovarian cancer who had been treated with 3 or more lines of chemotherapy1 The efficacy of LYNPARZA was investigated in a single-arm study of patients with deleterious or suspected deleterious germline BRCA-mutated (gBRCAm) advanced cancer. A total of 137 patients with measurable gBRCAm-associated ovarian cancer treated with three or more prior lines of chemotherapy were enrolled. Efficacy was based on objective response rate and duration of response.1 Objective response rate was defined as a ≥30% reduction in target lesion size, according to RECIST criteria, as measured by CT or MRI and confirmed at least 4 weeks later.2

34

%

OBJECTIVE RESPONSE RATE (95% CI: 26, 42)

0

10

20

30

PERCENTAGE OF PATIENTS WHO RESPONDED TO THERAPY

• The rate of partial response was 32% and the rate of complete response was 2%1

7.9

MEDIAN DURATION OF RESPONSE

MONTHS (95% CI: 5.6, 9.6)

Please see the following pages for additional Safety Information and Brief Summary of the full Prescribing Information.


Warnings and Precautions Myelodysplastic syndrome/Acute Myeloid Leukemia Myelodysplastic syndrome/Acute Myeloid Leukemia (MDS/AML) have been confirmed in 6 out of 298 (2%) patients enrolled in a single arm trial of LYNPARZA monotherapy, in patients with deleterious or suspected deleterious germline BRCA-mutated (gBRCAm) advanced cancers. In a randomized placebo controlled trial, MDS/AML occurred in 3 out of 136 (2%) patients with advanced ovarian cancer treated with LYNPARZA. Overall, MDS/AML were reported in 22 of 2,618 (<1%) patients treated with LYNPARZA. The majority of MDS/AML cases (17 of 22 cases) were fatal, and the duration of therapy with LYNPARZA in patients who developed secondary MDS/cancer-therapy related AML varied from <6 months to >2 years. All patients had previous chemotherapy with platinum agents and/or other DNA damaging agents. Monitor complete blood count testing at baseline and monthly thereafter. Do not start LYNPARZA until patients have recovered from hematological toxicity caused by previous chemotherapy (≤CTCAE Grade 1). For prolonged hematological toxicities, interrupt LYNPARZA and monitor blood counts weekly until recovery. If the levels have not recovered to CTCAE Grade 1 or less after 4 weeks, refer the patient to a hematologist for further investigations, including bone marrow analysis and blood sample for cytogenetics. If MDS/AML is confirmed, discontinue LYNPARZA. Pneumonitis Pneumonitis, including fatal cases, occurred in <1% of patients treated with LYNPARZA. If patients present with new or worsening respiratory symptoms such as dyspnea, fever, cough, wheezing, or a radiological abnormality occurs, interrupt treatment with LYNPARZA and initiate prompt investigation. If pneumonitis is confirmed, discontinue LYNPARZA. Embryo-Fetal Toxicity LYNPARZA can cause fetal harm when administered to a pregnant woman based on its mechanism of action and findings in animals. Olaparib was teratogenic and caused embryo-fetal toxicity in rats at exposures below those in patients receiving the recommended human dose of 400 mg twice daily. If the patient becomes pregnant while taking this drug, apprise the patient of the potential hazard to a fetus. Advise females of reproductive potential to avoid becoming pregnant while taking LYNPARZA. If contraceptive methods are being considered, use effective contraception during treatment and for at least one month after receiving the last dose of LYNPARZA.

Use in Nursing Mothers Nursing Mothers It is not known whether olaparib is excreted in human milk. Because many drugs are excreted in human milk and because of the potential for serious adverse reactions in nursing infants from olaparib, a decision should be made whether to discontinue nursing or to discontinue the drug, taking into account the importance of the drug to the mother.


Safety and tolerability of LYNPARZA • LYNPARZA 400 mg twice daily was evaluated as monotherapy in 223 patients with BRCA-mutated advanced ovarian cancer who had 3 or more prior lines of chemotherapy in 6 clinical trials1

Adverse Reactions Reported in ≥20% of Patients1

LYNPARZA 400 MG TWICE DAILY n=223 CTCAE GRADES 1-4 (%)

CTCAE GRADES 3-4 (%)

34

18

Abdominal pain/discomfort

43

8

Decreased appetite

22

1

Nausea

64

3

Vomiting

43

4

Diarrhea

31

1

Dyspepsia

25

0

66

8

26

0

Arthralgia/musculoskeletal pain

21

0

Myalgia

22

0

BLOOD AND LYMPHATIC DISORDERS

Anemia GASTROINTESTINAL DISORDERS

GENERAL DISORDERS

Fatigue/asthenia INFECTIONS AND INFESTATIONS

Nasopharyngitis/URI MUSCULOSKELETAL AND CONNECTIVE TISSUE DISORDERS

Please see accompanying Brief Summary of Full Prescribing Information. References: 1. LYNPARZA [package insert]. Wilmington, DE: AstraZeneca; 2014. 2. Eisenhauer EA, Therasse P, Bogaerts J, et al. New response evaluation criteria in solid tumours: revised RECIST guideline (version 1.1). Eur J Cancer. 2009;45(2):228-247.

LYNPARZA is a trademark of the AstraZeneca group of companies. ©2015 AstraZeneca. All rights reserved. 3118712 Last Updated 4/15


1

Laboratory Abnormalities

LYNPARZA 400 MG TWICE DAILY n=223 CTCAE GRADES 1-4 (%)

CTCAE GRADES 3-4 (%)

Decrease in hemoglobin (anemia)

90

15

Decrease in absolute neutrophil count (neutropenia)

25

7

Decrease in platelets (thrombocytopenia)

30

3

Decrease in lymphocytes (lymphopenia)

56

17

Mean corpuscular volume elevation

57

-

Increase in creatininea

30

2

a

Patients were allowed to enter clinical studies with laboratory values of CTCAE Grade 1.

The safety and tolerability of LYNPARZA were also evaluated in a randomized, placebo-controlled study1 • LYNPARZA 400 mg twice daily was evaluated as maintenance monotherapy in a randomized, placebo-controlled clinical trial of 96 patients with germline BRCA-mutated platinum-sensitive ovarian cancer who had received 2 or more lines of platinum-containing chemotherapy 1 • Frequently occurring adverse reactions and lab abnormalities were consistent with those seen in the 6 clinical trials, with the addition of back pain, headache, cough, rash, and dysgeusia 1

To learn more, including how to order LYNPARZA, please visit www.lynparza.com


The ASCO Post  |   JULY 25, 2015

PAGE 86

Patient’s Corner

My Faith Helped Me Choose a Way Forward With Cancer By Lisa Arzola, as told to Jo Cavallo

I

’ve been fortunate to be strong and healthy for most of my life. In fact, there is no history of any serious illness in my family, which is why I was so unprepared to hear the words “You have

breast cancer” 5 years ago. Not only was the diagnosis foreign to me, I had no idea what chemotherapy and radiation were—or even who an oncologist was— and why they might be important in my

care. To go from thinking I was completely healthy to being told I needed immediate surgery, followed by chemotherapy and radiation, was so frightening and bewildering that it took me several

weeks to figure out what was happening and what my course of action should be.

Hearing the Diagnosis

Trim: 7.625 x 10.5

LYNPARZA™ (olaparib) capsules, for oral use Brief Summary of Prescribing Information. For complete prescribing information consult official package insert. INDICATIONS AND USAGE Treatment of gBRCA-mutated advanced ovarian cancer Lynparza is indicated as monotherapy in patients with deleterious or suspected deleterious germline BRCA mutated (as detected by an FDA-approved test) advanced ovarian cancer who have been treated with three or more prior lines of chemotherapy. The indication is approved under accelerated approval based on objective response rate and duration of response [see Clinical Studies (14) in the full Prescribing Information]. Continued approval for this indication may be contingent upon verification and description of clinical benefit in confirmatory trials. DOSAGE AND ADMINISTRATION Patient Selection Select patients for the treatment of advanced ovarian cancer with Lynparza based on the presence of deleterious or suspected deleterious germline BRCA-mutations [see Indications and Usage (1) and Clinical Studies (14) in the full Prescribing Information]. Information on FDA-approved test for the detection of BRCA-mutations is available at http://www.fda.gov/companiondiagnostics. Recommended Dosing The recommended dose of Lynparza is 400 mg (eight 50 mg capsules) taken twice daily, for a total daily dose of 800 mg. Continue treatment until disease progression or unacceptable toxicity. If a patient misses a dose of Lynparza, instruct patients to take their next dose at its scheduled time. Swallow capsule whole. Do not chew, dissolve, or open capsule. Do not take capsules which appear deformed or show evidence of leakage [see How Supplied/Storage and Handling (16.2) in the full Prescribing Information]. Dose Adjustments for Adverse Reactions To manage adverse reactions, consider dose interruption of treatment or dose reduction. The recommended dose reduction is to 200 mg (four 50 mg capsules) taken twice daily, for a total daily dose of 400 mg. If a further final dose reduction is required, then reduce to 100 mg (two 50 mg capsules) taken twice daily, for a total daily dose of 200 mg. Dose Modifications for Use with CYP3A Inhibitors Avoid concomitant use of strong and moderate CYP3A inhibitors and consider alternative agents with less CYP3A inhibition. If the inhibitor cannot be avoided, reduce the Lynparza dose to 150 mg (three 50 mg capsules) taken twice daily for a strong CYP3A inhibitor or 200 mg (four 50 mg capsules) taken twice daily for a moderate CYP3A inhibitor [see Drug Interactions (7.2) in the full Prescribing Information]. CONTRAINDICATIONS None WARNINGS AND PRECAUTIONS Myelodysplastic syndrome/Acute Myeloid Leukemia Myelodysplastic syndrome/Acute Myeloid Leukemia (MDS/AML) have been confirmed in 6 out of 298 (2%) patients enrolled in a single arm trial of Lynparza monotherapy, in patients with deleterious or suspected deleterious germline BRCA-mutated (gBRCAm) advanced cancers. In a randomized placebo controlled trial, MDS/AML occurred in 3 out of 136 (2%) patients with advanced ovarian cancer treated with Lynparza. Overall, MDS/AML were reported in 22 of 2,618 (<1%) patients treated with Lynparza. The majority of MDS/AML cases (17 of 22 cases) were fatal, and the duration of therapy with Lynparza in patients who developed secondary MDS/cancer-therapy related AML varied from <6 months to >2 years. All patients had previous chemotherapy with platinum agents and/or other DNA damaging agents. Monitor complete blood count testing at baseline and monthly thereafter. Do not start Lynparza until patients have recovered from hematological toxicity caused by previous chemotherapy ( CTCAE Grade 1). For prolonged hematological toxicities, interrupt Lynparza and monitor blood counts weekly until recovery. If the levels have not recovered to CTCAE Grade 1 or less after 4 weeks, refer the patient to a hematologist for further investigations, including bone marrow analysis and blood sample for cytogenetics. If MDS/AML is confirmed, discontinue Lynparza. Pneumonitis Pneumonitis, including fatal cases, occurred in <1% of patients treated with Lynparza. If patients present with new or worsening respiratory symptoms such as dyspnea, fever, cough, wheezing, or a radiological abnormality occurs, interrupt treatment with Lynparza and initiate prompt investigation. If pneumonitis is confirmed, discontinue Lynparza. Embryo-Fetal Toxicity Lynparza can cause fetal harm when administered to a pregnant woman based on its mechanism of action and findings in animals. Olaparib was teratogenic and caused embryo-fetal toxicity in rats at exposures below those in patients receiving the recommended human dose of 400 mg twice daily. If the patient becomes pregnant while taking this drug, apprise the patient of the potential hazard to a fetus [see Use in Specific Populations (8.1) in the full Prescribing Information]. Advise females of reproductive potential to avoid becoming pregnant while taking Lynparza. If contraceptive methods are being considered, use effective contraception during treatment and for at least one month after receiving the last dose of Lynparza [see Use in Specific Populations (8.6) in the full Prescribing Information]. ADVERSE REACTIONS The following adverse reactions are discussed elsewhere in the labeling: • Myelodysplastic syndrome/Acute Myeloid Leukemia [see Warnings and Precautions (5.1) in the full Prescribing Information] • Pneumonitis [see Warnings and Precautions (5.2) in the full Prescribing Information]

I first noticed something was wrong

Clinical Trial Experience Because clinical trials are conducted under widely varying conditions, adverse reaction rates observed in the clinical trials of a drug cannot be directly compared to rates in the clinical trials of another drug and may not reflect the rates observed in practice. Lynparza 400 mg twice daily as monotherapy, has been studied in 300 patients with gBRCA-mutated advanced ovarian cancer, and 223 of these patients had received 3 or more prior lines of chemotherapy. In the 223 patients with gBRCA-mutated ovarian cancer who received 3 or more prior lines of chemotherapy (including 137 patients in Study 1 with measureable disease) [see Clinical Studies (14) in the full Prescribing Information] adverse reactions led to dose interruption in 40% of patients, dose reduction in 4%, and discontinuation in 7%. There were 8 (4%) patients with adverse reactions leading to death, two were attributed to acute leukemia, and one each was attributed to COPD, cerebrovascular accident, intestinal perforation, pulmonary embolism, sepsis, and suture rupture. Table 1 presents the frequency of adverse reactions reported in 20% of 223 patients (in 6 studies) with gBRCA-mutated advanced ovarian cancer who had received 3 or more prior lines of chemotherapy who were treated with Lynparza 400 mg twice daily. The median exposure to Lynparza in these patients was 158 days. Table 1 Adverse Reactions Reported in 20% of Patients with gBRCA-Mutated Advanced Ovarian Cancer Receiving Lynparza Adverse Reaction Blood and Lymphatic disorders Anemia Gastrointestinal disorders Abdominal pain/discomfort Decreased appetite Nausea Vomiting Diarrhea Dyspepsia General disorders Fatigue/asthenia Infections and infestations Nasopharyngitis/URI Musculoskeletal and Connective Tissue disorders Arthralgia/musculoskeletal pain Myalgia

3 or more lines of prior chemotherapy Grades 1-4 Grades 3-4 N=223 N=223 % % 34

18

43 22 64 43 31 25

8 1 3 4 1 0

66

8

26

0

21 22

0 0

Table 2 presents the frequency of abnormal laboratory findings in the 223 patients with gBRCA-mutated advanced ovarian cancer who had received three or more prior lines of chemotherapy receiving Lynparza 400 mg twice daily. Table 2 Laboratory Abnormalities Reported in Patients with gBRCA-Mutated Advanced Ovarian Cancer Receiving Lynparza Laboratory Parameter* 3 or more lines of prior chemotherapy Grades 1-4 Grades 3-4 N=223 N=223 % % Decrease in hemoglobin (anemia) 90 15 Decrease in absolute neutrophil count 25 7 (neutropenia) Decrease in platelets (thrombocytopenia) 30 3 Decrease in lymphocytes (lymphopenia) 56 17 Mean corpuscular volume elevation 57 Increase in creatinine* 30 2 * Patients were allowed to enter clinical studies with laboratory values of CTCAE Grade 1.

The following adverse reactions and laboratory abnormalities have been identified in 10 to <20% of the 223 patients receiving Lynparza and not included in the table: cough, constipation, dysgeusia, peripheral edema, back pain, dizziness, headache, urinary tract infection, dyspnea, and rash. The following adverse reactions and laboratory abnormalities have been identified in 1 to <10% of the 223 patients receiving Lynparza and not included in the table: leukopenia, stomatitis, peripheral neuropathy, pyrexia, hypomagnesemia, hyperglycemia, anxiety, depression, insomnia, dysuria, urinary incontinence, vulvovaginal disorder, dry skin/ eczema, pruritis, hypertension, venous thrombosis (including pulmonary embolism), and hot flush. Table 3 presents adverse reactions reported in 20% of patients from a randomized trial of Lynparza 400 mg twice daily as maintenance monotherapy compared to placebo in patients with platinum sensitive, relapsed, high-grade serous ovarian cancer following treatment with 2 or more platinum-containing regimens. Table 4 presents the laboratory abnormalities in patients from this randomized trial. Of the 96 patients with gBRCAmutation, 53 received Lynparza, and 43 received placebo. The median duration on treatment with Lynparza was 11.1 months for patients with a gBRCA mutation compared to 4.4 months for patients with gBRCA mutation on placebo. Adverse reactions led to dose interruptions in 26% of those receiving Lynparza and 7% of those receiving placebo; dose reductions in 15% of Lynparza and 5% of placebo patients; and discontinuation in 9% of Lynparza and 0% in placebo patients. One (2%) patient on Lynparza died as a result of an adverse reaction.


ASCOPost.com  |   JULY 25, 2015

PAGE 87

Patient’s Corner

when I unexpectedly became weak and faint while on my job as a home caregiver. A short while later, I noticed a large bruise on my right breast, but I didn’t relate the two incidents. Thinking I must have somehow bruised my breast while caring for my patient, I put it out of my mind. A few days later, I noticed a lump the size of a golf ball in my breast,

which seemed to be growing almost before my eyes. A physical examination and blood tests performed by my primary care physician revealed that I was severely anemic, which explained the bruise and my lightheadedness but not the growing tumor. I was sent to the emergency room of my local hospital for a blood transfu-

Trim: 7.625 x 10.5

LYNPARZATM (olaparib) capsules Table 3 Adverse Reactions Reported in 20% of Patients with gBRCA-Mutated Ovarian Cancer in the Randomized Trial Adverse Reactions Lynparza Placebo N=53 N=43 Grades 1-4 Grades 3-4 Grades 1-4 Grades 3-4 % % % % Blood and Lymphatic disorders Anemia 25 4 7 2 Gastrointestinal disorders Abdominal pain/discomfort 47 0 58 2 Decreased appetite 25 0 14 0 Nausea 75 2 37 0 Vomiting 32 4 9 0 Diarrhea 28 4 21 2 Dyspepsia 25 0 14 0 Dysgeusia 21 0 9 0 General disorders Fatigue (including asthenia, lethargy) 68 6 53 2 Infections and infestations Nasopharyngitis/Pharyngitis/URI 43 0 16 0 Musculoskeletal and Connective tissue disorders Arthralgia/Musculoskeletal pain 32 4 21 0 Myalgia 25 2 12 0 Back pain 25 6 21 0 Nervous system disorder Headache 25 0 19 2 Respiratory, Thoracic, Mediastinal disorders Cough 21 0 14 0 Skin and Subcutaneous Tissue Dermatitis/Rash 25 0 14 0 Table 4 Laboratory Abnormalities in Patients with gBRCA-Mutated Ovarian Cancer in the Randomized Trial Laboratory parameter* Lynparza Placebo N=53 N=43 Grades 1-4 Grades 3-4 Grades 1-4 Grades 3-4 % % % % Decrease in hemoglobin 85 8 58 2 Decrease in absolute neutrophil count 32 8 23 0 Decrease in platelets 26 6 19 0 Mean corpuscular volume elevation 85 44 Increase in creatinine* 26 0 5 0 * Patients were allowed to enter clinical studies with laboratory values of CTCAE Grade 1.

DRUG INTERACTIONS Olaparib is primarily metabolized by CYP3A. Anticancer Agents Clinical studies of Lynparza in combination with other myelosuppressive anticancer agents, including DNA damaging agents, indicate a potentiation and prolongation of myelosuppressive toxicity. Drugs that may Increase Olaparib Plasma Concentrations In patients (N=57), co-administration of itraconazole, a strong CYP3A inhibitor, increased AUC of olaparib by 2.7-fold. A moderate CYP3A inhibitor, fluconazole, is predicted to increase the AUC of olaparib by 2-fold. Avoid concomitant use of strong CYP3A inhibitors (e.g., itraconazole, telithromycin, clarithromycin, ketoconazole, voriconazole, nefazodone, posaconazole, ritinovir, lopinavir/ ritinovir, indinavir, saquinavir, nelfinavir, boceprevir, telaprevir) and moderate CYP3A inhibitors (e.g., amprenavir, aprepitant, atazanavir, ciprofloxacin, crizotinib, darunavir/ ritonavir, diltiazem, erythromycin, fluconazole, fosamprenavir, imatinib, verapamil). If the strong or moderate CYP3A inhibitors must be co-administered, reduce the dose of Lynparza [see Dosage and Administration (2.4) in the full Prescribing Information]. Avoid grapefruit and Seville oranges during Lynparza treatment [see Dosage and Administration (2.4) and Clinical Pharmacology (12.3) in the full Prescribing Information]. Drugs that may Decrease Olaparib Plasma Concentrations In patients (N=22), co-administration of rifampicin, a strong CYP3A inducer, decreased AUC of olaparib by 87%. A moderate CYP3A inducer, efavirenz, is predicted to decrease the AUC of olaparib by 50-60%. Avoid concomitant use of strong CYP3A inducers (e.g., phenytoin, rifampicin, carbamazepine, St. John’s Wort) and moderate CYP3A4 inducers (e.g., bosentan, efavirenz, etravirine, modafinil, nafcillin). If a moderate CYP3A inducer cannot be avoided, be aware of a potential for decreased efficacy of Lynparza [see Clinical Pharmacology (12.3) in the full Prescribing Information]. USE IN SPECIFIC POPULATIONS Pregnancy Pregnancy Category D [see Warnings and Precautions (5.3) in the full Prescribing Information] Risk summary Lynparza can cause fetal harm when administered to a pregnant woman based on its mechanism of action and findings in animals. Olaparib was teratogenic and caused embryo-fetal toxicity in rats at exposures below those in patients receiving the recommended human dose of 400 mg twice daily. If this drug is used during pregnancy, or if a patient becomes pregnant while taking this drug, apprise the patient of the potential hazard to the fetus and the potential risk for loss of the pregnancy. Animal Data In a fertility and early embryonic development study in female rats, olaparib was administered orally for 14 days before mating through to day 6 of pregnancy, which

sion. While I was there, a nurse suggested I have a mammogram, and that’s how I found out I had breast cancer. A physician in the hospital told me that a biopsy showed my cancer was stage III invasive ductal carcinoma and I needed immediate surgery. He delivered the news in such a callous way that I said I needed time to think about it and never went back.

2 resulted in increased post-implantation loss at a dose level of 15 mg/kg/day (with maternal systemic exposures approximately 11% of the human exposure (AUC0-24h) at the recommended dose). In an embryo-fetal development study, pregnant rats received oral doses of 0.05 and 0.5 mg/kg/day olaparib during the period of organogenesis. A dose of 0.5 mg/kg/day (with maternal systemic exposures approximately 0.3% of human exposure (AUC0-24h) at the recommended dose) caused embryo-fetal toxicities including increased post-implantation loss and major malformations of the eyes (anophthalmia, microphthalmia), vertebrae/ribs (extra rib or ossification center; fused or absent neural arches, ribs, and sternebrae), skull (fused exoccipital) and diaphragm (hernia). Additional abnormalities or variants included incomplete or absent ossification (vertebrae/sternebrae, ribs, limbs) and other findings in the vertebrae/sternebrae, pelvic girdle, lung, thymus, liver, ureter and umbilical artery. Some findings noted above in the eyes, ribs and ureter were observed at a dose of 0.05 mg/kg/day olaparib at lower incidence. Nursing Mothers It is not known whether olaparib is excreted in human milk. Because many drugs are excreted in human milk and because of the potential for serious adverse reactions in nursing infants from olaparib, a decision should be made whether to discontinue nursing or to discontinue the drug, taking into account the importance of the drug to the mother. Pediatric Use The safety and efficacy of Lynparza has not been established in pediatric patients. Geriatric Use In clinical studies of Lynparza enrolling 735 patients with advanced solid tumors [the majority (69%) of whom had ovarian cancer] who received Lynparza 400 mg twice daily as monotherapy, 148 (20%) of patients were aged 65 years. The safety profile was similar irrespective of age with the exception of AEs of CTCAE 3 which were reported more frequently in patients aged 65 years (53.4%) than those <65 years (43.4%). No individual adverse event or System Organ Class accounted for this observed difference. Females of Reproductive Potential Lynparza can cause fetal harm when administered to a pregnant woman [see Use in Specific Populations (8.1) in the full Prescribing Information]. Advise female patients of reproductive potential to avoid pregnancy while taking Lynparza. If contraceptive methods are being considered, use highly effective contraception during treatment with Lynparza and for at least one month following the last dose of Lynparza. Instruct patients to contact their healthcare provider if they become pregnant, or if pregnancy is suspected, while taking Lynparza. Hepatic Impairment The effect of hepatic impairment on exposure to Lynparza has not been studied. Patients with bilirubin >1.5 X ULN and AST/ALT 2.5 X ULN ( 5 X ULN in the presence of liver metastases) were excluded from Lynparza clinical trials. There are no data in patients with baseline hepatic impairment (serum bilirubin >1.5 X ULN) [see Clinical Pharmacology (12.3) in the full Prescribing Information]. Renal Impairment Based on preliminary data, a 1.5 fold increase in mean exposure (AUC) was observed in patients with mild renal impairment (CLcr = 50-80 mL/min) compared to patients with normal renal function (CLcr >80 mL/min). No dose adjustment to the starting dose is required in patients with CLcr of 50 to 80 mL/min, but patients should be monitored closely for toxicity. There are no data in patients with moderate or severe renal impairment (CLcr <50 mL/min) or patients on dialysis [see Clinical Pharmacology (12.3) in the full Prescribing Information]. OVERDOSAGE There is no specific treatment in the event of Lynparza overdose, and symptoms of overdose are not established. In the event of an overdose, physicians should follow general supportive measures and should treat symptomatically. 17 PATIENT COUNSELING INFORMATION SEE FDA-APPROVED PATIENT LABELING (MEDICATION GUIDE) • Dosing Instructions: Inform patients on how to take Lynparza [see Dosage and Administration (2.1) in the full Prescribing Information]. Lynparza should be taken twice daily. Instruct patients that if they miss a dose of Lynparza, not to take an extra dose to make up for the one that they missed. They should take their next normal dose at the usual time. Each capsule should be swallowed whole. Do not chew, dissolve, or open the capsule. Patient should not take Lynparza with grapefruit or Seville oranges. • MDS/AML: Advise patients to contact their healthcare provider if they experience weakness, feeling tired, fever, weight loss, frequent infections, bruising, bleeding easily, breathlessness, blood in urine or stool, and/or laboratory findings of low blood cell counts, or a need for blood transfusions. This may be a sign of hematological toxicity or a more serious uncommon bone marrow problem called ‘myelodysplastic syndrome’ (MDS) or ‘acute myeloid leukemia’ (AML) which have been reported in patients treated with Lynparza [see Warnings and Precautions (5.1) in the full Prescribing Information]. • Pneumonitis: Advise patients to contact their healthcare provider if they experience any new or worsening respiratory symptoms including shortness of breath, fever, cough, or wheezing [see Warnings and Precautions (5.2) in the full Prescribing Information]. • Pregnancy and Females of Reproductive Potential: Advise females to inform their healthcare provider if they are pregnant or become pregnant. Inform female patients of the risk to a fetus and potential loss of the pregnancy [see Use in Specific Populations (8.1) in the full Prescribing Information]. Advise females of reproductive potential to use effective contraception during treatment with Lynparza and for at least one month after receiving the last dose of Lynparza [see Warnings and Precautions (5.3) and Use in Specific Populations (8.1, 8.6) in the full Prescribing Information]. • Nursing Mothers: Advise patients not to breastfeed while taking Lynparza [see Use in Special Populations (8.3) in the full Prescribing Information]. • Nausea/vomiting: Advise patients that mild or moderate nausea and/or vomiting is very common in patients receiving Lynparza and that they should contact their healthcare provider who will advise on available antiemetic treatment options. Distributed by: AstraZeneca Pharmaceuticals LP, Wilmington, DE 19850 3079901 12/14 Issued: 12/2014

Experiencing a Miracle Finally, I found my way to a surgical oncologist who clearly explained my health situation and his recommendations for treatment. First, he would perform a lumpectomy to remove the tumor and several lymph nodes to test for spreading cancer cells and then wanted me to have adjuvant chemotherapy and radiation to kill any errant malignant cells. Worried about the damage chemotherapy and radiation might do to my body, I refused the treatment. “Okay,” said my oncologist. “Let’s see what the situation is after the surgery, and then we’ll make a decision.”

Lisa Arzola

When I saw my oncologist for a follow-up visit 7 days after my surgery, he gave me the good news: The pathology report showed that the surgical margin of the tissue and all seven lymph nodes he removed were clear of disease. I was cancer-free. “Lisa, I’ve been in practice for 25 years, and you are my first miracle,” he said. “You no longer have cancer, and I’m amazed.” He still recommended adjuvant chemotherapy and radiotherapy to be safe, but the decision, he said, was mine. I choose not to have additional therapy. I have a strong faith and believe that prayer helped steer me to the right decision, and I have no regrets.

A Caring Medical Team I had so little experience with illness and the world of medicine that I felt especially vulnerable. Until I finally found the surgical oncologist who treated me, none of the previous doctors I met with ever asked me what I knew about cancer or whether I had any questions about the suggested treatment. One nurse I saw, who I’m sure thought she was being helpful, shoved more than a dozen booklets about breast cancer into my hands, but without someone to help me navigate my way through all the information, I was completely lost. Hearing the words “you have cancer” will always be difficult to process, but having a caring medical team to help explain what those words mean makes that process easier. n Lisa Arzola lives in Long Beach, California.


The ASCO Post  |   JULY 25, 2015

PAGE 88

2015

2015 Oncology Meetings July APOS 12th Annual Conference and IPOS 17th World Congress of Psycho-Oncology July 28-August 1 • Washington, DC For more information: www.apos-society.org 16th Annual International Lung Cancer Congress® July 30-August 1 • Huntington Beach, California For more information: www.gotoper.com/conferences/ilc/ meetings/16th-International-LungCancer-Congress Best of ASCO® Boston July 31-August 1 • Boston, MA For more information: http://boa.asco.org/

August Advances in Cancer ImmunotherapyTM August 7 • Washington, DC For more information: www.sitcancer.org

Best of ASCO - San Francisco August 7-8 • San Francisco, California For more information: http://www.keck.usc.edu/events/6thannual-pain-managementsymposium-from-evidence-to-clinicalpractice/ 6th Annual Pain Management Symposium: From Evidence to Clinical Practice August 20-21 • Pasadena, California For more information: http://boa.asco.org/ World Congress on Cancer and Prevention Methods August 27-29 • Dubai, United Arab Emirates For more information: http://scientificfuture.com/ oncology-2015/

ASCO Multidisciplinary Cancer Management Course (MCMC) August 28-29 • Sao Paulo, Brazil For more information: www.asco.org/internationalprograms/multidisciplinary-cancermanagement-courses Best of ASCO - Chicago August 28-29 • Chicago, Illinois For more information: http://boa.asco.org/ European Society for Medical Oncology Academy 2015 August 28-30 • Oxford, United Kingdom For more information: www.esmo.org/Conferences/ ESMO-Academy-2015

September 2015 World Molecular Imaging Congress September 2-5 • Honolulu, Hawaii For more information: www.wmis.org/meetings/ International Palliative Care Workshop September 3-5 • Fez, Morocco For more information: www.asco.org/internationalprograms/international-palliativecare-workshops 25th World Congress of the International Association of Surgeons, Gastroenterologists, and Oncologists September 4-6 • Fuzhou, China For more information: www.csw-iasgo2015.org

The International Liver Cancer Association’s 9th Annual Conference September 4-6 • Paris, France For more information: www.ilca2015.org

16th World Conference on Lung Cancer September 6-9 • Denver, Colorado For more information: http://wclc2015.iaslc.org

ISEH 44th Annual Scientific Meeting September 17-19 • Kyoto, Japan For more information: www.iseh.org/?page=Meeting

25th World Congress of Lymphology September 7-11 • San Francisco, California For more information: www.lymphology2015.com

HPV 2015–30th International Papillomavirus Conference September 17-21 • Lisbon, Portugal For more information: www.hpv2015.org

29th Annual Canadian Association of Radiation Oncology (CARO) Annual Scientific Meeting September 9-12 • Kelowna, Canada For more information: www.caro-acro.ca American Society of Head and Neck Radiology (ASHNR) Annual Meeting September 9-13 • Naples, Florida For more information: http://ashnr.org/meetings/ ashnr-annual-meeting/ Perspectives in Melanoma XIX September 11-12 • Cleveland, Ohio For more information: http://imedex.com/perspectivesmelanoma-conference/index.asp 24th Annual Symposium on Molecular Pathology: Clinical Applications of Genomic Medicine September 16-17 • Troy, Michigan For more information: http://cmetracker.net/BEAUMONT/ The Inaugural International Cancer Immunotherapy Conference: Translating Science Into Survival September 16-19 • New York, New York For more information: http://www.aacr.org/Meetings/Pages/ MeetingDetail.aspx?EventItemID=54 &DetailItemID=309#.VXcJKVXtmko 18th Annual Meeting of the Chinese Society of Clinical Oncology (CSCO) September 16-20 • Xiamen, China For more information: www.csco.ac.cn American Society of Hematology (ASH) Meeting on Hematologic Malignancies September 17-19 • Chicago, Illinois For more information: www.hematology.org/Malignancies/

3rd Annual Hematology/Oncology Pharmacy Association (HOPA) Oncology Pharmacy Practice Management Program September 18-19 • Chicago, Illinois For more information: www.hoparx.org/education/2015Practice-Management-Program/2015practice-management-programwelcome.html 2nd International Symposium of the Cancer Research Center of Lyon (CRCL) September 21-23 • Lyon, France For more information: www.crclsymposium2015.fr 4th Annual Conference on Immunotherapy in Pediatric Oncology (CIPO2015) September 25-26 • Seattle, WA For more information: www.seattlechildrens.org/research/ childhood-cancer/CIPO-2015/ 2015 Breast Cancer Symposium September 25-27 • San Francisco, CA For more information: http://breastcasym.org European Cancer Congress (ECC 2015) September 25-29 • Vienna, Austria For more information: www.esmo.org/Conferences/ European-Cancer-Congress-2015 17th Annual International Meeting of the Institute of Human Virology September 27-30 • Baltimore, MD For more information: http://medschool.umaryland.edu/ ihvmeeting/default.html 5th World Congress on Cancer Therapy September 28-30 • Atlanta, Georgia For more information: http://cancer.global-summit.com/ america/ continued on page 90


In relapsed multiple myeloma,

TO DISRUPT THE CYCLE OF DIMINISHING RESPONSES,

IS DEPTH THE ANSWER ?

Relapsed multiple myeloma: A COMPLEX AND CHALLENGING DISEASE Today, there is no established cure for multiple myeloma (MM). It is characterized by a recurring pattern of relapse.1,2 The majority of patients with relapsed MM do not achieve optimal responses regardless of treatment regimen, and clinical benefit decreases with each subsequent line of therapy.3

Improving long-term outcomes is a primary goal of relapsed MM treatment.3 Relapsed multiple myeloma: AIMING FOR LONG-TERM SUCCESS Deep responses have been found to statistically correlate to progression-free survival across patient type and treatment setting.3,4 In an analysis of 1175 elderly patients with MM, the impact of complete response on long-term outcome was confirmed regardless of baseline patient characteristics, including age.1 Elderly patients and other high-risk patients with relapsed MM may achieve similar response rates to the rest of the population5; therefore, aiming for deep responses (complete response) for all patients should be the goal.1 Achieving and sustaining responses should be a priority in the treatment of relapsed MM.3 A prolonged or durable complete response is an independent posttreatment variable associated with improved survival.6 Continuous treatment, even after an initial response has been achieved, may further deepen the response.7

As the treatment landscape evolves, both deep and durable responses should be the expectation for relapsed multiple myeloma. LEARN MORE GO TO WWW.WHATMATTERSINMM.COM References: 1. Gay F, Larocca A, Wijermans P, et al. Complete response correlates with long-term progression-free and overall survival in elderly myeloma treated with novel agents: analysis of 1175 patients. Blood. 2011;117(11):3025-3031. 2. Mohty B, El-Cheikh J, Yakoub-Agha I, Avet-Loiseau H, Moreau P, Mohty M. Treatment strategies in relapsed and refractory multiple myeloma: a focus on drug sequencing and ‘retreatment’ approaches in the era of novel agents. Leukemia. 2012;26(1):73-85. 3. Chanan-Khan AA, Giralt S. Importance of achieving a complete response in multiple myeloma, and the impact of novel agents. J Clin Oncol. 2010;28(15):2612-2624. 4. Harousseau JL, Attal M, Avet-Loiseau H. The role of complete response in multiple myeloma. Blood. 2009;114(15):3139-3146. 5. Wildes TM, Rosko A, Tuchman SA. Multiple myeloma in the older adult: better prospects, more challenges. J Clin Oncol. 2014;32(24):2531-2540. 6. Barlogie B, Anaissie E, Haessler J, et al. Complete remission sustained 3 years from treatment initiation is a powerful surrogate for extended survival in multiple myeloma. Cancer. 2008;113(2):355-359. 7. Zago M, Oehrlein K, Rendl C, Hahn-Ast C, Kanz L, Weisel K. Lenalidomide in relapsed and refractory multiple myeloma disease: feasibility and benefits of long-term treatment. Ann Hematol. 2014;93(12):1993-1999. Onyx, Onyx Pharmaceuticals, and Onyx Pharmaceuticals logo are all trademarks of Onyx Pharmaceuticals, Inc. ©2015 Onyx Pharmaceuticals, Inc., South San Francisco, CA TROPIC-ONYX-101576 March 2015 Printed in USA


The ASCO Post  |   JULY 25, 2015

PAGE 90

2015

2015 Oncology Meetings continued from page 88

October Advances in Cancer ImmunotherapyTM October 2 • Nashville, Tennessee For more information: www.sitcancer.org/sitc-meetings/ aci2015/tn 5th International Breast Cancer Prevention Symposium October 2-3 • Le Gosier, Guadeloupe, French West Indies For more information: www.purdue.edu/breastcancer/

Congress of the International Society of Pediatric Oncology October 8-11 • Cape Town, South Africa For more information: http://siop2015.kenes.com

ESGO 2015-International Meeting of the European Society of Gynaecological Oncology October 24-27 • Nice, France For more information: http://esgo2015.esgo.org

Palliative Care in Oncology Symposium October 9-10 • Boston, Massachusetts For more information: http://pallonc.org

Lynn Sage Breast Cancer Symposium October 29-November 1 • Chicago, Illinois For more information: www.lynnsagebreastcancer.org

National Comprehensive Cancer Network (NCCN) 10th Annual Congress: Hematologic Malignancies™ October 16-17 • San Francisco, California For more information: www.nccn. org/professionals/meetings/ hematological/default.aspx

CAP ’15-The Pathologists’ MeetingTM (College of American Pathologists) October 4-7 • Nashville, Tennessee For more information: www.thepathologistsmeeting.org American College of Surgeons Clinical Congress October 4-8 • Chicago, Illinois For more information: www.facs.org/meetings_events/ future_congress/future

ASTRO’s 57th Annual Meeting October 18-21 • San Antonio, Texas For more information: www.astro.org/Meetings-andEvents/2015-Annual-Meeting/Index. aspx

30th Anniversary Annual Critical Issues in Tumor Microenvironment: Angiogenesis, Metastasis and Immunology October 5-8 • Cambridge, Massachusetts For more information: http://cmeregistration.hms.harvard. edu/events/30th-anniversaryannual-critical-issues-in-tumormicroenvironment-angiogenesismetastasis-and-immuno/ event-summary-567a0dd5664947b09 6f8a5fd33946e52.aspx

2015 International Cancer Education Conference October 21-23 • Tucson, Arizona For more information: http://2015.attendicec.org

20th World Congress on Advances in Oncology and 18th International Symposium on Molecular Medicine October 8-10 • Athens, Greece For more information: www.spandidos-publications.com/ pages/conference

ACCC 32nd National Oncology Conference October 21-24 • Portland, Oregon For more information: www.accc-cancer.org/meetings/ calendar.asp 13th Annual West Coast Colorectal Cancer Symposium October 23 • Seattle, Washington http://www.swedish.org/for-healthprofessionals/cme/conferences/ colorectal-cancer-symposium 53rd Annual Meeting of the Japan Society of Clinical Oncology (JSCO) October 24-26 • Kyoto, Japan For more information: www.jsco. or.jp/english/index/page/id/73

November NRCI Cancer Conference November 1-4 • Liverpool, UK For more information: http:// conference.ncri.org.uk Society for Immunotherapy of Cancer 30th Anniversary Annual Meeting November 4-8 • National Harbor, MD For more information: www.sitcancer.org/2015 JADPRO Live at APSHO for Advanced Practitioners in Oncology November 5-8, 2015 • Phoenix, AZ JW Marriott Desert Ridge For more information: jadprolive.com Advanced Breast Cancer Third International Consensus Conference November 5-7 • Lisbon, Portugal For more information: www.abc-lisbon.org 14th International Kidney Cancer Symposium November 6-7 • Miami, Florida For more information: http://registeruo.niu.edu/ iebms/wbe/wbe_p1_main. aspx?oc=40&cc=WBE4014167 ESMO Summit Americas 2015– Oncology Updates: From Evidence to Practice November 6-8 • Miami, Florida For more information: www.esmo.org/Conferences/ESMOSummit-Americas-2015 10th Annual New York Lung Cancer Symposium November 7 • New York, New York For more information: http://www. gotoper.com/conferences/nyl/ meetings/10th-Annual-New-YorkLung-Cancer-Symposium

Best of ASTRO November 13-14 • San Diego, California For more information: www.astro.org/Meetings-andEvents/2015-Best-of-ASTRO/Index. aspx 2015 Oncologic Emergency Medicine Conference November 13-14 • Houston, Texas For more information: http:// www.mdanderson.org/educationand-research/education-andtraining/schools-and-programs/ cme-conference-management/ conferences/d114243-2015oncologic-emergency-medicineconference.html Society for Integrative Oncology 12th International Conference November 14-16 • Boston, Massachusetts For more information: http://www.integrativeonc.org/ conference 12th International Conference of the Society for Integrative Oncology November 15-16 • Boston, Massachusetts For more information: www. integrativeonc.org/index.php/events Advances in Cancer ImmunotherapyTM November 18 • San Francisco, California For more information: www.sitcancer.org/sitc-meetings/ aci2015/casf 12th International Congress of the Society for Melanoma Research November 18-21 • San Francisco, California For more information: http://www. melanomacongress.com/ 20th Annual Scientific Meeting of the Society for Neuro-Oncology November 19-22 • San Antonio, Texas For more information: www.soc-neuro-onc.org ESMO Symposium on Immuno-Oncology November 20-21 • Lausanne, Switzerland For more information: www.esmo.org/Conferences/ Immuno-Oncology-2015


• Professional oncology social workers • Counseling and support groups

• Financial assistance • Resources and education

Access our services by telephone, face-to-face and online. All CancerCare services are free of charge.


The ASCO Post  |   JULY 25, 2015

PAGE 92

In Memoriam

Preeminent Cancer Researcher Brian E. Henderson, MD, Dies By Ronald Piana

Brian E. Henderson, MD

B

rian E. Henderson, MD, began his medical path as a researcher in virology, and as a young scientist, he ventured to Africa as part of a Centers for Disease Control team to study yellow fever. The better part of his esteemed medical career, however, was as one of the world’s most respected authorities in cancer epidemiology. Dr. Henderson died on June 20, 2015. He was 77 years old. He had been a Professor at the University of Southern California (USC) School of Medicine since 1970. From 1983 until 1994, he served as Director of the Kenneth Norris Jr. Comprehensive Cancer Center at USC, and from 2004 to 2007, he was Dean of USC’s Keck School of Medicine.

Early Life and Education Dr. Henderson was born on June 27, 1937, in San Francisco. His father, O’Brien Henderson, had emigrated from Ontario, Canada, in 1921 where he settled in the San Francisco area,

meeting a young woman who would soon become his wife. The couple moved to San Leandro, California, where Brian and his two siblings were born. Dr. Henderson graduated from San Leandro High School in 1954, where he was president of the science club and a star center on the high school’s football team. He would remain a lifelong lover of sports and a devoted fan of San Francisco’s pro teams, the baseball Giants and football 49ers. In 1957, Dr. Henderson graduated from the University of California, Berkeley, with a major in English. While attending the University of California,

to Africa to study yellow fever, and in 1969 he was part of a U.S. delegation on hemorrhagic fevers that went to the Soviet Union. He went on to join the first U.S. scientific delegation to the People’s Republic of China. However, in 1970, after President Nixon signed the National Cancer Act into law, Dr. Henderson turned his sights on cancer and joined the Keck School of Medicine as Associate Professor of Pathology. In 1972, Dr. Henderson set up the Los Angeles Cancer Surveillance Program—the county’s cancer registry—at USC, where it remains today as a much-coveted resource

In the course of my travels, I feel fortunate to have been taught and to have received more than I could possibly have given. —Brian E. Henderson, MD

San Francisco, Medical School in 1960, Dr. Henderson met Judith McDermott, a TWA stewardess. They would marry that same year. The couple moved to Chicago, where Dr. Henderson completed his studies, receiving his medical degree from the University of Chicago Medical School. Dr. Henderson completed his internship and residency at Massachusetts General Hospital, Boston.

Adventurous Spirit Early on in his career, Dr. Henderson’s adventurous spirit carried him

to researchers nationwide. The registry has elucidated countless medical issues, including the link between combination hormone replacement therapy and increased breast cancer risk in postmenopausal women as well as how intense smokers are more likely to develop aggressive bladder cancers. Two decades later, Dr. Henderson established the Hawaii–Los Angeles Multiethnic Cohort, an influential study following 215,000 people that has led to numerous influential findings on cancer risk factors. The

 In Memoriam

Brian E. Henderson, MD 1937 – 2015 

Cohort’s work continues today, with dozens of papers published from the study each year.

Intellectual Force of Nature In a statement, nationally recognized oncologist and researcher Stephen B. Gruber, MD, PhD, MPH, Director of the USC Norris Comprehensive Cancer Center, said: Brian Henderson was an intellectual force of nature who had many gifts to share both through science and in simple acts of kindness. The world is truly a better place because of Brian Henderson. He changed the course of American medicine and was among the first to recognize that many cancers could be understood, prevented, and managed with hormones. There is no question that he saved the lives of countless men and women.

In 1992, Dr. Henderson was inducted into the Institute of Medicine of the National Academy of Sciences. In 2004, he was one of the first scientific experts selected to the oversight committee for the California Institute of Regenerative Medicine. When asked about his career and work during an interview, Dr. Henderson said, “In the course of my travels, I feel fortunate to have been taught and to have received more than I could possibly have given.” He remained an active researcher and valued colleague until a few weeks prior to his death. Dr. Henderson is survived by his wife, 5 children, and 11 grandchildren. n


ASCOPost.com  |   JULY 25, 2015

PAGE 93

In Memoriam

Michael O’Malley, PhD, MPH, Former UNC Lineberger Associate Director, Dies at 64 In 1990, he took a leadership position at UNC Lineberger, where he served until his retirement in 2014. In recognition of his remarkable administrative

skill and research accomplishments, he was awarded the C. Knox Massey Award in 2007. A memorial service to honor Dr.

O’Malley will be held on Saturday, ­August 22 in the auditorium of the Genome Sciences Building (room G200) on the UNC campus. n

Early Bird Rate Michael O’Malley, PhD, MPH

M

ichael O’Malley, PhD, MPH, former Associate Director of the University of North Carolina (UNC) at Chapel Hill’s Lineberger Comprehensive Cancer Center and former Adjunct Associate Professor of Health Policy and Management at UNC’s Gillings School of Global Public Health, has passed away unexpectedly at the age of 64. Dr. O’Malley committed over 30 years of service to the state of North Carolina and was an exceptional leader, who many knew as a great “connector.” At the Gillings School, he served as a mentor for countless students over the years. He also Codirected the Cancer Control Education Program, an National Cancer Institute–funded pre- and postdoctoral training program in cancer prevention and control. Though he semi-retired last year, Dr. O’Malley still played a pivotal role in advising the cancer center leadership. He began his career in medical research and service roles in UNC School of Medicine and UNC Hospitals. This interest led him to obtain an MPH and a PhD in Health Policy and Management at the UNC Gillings School of Global Public Health.

NCCN 10th Annual Congress:

Hematologic Malignancies™

Expires September 11th

October 16 – 17, 2015 | San Francisco Marriott Marquis | San Francisco, CA

Register at NCCN.org/hem

NEW Location!

Attend 13 educational sessions featuring the latest advances in hematologic malignancies, plus 3 case-based panel discussions!

Co-Chairs:

Topics include*: • Advances in Waldenström’s Macroglobulinemia Ranjana H. Advani, MD Stanford Cancer Institute

• Bone Health in Patients with Multiple Myeloma • Diagnosis and Management of Castleman Disease • Evolving Targeted Management of Acute Myeloid Leukemia • Evolving Therapies for Follicular Lymphoma • Management of Acute Lymphoblastic Leukemia

Andrew D. Zelenetz, MD, PhD Memorial Sloan Kettering Cancer Center

• Management of HIV-associated Non-Hodgkin’s Lymphomas • Management of Multiple Myeloma • Management of Myelodysplastic Syndromes • Optimizing Small Molecular Inhibitor Therapy for Chronic Lymphocytic Leukemia • PET-Guided Treatment Approach for Advanced Stage Classical Hodgkin Lymphoma • Survivorship Issues: Late Effects of Curative Therapy in Lymphoma Survivors

 In Memoriam

• Targeted Therapies for Relapsed or Refractory Classical Hodgkin Lymphoma

Michael O’Malley, PhD, MPH

1951 – 2015 

• Patient Case Studies & Panel Discussions - New in 2015! *Agenda topics are subject to change.

Reserve your seat today:

NCCN.org/hem

This program is approved for AMA PRA Category 1 Credit™ and is also certified for nurses and pharmacists.

Sponsorship and exhibit opportunities available! For more information, e-mail exhibits@nccn.org. JNCCN-N-0209-0715


The ASCO Post  |   JULY 25, 2015

PAGE 94

In Memoriam

Irwin Rose, PhD, Nobel Laureate and Biochemist, Dies at 88

S

cientist and Nobel Laureate I­ rwin “Ernie” Rose, PhD, passed away June 2, 2015, after a long illness. He was 88. Dr. Rose shared the 2004 Nobel Prize for chemistry with Aaron Ciechanover, MD, DSc, and Avram

Hershko, MD, PhD, of the Israel Institute of Technology for their pioneering work in discovering the ubiquitin conjugating system. Dr. Rose conducted this research as a senior scientist at Fox Chase Cancer Center,

where he worked from 1963 until he retired in 1995. “Dr. Rose was a collaborator in the truest sense of the word,” said R ­ ichard Fisher, MD, President and CEO of Fox Chase Cancer Center. “He contributed

Irwin “Ernie” Rose, PhD

Need MOC Points?

ASCO’s Got You Covered MOC App Earn points quickly, easily, and conveniently without interrupting your busy schedule. Works with iPhone and Android devices. A mobile website is available for users on other platforms (including computers).

ASCO-SEP®, 4th Edition: MOC Course A subset of 60 multiple choice questions from the ASCO-SEP, 4th edition that include patient case information, educational links, and answer rationales.

Online Courses Several courses are available that include 30 case-based, multiple choice questions with detailed answer rationales and links to suggested readings.

Meeting-Based Opportunities Attending an ASCO meeting or viewing the Virtual Meeting? Earn MOC points for learning already being pursued. MOC Self-Assessment Activities available in conjunction with the Annual Meeting and Symposia.

Have questions about MOC?

WE’RE HERE TO HELP Email university@asco.org

university.asco.org/MOC

to many advances in the field of biochemistry, and we were extremely fortunate to have had him as an esteemed member of our scientific faculty.”

Overseeing a Biochemical Breakthrough Dr. Rose forged an unparalleled career as one of the world’s top biochemists. He earned a doctoral degree at the University of Chicago in 1952 and spent his career on research that revealed how ubiquitin molecules facilitate the breakdown of old and damaged proteins. These findings of the “kiss-of-death” mechanisms inside cells proved revolutionary, transforming the field of biology and ultimately fostering a new understanding of the molecular activity involved with cancer and other diseases. “Dr. Rose had a genius for asking the right questions,” said Jonathan ­Chernoff, MD, PhD, Deputy Director and Chief Scientific Officer at Fox Chase Cancer Center. “In the mid-1950s, when many scientists were interested in how proteins are synthesized, he became fascinated with the opposite issue—how are proteins degraded? With the collaboration of his Israeli colleagues, he cracked that problem with the discovery of the ubiquitin conjugating system. He was the quintessential scientist—perseverant, soft-spoken, and interested in science for science’s sake.” Dr. Rose’s laboratory discoveries led to the development of cancer drugs such as bortezomib (Velcade). “Dr. Rose was not interested in personal fame, and was oblivious to the politics of science; his total satisfaction came from solving intricate biochemical puzzles,” said Ann Skalka, PhD, William Wikoff Smith Chair in Cancer Research at Fox Chase Cancer Center. “Although he was an intellectual leader on the project that ultimately won him the Nobel, he took no personal credit. He was rather surprised at being recognized, but all of us at Fox Chase knew that the Nobel Committee had gotten it right.” n


ASCOPost.com  |   JULY 25, 2015

PAGE 95

In the Literature

Emerging Clinical Data on Cancer Management LEUKEMIA Durable Responses at 3-Year Follow-up for CLL Patients Receiving Single-Agent Ibrutinib At a median follow-up of 3 years, ibrutinib (Imbruvica) demonstrated continued activity with durable responses that improved in quality with extended treatment of patients with chronic lymphocytic leukemia (CLL). In addition, grade 3 toxicity and adverse events leading to treatment discontinuation diminished over time. Collectively, the data from the extended follow-up of 132 patients with symptomatic treatment-naive and relapsed/refractory CLL or small lymphocytic leukemia (SLL) “provide evidence that ibrutinib controls CLL disease manifestations and is well tolerated for an extended period,” John C. Byrd, MD, of The Ohio State University in Columbus, and colleagues reported in Blood. Single-agent ibrutinib, an orally administered inhibitor of Bruton’s tyrosine kinase, was approved by the U.S. Food and Drug Administration “for patients with CLL who have received at least one prior therapy and for all patients with del (17p) CLL,” the investigators noted. “As ibrutinib is a continuously administered oral once-daily therapy, data addressing the safety profile of ibrutinib over time, longer-term outcomes for those achieving partial response with lymphocytosis, and efficacy in patient subgroups become increasingly relevant.” The extension study included 31 patients aged ≥ 65 years who were symptomatic but treatment naive and 101 patients who had relapsed/refractory disease. All had completed a minimum of six treatment cycles of ibrutinib and had no evidence of disease progression. The median age of the patients was 68, and 43% were ≥ 70 years old. Most patients (74%) were male. “Treatment consisted of 420 or 840 mg/d of ibrutinib administered orally until progressive disease or poor tolerance,” the investigators stated. “Median time on treatment was 30 months (range, 0.3–44) for treatment-naive patients, with 81% remaining on study treatment, and 23 months (range, 0.3–45) for relapsed/ refractory patients, with 53% remaining on study treatment. For treatment-naive and relapsed/refractory patients, 81% and 46% received ibrutinib for > 2 years, respectively,” the researchers reported. “The primary endpoint of this analysis was safety, as assessed by the fre-

quency and severity of grade 3 adverse events, serious adverse events, and adverse events requiring dose reduction or discontinuation,” the authors explained. The most common adverse events ob-

served over 3 years of follow-up were hypertension, which occurred in 23% of treatment-naive and 20% of relapsed/refractory patients, and pneumonia, which occurred in 6% of treatment-naive and

25% of relapsed/refractory patients. “For grade 3 adverse events occurring in 5% of patients during years 1, 2, and 3 on therapy, the frequency of pneumonia, continued on page 96

The Association of Physician Assistants in Oncology presents

APAO 18th ANNUAL CONFERENCE

TOPICS INCLUDE: • An interactive bone marrow biopsy workshop • A free quality improvement workshop on the Seven Tools of Quality Improvement • Hill Day: A Chance for You to Speak With Legislators to Improve the PA Industry • Multidisciplinary lecture format with leading medical, surgical and radiation oncologists discussing the latest developments in oncologic care. • Opportunities to network with other PAs and NPs specializing in oncology. • Best of ASCO Lectures FACULTY (Includes Physicians, PAs, PharmDs, RNs, & NPs from): • Dana-Farber Cancer Institute • AAPA • UTHSC • Memorial Sloan Kettering Cancer Center • NorthShore University Health System • New York Presbyterian Hospital • University of Cailfornia • Steward Healthcare Network • Moffitt Cancer Center • University of Michigan

HOTEL INFORMATION: Westin Alexandria APAO Discounted Rate: $169/night plus tax. Call 703-253-8600 and be sure to mention APAO to get the discounted rate. Reserve your room by August 10, 2015 to secure the APAO discounted rate.

Westin Alexandria • Alexandria, VA • September 17-20, 2015

www.apao.cc

apao_ann15_ascoad.indd 1

6/30/15 2:29 PM


The ASCO Post  |   JULY 25, 2015

PAGE 96

In the Literature Emerging Clinical Data continued from page 95

neutropenia, thrombocytopenia, diarrhea, and fatigue generally decreased over time, whereas the frequency of hypertension and atrial fibrillation appeared constant. Overall, treatment-naive patients generally experienced fewer grade > 3 toxicities, particularly infectious and hematologic toxicities, compared with the previously treated cohort,” the investigators wrote. “Dose reductions due to adverse events were reported for 13 patients and occurred primarily during the first year of treatment,” the authors added. “The primary reasons for discontinuing therapy for all patients included disease progression in 22 patients (17%) and adverse events in 17 patients (13%).” Only four adverse events requiring treatment discontinuation “were possibly related to ibrutinib treatment,” the researchers noted, and all were resolved.

Response and Survival Additional endpoints included were overall response rate, achieved by 84% of previously untreated and 90% of relapsed/ refractory patients, and complete response, achieved by 23% of treatment-naive and 7% of relapsed/refractory patients. “Response quality evolves over time, as the median time to complete response was 21.2 months (range, 4.6–42.5),” the investigators observed. With extended followup, 94% of patients who achieved partial response with lymphocytosis converted to either complete or partial response. Median progression-free survival and overall survival times were not reached for all patients. At 30 months, the estimated progression-free survival rate was 96% for treatment-naive CLL/SLL patients and 69% for previously treated patients. The estimated 30-month overall survival rates were 97% for treatment-naive CLL and

79% for previously treated patients. “Progression remains uncommon, occurring primarily in some patients with relapsed del(17)(p13.1) and/or del(11) (q22.3) disease,” the investigators noted. The 28.1-month progression-free survival “observed in the 34-patient cohort of relapsed patients with del(17p), with a median of four prior therapies, surpasses that observed with any therapy published for this group, including those receiving first-line treatment. Ibrutinib represents a significant advance in the treatment options available for these patients,” the researchers wrote. “Notably, with 3 years of follow-up, 81% of treatment-naive patients continue to receive daily ibrutinib. There have been no further relapses for over 2 years. This gives some suggestion that the best outcomes may be seen when ibrutinib is administered as first-line rather than salvage therapy,” the authors stated. Byrd JC, et al: Blood. February 23, 2015 (early release online).

THYROID CANCER Thyroid Cancer Rarely Diagnosed in Those With Asymptomatic, Benign Nodules A prospective, multicenter, observational study involving 992 consecutive patients with one to four asymptomatic, sonographically or cytologically benign thyroid nodules found that “the majority of nodules exhibited no significant size change during 5 years of follow-up or actually decreased in size” and “thyroid cancer was rare,” occurring in just 0.3% of nodules. The authors of the study, published in JAMA, concluded, “These findings justify reconsideration of the current guideline recommendations for follow-up

©Michael Crawford/The New Yorker Collection/www.cartoonbank.com

of asymptomatic thyroid nodules.” Patients for the study were recruited at eight hospital-based thyroid-disease referral centers in Italy between 2006 and 2008. The mean age of the patients was 42.4 years, 82% were women, and 42.6% had a family history of nodular thyroid disease. Yearly thyroid ultrasound examinations assessed baseline nodule growth, the primary endpoint. “Size changes were considered significant for growth if an increase of 20% or more was recorded in at least two nodule diameters, with a minimum increase of 2 mm,” Cosimo Durante, MD, PhD, of the Universita di Roma Sapienza, Rome, explained. The secondary endpoints were sonographic detection of new nodules and the diagnosis of thyroid cancer during follow-up.

Major Findings Data analyzed during the first 5 years of follow-up, through January 2013, showed that thyroid cancer was diagnosed in five original nodules, and only two of those had grown. An incidental cancer was found at thyroidectomy in a nonvisualized nodule. New nodules developed in 93 patients (9.3%), and one cancer was detected. “Growth considered significant by American Thyroid Association standards was observed in only 15% of the patients,” the researchers reported. “Growth was slow, steady, and limited in magnitude, with a mean 5-year largest diameter increase of 4.9 mm, and which was generally restricted to the main nodule in patients with multinodular disease. Nodule size changes occurred early, starting from the 1-year follow-up visit.” Nodule growth was associated with the presence of multiple nodules, with odds ratios of 2.2 for two nodules, 3.2 for three nodules, and 8.9 for four nodules. Other factors associated with nodule growth included nodule diameters of 7.5 mm or more and male gender. An age of 60 years or older was associated with a lower risk of growth than an age younger than 45 years. Nodules shrank spontaneously in 184 individuals (18.5%). “Current guidelines suggest, based on expert opinion, repeating thyroid ultrasonography after 6 to 18 months and, if nodule size is stable, every 3 to 5 years,” the study authors noted. “The indolent behavior and limited growth observed in our study confirm that nodules that were benign based on initial fine-needle aspiration or subcentimeter and sonographically nonsuspicious can be safely managed with a second ultrasound examination 1 year after the first (early follow-up) and in the absence of changes, reassessment after 5 years (long-term follow-up). This

approach should be suitable for 85% of patients, whose risk of disease progression is low. Closer surveillance may be appropriate for nodules occurring in younger patients or older overweight individuals with multiple nodules, large nodules (> 7.5 mm), or both.” According to an accompanying editorial by Anne R. Cappola, MD, ScM, and Susan J. Mandel, MD, MPH, of the Perelman School of Medicine at the University of Pennsylvania, Philadelphia, “Thyroid nodules are pervasive, whereas thyroid cancer is not. The findings from Durante et al represent an important step in improving the efficiency and mitigating the expense of follow-up for the vast majority of thyroid nodules that are either cytologically or sonographically benign.” These authors pointed out that the study has four important implications for the follow-up of thyroid nodules. “First, these prospective data provide reassurance about the validity of a benign cytology result obtained by ultrasound-guided fine-needle aspiration and confirm a very low false-negative rate, at 1.1%,” they wrote. “Second, the practice of routine sonographic surveillance with repeat fineneedle aspiration for growth, as recommended by published guidelines, is not the most efficient strategy to detect the very small number of missed cancers among previously sampled cytologically benign nodules. The one-size-fitsall approach simply does not work. Instead, surveillance strategies should be individualized based on a nodule’s sonographic appearance,” they continued. “Third, many nodules detected on ultrasound are small (ie, < 1 cm) and not sonographically suspicious. In fact, 54% of nodules followed up in this study were initially classified as benign not through fine-needle aspiration but because they were smaller than 1 cm and lacked suspicious sonographic features. How reliable is the absence of these features at predicting benign disease? The answer is excellent.” As the fourth and final implication, the editorialists noted that although 69% of the nodules remained stable in size, “size increase was not a harbinger of malignancy, especially if the nodule had no sonographically suspicious features.… An important consideration, however, was that the Italian population studied had mildto-moderate iodine insufficiency, which could differ in thyroid nodule growth rates from iodine-sufficient populations, such as in the United States.” n Durante C, et al: JAMA 313:926-935, 2015. Cappola AR, Mandel SJ: JAMA


CALL FOR ABSTRACTS

Cancer Survivorship Symposium: Advancing Care and Research

A Primary Care and Oncology Collaboration JANUARY 15-16, 2016

SAN FRANCISCO MARRIOTT MARQUIS, SAN FRANCISCO, CA

SUBMIT BY SEMPTEMBER 1, 2015

Submit an abstract for presentation at the inaugural Cancer Survivorship Symposium: Advancing Care and Research. This innovative educational event will feature research presentations, educational sessions, keynote lectures, and networking events to address the needs of clinicians, researchers, and health professionals who are interested in cancer survivorship care.

ABSTRACT SUBMISSION DEADLINE: SEPTEMBER 1, 2015, AT 11:59 PM (EDT) The Program Committee seeks abstracts on the following topic areas: CARE COORDINATION AND FINANCIAL IMPLICATIONS

LATE-AND LONG-TERM EFFECTS/ COMORBIDITIES

• Chronic Care Management • Cost of Care • Educational Curriculum • Financial Toxicity • Hospice • Issues, Trends, and Statistics in Health Care Access and Survivorship • Models of Care/Medical Homes • Payment/Reimbursement Models • Quality Assurance • Survivorship Care Plans

• Fertility Preservation • Long-term Complications/Sequelae of Treatment (Noncancer) • Onco-cardiology • Rehabilitation Services and Function • Screening for Late-/Long-term Effects • Short-term Complications • Symptom Management

COMMUNICATION • Communicating About Advanced Care Planning and End-of-Life Decisions • Communicating with Patients/Families/ Caregivers • Oncology/Primary Care Communication Coordination

HEALTH PROMOTION • Complementary Therapy, Vitamins, NSAIDs, etc. in Cancer Survivors • Energy Balance: Diet, Exercise, and Metabolic Syndrome • Smoking Cessation

PSYCHOSOCIAL ISSUES • Addressing the Emotional Needs of Families and Caregivers • Psychological and Social Well-being • Quality-of-Life Issues • Spirituality

RECURRENCE AND SECONDARY MALIGNANCIES • Detection of Recurrence and Secondary Malignancies • Genetic Risk Assessment for Prevention of Second Primaries and Cancer Recurrence • Prevention of Secondary Cancers • Treatment of Recurrent Cancer

survivorsym.org


The ASCO Post  |   JULY 25, 2015

PAGE 98

Perspective Collaborating Toward a Cure continued from page 1

biotech and pharmaceutical industries, academia, and the cancer community to inject speed and efficiency into the process of discovering and developing new drugs for the t­reatment of multiple myeloma. Through this collective approach, we’ve seen the development of seven new treatments—breakthrough innovations that have almost tripled patients’ lifespan and have given patients such as journalist Tom Brokaw and thousands of others living with this incurable disease a fighting chance. Now, we believe this same collaborative approach can be used to usher in a new era of precision medicine and its promise of a cure.

drug targets, like BRAF, previously never linked to myeloma but for which treatments have already been approved for melanoma or are now in clinical trials. Today, through the CoMMpass study, our team is tracking the progress of 1,000 patients for at least 5 years, analyzing their genetic and clinical

explore new and potentially more effective ways of treating myeloma. (To access our Researcher Gateway, visit https://research.themmrf.org/.) At the same time, we have built a worldwide network of research institutions and cancer centers that are working collectively to test an arsenal of new

At the Multiple Myeloma Research Foundation, collaboration is not just a priority; it is the only way we do things. —Kathy Giusti

Making Precision Medicine a National Priority

The Genomics of Myeloma We began by building a robust data ecosystem, including the first multicenter multiple myeloma tissue bank. This allowed us to sequence the myeloma genome and then launch the MMRF CoMMpass (Relating Clinical Outcomes in Multiple Myeloma to Personal Assessment of Genetic Profile) study, the largest long-term genomic research study ever conducted in myeloma to have data outputs placed in the public domain. Collaborating with key partners like the Translational Genomics Research Institute (TGen) and the Broad Institute of MIT and Harvard, we uncovered deeper insights into the disease’s biology, which revealed important new

ping up to share knowledge they have gained on their cancer journeys with fellow patients—including sequencing data, when available—so they can learn which type of myeloma they have and share their data to advance research. We consider these patients as partners in our search for better, more targeted treatments. Our future hope is that these same patients, as they eventually have their genomes sequenced and learn which, if any, mutations they have, can connect with other patients, engage with researchers who specialize in that particular area of genomics, and sign up for clinical trials that might be appropriate for them.

data to better understand how certain factors influence response to treatment and disease progression and to generate hypotheses for genomically informed clinical trials. We then openly and freely shared these data through a global learning network, so brilliant minds from all corners of the scientific community— such as experts at the big data analytics company GNS Healthcare—can collaboratively decipher patterns for further exploration. This network was built to democratize data, allowing researchers and clinicians worldwide to share and contribute to a growing knowledge base that can be used to generate hypotheses for clinical trials and rapidly

drugs and combination therapies. So far, 30 new therapies have been in clinical trials within our network, including immunotherapies, targeted therapies, and drugs that work in entirely new ways to stop cancer in its tracks. We are also working closely with partners in the pharmaceutical and biotech industries and regulatory officials to leverage new clinical trial designs to speed these new treatments to patients.

Partnering With Patients Finally, we are building a massive community of patients, who are every bit a part of curing the disease as scientists. These patients are step-

San Simeon, California

Mating season at the Piedras Blancas Rookery in San Simeon, California. Thousands of elephant seals convene on the shore for birthing, breeding, and barking. It’s a noisy crowd! Photo by Chase Doyle, Copyright 2015. ChaseDoyle.com. Send your photo and caption to editor@ASCOPost.com.

This approach is so promising to advancing more effective treatments and cures for diseases like cancer that President Barack Obama has made precision medicine a national priority, and it is an honor to serve on the President’s advisory board for the groundbreaking Precision Medicine Initiative. Launched with a $215 million investment in the President’s 2016 budget, the Precision Medicine Initiative aims to accelerate biomedical discoveries and provide clinicians with new tools, knowledge, and therapies to select which treatments will work best for individual patients. The near-term goal is to create more and better treatments for cancer. In the longer term, the project would provide information on how to individualize treatment for a range of diseases. At the heart of the program is the national study involving the health records and DNA of 1 million volunteers to determine how peoples’ genes, environment, and lifestyle affect their health. The $130 million study will recruit new volunteers as well as merge data from more than 200 large ongoing American health studies, together involving at least 2 million people. The investment also allocates $70 million for DNA-driven research on cancer and another $10 million for the U.S. Food and Drug Administration to improve the regulation of genome tests. These programs speak to the incredible power of working collaboratively and adapting and maximizing the use of new technologies to ensure continued progress. Only then will the promise of personalized cancer care become a reality—not years from now but today. We look forward to drawing insights from one another, as we work urgently toward a cure. n Disclosure: Ms. Giusti reported no potential conflicts of interest.


Tell Your NP & PA to

REGISTER TODAY!

REGISTER BY SEPTEMBER 18 AND SAVE!

A CE EVENT FOR

Advanced Practitioners

in Oncology NOVEMBER 5-8, 2015 JW Marriott Desert Ridge Phoenix, Arizona Learn more at jadprolive.com

This CE/CME/CEU accredited conference is jointly provided by:


In cancer immunotherapy research,

Science is changing the score. Starting with the PD-L1 pathway, Genentech is taking a dynamic approach to research by exploring how to modulate the antitumor immune response.

Find out more at Explore.ResearchCancerImmunotherapy.com.

Š 2015 Genentech USA, Inc. All rights reserved. PDL/032315/0024(2) Printed in USA.


Issuu converts static files into: digital portfolios, online yearbooks, online catalogs, digital photo albums and more. Sign up and create your flipbook.